libvolume5.xyzlibvolume5.xyz/law/ballb/3rdyear/semester5/corporatelaw/corporate... · abe advanced...

398
Advanced Diploma in Business Administration Study Manual CORPORATE FINANCE The Association of Business Executives William House 14 Worple Road Wimbledon London SW19 4DD United Kingdom Tel: + 44(0)20 8879 1973 Fax: + 44(0)20 8946 7153 E-mail: [email protected] www.abeuk.com

Upload: truongtruc

Post on 10-Mar-2018

221 views

Category:

Documents


5 download

TRANSCRIPT

Page 1: libvolume5.xyzlibvolume5.xyz/law/ballb/3rdyear/semester5/corporatelaw/corporate... · ABE Advanced Diploma in Business Administration Study Manual CORPORATE FINANCE Contents Study

Advanced Diploma in

Business Administration

Study Manual

CORPORATE FINANCE

The Association of Business Executives

William House • 14 Worple Road • Wimbledon • London • SW19 4DD • United Kingdom Tel: + 44(0)20 8879 1973 • Fax: + 44(0)20 8946 7153

E-mail: [email protected] • www.abeuk.com

Page 2: libvolume5.xyzlibvolume5.xyz/law/ballb/3rdyear/semester5/corporatelaw/corporate... · ABE Advanced Diploma in Business Administration Study Manual CORPORATE FINANCE Contents Study

© Copyright assigned to ABE

All rights reserved

No part of this publication may be reproduced, stored in a retrieval system, or transmitted in any form, or by any means, electronic, electrostatic, mechanical, photocopied or otherwise, without the express permission in writing from The Association of Business Executives.

Page 3: libvolume5.xyzlibvolume5.xyz/law/ballb/3rdyear/semester5/corporatelaw/corporate... · ABE Advanced Diploma in Business Administration Study Manual CORPORATE FINANCE Contents Study

ABE Advanced Diploma in Business Administration

Study Manual

CORPORATE FINANCE

Contents

Study Unit

Title Page

Syllabus i

1 The Context of Corporate Finance 1 Basic Principles of Companies 3 Financial Objectives 4 Corporate Governance 8 Corporate Financial Management 22

2 Company Performance, Valuation and Failure 31 Ratio Analysis 33 Using Ratio Analysis 40 Introduction to Share Valuation 48 Methods of Share and Company Valuation 49 Predicting Company Failure 55 Capital Reconstruction Schemes 59

3 Acquisitions and Mergers 61 Company Growth 63 The Regulation of Takeovers 67 The Acquisition/Merger Process 71 Measuring the Success and Failure of Mergers and Takeovers 74 Disinvestment 78

4 Financial Markets 83 Stock Markets 84 Other Sources of Finance 90 Other Financial Markets 92 Recent Changes in Capital Markets 93 Impact of the Markets on Market Decisions 94

Page 4: libvolume5.xyzlibvolume5.xyz/law/ballb/3rdyear/semester5/corporatelaw/corporate... · ABE Advanced Diploma in Business Administration Study Manual CORPORATE FINANCE Contents Study

5 Sources of Company Finance 95 Share Capital 97 Methods of Issuing Shares 100 Share Repurchases 107 Debt and Other Forms of Loan Capital 108 Short-Term Finance 117 International Capital Markets 118 Finance and the Smaller Business 121

6 Cost of Finance 125 Investors and the Cost of Capital 127 Cost of Equity 128 Cost of Debt Capital 131 Cost of Internally Generated Funds 135 Weighted Average Cost of Capital 136 Assessment of Risk in the Debt Versus Equity Decision 138 Cost of Capital for Other Organisations 140

7 Portfolio Theory and Market Efficiency 143 Risk and Return 144 The Impact of Diversification 146 Portfolio Composition 150 The Application of Portfolio Theory 157 Market Efficiency 159

8 The Capital Asset Pricing Model 171 Risk, Return and CAPM 172 Calculation of Betas 178 Validity of the CAPM 179 Practical Applications of CAPM 180 The Arbitrage Pricing Model 182

9 Capital Structure 187 Capital Gearing 188 Factors Determining Capital Structure 192 Theory of Capital Structure 196 Capital Gearing and the Effects on Equity Betas 202

10 Corporate Dividend Policy 205 Key Influences on Dividend Policy 206 Theories of Dividend Policy 211 Practical Aspects of Dividend Policy 213

Page 5: libvolume5.xyzlibvolume5.xyz/law/ballb/3rdyear/semester5/corporatelaw/corporate... · ABE Advanced Diploma in Business Administration Study Manual CORPORATE FINANCE Contents Study

11 Working Capital and Short-Term Asset Management 217 Working Capital 219 Overtrading 222 Cash Management 224 Management of Stocks 230 Management of Debtors 235 Creditor Management 242 Short-Term Finance and Investment 243

12 Capital Investment Decision Making 1: Basic Appraisal Techniques 255 Future Cash Flows and the Time Value of Money 257 Return on Investment (Accounting Rate Of Return) 258 Payback 259 Discounted Cash Flow 260 Net Present Value (NPV) 261 Internal Rate of Return 267 Cost/Benefit Ratio 270 Comparison of Methods 270 Impact of Taxation on Capital Investment Appraisal 271 Appendix: Discounting Tables 274

13 Capital Investment Decision Making 2: Further Considerations 285 Allowance for Risk and Uncertainty 286 Impact of Inflation on Investment Appraisal 288 Capital Rationing 289 Lease Versus Buy Decisions 291 Post-Appraisal Audit 293 Adjusted Present Value (APV) 293 Use of the Capital Asset Pricing Model 296 Worked Examples 297

14 Managing Exposure to Risk – Financial Derivatives 319 Principles of Hedging 321 Interest Rates, Risk and Exposure 325 Internal Techniques of Managing Interest Rate Exposure 328 Futures Contracts 329 Forward Rate Agreements (FRAs) 332 Interest Rate Swaps 333 Options 335

15 International Trade and Finance 351 Theory and Practice of International Trade 353 International Investment 358 Finance and International Trade 362 Exchange Rates 372 Risk and International Trade/Finance 379 Internal Methods of Managing Exchange Rate Risk and Exposure 380 External Methods of Managing Exchange Rate Risk and Exposure 382

Page 6: libvolume5.xyzlibvolume5.xyz/law/ballb/3rdyear/semester5/corporatelaw/corporate... · ABE Advanced Diploma in Business Administration Study Manual CORPORATE FINANCE Contents Study
Page 7: libvolume5.xyzlibvolume5.xyz/law/ballb/3rdyear/semester5/corporatelaw/corporate... · ABE Advanced Diploma in Business Administration Study Manual CORPORATE FINANCE Contents Study

i

© Copyright ABE

Advanced Diploma in Business Administration

Corporate Finance

Syllabus

Aims 1. Understand the application of capital budgeting and investment appraisal techniques applied to

complex situations.

2. Understand the financing decision and the uses and limitations of the weighted average cost of capital. The Modigliani Miller proposition and optimal capital structures.

3. Understand the portfolio approach to risk and uncertainty and the relationship between risk and return.

4. Understand the issues surrounding dividend and distribution policies.

5. Demonstrate a knowledge of the various business valuation techniques.

6. Understand the problems associated with foreign exchange management.

Programme Content and Learning Objectives

After completing the programme, the student should be able to: 1. The broad environment of financial decision-making involving corporate objectives and

functioning of the capital markets:

Evolution of financial management in response to economic and other external factors e.g. technical developments and inflation resulting in the globalisation of financial decision making. The agency problem, finance theory in practice incorporating market and other imperfections. Financial markets, market efficiency and short-termism.

2. The capital investment decision in practice, including unequal project lives, effects of inflation and risk:

Identifying the relevant information in investment analysis. Evaluating replacement and other investment decisions. The problems of inflation and taxation. Replacement decisions. The control process as part of the investment decision. Post audit reviews.

3. The impact of risk and uncertainty on the financial decision-making process and the contribution of modern portfolio theory:

The need to understand how uncertainty affects investment decisions. Managers’ attitudes to risk. Mechanics of portfolio construction. Determination of appropriate risk premiums to incorporate into discount rates. The capital asset pricing model and its limitations.

4. The financing decisions of firms, including leverage, debt/equity ratios and leasing decisions:

Treasury management and working capital policy. Short term asset management. Gearing and the advantages of debt capital. Factors to be taken into account when designing a capital

Page 8: libvolume5.xyzlibvolume5.xyz/law/ballb/3rdyear/semester5/corporatelaw/corporate... · ABE Advanced Diploma in Business Administration Study Manual CORPORATE FINANCE Contents Study

ii

© Copyright ABE

structure policy. The Modigliani and Miller contribution. Buy or lease decisions, effects on overall financing strategy.

5. The role of financial markets in the provision of finance and the efficiency of the markets in the pricing of securities:

The key characteristics of the main sources of finance. The efficient market hypothesis and its practical implications. The derivatives market including the role of options, futures and swaps.

6. The dividend policy debate, including the implications of particular dividend distribution policies on the future of the firm:

Competing views about the role of dividend policy. The implications of a change in dividend payouts. The “information content” of dividends. The theory of dividend policy. Market imperfections and the clientele effect.

7. Evaluation of acquisitions and mergers and the problems associated with the valuation decision:

The reasons behind the choice of merger or acquisition as a strategic option. Financing of acquisitions. The valuation problem particularly related to unquoted companies. Evaluation of the success of a merger or take-over.

8. The impact of international trade on financial decisions, including the impact of exchange rates and the problems both financial and non-financial of international investment:

Economic theory involving the operation of international markets. Understand the three forms of risk involved in international trade: currency risk, translation risk and economic risk. Internal and external hedging of these risks.

Method of Assessment By written examination. The pass mark is 40%. Time allowed 3 hours.

The question paper will contain:

Six questions of which four must be answered. All questions carry 25 marks.

Further Reading The ABE is keen to encourage students to read around their subjects although your study manual provides complete coverage of the syllabus for the examination. If you have time available once you have worked through the manual, you may wish to consult one or more relevant books from the ABE’s suggested reading list which can be found in each subject syllabus. Advanced Diploma students especially should supplement their study of the manual with wide reading of relevant journals, quality newspapers and contemporary media sources.

Recommended Reading � R Pike, and P Neale, Corporate Finance and Investment; 3rd edition, 1999 (Prentice Hall)

Additional Reading � G Arnold, Corporate Financial Management; 1998 (Pitman)

Page 9: libvolume5.xyzlibvolume5.xyz/law/ballb/3rdyear/semester5/corporatelaw/corporate... · ABE Advanced Diploma in Business Administration Study Manual CORPORATE FINANCE Contents Study

1

© Licensed to ABE

Study Unit 1

The Context of Corporate Finance

Contents Page

Introduction 3

A. Basic Principles of Companies 3 Types of Company 3 Regulatory Framework for Companies 4

B. Financial Objectives 4 The Prime Objective 4 Valuation of Companies 5 Shareholder Value Analysis (SVA) 6 Long-term Versus Short-term Objectives 6 Objectives of Multi-National Companies 6 Objectives of Public Sector Organisations 6

C. Corporate Governance 8 Company Stakeholders 8 Management/Shareholder Relationship and Agency Theory 13 The Cadbury Report 13 The Greenbury Report 17 Hampel Committee Report 18 The Combined Code 19 The Turnbull Report 21 Directors’ Emoluments 21

D. Corporate Financial Management 22 Financial Decision Making 22 Financial Functions in Organisations 23 The Role of the Finance Manager 24 Planning 25 Forecasting 26

(Continued over)

Page 10: libvolume5.xyzlibvolume5.xyz/law/ballb/3rdyear/semester5/corporatelaw/corporate... · ABE Advanced Diploma in Business Administration Study Manual CORPORATE FINANCE Contents Study

2 The Context of Corporate Finance

© Licensed to ABE

Budgeting 27 Cash Management 28 Economic and Government Influences 29

Page 11: libvolume5.xyzlibvolume5.xyz/law/ballb/3rdyear/semester5/corporatelaw/corporate... · ABE Advanced Diploma in Business Administration Study Manual CORPORATE FINANCE Contents Study

The Context of Corporate Finance 3

© Licensed to ABE

INTRODUCTION

Corporate finance covers a wide range of topics and functions within an organisation. The three main areas we will look at in this course relate to answers to the following questions:

� Which investments should the firm undertake?

� How, where, when and how much finance should be raised?

� How should the firm’s profits be used or distributed?

These questions are more commonly referred to as:

(a) The investment decision

(b) The financing decision

(c) The dividend decision.

In making such decisions, the firm must ensure that it achieves its objectives. Central to this first unit, then, is the issue of what the objectives of companies are. This is our first main area of study.

The prime objective is often stated as the maximisation of shareholder wealth. This would imply that companies must be run in the interests of shareholders. However, there are a range of interests involved in the way in which companies are managed. We shall examine these in the second main section of the unit and consider, in particular, the importance of the stakeholder concept and the tensions that arise from the different interests involved.

Finally, we turn to the scope of corporate financial management. We shall develop the issues of financial decision making referred to above and consider their implications for the range of financial functions carried out in modern organisations and the roles required of the finance manager.

A. BASIC PRINCIPLES OF COMPANIES

We shall start by reviewing two fundamental concepts relating to companies which underpin much of our studies.

Types of Company A company is a separate legal entity, which means that it may take legal action against its shareholders or vice versa. Limited liability companies have capital divided into shares. If a shareholder has paid in full for his or her shares, then liability is limited to those shares. This is the concept of limited liability.

The two main classes of limited company are public and private companies. We will look at the main features of each class.

Company legislation defines a public company as one which has:

� An authorised share capital of at least £50,000;

� A minimum membership of two (there is no maximum);

� A name ending with “public limited company” or plc.

Not all public companies have shares which are traded on the Stock Exchange. Those traded on the Stock Exchange are known as quoted or listed companies.

Page 12: libvolume5.xyzlibvolume5.xyz/law/ballb/3rdyear/semester5/corporatelaw/corporate... · ABE Advanced Diploma in Business Administration Study Manual CORPORATE FINANCE Contents Study

4 The Context of Corporate Finance

© Licensed to ABE

A private company can be formed by two or more persons. They are often smaller or family owned businesses. A private company:

� Can have an authorised share capital of less than £50,000;

� Cannot offer its shares for sale to the general public.

You may know of private companies which have become public companies and have started to trade on the Stock Exchange. An example was the clothing retailer Laura Ashley which started life as a family owned private company.

Regulatory Framework for Companies The main legislation regulating companies is the Companies Act 1985 and the Companies Act 1989. The 1989 Act adds to and amends the 1985 Act. For example, the 1989 Act contains new legislation relating to the control of company auditors. Legislation on the duties and appointment of auditors was already set out in the 1985 Act.

In addition, UK company law must also incorporate company law directives issued by the Council of the European Union. For example, the European Eighth Directive on company law required more direct control of auditors and therefore the Companies Act 1989 introduced the concept of supervision of auditors.

Companies are also regulated in other ways:

� The production of company financial statements must be prepared in accordance with UK accounting standards. These are issued by the Accounting Standards Board (ASB). Recent legal opinion has now established accounting standards as a source of law.

� Another key area of regulation for the privatised utility companies are the consumer watchdogs – for example, Oftel which regulates British Telecom.

� Case law is also a very important aspect of company regulation. Try to think of examples from your legal studies.

B. FINANCIAL OBJECTIVES

The Prime Objective The underlying assumption of the theory of finance states that:

“the main objective of the firm is the maximisation of shareholder wealth in the long term”.

In order to maximise shareholder wealth the management must maximise the value of the firm, because the legal owners of the company are the shareholders, and all surplus value after creditors and other liabilities have been met belongs to them. Thus the greater the value of the firm after liabilities, the greater the wealth of the shareholders. The value of the firm and shareholder wealth are represented by the market price of the company’s shares, which is the amount a shareholder could obtain for selling his part of the business as a going concern. (We shall discuss this in greater detail later.)

It may seem to be a strange choice of major objective, but perhaps by thinking about some other likely objectives you will appreciate why the maximisation of shareholder wealth is the major objective of firms.

Page 13: libvolume5.xyzlibvolume5.xyz/law/ballb/3rdyear/semester5/corporatelaw/corporate... · ABE Advanced Diploma in Business Administration Study Manual CORPORATE FINANCE Contents Study

The Context of Corporate Finance 5

© Licensed to ABE

(a) The maximisation of company profits is often considered to be a major objective of firms. Clearly it is important, but even when there are rising profits the value of shares (and thus shareholder wealth) can fall. Can you think of how this might happen?

There are several ways in which it may occur; one is when a company raises additional share capital in order to fund an investment to increase profits, but may cause earnings per share to fall (there being more shareholders to share in the profits), thus resulting in a fall in share price. Consider the following example.

A company currently has 200,000 shares in issue and has expected profits of £50,000, thus EPS (earnings per share) are 25p. If the company issues a further 100,000 £1 shares to invest in a project which will give a 10% return on investment, then expected profits will increase by £10,000. However, there are now 300,000 shares in issue, and the EPS has fallen to 20p (£10,000 + £50,000/300,000 shares). The falling EPS causes the share price to drop, and shareholder wealth is therefore also reduced.

(b) Another objective you might feel to be important is the maximisation of balance sheet or asset values. Whilst a company’s balance sheet is important to investors, you will discover from this course and your accountancy studies that balance sheets do not reflect a true and up-to-date valuation of the company and its assets, and thus cannot be relied upon to determine the worth of the company.

All of the objectives we have considered so far are financial objectives. In addition, a company will have important non-financial objectives which include:

� Raising the skills of the workforce, perhaps through training and appraisal;

� Adhering to environmental legislation; for example, by reducing pollution emissions;

� The provision of a quality service to customers.

Financial and non-financial objectives are both important to a company. They may sometimes be in conflict but often they are complementary. For example, training the workforce will increase costs initially but should result in increased production which will generate additional profit for the company.

Valuation of Companies In order to achieve the main objective of maximising shareholder wealth we have to determine exactly how we value companies and their shares. Shareholder wealth obtained from a company is measured by increases in the price of shares above the price the shareholder paid for them (capital gains) and dividends received. You will see later that the price of a share is strongly affected by expectations of future dividends (the higher the expected dividends the higher the share price), and thus we can conclude that shareholder wealth can be maximised by maximising a company’s share price.

Management should therefore set itself financial targets directly related to maximising shareholder wealth, but how can this be done?

� The price of a company’s shares reflects the future earnings of the company so, in order to maximise shareholder wealth, the company must invest in those projects which give the highest value over time. We will look at how this is achieved in practice in module 887b.1.3.

� Increasing earnings per share and dividends per share also increases the share price, and firms should take decisions which allow a potential for maximisation of future dividends and earnings.

Page 14: libvolume5.xyzlibvolume5.xyz/law/ballb/3rdyear/semester5/corporatelaw/corporate... · ABE Advanced Diploma in Business Administration Study Manual CORPORATE FINANCE Contents Study

6 The Context of Corporate Finance

© Licensed to ABE

� By maximising profits – whilst we noted that maximising profits does not always increase shareholder wealth, in general it does and firms should aim to achieve this whilst considering the points raised above. Firms, however, should take care not to take undue risks when attempting to maximise their profits.

Shareholder Value Analysis (SVA) SVA utilises the concept of NPV (net present value, which we shall discuss later) and argues that the value of an organisation is the net present value of its net future cash flows discounted at its appropriate cost of capital.

SVA states that managers should concentrate on the value drivers, which are the factors which maximise shareholder value. They include:

� Growth in sales

� Profit margins

� Investment in fixed assets

� Investment in working capital

� The cost of capital

� The tax rate.

Management must identify the value drivers, cash flows and risks that result from investment options and aim to maximise value in the long term. Whilst this approach is popular in several companies (well-known examples being Disney and Pepsi) it relies on subjective judgments of cash flows in the future, and so does not have universal appeal.

Long-term Versus Short-term Objectives Unfortunately, the Stock Market is often more concerned with short-term increases in share prices rather than the maximisation of long-term profits (short-termism) e.g. choosing the project with the higher profits in the first year rather than over the life of projects. Often companies have to trade off short-term gains (e.g. achieving an earnings figure for a financial year) against acting in the best interests of the company in the long term (e.g. investing in future training and development expenditure).

Objectives of Multi-National Companies The objectives of multinational companies (MNC) are similar to those of other organisations but may be more complicated due to the number of differing views and requirements of the different countries they are based in.

Objectives of Public Sector Organisations Characteristics of organisations generally considered to be within the public sector include being non-profit making, with the Government accepting full, or a degree of, responsibility for their performance and exercising some measure of control over their activities. Broadly the public sector encompasses central government departments, local authorities, the Health Service, the police, and public bodies which receive their principal financing from central and local government (e.g. the Arts Council, the Fire Service) plus nationalised industries (see below).

These bodies are statutory organisations created by Acts of Parliament. The appointment of some of the members of the organisation is a matter for the Executive. It may be that a minister has statutory

Page 15: libvolume5.xyzlibvolume5.xyz/law/ballb/3rdyear/semester5/corporatelaw/corporate... · ABE Advanced Diploma in Business Administration Study Manual CORPORATE FINANCE Contents Study

The Context of Corporate Finance 7

© Licensed to ABE

powers to make appointments, such as in nationalised industries, or administrative power, e.g. making appointments to advisory committees.

Public sector organisations will be funded either wholly or in part by money provided by Parliament. Care must be taken, however, to distinguish between those organisations financed by Parliament and those which simply receive grant aid to assist them with an investment programme.

There are several differences which may exist between public bodies and commercial organisations.

(a) Many public organisations may have monopolies in either a service or geographical area.

(b) Although prices may be charged for some public services, they are rarely related to profit-making objectives and sometimes fail to cover the full economic cost. In general, the public sector does not use the price mechanism to test whether the public want the services provided. Instead, the criteria applied tend to be based on the political judgment of elected representatives under the constraints of the political mechanism of elections, pressure groups and consultative processes.

(c) The public sector exists to serve the community and, in the field of accounting, the stewardship of funds is often more important than the profit motive. However, the responsibilities of the financial manager and the need to exercise good financial public relations are as important as in commercial organisations. In order to replace the profit motive as a yardstick performance measures have been developed in order to ensure that the three “Es” of efficiency, economy and effectiveness are achieved, and to protect public money.

In an attempt to improve the performance of central government departments, the Government has introduced more than 650 performance targets (public service agreement (PSA) targets).

Within these overall points about the public sector in general, we should also recognise a number of particular aspects and developments relating to specific types of organisation.

(a) Nationalised Industries

In recent years the number of nationalised industries has been reduced due to the privatisation programme of the Conservative Governments of 1979-1997. However, there are still some large organisations remaining in this category, including the Post Office (although this may soon be privatised). The objectives of such organisations are generally social or service-led.

They are funded by borrowing from the capital markets and the Government. Whilst the maximisation of profits is not their main objective they generally have to obtain set financial targets, perhaps to maintain required subsidies at a set level or below. One set of financial targets which nationalised industries have to consider are external financing limits (EFLs), with the profitable organisations paying funds to the Government, and there being restrictions on the amounts that non-profitable firms can borrow. EFLs are the controls on finance between the nationalised industries and the Government.

In general nationalised industries in the UK have been expected to aim to achieve a rate of return (before interest and tax) on new investment programmes of 5% in real terms. The rate of return is measured by current cost operating profit as a proportion of the net replacement cost of assets employed. Nationalised industries also have other performance measures, including cost reductions and efficiency gains which they have to achieve.

(b) Government Departments

One major change in this area within the UK recently has been the formation of executive agencies to carry out specific functions, such as the Contributions Agency. They are expected

Page 16: libvolume5.xyzlibvolume5.xyz/law/ballb/3rdyear/semester5/corporatelaw/corporate... · ABE Advanced Diploma in Business Administration Study Manual CORPORATE FINANCE Contents Study

8 The Context of Corporate Finance

© Licensed to ABE

to achieve a set level of service and are answerable to the Government for their service levels but are managed independently on business-led lines.

(c) Private Finance Initiative (PFI)

The Private Finance Initiative (PFI) was introduced in 1992 as a means of obtaining private finance for public sector long-term capital projects, e.g. the building of prisons, schools and hospitals. The current government is committed to developing this approach across a wide range of public services. A measure of the commitment to the PFI can be demonstrated by the £3 billion of PFI credits available from 1997/98 to 2000/02.

A new Commission on Public Private Partnerships was set up in Autumn 1999 – the Institute of Public Policy Research Commission (IPPR). It will examine questions about specific forms of partnership between private sector firms and public sector organisations. For example, how can private firms involved in partnerships be made accountable to the public? How does this accountability fit in with achieving the best value for money?

(d) Not for Profit Organisations

The prime objectives of organisation such as charities are not concerned with profit-making, but with services, e.g. to offer a service such as training guide dogs for the blind, or to fund research into cancer treatments. They do, however, operate within financial constraints and must work within the funds they obtain.

All not-for-profit organisations also strive, as do many commercial ones, to obtain the three Es of economy, efficiency and effectiveness.

C. CORPORATE GOVERNANCE

Shareholders are the owners of a company and it is important to remember that the maximisation of their wealth is the prime objective of companies in the private sector. This is the underlying concept in the theoretical parts of this course. However, they are not the only groups with an interest in the company and the interplay of factors in the governance of a company is a key concept.

Company Stakeholders In practice companies often have multiple objectives (both financial and non-financial) involving various stakeholder groups, which prevent the maximisation of shareholder wealth. The different stakeholder groups in an organisation were identified in 1975 by the Corporate Report (ASC) which dealt with their objectives and specific requirements from accounting information. Clearly, different users will look at the company in different ways, and the objectives of organisations have to be designed to satisfy their varying needs, with the objectives of one group often also applying to another group. The objectives of different groups may conflict, and compromises will have to be made.

We consider the interests of some of the stakeholder groups below, but you should try to think of other points yourself.

(a) Banks and Other Lenders

This group includes anyone who makes a loan or other financial accommodation available to an organisation, examples being debenture-holders, finance companies, building societies and venture capitalists.

The main concern of this group is the safety of the investment; lenders expect to get their money back within an agreed period and to make a profit. In order to maintain the safety of the

Page 17: libvolume5.xyzlibvolume5.xyz/law/ballb/3rdyear/semester5/corporatelaw/corporate... · ABE Advanced Diploma in Business Administration Study Manual CORPORATE FINANCE Contents Study

The Context of Corporate Finance 9

© Licensed to ABE

investment they want to ensure that the level of debt to equity does not become too high, because increases in the level of debt increase the risk of insolvency of the firm, with the firm being unable to pay the required interest payments. Short-term lenders are especially concerned with the ability and willingness (known as “corporate integrity”) to repay the liability from cash generated by the business. Long-term lenders may place a restrictive trust deed or set financial guidelines, e.g. a set proportion of working capital, in order to ensure their investment remains safe.

(b) Business-contact Group (includes Debtors and Creditors)

This group includes suppliers, competitors and all other business affected by an organisation’s activities. Their objectives include ensuring that the firm deals honestly, does not misuse any monopoly powers and pays its bills promptly within the terms of the trading agreement.

The group will be interested in developing long-term strategic relationships and the continuity of trading opportunity with an organisation which is financially stable with minimal administration. Customers of the organisation will be concerned with having a supplier who is reliable, and who provides a constant supply of the product (when required) of consistent quality with good, efficient service at a fair price. Customers will also be concerned with the level of service they are receiving, the value for money, and the safety of the goods they receive.

Competitors are also included in this group, and include those who may be interested in acquiring the business as well as those who are rivals in trade. The group will require as much information about the company and its finances as possible, although the company will not wish them to have such information, and secrecy may conflict with the needs of other groups.

(c) Public

The needs of the general public can take many forms, e.g. sections of the general public may wish to see a restriction on contributions to political parties, charities or social groups, or a restriction on the business activities carried out with, or in, a particular country. Another example is where local residents are interested in the amount of investment and degree of control that an entity has in their own community and its ultimate effect on their local environment. When public money assists the enterprise the public may wish to see the return in profitability, jobs and services.

(d) Government

The Government (and, indeed, the public) wish to ensure that the organisation adheres to the law, pays the correct amount of taxes and other financial charges levied upon it by government bodies, and provides the statistical and other information required in order to ensure control over its (the Government’s) own economic policy. The Government will also be interested in ensuring that the organisation respects its social and environmental commitments.

Moreover, the Government has a desire to regulate some of the privatised utilities to prevent them abusing their monopoly powers. For this reason it has set up consumer “watchdogs”, e.g. Oftel, which regulates British Telecom, to oversee such companies. The watchdogs may, amongst other things, limit price levels which can conflict with a company’s desire to maximise profits.

(e) Financial Analysts and Advisors

This group will comment upon the progress, or otherwise, of the entity. In order to do so they will need the fullest possible information in whichever field their interest lies. Their requirements may mirror those of any of the other user groups. They will, however, have the

Page 18: libvolume5.xyzlibvolume5.xyz/law/ballb/3rdyear/semester5/corporatelaw/corporate... · ABE Advanced Diploma in Business Administration Study Manual CORPORATE FINANCE Contents Study

10 The Context of Corporate Finance

© Licensed to ABE

key objective of ensuring compliance with accounting standards to provide for uniformity to the presentation of information and the easier comparison with other organisations.

(f) Employees and Management

Employees will be concerned with the remuneration they receive from the company, their working conditions and security of employment. They may also be concerned with other factors such as training and career development prospects within the firm; benefits in kind such as company cars; company pension and redundancy provisions; and the potential for future expansion of jobs for themselves and their friends and families.

(g) Shareholders and Investors

Shareholders and investors are obviously an important stakeholder group, being the owners of the business. In order to meet the needs of shareholders management must:

� Maximise their wealth (shown by the growth in share price and the payment of dividends).

� Achieve a specific level of earnings, earnings per share and dividends per share. Note that some shareholders prefer high dividends and some prefer capital gains (see later study unit) but the needs of the majority should be met as far as possible.

� Stick to a preset target for operating profitability represented by either a set return on capital employed or a profit/sales ratio (also discussed later).

� Expand the business when feasible – to be a worthwhile investment, growth, level of risk, return on investment and profitability in relation to competitor businesses and other investment opportunities will be expected to be at an appropriate level.

� Maintain the security (as far as is consistent with profit-making) of the shareholder’s investment. (The risk-return trade off is discussed in more detail in a later study unit.) This includes considering the fact that shareholders have different risk preferences and thus prefer different levels of gearing.

� Satisfy the investor that the company has sufficient cash flow to accommodate its plans and avoid future potentially fatal liquidity problems.

� Give details of political, charitable or social donations in order to allow shareholders to decide whether the convictions of the management are in line with their own views.

This is not an exhaustive list of management objectives in respect of shareholder interests and you may be able to think of several others. A company therefore has to know who its major shareholders are and what their objectives for the company are, and concentrate on achieving those objectives. Such knowledge would also help to explain recent price movements when shareholdings change hands, and might help in fighting off a takeover bid.

Companies may have only a few shareholders (e.g. a private family company) or they may have many small shareholders (e.g. some of the privatised utilities). Advantages of having a large number of shareholders include a reduced risk of one shareholder obtaining a controlling interest; greater market activity in the firm’s shares and thus the likelihood of vast price movements caused by one shareholder selling his shares is also reduced; and takeover bids are easier to frustrate. Against this, however, will be increased administration costs covering statutory requirements of information to shareholders, and it may be more difficult to meet all shareholders’ conflicting objectives.

Page 19: libvolume5.xyzlibvolume5.xyz/law/ballb/3rdyear/semester5/corporatelaw/corporate... · ABE Advanced Diploma in Business Administration Study Manual CORPORATE FINANCE Contents Study

The Context of Corporate Finance 11

© Licensed to ABE

Many decisions in financial management are taken in a framework of conflicting stakeholder viewpoints. For example, consider the stakeholders and the related financial management issues involved in the following situations.

� A private company converting into a public company

The stakeholders will include:

(a) Shareholders of existing private company;

(b) Shareholders of new public company;

(c) Employees and management.

Some of the key financial management issues will be:

(i) Who will gain a controlling interest in the new company?

(ii) Will the company be administered differently, perhaps as family owner shareholders no longer have day to day involvement in running the company? How will it affect terms and conditions of employees?

(iii) How will the conversion affect maximisation of shareholder wealth?

� A highly geared company, such as Eurotunnel, attempting to restructure its capital finance

The stakeholders will include:

(a) Debenture holders;

(b) Banks and other lenders;

(c) The government;

(d) Shareholders.

Some of the key financial management issues will be:

(i) Shareholders will be concerned about the effects of additional gearing on the company’s ability to pay dividends, which may conflict with the government’s objective of ensuring financial stability.

(ii) As this is a large public interest project (Eurotunnel) the government will want to see financial stability to ensure that the company can complete the project without financial collapse.

(iii) Debenture holders are concerned to ensure that the company will have sufficient cash flow to meet interest payments as they fall due.

� A large conglomerate “spinning off” its numerous divisions by selling them, or setting them up as separate companies, e.g. Hanson

The stakeholders will include:

(a) Employees and management;

(b) Debtors and creditors;

(c) Shareholders.

Some of the key financial management issues will be:

(i) The security of jobs for employees and management in the new companies, which may conflict with the aim of shareholders to maximise wealth.

Page 20: libvolume5.xyzlibvolume5.xyz/law/ballb/3rdyear/semester5/corporatelaw/corporate... · ABE Advanced Diploma in Business Administration Study Manual CORPORATE FINANCE Contents Study

12 The Context of Corporate Finance

© Licensed to ABE

(ii) Customers (debtors) will be concerned about the quality of the product and whether the new structure will affect this.

(iii) Suppliers (creditors) will want to know the liquidity of separate companies and their ability to pay outstanding debts. Also how will outstanding debts be settled if divisions are sold?

� Japanese car-makers, such as Nissan and Honda, building new car plants in other countries

The stakeholders will include:

(a) Shareholders;

(b) Employees and management;

(c) Government;

(d) Public.

Some of the key financial management issues will be:

(i) The public may be concerned that they have no control over foreign companies setting up in their local areas, which may conflict with the aims of government in encouraging investment by overseas companies.

(ii) The government may grant development finance and incentives to incoming companies.

(iii) The shareholders of the Japanese companies will be concerned about the security of their investment overseas.

(iv) Japanese management may be concerned about different pay and conditions if they are sent to manage the overseas plants.

� A public company offering to run the UK national lottery for free rather than for profit

The stakeholders will include:

(a) The government;

(b) The public;

(c) Shareholders;

(d) Financial analysts.

Some of the key financial management issues will be:

(i) The government will be concerned about the company’s objectives if profit is not the obvious one.

(ii) Shareholders will want to know how this non-profit making venture will affect dividends and the value of shares. After all, their main objective is maximisation of shareholder wealth.

(iii) Financial analysts will study the possible effects on the company’s value if it gains the contract to run the lottery.

(iv) The public will want to know what percentage of takings will be donated to good causes and how much will be retained by the company for administration and investment in equipment.

Page 21: libvolume5.xyzlibvolume5.xyz/law/ballb/3rdyear/semester5/corporatelaw/corporate... · ABE Advanced Diploma in Business Administration Study Manual CORPORATE FINANCE Contents Study

The Context of Corporate Finance 13

© Licensed to ABE

Management/Shareholder Relationship and Agency Theory The skill and experience of the senior management board (and to a lesser extent its subordinate management) are important to shareholders as they are employed to manage the shareholders’ investment on their behalf. There must be trust in the integrity and ability of the managers; a dynamic board of management can make a significant difference to the performance of a business and the way the market views it.

An agency relationship exists where one person (an agent) acts on the behalf of another (the principal). The management/shareholder relationship is an example of an agency relationship. Goal congruence occurs when the objectives of the agents match those of the principals. The agency problem is the conflict that arises from the separation of management and ownership in many companies, leading to a lack of goal congruence. The financial and other rewards of managers (agents) may not be linked to the shareholders’ (principals) financial return. In theory management should not be able to act contrary to the wishes of shareholders because shareholders can dismiss the managers or sell their shares. Unfortunately it is often not the case. Small shareholders frequently have little knowledge about the running of the business and little power to alter its execution; and the large institutional shareholders have often been passive and uninvolved.

However, a series of “corporate raids” in the late 1980s, when firms acquired and then asset stripped managerially-focused companies believing them to be undervalued, has led to the large institutional shareholders considering the actions of management more carefully.

A number of incentive schemes have been introduced in an attempt to encourage goal congruence between management and shareholders. The most popular is the stock option scheme. This allows senior management up to a certain number of the company’s shares at a fixed price at a specified time in the future. The management therefore have a financial incentive to act in ways to maximise the share price, which benefits all shareholders. However, such schemes are of doubtful benefit – management do not have to buy shares if the price has fallen; and the schemes can lead to volatility in the share price which is counter to the principle of a stable share price which many shareholders desire.

Another popular scheme involves profit-related incentives in which bonuses are based on the annual growth in earnings per share, measured against a pre-set target such as companies in the sector. However, you will appreciate that accounting figures can easily be manipulated and can also be affected by external factors such as a change in tax rates. Such measures therefore only give a partial (and perhaps misleading) picture of managements’ activities.

The Cadbury Report The Committee on the Financial Aspects of Corporate Governance (known as the Cadbury Committee, after its Chairman) was set up in May 1991 by the Financial Reporting Council (FRC), the London Stock Exchange and the accounting profession, in response to increasing public concern over the management of large companies and professional investors’ low levels of confidence in financial reporting and auditing. Concerns included lack of direction and control of organisations by the boards, a lack of true auditor independence, and an increase in litigation and damages awarded against companies. These concerns were increased by a number of highly publicised company “scandals” and collapses – examples you may be familiar with included Guinness, Blue Arrow, BCCI and Barings Bank. Moreover, there has been public disquiet regarding the large salary increases awarded to directors of privatised utility companies and the ease with which former Conservative cabinet ministers appeared to be able to secure well-paid jobs in companies which they had previously regulated or privatised whilst in government office.

Page 22: libvolume5.xyzlibvolume5.xyz/law/ballb/3rdyear/semester5/corporatelaw/corporate... · ABE Advanced Diploma in Business Administration Study Manual CORPORATE FINANCE Contents Study

14 The Context of Corporate Finance

© Licensed to ABE

The aim of the Cadbury Committee was “to bring forward proposals to promote good financial corporate governance, without stifling entrepreneurial drive or impairing companies’ competitiveness”. “Corporate governance” was defined by the committee as “the system by which companies are directed and controlled” and is the responsibility of the directors of the company. The Committee intends to consider the responsibilities of each group involved in the financial reporting process including:

� The links between board, auditors and shareholders;

� The role and responsibilities of audit and the auditors;

� The need for audit committees, their functions and membership;

� The type and frequency of information required by shareholders and other parties with a financial interest;

� The role and responsibilities of executive and non-executive directors as regards the reporting of financial performance.

The heart of the Committee’s recommendations was a Code of Best Practice, to be adopted by the directors of all UK public companies, with all company directors to be guided by it. Some allowances are made for the way in which it might be implemented in different companies.

You will see later that the Cadbury Code of Best Practice has been incorporated into the Combined Code. However, the recommendations of the Cadbury Committee are so important in the development of corporate governance in the UK that we will look at them in detail.

The Code of Best Practice

The major points are as follows:

(a) The Board

There should be a clearly accepted division of responsibility at the head of a company ensuring a balance of power and authority. In cases where the Chief Executive is also the Chairman there should be strong independent executives on the board with their own appointed leader.

The calibre and number of non-executive directors should be such that their views carry significant weight on the board.

Boards should meet regularly and have a formal schedule of matters reserved for their decision to ensure that the direction and control of the company remain firmly in their hands, including the monitoring of the executive management.

All directors should have access to the advice and services of the company secretary, who is responsible to the board for ensuring that board procedures are followed and that applicable rules and regulations are complied with. Any question of the removal of the company secretary should be a matter for the board as a whole.

(b) Executive Directors

Directors’ total emoluments and those of the Chairman and the highest-paid UK director should be fully disclosed and split into their salary and performance-related elements, with an explanation of the basis on which performance is measured.

Executive directors’ pay should be subject to the recommendations of a remuneration committee made up wholly or mainly (and preferably chaired) by non-executive directors. Directors’ service contracts should not, unless approved otherwise by shareholders, exceed three years.

Page 23: libvolume5.xyzlibvolume5.xyz/law/ballb/3rdyear/semester5/corporatelaw/corporate... · ABE Advanced Diploma in Business Administration Study Manual CORPORATE FINANCE Contents Study

The Context of Corporate Finance 15

© Licensed to ABE

(c) Controls and Reporting

Boards must establish effective audit committees (see below). The chairmen of audit and remuneration committees should be responsible for answering questions at the annual general meeting.

The board should ensure that an objective and professional relationship is maintained with the auditors. The board must explain their responsibility for preparing the accounts next to a statement by the auditors regarding their reporting responsibilities.

The Code requires that directors report on the effectiveness of the company’s system of internal control, and state that the business is a going concern, with supporting assumptions or financial qualifications if necessary. Actions to be taken by companies to ensure they are able to give a positive account of their control arrangements include:

� Deciding whether to report in advance of the effective date (periods beginning on or after 1 January 1995).

� Identifying the main elements of the control framework of the business (including principal risks) and determining the nature of evidence which the board will need to evaluate the effectiveness of the control process.

� Developing a plan of action to analyse the risk and control framework and to obtain evidence for the analysis.

� Drafting disclosures for internal discussion.

� Companies are expected to treat the code as an opportunity to review the effectiveness of their business control and risk management in order to demonstrate that the board has done all it reasonably can to avoid failure of controls, thereby retaining full and effective control.

The board’s duty is to present a balanced and understandable assessment of their company’s position. Balance sheet information should be included with the interim report, which should be reviewed by the external auditors but need not be subject to a full audit. As part of all annual reports for years ending after 30 December 1992 the Committee recommended that UK-registered listed companies make a statement about their compliance with the Code, explaining any failure to do so.

(d) Shareholders

Both boards and shareholders were encouraged by the Committee to consider how to improve the effectiveness of general meetings. The way in which institutional shareholders use their power to influence standards of corporate governance was noted as fundamentally important. The Committee looked to the institutions, backed by the Institutional Shareholders Committee, to use their influence to ensure that companies comply with the Code.

(e) Auditing

The annual audit is described as “one of the cornerstones of corporate governance”. Several minor recommendations were made to ensure its effectiveness and objectivity:

� Audit Effectiveness

Audit effectiveness should be increased by clarifying the respective responsibilities of directors and auditors for preparing and commenting on financial statements, and by developing audit practice in areas such as internal control, going concern, fraud and other illegal acts. Specific recommendations are that the directors should report on the

Page 24: libvolume5.xyzlibvolume5.xyz/law/ballb/3rdyear/semester5/corporatelaw/corporate... · ABE Advanced Diploma in Business Administration Study Manual CORPORATE FINANCE Contents Study

16 The Context of Corporate Finance

© Licensed to ABE

effectiveness of their system of internal financial control and the auditors should comment upon this statement. Note the focus on financial controls only rather than the whole range of controls including operating controls.

Directors should specifically state that the company is a going concern, giving their supporting assumptions or qualifications. Auditors should also comment on this statement.

Government should also consider extending the statutory protection already available to auditors in regulated sectors (banks, building societies, insurance, etc.) to that of all auditors so they may freely report reasonable suspicion of fraud to the authorities.

� Audit Objectivity

Both the board and auditors have a responsibility to ensure that the relationship between them is professional and objective. The relationship will be backed by the development of more effective accounting standards through the work of the Accounting Standards Board (ASB); by the formation of audit committees; by full disclosure of fees paid to audit firms for non-audit work; and by a periodical change of audit partner. To ensure they would be put into practice the Committee recommended that the 1991 regulations under the Companies Act on the disclosure of fees for non-audit work be amended, and that the accountancy profession drafts guidelines on the rotation of audit partners.

� Audit Committees

The Committee stated that the board should establish an audit committee of at least three non-executive directors with written terms of reference which deal clearly with their authority and duties.

Listed companies were allowed a maximum of two years to establish an effective audit committee. Membership should be confined to non-executive directors (the majority should be wholly independent of the company), details being given in the annual report. The audit committee should meet a minimum of three times a year and should have formal terms of reference covering duties, authority and membership.

(f) Non-executive or Outside Directors

Non-executive directors should bring an independent judgment to bear on issues of strategy, performance and resources including key appointments and standards of conduct.

The Committee recommended that a majority of non-executive directors should be independent and free of any business or financial connection with the company (apart from their fees and shareholdings). Fees should reflect the time which they commit to the company, but they should receive no pension or share options as part of their service. The Code also suggested that an agreed procedure should be in place for non-executives to take independent professional advice at the company’s expense. The selection of non-executive directors should be by a formal process, for a specified term, and their nomination should be a matter for the board as a whole. However, the report does not discuss the action that should be taken in the event of a non-executive director resigning or being released.

The independence of non-executives must be transparent. Fees should be such that part-time rather than full-time involvement is encouraged and, since resignation is the ultimate sanction of the non-executive director, the fees should not be so large that the non-executive is dependent upon them.

Page 25: libvolume5.xyzlibvolume5.xyz/law/ballb/3rdyear/semester5/corporatelaw/corporate... · ABE Advanced Diploma in Business Administration Study Manual CORPORATE FINANCE Contents Study

The Context of Corporate Finance 17

© Licensed to ABE

ProNED is a service sponsored by the Institute of Directors that actively campaigns for the increased use and profiling of non-executive directors. It has become increasingly active in using its contacts to find willing and able non-executive directors. One highly publicised example is Amstrad, where outside directors were appointed in 1993 when Alan Sugar acquiesced to outside shareholder pressure.

In summary, the recommendations of the Cadbury Report, as encompassed by the Code of Best Practice, were that listed companies should include in their accounts full and clear disclosure of directors’ total emoluments and those of the Chairman and the highest-paid director. The disclosures should include pension contributions and stock options. Performance-related elements, and the basis upon which performance is measured, should be shown separately.

With effect from 23 April 1993 the Stock Exchange stated that UK-incorporated listed companies must state in their accounts for accounting periods ending after 30 June 1993, whether they have complied with the Code throughout the accounting period in addition to the other continuing obligations. Any failure to comply must be stated, along with reasons for the non-compliance. The compliance statement must be reviewed by the auditors. Other UK companies should adopt the Code at the earliest practicable date.

In addition, from 1 January 1995 directors’ statements should include the following:

(a) An acknowledgment that directors are responsible for the system of internal financial control including the main procedures established which are designed to provide effective financial control.

(b) An explanation that the system can only provide a reasonable level of control.

(c) Confirmation that the directors have reviewed the effectiveness of their present internal financial control.

The Greenbury Report The Greenbury Report, published by the Greenbury Committee in July 1995, goes beyond the Cadbury Code of Best Practice in establishing principles for determining directors’ pay and disclosures on pay to be given in the annual accounts and company reports.

The Greenbury Code recommends that a remuneration committee should be established comprising solely non-executive directors – though the Chief Executive or Chairman may be asked for advice – and that this committee should determine executive directors’ remuneration. The Code also recommends that directors should have service contracts limited to one year.

Other important recommendations are:

� Public companies should publish an audited statement detailing compliance with the Greenbury Code under Stock Exchange rules.

� The remuneration committee should report to shareholders via the annual report and accounts. Full details should be included of directors’ remuneration:

(i) Basic salary

(ii) Benefits in kind

(iii) Annual bonuses

(iv) Long-term incentive schemes.

The majority of the Greenbury Code principles have been included in the Listing Rules of the Stock Exchange.

Page 26: libvolume5.xyzlibvolume5.xyz/law/ballb/3rdyear/semester5/corporatelaw/corporate... · ABE Advanced Diploma in Business Administration Study Manual CORPORATE FINANCE Contents Study

18 The Context of Corporate Finance

© Licensed to ABE

Hampel Committee Report Sir Ronald Hampel was given the task of continuing the work of Sir Adrian Cadbury on corporate governance. The final report of the Hampel Committee was issued in February 1998. Sir Ronald summed up the essence of his committee’s report by saying that “Good governance requires judgment, not prescription and for that reason I believe it is in business’ own interest to conform, and that it will”. Some features of this report relating to corporate governance are:

� Most non-executive directors should be independent and their independence should be identified in the annual report.

� Directors should receive appropriate training.

� The roles of chairman and chief executive should be separate.

� A senior non-executive director should be appointed to deal with shareholders’ concerns. The name of the director should be identified in the annual report.

� The practice of paying non-executive directors using company shares is not recommended, although there is nothing against it in principle.

� Directors should be on contracts of one year or less.

� A remuneration committee of the board should be established, made up of independent non-executive directors. The committee should make decisions on the pay packages of executive directors and the framework of executive pay.

� Companies should include in the annual report a narrative account of how they apply broad principles and should explain their policies. Any departure from best practice should be justified in the report.

� The creation of an internal audit department is recommended.

� Directors should review the effectiveness of the company’s internal controls (not just financial controls) but need only report publicly on the system rather than its effectiveness.

The report has not been well received by many commentators, who include shareholders, finance directors and accountancy professional bodies. For example, the committee’s recommendations do not force directors to identify major organisational risks or make establishment of internal audit compulsory.

The Stock Exchange has a code of practice which incorporates the recommendations of the Cadbury, Greenbury and Hampel reports (see later). The Stock Exchange will be responsible for overseeing adherence to the code. The government has indicated that if companies do not adopt best practice, it may take action to introduce legislation on corporate governance. In particular, continuing large pay awards to directors and extensive share option schemes may prompt the government to take action.

Some professional bodies have called for the government to consider setting up an independent body to review corporate governance and to take action if a company breaches Stock Exchange listing rules. The Hampel Committee, however, saw no need for such a body and recommended that the existing Financial Reporting Council (FRC) should monitor the need for reviews of corporate governance in the future.

This is an important area of current debate and you should follow developments in the financial press.

Page 27: libvolume5.xyzlibvolume5.xyz/law/ballb/3rdyear/semester5/corporatelaw/corporate... · ABE Advanced Diploma in Business Administration Study Manual CORPORATE FINANCE Contents Study

The Context of Corporate Finance 19

© Licensed to ABE

The Combined Code In June 1998 the London Stock Exchange published the Hampel Committee Principles of Good Governance and the Code of Practice (the Combined Code). It replaces the Cadbury and Greenbury Codes but incorporates aspects of both of them.

You should study this section carefully as it is an important topic area.

(a) The Principles of Good Governance

� Directors and the Board

(i) There should be an effective board to lead and control the company.

(ii) Running the board and running the business are separate tasks. There must be a clear division of responsibilities at the head of the company. No one individual should have unfettered powers.

(iii) There should be a balance of executive and non-executive directors (NEDs) including independent NEDs. No individual or group should dominate the board.

(iv) Timely and quality information should be provided to the board.

(v) There should be a formal and transparent procedure for board appointments (usually a nomination committee of mainly NEDs).

(vi) Directors should be re-elected at least every three years.

� Directors’ Remuneration

(i) Remuneration should be sufficient to attract and retain the directors needed but no more than necessary. Executive directors’ remuneration should be partly linked to corporate and individual performance.

(ii) There should be a formal and transparent procedure for developing remuneration policy and individual packages, i.e. a remuneration committee of NEDs. Directors should not be involved in deciding their own remuneration.

� Relations with Shareholders

(i) The board should encourage dialogue on objectives with institutional shareholders.

(ii) The annual general meeting (agm) should be used to communicate with private investors and encourage their participation.

� Accountability and Audit

(i) There should be a balanced, understandable assessment of the company’s position and prospects.

(ii) There should be a sound system of internal control to safeguard the shareholders’ investment and the company’s assets.

(iii) There should be formal and transparent arrangements to apply the above two principles and maintain relationships with the auditors.

The Combined Code requires an explanation in a listed company’s annual report of how these principles have been applied. A major impact of the Combined Code is that a company must review the effectiveness of all controls, not just financial controls.

Page 28: libvolume5.xyzlibvolume5.xyz/law/ballb/3rdyear/semester5/corporatelaw/corporate... · ABE Advanced Diploma in Business Administration Study Manual CORPORATE FINANCE Contents Study

20 The Context of Corporate Finance

© Licensed to ABE

(b) Examples of Contents of Combined Code

� Directors

(i) The Board must comprise at least one third non-executive directors.

(ii) There must be a senior independent director (not the Chairman) to whom shareholders can raise their concerns.

(iii) A nomination committee is strongly recommended.

(iv) All directors should receive training when first appointed.

� Directors’ Remuneration

(i) A significant proportion of executive remuneration should be linked to corporate and individual performance.

(ii) The remuneration report should be issued in the name of the board and not just the remuneration committee.

� Accountability and Audit

(i) Financial reporting provisions apply to all price-sensitive public reports and reports to regulators.

(ii) The definition of “internal control” covers all controls, not just financial controls.

(iii) The majority of the audit committee should be independent non-executive directors.

(iv) The audit committee should review the scope, results, cost-effectiveness, independence and objective of external audit.

(v) The company should review the need for internal audit.

(c) Matters to be Reported Publicly by Companies

� How the company applies the principles

� Whether or not the company complies with detailed provisions; any exceptions must be explained

� Justify combined posts of chairman and chief executive

� Give the names of:

(i) The chairman, chief executive, senior independent director and other independent directors

(ii) The chairman and members of the nomination committee

(iii) Members of the remuneration committee

(iv) Members of the audit committee

(v) Biographies of directors submitted for election or re-election

� Remuneration policy and details of the remuneration of each director

� Explain the directors’ responsibility for preparing the accounts

� Whether the business is a going concern, together with the supporting assumptions or qualifications

Page 29: libvolume5.xyzlibvolume5.xyz/law/ballb/3rdyear/semester5/corporatelaw/corporate... · ABE Advanced Diploma in Business Administration Study Manual CORPORATE FINANCE Contents Study

The Context of Corporate Finance 21

© Licensed to ABE

� State that directors have reviewed the effectiveness of internal controls

The Turnbull Report Guidance for directors on the scope, extent and nature of the review of internal controls was issued by the Institute of Chartered Accountants in England and Wales in late September 1999 in the form of the Turnbull Report – “Internal Control: Guidance for Directors of Listed Companies in the UK”. This guidance has the support and endorsement of the Stock Exchange.

The Turnbull Report states that:

“A company’s system of internal control has a key role in the management of risks that are significant to the fulfilment of its business objectives”.

The company’s system of internal control should:

� Be embedded within its operations and not be treated as a separate exercise;

� Be able to respond to changing risks within and outside the company; and

� Enable each company to apply it in an appropriate manner related to its key risks.

The Report makes it clear that the board of a company is ultimately responsible for its system of internal control. It will normally delegate to management the task of establishing, operating and monitoring the system. The board must, however, regularly assure itself that:

(a) Appropriate processes are functioning effectively to monitor the risks to which the company is exposed; and

(b) The system of internal control is effective in reducing those risks to an acceptable level.

Reviewing the effectiveness of internal control is the responsibility of the board although the board may delegate to the audit committee certain aspects of the review work. The board is also responsible for preparing an annual statement on internal control. This statement should explain how the company has maintained a sound system of internal control and should report to shareholders that it has reviewed the effectiveness of the system.

You should follow developments in this area in the financial press.

The Institute of Chartered Accountants in England and Wales has prepared a briefing for directors and managers to give practical advice on implementing the Turnbull Report and setting up good risk management and internal control. It recommends that listed companies should:

� Not delay in implementing the Turnbull Report

� Ensure that management at all levels in the company are committed to the Report’s implementation

� Prepare a plan and identify company objectives

� Identify the main risks to achievement of these objectives

� Establish a risk management framework and associated control strategy

� Monitor progress continuously

Directors’ Emoluments There are requirements for disclosure of directors’ remuneration in the annual financial statements. These are the Company Accounts (Disclosure of Directors’ Emoluments) Regulations 1997 and they apply to all listed and unlisted companies.

Page 30: libvolume5.xyzlibvolume5.xyz/law/ballb/3rdyear/semester5/corporatelaw/corporate... · ABE Advanced Diploma in Business Administration Study Manual CORPORATE FINANCE Contents Study

22 The Context of Corporate Finance

© Licensed to ABE

The principal change relates to the disclosure of directors’ aggregate emoluments and other benefits. Requirements to show directors’ emoluments in bands of £5,000 and details of the chairman’s emoluments have been abolished. The emoluments of the highest paid directors must still be disclosed although the details have changed.

The following must be disclosed in notes to the accounts:

� The aggregate amount of emoluments paid to or receivable by directors in respect of qualifying services.

� The aggregate amount of directors’ gains on the exercise of share options.

� The aggregate of:

(i) The amount of money paid to or receivable by directors under long term incentive schemes in respect of qualifying services; and

(ii) The net value of assets (other than money or share options) received or receivable by directors under these schemes.

� The aggregate value of any company contributions paid, or treated as paid, to a pension scheme in respect of directors’ qualifying services.

D. CORPORATE FINANCIAL MANAGEMENT

In this final section, we shall consider the scope of financial management within the modern company.

Financial Decision Making We noted earlier that corporate finance deals with three decisions which a firm must undertake.

� Which investments should the firm undertake? – the investment decision

� How, where, when and how much finance should be raised? – the financing decision

� How should the firm’s profits be used or distributed? – the dividend decision.

(a) The Investment Decision

The investment decision involves a firm in choosing which projects to invest or disinvest in. It can include internal decisions concerned with current areas of the firm’s involvement, and external decisions concerned with expansion or contraction of the firm by takeover, merger or disinvestment.

(b) The Financing Decision

An organisation is funded by a combination of debt (both long and short term) and equity (share capital). The financing decision involves deciding on the level of funds required, which type or types of funds to raise, and the raising of funds. In deciding the level and type of funds to raise the firm has to know the cost and risk involved in each particular type of funds. The cost involved is the opportunity cost to the provider of the funds; the risk involved in raising finance is the possibility of negative returns on the investment (risk is dealt with in more detail in a later study unit). In making the financing decision a trade-off is often made between keeping as low a level of funds as possible to aid profitability (by limiting the costs of servicing those funds, i.e. interest payments and administration of share registers) and ensuring the firm has sufficient funds to remain solvent.

Page 31: libvolume5.xyzlibvolume5.xyz/law/ballb/3rdyear/semester5/corporatelaw/corporate... · ABE Advanced Diploma in Business Administration Study Manual CORPORATE FINANCE Contents Study

The Context of Corporate Finance 23

© Licensed to ABE

(c) The Dividend Decision

This is the trade-off between retaining profits in the business and distributing them to shareholders.

Whilst initially you will study the three decisions separately, always remember that in practice they are interconnected and their interrelationship must not be ignored. Before reading further try to think of some likely interrelationships.

One common interrelationship is between investment decisions and the financing decision. If a firm decides to invest in a major new project it may have to raise additional finance, having to make a choice with regard to the type and level of finance to acquire. Moreover, the cost of the funds will affect the viability of the project. Another common interrelationship is between the financing decision and the dividend decision – if a firm pays out a high percentage of its profits as dividends it may need to raise additional finance and – as we discussed earlier – it may not be in the shareholders’ best interest. Clearly these are only two examples and whenever a decision is made in one of the areas it will have an impact on the other areas. We shall cover the interrelationships of the decisions in greater detail later in the course.

Financial Functions in Organisations The role of the finance function within an organisation is considerable and includes the following aspects:

� The determination of the volume of financial resources required.

� The acquisition of the required financial resources, either internally through profit retention and dividend policy, or externally through share issues, debenture issues or loans.

� The maintenance of an optimum mix of funding, bearing in mind the potential impact of the capital structure on the market value of the business.

� The needs of the providers of finance have to be considered, including their required return on investment and the maximisation of the value of their shares. If the company is floated on the Stock Market its rules and regulations must be adhered to. A financial manager has a duty to value a business in a manner which shows its true worth, and he should take into account the effect of price changes over time on the valuation of the business. The financial manager must take account of factors which determine share prices, particularly those over which he has control. The business should not only be maximising the wealth of its shareholders, but it should also be seen to be doing so, which requires some degree of financial public relations by the financial managers, in order to maintain a good financial image of the firm to outside sources of investment.

� Assessment and valuation of investment opportunities to ensure that the generated resources are employed efficiently. This includes the investment of surplus cash in short-term investments. In order to do the latter the financial manager must have a thorough understanding of the workings of the “City” and investment markets generally.

� The assessment of the optimum amount of assets provided, including the calculation of fixed assets, current assets and liquid resources wanted by the business. Plans should cater for seasonal fluctuations as well as medium- to long-term strategic requirements and plans of the organisation.

� The finance manager should ensure that suitable control systems are employed for the authorisation of expenditure on fixed assets, to ensure that stocks of raw materials, finished goods and work in progress are kept to a level which is the lowest possible to be consistent with

Page 32: libvolume5.xyzlibvolume5.xyz/law/ballb/3rdyear/semester5/corporatelaw/corporate... · ABE Advanced Diploma in Business Administration Study Manual CORPORATE FINANCE Contents Study

24 The Context of Corporate Finance

© Licensed to ABE

efficiency, and to ascertain that the cash available in the business is used as fully as possible throughout the year. The control function also covers investments, and the maintenance of a reasonable balance between credit taken and credit given.

� The control of liquidity, by ensuring sufficient working capital within the company taking account of the timing of future plans for growth and fixed asset purchases.

� In all but the smallest of organisations there should be provision for internal audit to ensure that control systems are working and to help prevent and detect errors and fraud.

� The financial manager should ensure that all risks capable of being calculated are covered by insurance, including cover in respect of accident, fire and consequential loss and, if appropriate, credit insurance.

� The business must comply with statutory requirements including those of central and local government and the European Union, and for a listed company with those of the Stock Exchange. Accounts must show a true and fair view of the position of the business at the time they are drawn up. Moreover, they should be capable of being interpreted in a way that gives the reader the correct impression. Problems arising from reconstructions and amalgamations of companies also form a major part of the role of the financial manager.

The Role of the Finance Manager The tendency to internalise corporate finance means that the financial manager of the major or multinational company must become an expert in a wide variety of areas including the responsibilities we described above. He must also remain up-to-date in a world-wide market which is rapidly changing. It is a massive task and, to combat the resulting problems, larger companies have typically created specialist functions, each reporting to the Financial Director. They are senior roles and their functional responsibilities are discussed below. In the biggest organisations, responsibilities would be further broken down and delegated. Smaller organisations may not require all the functions, and one person may take responsibility for more than one area of work. The job titles in any one organisation may differ from those listed below, but the functions remain the same.

The different functions are:

� The Controller

This role encompasses capital budgeting and investment appraisal, stock and credit control, the selection of short-term investments and the internal and external audit functions.

� The Accountant

The accountant will be responsible for the records of financial data and for producing management reports and company accounts. The role is split into the disciplines of management and financial accounting covered elsewhere in your studies.

� The Treasurer/Cashier

This involves budgeting for cash flows, procuring adequate liquid funds and the physical security of cash resources. In a business which deals internationally this function will also include foreign currency management.

� Financial Strategist

This involves procuring and managing the correct volume and optimum mix of funds, whilst organising a suitable channel of communication with external bodies such as government and investors and, as a member of general management, in meeting the demands of fellow functional managers.

Page 33: libvolume5.xyzlibvolume5.xyz/law/ballb/3rdyear/semester5/corporatelaw/corporate... · ABE Advanced Diploma in Business Administration Study Manual CORPORATE FINANCE Contents Study

The Context of Corporate Finance 25

© Licensed to ABE

� Corporate Planner

The responsibilities of this role include the development of strategic and business plans. In some organisations it is the responsibility of a dedicated director dealing with planning.

Often smaller firms do not have a full-time financial manager, relying simply on regular visits by a member of the firm’s external audit staff. At least some businesses lack an awareness of the degree of financial expertise required for their business to operate effectively. Even in firms that do employ a financial manager, many limit his key responsibilities to the production of accounting information and recording of financial data.

The finance director may also be responsible for general administration and/or information technology, depending on the size of the organisation.

When considering financial functions in organisations remember that the financial manager must, to some extent, be concerned with the way in which finance interacts with the other activities in an organisation such as production and personnel. The financial manager must ensure that the individual objectives of each function do not conflict with the overall corporate objective of the business. For example, the marketing manager who seeks to increase sales or market share must do so within budgetary constraints and profitably in the long term. Budgeting is a useful way of coordinating all functional activities – we will discuss budgeting in more detail later in this study unit.

Planning There are a number of different types of planning.

(a) Strategic Planning

Strategic planning is the process by which the objectives of an organisation are made or changed. Examples of decisions involving strategic planning include deciding what to produce and sell and where; whether to merge with another company; and what overall profitability targets to set for the organisation.

Strategies are likely to exist at a number of levels in an organisation and strategic planning must take this into account. Many management theorists have identified three different levels of corporate strategy:

� The corporate level, which is concerned with what type of business the company should be in. The company may specialise in one product or operate in diverse markets. Decisions would include considering whether to widen the range of products or move into different geographical areas.

� Competitive or business strategy, which focuses on how to compete in a particular market; for example, a company may consider a strategy of modernisation and rationalisation of its factories to help it compete in a fabric manufacturing market.

� Operational strategies, which look at how the different functions of the company contribute to other levels of strategy; for example, in seeking to be competitive a company will consider price and marketing of its products.

Strategic planning involves generation of options which are then evaluated and choices made. Management will select the options they intend to pursue and use them to form the basis of strategic plans.

(b) Tactical Planning

Management or tactical planning deals with the achievement of strategic plans utilising the organisation’s resources in the most effective and efficient manner. It involves setting budgets

Page 34: libvolume5.xyzlibvolume5.xyz/law/ballb/3rdyear/semester5/corporatelaw/corporate... · ABE Advanced Diploma in Business Administration Study Manual CORPORATE FINANCE Contents Study

26 The Context of Corporate Finance

© Licensed to ABE

and performance targets for departments, determining the organisation’s structure and developing and launching products and their marketing campaigns.

(c) Operational Planning

Operational planning deals with the achievement of tactical plans by carrying out specific tasks in the most effective and efficient manner.

The dividing lines between strategic and tactical planning, and tactical and operational planning are often unclear, and some decisions involve more than one type of planning. The level at which the decision is taken is generally a guideline as to the type of planning being undertaken, as is the frequency of the decisions. Operational decisions are often taken several times a day by lower-level management and supervisory staff; tactical decisions are generally taken at regular weekly, monthly or yearly intervals by middle and senior management; whereas strategic planning is generally undertaken by senior management and board members, and at infrequent and irregular intervals.

Knowing what is to be accomplished within a specific time-scale is a basic requirement of the planning process, which is necessary to establish how the required results are to be achieved. The financial manager should provide a focal point for functional managers throughout the process, emphasising the importance of financial matters in developing and implementing plans. Especially important in the planning process are the source and timing of cash requirements. The shape of the balance sheet may also be an important issue for the private company whose shareholders wish to realise a part of their investment by floating the company on the Stock Market at a later date. Similarly, a firm wishing to sell the entire business may be very concerned about the size and value of its asset base.

Forecasting In order to develop plans with any degree of accuracy a company must forecast the variables included in plans. Can you think what they might be?

The variables to be forecasted include:

� Demand for, and sales revenue from, the company’s products (analysing each product separately)

� Costs of raw materials

� Wage and salary costs

� Other expense costs

� Competition in the product and supplier markets

� Potential for new product or production methods development, including the results of research and development

� Interest rates and the cost of capital

� Product and safety legislation

� Availability of skilled workers

� Rate of inflation

� Economic growth rates

� Changes in the political environment, including industry regulation

� Exchange rates

Page 35: libvolume5.xyzlibvolume5.xyz/law/ballb/3rdyear/semester5/corporatelaw/corporate... · ABE Advanced Diploma in Business Administration Study Manual CORPORATE FINANCE Contents Study

The Context of Corporate Finance 27

© Licensed to ABE

� Taxation

� Dividend policy

� Assets and liabilities

� Potential for new sales and marketing methods.

Plans should include contingencies for external factors which may occur (also known as sensitivity analysis). Lenders will usually act with extra caution when the economy emerges from recession, because of their potential exposure to the risks of second-round failure (see later in this study unit). Modern computer modelling techniques can help with forecasting. The computer model can be built to assess the effect of changes in all variables and produce scenarios which match the change in the variables.

The actual outcomes of the above must be compared to the forecasts and the variations from the predictions fed back into the planning process to help improve the accuracy of future forecasts.

Budgeting We have already seen how a business implements its long-term plans via tactical and operational plans; often, they are expressed as budgets.

Budgets are quantitative and/or financial statements of the plans of an organisation which are prepared and approved before the period to which they relate.

Budgets can be set for long periods of time covering several years, or for short periods covering days or hours. Often the very short-term plans are expressed in quantitative rather than financial terms and may deal with the production or service process. Similar to the different levels of planning, the longer the period of time covered by a budget the less precise the plans. The common length of time for most budgets, and the budgets you will probably meet at work, are those of one year.

Budgets will be set for a wide variety of areas including all parts of sales and production. A company, a group of companies, or other organisation will have a master budget. A master budget includes an approved summary of production, sales and costs for the period, showing the budgeted trading and profit and loss account, cash flow statement and the balance sheet. In the area of financial planning an organisation will have some, or all, of the following budgets:

� Financial returns on investments

� Cash flow planning

� Profitability

� Sources of finance

� Capital structure

� Working capital control

� Arrangements for banking

� Tax planning

� Foreign currency management

� Changes in asset structures

� Funding planned takeovers

Part of the budgeting and control process is the monitoring of actual results against those forecast in the budgets and other plans. It is unlikely that the plans will be met precisely, and management has to

Page 36: libvolume5.xyzlibvolume5.xyz/law/ballb/3rdyear/semester5/corporatelaw/corporate... · ABE Advanced Diploma in Business Administration Study Manual CORPORATE FINANCE Contents Study

28 The Context of Corporate Finance

© Licensed to ABE

set a tolerance limit for deviations (or variances as they are known in accountancy and finance). The tolerance limit will vary between organisations and between different items of income and expenditure. Variances falling outside the limits must be investigated to see whether there is a problem that needs correcting, an opportunity which can be capitalised, the forecasts need revising in view of unforeseen circumstances, or whether the variance happened purely by chance.

A major part of the process of analysing variances is that the results of the analysis will be used with other new information to update plans and budgets for the future. Indeed, part of strategic planning involves the continual updating of plans as new information becomes available.

Cash Management (a) Cash Flow Planning

In order to understand cash management you should be aware of the difference between profits and cash flow. From your accountancy studies you will be aware that profit is the amount by which income exceeds expenditure when both are matched on a time basis. However, cash flow is the actual flow of cash in and out of the organisation with no adjustments made for prepayments or accruals.

An organisation must ensure that it has sufficient liquid funds to pay its bills as they become due. A business which has insufficient cash may be forced into liquidation by its unpaid creditors, even if it is profitable. Management must therefore plan and control cash flow to prevent liquidation. In the short term this is done by cash flow budgeting, which can be daily, weekly, monthly or yearly, ensuring that the organisation has sufficient cash inflows to meet its outflows as they become due. If a shortage is expected, then the firm can arrange finance, perhaps by increasing its overdraft, to overcome the problem. If there is a short-term surplus of funds then they should be invested in short-term marketable securities. Fluctuations in cash can arise for a variety of reasons, a major one being seasonal fluctuations in trade.

A significant part of strategic planning is the setting of long-term financial plans, setting out the medium and long-term financial objectives. The plans can allow the business to judge whether or not it will achieve its financial objectives, e.g. the repayment of loans, what finance needs it may have in the long term and short term, and whether there is any surplus cash which should be invested.

Strategic cash flow planning is basically long-term cash flow budgeting, except that there are greater uncertainties about cash flows due mainly to the longer planning horizon. The cash flows which result from planning must be consistent with the firm’s financing, investment and dividend policies. The firm will be able to respond to cash flow shortages or surpluses in a planned way – raising or investing the required amount of finance in the way most optimal for the company. This includes taking advantage of changing interest rates and economic climates to the benefit of the company. Some businesses will be at risk of failure due to insufficient financial resources to accommodate their necessarily increased investment when the upturn in the economy does arrive. Such failure is known as the “second-round” failure.

However, there may be unexpected changes in the business cash flow patterns, such as a slump in trade, which can not be forecasted for, so an organisation must have sufficient cash to cover such eventualities. Cash flow planning which considers the ability of an organisation to overcome such cash flow deficits is called strategic fund management.

Strategic fund management may deal with such unexpected changes by cutting dividend payments; improving working capital management by increasing creditors or the overdraft level or by cutting debtors and stocks; or by selling assets which are not required by the core

Page 37: libvolume5.xyzlibvolume5.xyz/law/ballb/3rdyear/semester5/corporatelaw/corporate... · ABE Advanced Diploma in Business Administration Study Manual CORPORATE FINANCE Contents Study

The Context of Corporate Finance 29

© Licensed to ABE

activities of the organisation. (Those assets which are most marketable, such as short-term marketable securities, would be sold before those which would take longer to realise, such as land and unused machinery.)

Whilst a company should ensure that it has sufficient cash to cover unforeseen circumstances, holding too much cash is inefficient because of the opportunity cost of income foregone (either from placing short-term surpluses in marketable securities, or from the investment in projects earning a rate of return higher than the return required by the supplier of finance for longer-term surpluses). If the company cannot invest long-term surplus money in projects which receive higher returns than placing the money in a bank account, then they should return the money to the shareholders to allow them to utilise the money in the way they consider optimal. This can be done either by paying out higher dividends or by repurchasing the company’s shares.

(b) Capital Structure and Cash Flow

The capital structure of an organisation is the way in which its assets are financed, i.e. the levels, types and proportions of equity, debt capital, long- and short-term liabilities. Obviously, when the business is growing it will require additional capital to fund its increased assets.

The working capital of the business is made up of the more permanent current assets plus the fluctuating current assets less current liabilities. Different levels, and types, of long- and short-term sources of finance can be used to fund the fixed assets and the working capital of the business. The method chosen will have an impact on the cash flow of the organisation.

An aggressive approach to the financing of working capital is to finance all fluctuating current assets and some more permanent ones from short-term sources. This may be beneficial to the company if the short-term funds are cheaper than the equivalent long-term ones, but increases the likelihood of liquidity and cash flow problems.

A conservative approach uses long-term financing to fund all permanent assets and some fluctuating current assets. In fact it is only when current assets are large that short-term financing is necessary – at other times there may be surplus cash to invest. The company would probably invest this in marketable short-term securities, especially if the amount is significant.

A balanced approach is to fund permanent assets from long-term sources and fluctuating assets from short-term sources.

The method chosen will be a choice for senior management and will reflect their overall policy and plans for the organisation. However, a business may be forced to adopt a sub-optimal approach (from their viewpoint) due to restrictions in their ability to raise the “correct” type of funds. As in all areas, the policy adopted by the firm should match the expectations of the shareholders (see Study Unit 1 if you wish to remind yourself of this topic).

Moreover, the market’s view of the company’s prospects and abilities will determine the level of debt investors will be willing to lend the company. The nature of the industry that the company operates in will also affect the level of debt that the market will consider prudent – the more volatile the sector, the lower the level of gearing which would be advisable.

Economic and Government Influences The operations of an organisation are subject to a whole series of constraints imposed by competitors, customers, trade unions, the general public and statutory bodies. As the social environment has evolved, government has become more involved in the operation of business – issuing laws,

Page 38: libvolume5.xyzlibvolume5.xyz/law/ballb/3rdyear/semester5/corporatelaw/corporate... · ABE Advanced Diploma in Business Administration Study Manual CORPORATE FINANCE Contents Study

30 The Context of Corporate Finance

© Licensed to ABE

regulations, directives, voluntary policies and codes of practice covering most areas of commercial life.

The area is rapidly changing, and we cannot possibly hope to cover every aspect affecting companies, e.g. in areas such as health and safety; instead we will highlight some of the more important pieces of legislation and government policy which directly impact in the area of finance.

(a) The Companies Acts (1985 and 1989)

These contain various legal requirements for a company operating its affairs in the UK including, in the field of accounting and finance: requirements for depositing accounts; annual returns; registration of changes with the Registrar of Companies; requirements for disclosure of information in the published accounts; and procedures for the winding-up of a company. The legislation lays down the minimum standards with which the company must comply.

(b) The European Union (EU)

The EU provides a huge area of government involvement in corporate laws. The Council of Ministers can publish requirements that impose obligations upon all companies in member countries, even overriding the laws in those member states. However, the Community does provide the opportunity for additional markets and greater availability of financial resources, including grant aid. It is essential that you keep up-to-date and well informed about EU developments by reading business journals and newspapers. In particular, the debate about when or whether the UK should adopt the Euro is one to follow closely.

(c) Government Monetary Policy

Monetary policy is the manipulation by the Government of interest rates and/or the supply of money in an attempt to influence the economy, and such economic variables as growth, inflation and the balance of trade. However, in May 1997 the UK Labour Government surrendered its setting of interest rates to the Bank of England, in line with many other European countries. Nevertheless, the Bank of England will still manipulate interest rates to achieve the desired levels of major economic variables, including inflation, and this clearly, along with changes in the money supply, will have an impact on the cost and availability of capital to an organisation. Moreover, as with fiscal policy discussed below, monetary policy will also impact on other areas of a firm’s operation including the ability to export the firm’s goods and the cost of imported raw materials and components. Although there have been reductions in UK interest rates, the levels are still higher than in many other European countries and this is causing major concern for UK companies that export goods as cheaper alternatives are available from suppliers in other countries.

(d) Fiscal Policy

Fiscal policy is the alteration by the Government of the levels of taxation or the level of government spending in order to affect economic variables such as unemployment and inflation. It will obviously have an impact on the organisation – not just in the levels of taxation which it pays and in the availability and level of grants, but also in the overall level of demand in the economy and for the firm’s products or services.

Page 39: libvolume5.xyzlibvolume5.xyz/law/ballb/3rdyear/semester5/corporatelaw/corporate... · ABE Advanced Diploma in Business Administration Study Manual CORPORATE FINANCE Contents Study

31

© Licensed to ABE

Study Unit 2

Company Performance, Valuation and Failure

Contents Page

Introduction 33

A. Ratio Analysis 33 Liquidity 33 Profitability 34 Debt and Gearing 36 Investor Ratios 38 Miscellaneous Items 40

B. Using Ratio Analysis 40 Analysing Company Performance 41 Problems with the Use of Ratios 45 Uniform Cost Accounting 45 The Centre for Inter-firm Comparison 45

C. Introduction to Share Valuation 48

D. Methods of Share and Company Valuation 49 ARR (Accounting Rate of Return) Method 49 P/E Method 49 Net Asset Method 50 Super-profits Method 51 Dividend Yield Method 52 Discounted Future Profits 54 The Berliner Method 55 Note re CAPM 55

(Continued over)

Page 40: libvolume5.xyzlibvolume5.xyz/law/ballb/3rdyear/semester5/corporatelaw/corporate... · ABE Advanced Diploma in Business Administration Study Manual CORPORATE FINANCE Contents Study

32 Company Performance, Valuation and Failure

© Licensed to ABE

E. Predicting Company Failure 55 Company Information Services 55 The Z Score 56 Other Models 57 Other Indicators 58 Problems with Prediction Models 58

F. Capital Reconstruction Schemes 59 Reasons for Capital Reconstruction 59 Principles of Capital Reconstruction 59

Page 41: libvolume5.xyzlibvolume5.xyz/law/ballb/3rdyear/semester5/corporatelaw/corporate... · ABE Advanced Diploma in Business Administration Study Manual CORPORATE FINANCE Contents Study

Company Performance, Valuation and Failure 33

© Licensed to ABE

INTRODUCTION

A company’s financial position will affect its plans, and its ability to carry out those plans, in the key areas of financing, investment and dividend policy. Shareholders and other interested parties will thus be interested in the profitability of the company, its ratio of debt to equity, its liquidity and other measures of a company’s financial performance. Company reports and financial statements can be used to assess the performance of companies by the use of ratio analysis.

When considering ratios it is important that a number of years are looked at to obtain as meaningful a picture as possible, and also to compare the organisation with others which are similar in size and industry. The ratios chosen should be relevant to the organisation in question, e.g. stock turnover would not be relevant to a service organisation with little or no stock. The wider economic and environmental context the firm is operating in must also be considered.

A. RATIO ANALYSIS

This should be a revision section for you, ratio analysis having been covered in your earlier studies. The ratios we shall consider can be grouped into four main types:

� Liquidity

� Profitability

� Debt and gearing

� Investor.

We will now consider the main ratios under each heading.

Liquidity A company may be profitable but not necessarily liquid and able to pay its obligations when required – failure to do so may lead to the company being wound-up. The ratios and figures under this heading indicate the extent to which a company can meet its current liabilities as they become due. The common liquidity ratios are:

� The current ratio, which is calculated as:

sliabilitieCurrent assetsCurrent

� The acid test ratio, which is calculated as:

sliabilitieCurrent stock less assetsCurrent

These two ratios show the liquid resources available to pay the short-term liabilities, low ratios indicating potential cash flow problems. The quick (acid test) ratio is often calculated because of the length of time it may take to convert stock into cash, this ratio giving the truer picture of the liquid assets of the organisation. The yardsticks which an organisation’s ratios are traditionally compared to are 2:1 for the current ratio and 1:1 for the acid ratio. However, the yardsticks should be viewed in relation to the organisation in question, and in relation to other ratios – a company with high stock turnover can have a healthy liquidity position with an acid ratio of less than 1.

Page 42: libvolume5.xyzlibvolume5.xyz/law/ballb/3rdyear/semester5/corporatelaw/corporate... · ABE Advanced Diploma in Business Administration Study Manual CORPORATE FINANCE Contents Study

34 Company Performance, Valuation and Failure

© Licensed to ABE

When considering current and acid ratios remember that high results may indicate overstocking, poor collection of debtors or that the company has excessive cash; in such cases action should be taken to determine why there is a high ratio and steps taken (if appropriate) to correct the situation.

� The debtors’ payment period, which is calculated as:

365salesCredit

debtors Trade ×

This shows the length of time taken by a company’s debtors to pay their bills. In general companies give 30 days’ credit on invoices and this can be used as a yardstick. The resultant ratio, however, must be viewed in the light of any seasonal variations which may be present in the figures used to calculate the ratio. Industry norms and the type of customers the firm has also need to be considered in determining a yardstick to use (e.g. a high level of overseas customers may mean that terms longer than 30 days may be given).

� The stock turnover ratio, which is calculated as:

365sales ofCost stock Average ×

It shows the number of days that stocks are held; as with the debtors’ payment period care must be taken to note any seasonal fluctuations contained in the figures used, for which reason it is better to look at the trend in this figure. An increase may indicate a slow down in sales or that the firm is overstocking.

Stock turnover ++++ debtor payment period give a good indication of the cash conversion period.

The ratio can also be calculated to show the number of times average stock is turned over in a year:

stock Averagesales ofCost

� In addition, calculations by Beaver, Lev and others in the USA have shown that the most significant single index of solvency is provided by establishing a trend line over a period, from the ratio:

debt Total flowCash

Profitability The primary (or most frequently used) ratio is return on capital employed (ROCE). This is usually calculated as:

employed Capital taxationandinterest before activitiesordinary on Profit

or employed Assets

taxationandinterest before activitiesordinary on Profit

This provides a measure as to how the investment of capital in the company is being rewarded. The result can be compared to the cost of capital and the returns available elsewhere reflected in interest rates and other companies’ ROCE figures. You can see a breakdown of this ratio in Figure 2.1.

Page 43: libvolume5.xyzlibvolume5.xyz/law/ballb/3rdyear/semester5/corporatelaw/corporate... · ABE Advanced Diploma in Business Administration Study Manual CORPORATE FINANCE Contents Study

Company Performance, Valuation and Failure 35

© Licensed to ABE

Figure 2.1: Breakdown of ROCE

Profit/Assets employed

Assets/Sales Profit/Sales

Current

assets/Sales Fixed

assets/Sales Admin.

costs/Sales Production

Costs/Sales

Selling & distribution costs/Sales

Stock

Land & buildings

Labour

Debtors

Plant & equipment

Material

Cash

Fixtures & fittings

Expenses

Vehicles

However, as with many other ratios, there is no one agreed method of calculating it. Problems in its calculation include the following:

(a) Should profit be pre-tax or post-tax? Shareholders will prefer post-tax because this is the money available to pay dividends with; management will prefer pre-tax unless they are responsible for minimising the company’s tax liability.

(b) Should non-recurring items, e.g. profit on the sale of an asset or arising from an insurance claim, be included?

(c) Should non-trading profits, such as rents and investment income, be included?

(d) Should total assets or net assets (net assets = total assets minus current liabilities) be used as the capital employed figure? Often a company has a permanent bank overdraft (which is included in the current liabilities) and therefore should be considered to be part of capital employed.

(e) Should intangible assets such as goodwill be included in the capital employed figure?

(f) How should assets be valued:

� At cost?

� At written-down book values?

� At replacement values?

� At current market values?

(Remember – the understatement of a company’s assets can produce an artificially high ROCE.)

(g) Which balance sheet date and profit figures are relevant? Should the capital employed be that at the start or end of the year, or some average figure?

Page 44: libvolume5.xyzlibvolume5.xyz/law/ballb/3rdyear/semester5/corporatelaw/corporate... · ABE Advanced Diploma in Business Administration Study Manual CORPORATE FINANCE Contents Study

36 Company Performance, Valuation and Failure

© Licensed to ABE

The method chosen to calculate ROCE depends on the individual company. There is some evidence that companies often choose the set of values which gives them the highest ROCE but, whichever method is chosen, you must be consistent between years and companies to allow comparability. Industry norms are also important, e.g. service industries tend to have higher profit margins than manufacturing industries.

Other common profitability ratios are:

� Gross trading profit : Sales

� Net trading profit : Sales

(It is useful to compare trends in these two measures against each other to provide an indication as to how well expenses are being controlled.)

� Net profit : Equity capital

� Net profit : Working capital

� Sales : Capital employed (expressed as a number of times)

� Fixed asset turnover rate, measured by Sales : Fixed assets (expressed as a number of times) with a possible breakdown to asset class.

� Current asset turnover rate, which is subdivided into the following:

(i) Sales : Total current assets

(ii) Sales : Debtors

(iii) Sales : Stocks held

In this calculation the figure of sales may be replaced by the cost of sales (if known) since stocks at cost value remove the potentially distorting effects of selling price changes in response to market conditions.

Note: The Du Pont Index is a variation on the primary ratio:

employed CapitalSales

SalesProfit ×

made up of the secondary ratios – the profit margin and asset turnover. Profit may be calculated before interest and taxation or just as pre-tax profit. The first shows the level of profit achieved on sales and the second shows how well assets are being used to generate sales. Often there is a trade-off between profit margin and turnover – high profit levels may lead to low sales and vice versa. Used in a series of ratios over a period of time this provides more information than the basic ROCE ratio.

An obvious check on profitability is to look at the level of profit or loss shown in the accounts and the change from previous years.

Debt and Gearing � The gearing ratio is expressed as Debt capital : Equity capital. Again there is no one accepted

method of calculating this ratio – some analysts prefer to use long-term debt only, whilst others prefer to use all debt (excluding provisions) in a company’s structure. (The latter is often called the debt ratio.) Similarly, there is no agreement as to whether balance sheet or market values should be used.

Page 45: libvolume5.xyzlibvolume5.xyz/law/ballb/3rdyear/semester5/corporatelaw/corporate... · ABE Advanced Diploma in Business Administration Study Manual CORPORATE FINANCE Contents Study

Company Performance, Valuation and Failure 37

© Licensed to ABE

There is no absolute “correct” level of gearing, although an often quoted benchmark for the debt ratio is 50%; the resultant figure again should be considered in line with previous years and industry norms.

The significance of the gearing ratio is the extent to which profit fluctuations are borne by the equity holders. The higher the level of gearing the greater the impact on shareholder wealth of changes in profit levels (see later in course). Moreover, a high level of debt makes future borrowing more difficult.

Example

Gearing ratio is frequently calculated using the formula:

Gearing ratio = Reserves + capital shareOrdinary

capital dividend Fixed +interest Fixed

The following is an extract from the balance sheet of Denton Ltd as at 31 December 200X:

£000

Creditors: Amounts falling due after more than one year

8% debentures 10,000

Capital and Reserves:

Ordinary share capital (£1 ord shares) 30,000

10% Preference shares 15,000

Reserves 23,000

The gearing ratio is:

000,23000,30000,15000,10

++ = 0.47 : 1

Remember, however, that this is only one method of calculating the gearing ratio, and you should always make this clear in using it.

Other ratios in this group include:

� Equity interests : Net assets

� Debentures : Net assets

These two ratios provide an indication of the cover of fixed assets to the particular type of capital investment. Moreover, by establishing a ratio of Fixed assets : Equity capital you can see to what extent the shareholders “own” the fixed assets.

� Debenture interest cover measures the “safety” of the interest payment – the higher the cover the more dependable the payment is. It is calculated by dividing the Net profit before tax and debenture interest by the Debenture interest payable. For example, if the profit before tax and interest was £15,000 and the debenture interest was £3,000 then it would be 15/3 = 5 times covered.

� The cash flow ratio, measured as:

)provisions (includingdebt Totalstatement) flowcash company thefrom(taken flowin cash annualNet

Page 46: libvolume5.xyzlibvolume5.xyz/law/ballb/3rdyear/semester5/corporatelaw/corporate... · ABE Advanced Diploma in Business Administration Study Manual CORPORATE FINANCE Contents Study

38 Company Performance, Valuation and Failure

© Licensed to ABE

This shows the ability of an organisation to meet its commitments, with changes in the ratio showing changes in the cash position of the firm.

Investor Ratios � A major ratio in this class is that of earning per share (EPS) calculated as:

Net profit after tax, debenture interest, extraordinary items, minority interests and preference dividends/No. of ordinary shares in issue and ranking for dividends. It is measured in pence.

Investors wish to see growth in the EPS in order to fund investment and increases in dividend payments. An inability to sustain a level of EPS could have a negative impact on the level of a company’s dividend.

When looking at the trend of the EPS over time changes in capital structure, such as the issuing of new shares or the conversion of convertible loan stock, need to be considered. Similarly, when comparing different companies differences in their number of issued shares should be considered. In the former case it would be useful to calculate the fully diluted earnings per share which takes into account all capital instruments ranking as equity shares now or in the future; for example, convertible loan stock or share options. This also gives investors an indication of the effects of the future exercise of share options, warrants and such like (including in the numerator the savings in financing such instruments and the additional profits to be earned from utilising the funds raised in the business).

� Dividend cover shows how many times the declared dividend could be paid out of distributable profits. For example, if a company’s profit after tax and debenture interest was £40,000 and a dividend of £32,000 was declared, the dividend cover would be:

dividend Declaredinterest debenture andafter tax Profit =

3240 = times covered.

If preference share dividends are payable these, too, form a prior deduction when considering dividend cover on ordinary shares. The ratio shows the proportion of distributable profits being paid out and indicates the risk that if earnings fall this level of payout could not be maintained. A high cover may indicate that the firm is investing in future growth.

Dividend cover can also be calculated using the EPS:

Dividend cover = shareordinary per Dividend

EPS

� The dividend yield of a company’s shares is important because it shows the return the investor receives on the market value of shares (declared dividend is based on the nominal (or par) value of shares). The shareholder can compare the yields between different investments to help determine the value for money of his shares in the company. Dividend yield is calculated as:

100shareper ueMarket valshareper dividend Gross ×

Dividends paid to shareholders in the UK are net dividends (after tax); to determine the gross dividend figure to use in the above calculation we have to adjust for taxation, using the following formula:

dividend Gross(%) tax income of rateLower 100100 shareper dividendNet =

−×

Page 47: libvolume5.xyzlibvolume5.xyz/law/ballb/3rdyear/semester5/corporatelaw/corporate... · ABE Advanced Diploma in Business Administration Study Manual CORPORATE FINANCE Contents Study

Company Performance, Valuation and Failure 39

© Licensed to ABE

Example

Thomas plc has just declared a net dividend of 10p per share. If the current share price of Thomas is 135p, what is the dividend yield?

Gross dividend = (%) tax income of rateLower 100

100 shareper dividendNet −

×

= 20(%)10010010

−×

= 12.5p

Dividend yield = 100shareper ueMarket valshareper dividend Gross ×

= 10013512.5 ×

= 9.259

� Interest yield is the equivalent of dividend yield for the return on the market price of loan stock.

yieldInterest 100stockloan of ueMarket val tax)(beforeinterest Gross =×

Be careful not to confuse interest yield with the coupon rate, which is the return on the face value of the debt.

The interest yield is generally higher than the dividend yield. This is because shareholders expect to receive capital gains on their shares. When capital gains and dividend yield are added together they should produce a higher return than interest yield, reflecting the greater risk of holding shares (risk is considered in detail in a later study unit).

� Earnings yield is the equivalent of dividend yield for the return on the market price of earnings per share.

yield Earnings100share of ueMarket val

tax)(before earnings Gross =×

The gross value of the EPS is used in order to allow comparability with dividend yield.

� The price earnings (P/E) ratio compares the market price of a share to the earning per share and is expressed as:

)(after tax shareper EarningspriceMarket

Note that the net basis is used rather than the gross basis we encountered in earlier ratios. For example, if ABC plc has 200,000 ordinary shares of £1 which are currently quoted in the market at £1.70, and its net earnings for the year were £45,000, the P/E ratio would be:

=0,000)(45,000/20

1.70 7.56.

An investor purchasing the shares at £1.70 would, in other words, be paying 7.56 times the annual earnings on those shares. The level of a company’s P/E ratio is seen as a reflection of the market’s views on the prospects of a company. A company with a higher P/E ratio than another may have better growth prospects or more secure earnings. This is because the P/E

Page 48: libvolume5.xyzlibvolume5.xyz/law/ballb/3rdyear/semester5/corporatelaw/corporate... · ABE Advanced Diploma in Business Administration Study Manual CORPORATE FINANCE Contents Study

40 Company Performance, Valuation and Failure

© Licensed to ABE

ratio should remain constant over time, and an increased EPS will result in an increased market price and thus an increased P/E ratio. However, the only real value of the P/E ratio is that it shows the relationship between earnings and market price for a company, which may be difficult to interpret when market prices are fluctuating widely due to circumstances outside the control of the company, such as changes in interest and exchange rates.

Miscellaneous Items The following ratios may also be of use to stakeholders when analysing reports:

� Value added per employee

� Sales per employee

� Asset structure – this involves calculating the varying proportions in which the assets are structured, for example:

Fixed assets 35%Investments 5%Net current assets 60%

100% � Sources of asset structure, for example:

Ordinary capital and reserves 50%Debt capital 45%Net current liabilities 5%

100% Alternatively the asset structure could show gross current assets, while gross current liabilities are shown in the source of asset structure. The two could then be compared to see to what extent outside interests “own” the company assets.

� Proportions of shareholders’ interests, for example:

Preferential capital 10%Ordinary capital 50%Capital reserves 8%General reserves 20%Specific reserves 12%

100%

B. USING RATIO ANALYSIS

It would not be possible to list every single ratio capable of calculation. The important point to note is that different groups will be interested in using different ratios to reflect their particular interest in the company. You must also remember that ratios should always be considered as part of a trend, and once calculated they require careful interpretation. Companies often give information on ratios in their five and ten year summaries, but the full set of accounts is essential to allow a full comparison – they are generally only available for the year of the accounts and the previous years.

Page 49: libvolume5.xyzlibvolume5.xyz/law/ballb/3rdyear/semester5/corporatelaw/corporate... · ABE Advanced Diploma in Business Administration Study Manual CORPORATE FINANCE Contents Study

Company Performance, Valuation and Failure 41

© Licensed to ABE

Analysing Company Performance We will now study the use of ratio analysis by considering a detailed example. It is important to remember that you will generally be asked for an interpretation of the ratios identified, which means you must comment not only on individual ratios, but also on the composite position disclosed by your analysis, including changes in profit before and after tax and turnover. You must also remember, where possible, to look at a selection of ratios with at least one from each of the four main groups we have discussed.

Example

The summarised accounts of New Ideas plc are as follows:

Balance Sheets as at 30 April

Year 2 Year 1 £000 £000

Fixed assets (net) 6,401 2,519 Current assets

Stock 25,426 20,231 Debtors 21,856 20,264 Balance at bank 2,917 6,094

56,600 49,108

Ordinary shares of 50p 5,000 5,000 Revenue reserves 14,763 12,263 Deferred taxation 5,433 3,267 10% Debenture loans 10,000 10,000 Creditors: Amounts falling due within one year

Trade creditors 18,762 16,431 Taxation 1,642 1,247 Dividends 1,000 900

56,600 49,108

Results for the year ended 30 April

Year 2 Year 1 £000 £000 Sales 264,626 220,393 Trading profit 9,380 8,362 Interest payable 1,000 1,000 Taxation 4,380 3,642 Dividend 1,500 1,400

Page 50: libvolume5.xyzlibvolume5.xyz/law/ballb/3rdyear/semester5/corporatelaw/corporate... · ABE Advanced Diploma in Business Administration Study Manual CORPORATE FINANCE Contents Study

42 Company Performance, Valuation and Failure

© Licensed to ABE

The following additional information is provided.

(a) The ordinary shares are quoted at £1.20.

(b) New Ideas plc requires £16 million for an investment project and is considering one of the following:

(i) The issue to shareholders of £16 million 10% convertible (£1 for 1 share) debentures at par.

(ii) A rights issue at 80p.

(iii) The sale in the market of £16 million 13% debentures at par.

You are required to:

(a) Calculate from the balance sheet and results:

(i) Two ratios particularly significant to creditors.

(ii) Two ratios particularly significant to management.

(iii) Two ratios particularly significant to shareholders.

(b) Comment briefly on the change between Year 1 and Year 2 in the ratios you have calculated.

(c) Calculate the immediate effect of the three schemes on the gearing of the company.

(d) Calculate the effect of the three schemes on the earnings per share, on the assumption that the Year 2 profits from the existing assets will be maintained and that the £16m net investment will produce profits of £3.5m before tax and interest. The rate of tax can be assumed at 50% (this is not the current rate but is used for ease of calculation).

Answer

Year 2 Year 1

(a) (i) Ratios significant to creditors

� Current ratio = sliabilitieCurrent

assetsCurrent

Year 2 50,199 : 21,404 2.35 : 1

Year 1 46,589 : 18,578 2.51 : 1

� Liquidity ratio = sliabilitieCurrent Stock assetsCurrent −

Year 2 24,773 : 21,404 1.16 : 1

Year 1 26,358 : 18,578 1.42 : 1

(ii) Ratios significant to management

� Activity ratio = Sales

profittax -Pre

Year 2 8,380 : 264,626 3.17%

Year 1 7,362 : 220,393 3.34%

Page 51: libvolume5.xyzlibvolume5.xyz/law/ballb/3rdyear/semester5/corporatelaw/corporate... · ABE Advanced Diploma in Business Administration Study Manual CORPORATE FINANCE Contents Study

Company Performance, Valuation and Failure 43

© Licensed to ABE

Year 2 Year 1

� Profitability ratio = assetsNet profittax -Pre

Year 2 8,380 : 35,196 23.8%

Year 1 7,362 : 30,530 24.1%

(iii) Ratios significant to shareholders

� Return on capital employed = funds rs'Shareholdeprofittax -After

Year 2 4,000 : 19,763 20.2%

Year 1 3,720 : 17,263 21.6%

Note that after-tax profit is used as this is preferred by shareholders.

� Dividend cover ratio = Dividend

interest debenture andafter tax Profit

Year 2 4,000 : 1,500 2.7 : 1

Year 1 3,720 : 1,400 2.7 : 1

(b) Comments on ratios

In spite of an increase in sales of 20% and an increase in pre-tax profits of 13.8%, the ratios mentioned show a marginally unfavourable trend between Year 1 and Year 2. Among the unfavourable trends, the change in the liquidity ratio may cause concern to creditors.

(c) Effect of fund-raising schemes on gearing

Gearing is:

reserves) + capital (shareEquity shares Preference + capitalLoan

This is currently:

50.6%.19,76310,000 =

(i) Issue of £16m 10% convertible debentures changes the gearing to:

131.6%19,76326,000 =

(ii) By implementing a rights issue of ordinary shares, the gearing is reduced to:

28.0%35,76310,000 =

(iii) By issuing £16m 13% debentures, the gearing is the same as under (i) above.

It is considered that schemes (i) and (iii) represent a dangerously high level of gearing.

Page 52: libvolume5.xyzlibvolume5.xyz/law/ballb/3rdyear/semester5/corporatelaw/corporate... · ABE Advanced Diploma in Business Administration Study Manual CORPORATE FINANCE Contents Study

44 Company Performance, Valuation and Failure

© Licensed to ABE

(d) Effect of fund-raising schemes on Earnings per Share

The current EPS is calculated as follows:

£000 £000Trading profit 9,380less: Interest payable 1,000 Taxation 4,380 5,380

Net profit 4,000

Earnings per share = 40p10,0004,000 =

(i) On issue of £16m 10% convertible debentures:

£000 £000Net profit as before 4,000add: Additional profit 3,500less: Tax @ 50% 1,750 1,750

5,750less: Debenture interest (10% on £16m) 1,600 Tax @ 50% 800 800

4,950

Earnings per share = 49.5p10,0004,950 =

(ii) On issue of 20 million ordinary shares (at 80p each = £16m):

£000 £000Net profit as before 4,000add: Additional profit 3,500less: Tax @ 50% 1,750 1,750

Net profit 5,750

Earnings per share = 19.2p30,0005,750 =

(iii) On issue of £16m 13% debentures:

£000 £000Net profit as before 4,000Additional profit (net) 1,750

5,750less: Debenture interest (13% on £16m) 2,080 Tax @ 50% 1,040 1,040

4,710

Earnings per share = 47.1p10,0004,710 =

Page 53: libvolume5.xyzlibvolume5.xyz/law/ballb/3rdyear/semester5/corporatelaw/corporate... · ABE Advanced Diploma in Business Administration Study Manual CORPORATE FINANCE Contents Study

Company Performance, Valuation and Failure 45

© Licensed to ABE

Note that potential problems can also be detected from the company’s accounts by carefully reading the reports that accompany the figures and the use of significant financial ratios. We shall look at this in more detail in later in the course.

Ratios can also be used in inter-firm comparisons. Inter-firm comparisons involve the contrasting of the results of a company with one or more other companies in order to help assess their relative performances.

Problems with the Use of Ratios The main difficulty with the use of ratios is the question: “Are we really comparing like with like?”.

Even where the comparison is between two companies of roughly equal size and ambition, there will always be an element of doubt that the comparison has true validity because of alternative accounting policies which can be adopted in areas such as depreciation and the use of off-balance sheet items. One of the biggest problems in all forms of comparison is the differences which occur in the structure and culture of businesses, the economic and general environment making strict comparison extremely difficult. Similarly, when making comparisons over time the impact of inflation must be considered because of its impact on turnover, earnings, profit and asset values. However, the comparisons are useful in helping to judge stewardship and the return on the investment relative to others available.

If the accounts are different to those of the industry or segment to which they belong, further investigation may be required by the financial manager or analyst.

Uniform Cost Accounting Attempts have been made in the past to compare costs and data of different firms through the medium of uniform cost accounting, resulting in various degrees of success. In the process of inter-firm comparison the following common features are often standardised for a particular industry:

� Accounts classification and codes

� Methods of recovering overhead costs

� The division of departments into producing and service cost centres

� The accounting system employed, e.g. whether or not interlocking accounts are used

� Uniformity in charging depreciation

� Methods of costing joint projects and products

� The manner in which the “normal capacity” is determined for the purposes of overhead recovery.

Often other industry-dependent factors will also have to be standardised. Each system will be “tailored” to suit the individual needs of the particular business, but at the same time the uniformity of the approach and procedures will be maintained.

Once uniformity of methods has been defined, use can be made of inter-firm comparisons. This can be done at a variety of levels, for instance by an industry itself, by an individual company both intra- and inter-firm, by a management professional body, or by researchers in a university.

The Centre for Inter-firm Comparison Established by the Institute of Management and the British Productivity Council in 1959, the Centre has evolved the pyramid method of selecting ratios. Under this method the principal ratios are listed at the top of the pyramid, starting with return on capital employed which is referred to as the primary

Page 54: libvolume5.xyzlibvolume5.xyz/law/ballb/3rdyear/semester5/corporatelaw/corporate... · ABE Advanced Diploma in Business Administration Study Manual CORPORATE FINANCE Contents Study

46 Company Performance, Valuation and Failure

© Licensed to ABE

ratio. This is taken to be the key indicator of company performance and profitability. The ratio is then broken down into a number of subordinate ratios as shown in Figures 2.2 and 2.3. (All ratios shown in the pyramid are compiled for the industry as a whole.)

Figure 2.2: Pyramid Diagram of Ratios – Certain Distributive Trades (as devised by the Centre for Inter-firm Comparison)

The overall profitability of the operations of the business, and the overall success of its management, depend on the ratio:

employed assets Total

taxbeforeProfit

Differences between constituent parts of this ratio could arise from:

Differences in the ratio or Differences in the ratio

Sales taxbeforeProfit

employed assets TotalSales

Inter-firm variations could arise from Inter-firm variations could arise from

the ratio of: or the ratio of: or the ratio of:

Salesprofit Gross

various ratios relating

departmental coststo sales

assets Fixed

Sales assetsCurrent

Sales

Sales v Current assets

may be affected by stock turnover or debtor

turnover

Firms can raise their gross profits either by achieving a higher volume of sales of items earning high gross profits, or by adjusting prices within market constraints. The extent to which either of them affects total gross profit will be indicated by the inter-firm comparison of ratios showing:

(a) The composition of sales made;

(b) The gross profit achieved on different products.

Page 55: libvolume5.xyzlibvolume5.xyz/law/ballb/3rdyear/semester5/corporatelaw/corporate... · ABE Advanced Diploma in Business Administration Study Manual CORPORATE FINANCE Contents Study

Company Performance, Valuation and Failure 47

© Licensed to ABE

Figure 2.3: Pyramid Diagram of Ratios – Manufacturing Industries (as devised by the Centre for Inter-firm Comparison)

employed Assets

profit Operating

Prim

ary

ratio

Salesprofit Operating

employed AssetsSales

Supp

ortin

g ra

tios

Salessold goods of costsFactory Sales

coststion Administraassets Fixed

Sales assetsCurrent

Sales

Sales

costson distributi and Marketing Gen

eral

ex

plan

ator

y ra

tios

Production costs

Sales value of production

stocks Materialcostat Sales

progressin Work costat Sales

costat stocks goods Finishedcostat Sales

DebtorsSales

Spec

ific

supp

ortin

g ra

tios

production of valueSalescosts materialDirect

production of valueSalescostslabour Direct

production of valueSalesoverheads Production

Once the ratios are compiled for the industry as a whole, they will be prepared for the participating firm. The resultant ratios are then compared and any material deviations investigated.

For example, if the primary ratio shows that the rate of return for the company is less than for the industry as a whole, one or more of the subordinate ratios that make it up must be affected. This type of comparison will indicate those work areas which are “up to standard” as well as those which are either under, or over, performing. Senior management can then focus its attention on the specific areas which have been identified.

Page 56: libvolume5.xyzlibvolume5.xyz/law/ballb/3rdyear/semester5/corporatelaw/corporate... · ABE Advanced Diploma in Business Administration Study Manual CORPORATE FINANCE Contents Study

48 Company Performance, Valuation and Failure

© Licensed to ABE

Advantages and Disadvantages of Inter-firm Comparisons

Advantages Disadvantages

Participating firms can see their efficiency in comparison with others.

It is often difficult to obtain uniformity of procedure, methods and definitions.

Correcting action to address weaknesses can be made in good time.

Some companies hide or refuse to release key data required in the process.

By using a specialist, confidential agency, the fear that personal company data will pass to a competitor is removed.

The nature of a company’s operations may be too diverse for true comparison.

Major customers’ and suppliers’ accounts can be compared to measure their future stability and plans made where serious weaknesses are detected.

The process can be time-consuming and requires specialist skills.

C. INTRODUCTION TO SHARE VALUATION

Whilst for quoted companies share values can always be found from market prices on the Stock Exchange, it may be necessary to calculate another valuation for a quoted company’s shares in the process of a takeover bid. (We shall consider this in more detail in the next study unit.)

Unquoted companies do not have a market price for their shares, and may have to estimate the value for them in the following situations:

� The shares are to be sold.

� The shares may have to be valued for taxation purposes.

� Shares may be used as collateral for a loan.

� The company may wish to be quoted on the Stock Market and wants to fix an issue price.

� The relative values of shares involved in a merger may need to be assessed in order to determine the relative prices.

With mergers and takeover bids in the case of a quoted company, the minimum price which can be suggested will be the current market price of the shares. Often the final, negotiated price will be around 20% higher than this minimum in order to encourage the shareholders to part with their shares.

Page 57: libvolume5.xyzlibvolume5.xyz/law/ballb/3rdyear/semester5/corporatelaw/corporate... · ABE Advanced Diploma in Business Administration Study Manual CORPORATE FINANCE Contents Study

Company Performance, Valuation and Failure 49

© Licensed to ABE

D. METHODS OF SHARE AND COMPANY VALUATION

In this section we will look at company valuation in addition to share valuation. You will see that a number of different methods can be used to value a company. Valuations using these methods can often differ significantly from the total of shareholders’ funds on a company balance sheet.

ARR (Accounting Rate of Return) Method The ARR method uses the following formula to calculate the value of the shares:

employed capitalon return Required

profits future Estimated = Value

The required rate of return used is the accounting rate of return. You must be careful not to confuse this method with the P/E method below which uses the market rate of return.

If you use this method to value a takeover bid, it may be necessary to adjust the estimated future profit figures for expected changes resulting from the takeover, such as economies of scale achieved due to rationalisation.

Example

Smith plc is considering acquiring Jones plc whose current post-tax earnings are £5m. The board of Smith plc feel that they could reduce operating costs significantly in Jones plc, which will increase earnings by £0.5m. The increased size of the firm means that its current loans can be renegotiated at a lower level of interest, saving £0.25m per annum and the level of remuneration paid to directors will be cut by £0.15m. If the post-tax accounting rate of return of Smith plc is 20%, calculate the value of Jones plc.

Answer

First let us calculate the estimated future profits:

Current earnings £5.00madd: Operating savings £0.50m Savings in interest costs £0.25m Savings in directors’ remuneration £0.15m

Estimated future profits £5.90m

Valuation = employed capitalon return of rate Required

profits future Estimated

= 20%

£5.9m = £29.5m

The ARR is felt to represent the maximum value of a company – it being the value of the company after any post-acquisition reorganisation and efficiency drives have occurred. As with all the other methods it should not be used in isolation.

P/E Method This method calculates the value of a company’s shares by using the following formula:

Market value per share = EPS × P/E ratio

Page 58: libvolume5.xyzlibvolume5.xyz/law/ballb/3rdyear/semester5/corporatelaw/corporate... · ABE Advanced Diploma in Business Administration Study Manual CORPORATE FINANCE Contents Study

50 Company Performance, Valuation and Failure

© Licensed to ABE

This method makes use of a company’s level of earnings to calculate its value; the EPS used can either be an historical one, an average of past figures, or a prediction of a future figure. The latter is the best but care must be taken when using forecasts, especially with the figures used for growth in earnings. Similarly an appropriate P/E ratio should be used. The P/E figure used depends upon:

(a) How secure the earnings of the company are – the more secure the earnings, the higher the P/E ratio. Companies with high gearing levels tend to have lower P/E ratios reflecting greater financial risk.

(b) Expectations of future profits – the higher the expected earnings, the higher the P/E ratio. Adjustments may be made for past profit trends and the reliability of the estimates. (Expectations of future profits can be calculated using the discounted cash flow techniques which we shall discuss later in the course.)

(c) Companies which are unquoted generally have a P/E of between 50% and 60% of a company which is quoted on the Stock Exchange and around approximately 70% of shares quoted on the AIM, reflecting their reduced marketability and smaller size. However, an unquoted company with earnings of £300,000 or more and growing at a regular rate may have a higher P/E ratio because it may be able to be quoted on the AIM.

(d) General financial and economic conditions.

(e) The industry or industries which the firm is in and the prospects of those sectors.

(f) Liquidity and asset backing, including the nature of assets – specialised assets with a restricted resale market may reduce the P/E ratio.

(g) The make-up of the shareholders and the financial status of any major shareholders.

(h) Companies dependent on one or two key individuals and their skills may have their P/E ratio lowered.

Sometimes the P/E ratio of a company being acquired may be increased to reflect the improvements the predator thinks they can introduce into the “victim” company, although often such improvements are not realised.

Example

Sinbad plc is considering acquiring Flower Ltd. Sinbad plc’s shares have been quoted recently at an average of £6.40 and the recently published EPS of the company is 40p. Flower Ltd has 100,000 shares and a current EPS of 50p. Suggest an offer price for Flower Ltd.

Answer

First we have to decide a reasonable P/E ratio. The P/E ratio for Sinbad plc is 640/40 = 16. Assuming Flower Ltd is in the same industry its P/E ratio can be based on Sinbad plc’s P/E ratio, adjusted for the fact that it is not quoted, its growth prospects, and riskiness of its earnings. (If Flower Ltd is in a different industry then a typical P/E ratio for that industry could be used as a basis for the calculations.)

Using Sinbad plc a P/E ratio for Flower Ltd can be estimated as, for example, 16 × 50% = 8. A value for the shares can then be calculated as 8 × 50p = £4. This price would be the basis for negotiations on the value of the company.

Net Asset Method The premise that the value of a class of a company’s shares is equal to the net tangible assets of the company attributable to those shares is the basis of this method of calculating share values. Intangible

Page 59: libvolume5.xyzlibvolume5.xyz/law/ballb/3rdyear/semester5/corporatelaw/corporate... · ABE Advanced Diploma in Business Administration Study Manual CORPORATE FINANCE Contents Study

Company Performance, Valuation and Failure 51

© Licensed to ABE

assets are only included in the calculation if they have a recognisable market value, e.g. a copyright. To calculate the value of a share we simply divide the value of the assets attributable to a class of shares by the number of shares in the class.

Whilst this may seem to be an easy method in principle, in practice it can be quite difficult, the problems arising from arriving at a value for net assets. The problems include the following:

� Are the assets to be valued on a going concern or break-up basis?

� Are any assets covered by prior charges?

� How can the assets be valued – is a professional valuation required?

� What are the costs of sale – redundancy, taxation charges on disposal?

� Have all the liabilities been identified and correctly valued, including contingent liabilities?

If you are given the information to do so in an exam question on valuation, always calculate the net assets per share. There are two reasons for this:

(a) The value shows the amount a shareholder could expect to receive if the company went into liquidation. A potential shareholder could compare the asking price for the shares with the net assets per share value to calculate the maximum possible loss if the company fails to provide the promised dividends and earnings figures.

(b) An adjustment may be required in a scheme of merger to the value of the companies’ shares to reflect differences in asset-backing – shares with higher net assets per share figures could be expected to gain a higher price.

Unless otherwise told in an exam situation you should use the balance sheet figures provided, adjusted for intangible assets, to calculate the share values. However, you should list any concerns that you have along the above lines regarding the figures given.

Super-profits Method This method applies a “fair return” to the net tangible assets of the company and compares this figure with expected profits. The difference between the two is the firm’s goodwill. The purchase price is taken to be the value of the net tangible assets plus a fixed number of years’ goodwill or “super-profit”. Unfortunately both the rate of return and the number of years’ super-profits purchased are arbitrary figures with no theoretical underpinning. The method is not used so frequently in current practice, perhaps because of these problems.

Example

L Ltd wishes to acquire N Ltd. N has present earnings of £15,000 and net tangible assets of £60,000. The fair return figure decided upon by L Ltd’s board for N Ltd is 15%, and they decide to offer a price based on the super-profits method of valuation taking goodwill at two years’ super-profits. What price do they offer N’s shareholders?

Answer

First calculate super-profits and goodwill: £ Actual profits 15,000less: Fair return on net tangible assets (15% × £60,000) 9,000

Super-profits 6,000

Page 60: libvolume5.xyzlibvolume5.xyz/law/ballb/3rdyear/semester5/corporatelaw/corporate... · ABE Advanced Diploma in Business Administration Study Manual CORPORATE FINANCE Contents Study

52 Company Performance, Valuation and Failure

© Licensed to ABE

Therefore, Goodwill (2 × £6,000) = £12,000

Value of N Ltd = Net tangible assets + Goodwill

= £60,000 + £12,000 = £72,000

Dividend Yield Method The model used under this method varies with the assumptions used. The simplest model assumes that dividends will remain at a constant level in the future. The value of a company’s shares can be calculated using the formula:

Market value = % shares on the yield)(or return Expected

pencein Dividend

Example

Tinkeywinkey Ltd’s shareholders expect a dividend yield of 12% and have been told that dividends per share for the foreseeable future will be 20p. Calculate the value of Tinkeywinkey’s shares if they have 100,000 in circulation.

Answer

Using the above formula to calculate the value of one share:

Value = 166.67p.12%20

return Expectedpencein Dividend

==

The value of all 100,000 shares = value of one share × number of shares in issue so the value of the company = 166.67p × 100,000 = £166,670.

In other words, a shareholder in Tinkeywinkey who accepted a yield of 12% on an investment of £1.67, would be prepared to pay £1.67 for a share which paid him a dividend of 20p, or 12% on a nominal value of £1.

However, as we will see in a later study unit, shareholders prefer a constant growth in their dividends. In order to reflect this in valuing the company using a dividend method we have to predict future growth in dividends – which generally reflects predicted changes in a company’s earnings. When the expected growth figure has been determined we can calculate the value of the company’s shares using the Dividend Growth Model or Gordon’s Model of Dividend Growth.

This model states:

Po = g) r(

g) + (1do

where: Po = the current ex dividend market price

do = the current dividend

g = the expected annual growth in dividends

r = the shareholder’s expected return on the shares

The expression do(1 + g) represents the expected dividend in the next year.

Example

Poh Ltd is expecting to pay a dividend of 20p this year, increasing at a rate of 5% per annum. If its shareholders have a required return of 15%, calculate the current market price.

Page 61: libvolume5.xyzlibvolume5.xyz/law/ballb/3rdyear/semester5/corporatelaw/corporate... · ABE Advanced Diploma in Business Administration Study Manual CORPORATE FINANCE Contents Study

Company Performance, Valuation and Failure 53

© Licensed to ABE

Answer

Using the formula:

Po = g) r(

g) + (1do

= 0.05)(0.150.05)20(1

−+ = 210p

The dividend yield method is often used when valuing small shareholdings in unquoted companies. The reasoning behind the model is that such shareholders, being unable to influence a company’s earnings to any extent, will only be really interested in the dividends they receive from holding their shares. This method assumes that a share price is equal to the value of all the dividends it will attract during the time it is held, plus the amount received when it is sold (the sale price will reflect future dividends expected at the point of sale).

Amounts of cash received in the future are worth less than cash received today, so we must discount the future values to compensate and express them in terms of their equivalent value today. The discount rate used is the cost of the capital provided (which is the yield the investor expects to receive from his investment in the company). (We will cover this topic in much greater detail later in the course.)

This discounting can be expressed as:

Po = 33

33

221

)r1(P

)r1(D

)r1(D

)r1(D

++

++

++

+

where D = dividend (i.e. D1 is dividend in next year)

To calculate D1, D2, D3 , etc.:

D1 = do (1 + g)2

D2 = do (1 + g)3

D3 = do (1 + g)4

This model can be expanded to allow for potential growth in the dividend rate, and can be simplified to the dividend growth model shown above:

Po = g) r(

g) + (1do

Example

Dipsey Ltd, whose shareholders require a return of 20%, expects to pay no dividends for the following three years, but then expects to be able to pay a dividend of 10p per share for the foreseeable future. What is the value of its shares?

Answer

There is no return in the first three years so the price is:

0 + 0 + 0 + 4(1.20)10p + 5(1.20)

10p + .......

Because the cash flows continue into the foreseeable future this will be the same as:

0.20

p10 at time t3

Page 62: libvolume5.xyzlibvolume5.xyz/law/ballb/3rdyear/semester5/corporatelaw/corporate... · ABE Advanced Diploma in Business Administration Study Manual CORPORATE FINANCE Contents Study

54 Company Performance, Valuation and Failure

© Licensed to ABE

The present value of £1 a year forever at r% growth is r1 .

Therefore the price today = (0.20)p10 × 3(1.20)

1 = (1.20)50p

3 = 28.94p (say 29p)

Note that growth will usually be expressed as a percentage.

Discounted Future Profits This method is sometimes used when a company intends to purchase another’s assets and invest in improvements in order to increase future profits. It is best illustrated using an example.

Example

Bear plc is proposing to acquire Lion plc who is currently just breaking even. Bear feels that the investments it plans to make should lead to the following after-tax figures (ignoring any price paid) for Lion:

Year Earnings £000 1 85 2 88 3 92 4 96 5 96

Bear wishes to recover its investment within five years. If the after-tax cost of capital is 12.5%, what is the maximum price Bear should be prepared to pay?

Answer

The maximum price is the one where the discounted future earnings exactly equal the purchase price paid.

Year Earnings Discount Factor Present Value (Earnings × Discount Factor)

£000 £ 1 85 0.893 75,905 2 88 0.797 70,136 3 92 0.712 65,504 4 96 0.636 61,056 5 96 0.567 54,432

Σ Present Value 327,033

Therefore the maximum purchase price would be £327,033. (Don’t worry if you do not understand the discount factors at this stage; they will be fully explained later in the course.)

Page 63: libvolume5.xyzlibvolume5.xyz/law/ballb/3rdyear/semester5/corporatelaw/corporate... · ABE Advanced Diploma in Business Administration Study Manual CORPORATE FINANCE Contents Study

Company Performance, Valuation and Failure 55

© Licensed to ABE

The Berliner Method This method is calculated by using the average of share prices obtained using the net assets method and the earnings methods (see above).

This method is also known as the free cash flow approach. The method may be difficult to adopt in practice as it needs forecasts of working capital (see later in the course) and taxation to ensure that estimates of future cash flows and their timings are accurate.

Note re CAPM The Capital Asset Pricing Model is a further method of valuing shares. It is used especially to determine the required yield on equity when the shares are being priced before a Stock Market listing. We shall cover this topic in a later study unit, but we mention it here to remind you to include it in your revision of this stage.

E. PREDICTING COMPANY FAILURE

We have seen that ratio analysis is used to assess several factors of the firm’s performance including the financial stability, the return received from investment and the efficiency and effectiveness of management. We have also seen that inter-firm comparison can be a useful way of identifying the strengths and weaknesses of different companies, and as a tool to increase efficiency. It is also useful in predicting and identifying corporate failure.

A business that is highly dependent on one major customer or one major supplier, must monitor that company’s activities in order to detect any early warning signs of potential collapse which leave those who deal with it with potentially serious problems. A further reason for monitoring the progress of companies is when the managers of the firm are seeking a takeover opportunity, at which time they may be reviewing certain businesses to see which could be the most suitable targets for acquisition.

The problem of corporate collapse is a serious one; empirical evidence shows that for every company which is in receivership there are three or four others with serious cash constraints. Many such businesses are at risk of second-round failure, which we discussed in the previous unit.

Avoiding the pitfalls which the managers of such firms are likely to face will require, among other things, careful control of debtor and creditor relationships. The quality of credit control in the firm may, in these circumstances, mean the difference between survival and disaster, by ensuring that the firm always has sufficient funds to pay its obligations thus avoiding liquidation. Time spent in credit control should be regarded as equally important as time spent developing new business.

Company Information Services Besides specialists in the field of inter-company comparison, there are several organisations that provide company information services. They include:

(a) Performance Analysis Services

Performance Analysis Services was formed to collect and collate results of some 800 of Britain’s largest, listed, industrial companies. From data collected, the company has developed ways of predicting firms that appear to be most at risk of receivership.

(b) Dun and Bradstreet

Dun and Bradstreet are suppliers of financial information and related services and they, too, have developed an early warning system on possible company failures. The service is based on Stubbs Gazette, which is used as a key source of information, providing up-to-date information

Page 64: libvolume5.xyzlibvolume5.xyz/law/ballb/3rdyear/semester5/corporatelaw/corporate... · ABE Advanced Diploma in Business Administration Study Manual CORPORATE FINANCE Contents Study

56 Company Performance, Valuation and Failure

© Licensed to ABE

on compulsory winding-up orders, voluntary liquidations, court judgments, mortgages and charges.

(c) Institute of Chartered Accountants in England and Wales (ICAEW)

ICAEW has produced an operational guideline on the continuity of business, which identified a number of factors which put continuity into question:

� Loss of key management and staff

� Significantly higher stock levels, without the apparent source of finance to pay for them

� Regular work stoppages and labour disputes

� Dependence on a single product or project

� Dependence on a single supplier, or a large customer

� Outstanding legal proceedings

� Political risks

� Technical obsolescence

� Loss of a major franchise or patent

� History of poor performance within the industry.

Clearly it is the trend towards the regular occurrence of several factors which is important, not a single “one off” event in isolation. If there should be a build up, then it could lead to a situation of “crisis management”, i.e. where managers are dealing with one serious problem after another rather than focusing on planning for the future needs of the business.

In extreme circumstances, managers may have no option but to slim down the entity resulting in lower, but more manageable, operating capacity. However, not every manager will be willing to sacrifice the prestige of running a business of a certain size. Many problems stem from the failure of managers to react in time (if at all).

The Z Score The analysis of financial ratios is generally concerned with the assessment of the effectiveness and efficiency with which a company deploys its resources. This is in itself an attempt to measure the overall financial stability of the organisation – you will remember that this is a principal duty of the financial manager.

An American, E.I. Altman, researched the use of ratio analysis in order to examine ratio correlation and business failure. In his research he looked simultaneously at several ratios to attempt to predict business failure. He used some 22 accounting and non-accounting ratios which he applied to a selection of failed and continuing US companies. From this initial analysis, he determined that there were five key indicators of impending failure, and he used them to formulate the Z Score.

Using the Z score, Altman predicted that firms with a score below a certain level were much more likely to fail than those with higher scores, and he identified “middle ground” in which the outcome of the company’s future was uncertain.

Page 65: libvolume5.xyzlibvolume5.xyz/law/ballb/3rdyear/semester5/corporatelaw/corporate... · ABE Advanced Diploma in Business Administration Study Manual CORPORATE FINANCE Contents Study

Company Performance, Valuation and Failure 57

© Licensed to ABE

Altman’s model emerged in 1986 as:

Z = 1.2X1 + 1.4X2 + 3.3X3 + 0.6X4 + 1.0X5

where: X1 = Working capital/total assets

X2 = Earnings/total assets

X3 = Earnings before tax and interest/total assets

X4 = Market value of equity/book value of total debt

X5 = Sales/total assets

A Z score of 2.7 or more indicates non-failure, whereas a score of 1.8 or less may imply impending failure. You will not be expected to calculate Z scores in your examination.

Altman’s score has received criticism. The size of the sample was considered small and only related to US companies. Other research since Altman’s has produced differing results despite being based on similar principles.

It is, however, important to recognise that Altman’s work, and his scoring method, represent an important theoretical approach to the way of looking at business failure. The current view on linking financial ratios and business failure generally suggests that where a firm has failed, the financial ratios show significant signs of decline, retrospectively. Prior to the deterioration and again in retrospect, the financial ratios of a failed firm can indicate to some degree why the failure occurs.

Although there has been no general acceptance of a single Z Score model, several variations of the original have been used with a certain amount of success. Many banks use a form of Z Scoring to monitor the progress of their corporate customers, but the models are computer-based and considerably more sophisticated than Altman’s original.

Other Models (a) Argenti

This model, also known as Argenti’s failure model, is based on the calculation of a company’s scores in the areas of defects of the company, management mistakes and symptoms of failure.

Company defects include a passive board, an autocratic Chief Executive and weak budgetary control. Managerial mistakes include high levels of gearing and the failure of a large (in relation to the company) project. Overtrading (the business expanding too quickly for its level of cash funding and thus having insufficient liquid funds to pay creditors) is also noted as a major managerial mistake. Symptoms of decline include deteriorating ratios, quality and staff morale and the use of window dressing. Each of these three areas has a danger mark.

Argenti based his model on past company data and, as with the other models, it is difficult to assess its predictive ability.

(b) Taffler

Taffler has developed a model to predict business failure which is based on a series of ratios:

� Sales/total assets

� The current ratio

� The reciprocal of the current ratio

� Earnings before tax/current liabilities.

There is at present insufficient evidence to determine this model’s predictive qualities.

Page 66: libvolume5.xyzlibvolume5.xyz/law/ballb/3rdyear/semester5/corporatelaw/corporate... · ABE Advanced Diploma in Business Administration Study Manual CORPORATE FINANCE Contents Study

58 Company Performance, Valuation and Failure

© Licensed to ABE

(c) Beaver

Empirical research conducted by Beaver found that the best prediction of corporate failure is a low cash flow/borrowings ratio, and the poorest measure for forecasting failure is the current ratio (current assets/current liabilities).

Other Indicators One quick method of predicting company failure is the use of the liquidity ratios – proponents of this method state that a current ratio of below 2 : 1 and an acid (quick) ratio of less than 1 : 1 means that the firm is illiquid and liable to fail. However, as we discussed earlier, these ratios need to be viewed in the light of the nature of the business and the overall position of the firm. Moreover, empirical evidence (see Beaver above) has found that these ratios, and trends in them, give little or no indication of eventual business failure. A worsening liquidity position could, however, indicate that the firm is having problems and its financial and operational performance should be carefully scrutinised.

Other accounting information which may provide indications of financial difficulties include important post balance sheet events, large contingent liabilities and large increases in intangible assets.

Non-accounting information contained in the annual report may also provide indications of company problems, as may changes in the composition of the board if the more able directors have left. In addition, the Chairman’s Report, although generally optimistic (and not audited), may discuss current and future unsolved problems.

External events such as legislation, political changes at home and abroad, competitors’ actions, and changes in economic variations (e.g. interest and exchange rates) may also give forewarning of potential problems for a firm. In addition, newspapers and journals may report on the financial and other difficulties a firm is or may be experiencing.

Problems with Prediction Models There are several problems with the use of the above models:

� Many assumptions have to be made in interpreting information, including the methods of accounting which have been used. Indeed, any limitations in the accounting data used will also affect the models.

� It is difficult to value the equity in a private company, which may make the use of the models for private company analysis difficult and extremely subjective.

� The information they use reflects the past and thus is out of date. The problem is exacerbated by the delay in the publication of company accounts.

� Prediction models take little or no account of economic conditions occurring when they are used, and the effect that economic variables may have on the figures used in their models.

� It is very difficult to define corporate failure because companies which would otherwise have been liquidated can be “rescued” or taken over. Similarly, businesses may close for reasons other than failure, e.g. a private company may cease to operate because the owner-manager wishes to retire.

� Companies may manipulate the measures used in the models in order to prevent predictions of failure.

Page 67: libvolume5.xyzlibvolume5.xyz/law/ballb/3rdyear/semester5/corporatelaw/corporate... · ABE Advanced Diploma in Business Administration Study Manual CORPORATE FINANCE Contents Study

Company Performance, Valuation and Failure 59

© Licensed to ABE

F. CAPITAL RECONSTRUCTION SCHEMES

It is not unusual for companies to be liquidated, the principal reasons for this being that:

� The company does not have enough cash to pay its liabilities and its creditors apply to the court for it to be wound up – this may happen even if the company is profitable.

� A company may not be profitable enough to continue in business (or it may be acquired by another company).

However, a company may be able to prevent liquidation if it can devise a capital reconstruction scheme attracting new capital and/or persuade its creditors to “change” their loan to the business into company securities, thus allowing the business to continue trading. The business must be able to show that its problems are only temporary and that the scheme will allow it to become profitable and/or improve its cash flow situation. Negotiating such schemes is often difficult, but a failure to do so may lead to the business being liquidated.

We should distinguish between the term reconstruction, which is used to refer to major changes to capital as a prelude to a merger, and capital restructuring. Capital restructuring (or reorganisation) is the term usually used when there is only one company concerned and the rights of its members, and sometimes of its creditors, are varied by an alteration of its capital structure but with the existing company continuing in business.

Reasons for Capital Reconstruction There are a number of reasons why a restructuring may be necessary:

� The company may have become too highly geared and a solution may be to issue equity in place of debt capital.

� The existing capital structure may have become over-complicated with perhaps too many classes of shareholder with different rights to each class. They can be consolidated into one or two classes, but care must be taken to ensure that the relative voting strength remains in the same proportion.

� Capital with prior rights may carry a high fixed dividend which then gives a misleading impression of the company, or preference shareholders may have control of the company. In such cases the structure should be reorganised into a more convenient nature.

� The company may decide to replace preference shares with debentures in order to reduce the corporation tax on the company’s shares.

Principles of Capital Reconstruction There are a number of points of principle in the design of a scheme of reconstruction.

(a) Firstly, if the company is having problems it is likely to require more finance which may come from either existing shareholders or a bank, generally in the form of equity finance, although some may be in the form of loan stock. This new equity may replace existing share capital, or may have a different nominal value to it. Those providing such finance will require profit and cash flow forecasts to show how the business can be turned round and provide a good return for their additional money. In such cases it is wise to maintain the income position of a particular class under the scheme as far as it is possible. Often more income will be offered as an incentive to the holders of a particular security to agree to the capital restructuring.

Page 68: libvolume5.xyzlibvolume5.xyz/law/ballb/3rdyear/semester5/corporatelaw/corporate... · ABE Advanced Diploma in Business Administration Study Manual CORPORATE FINANCE Contents Study

60 Company Performance, Valuation and Failure

© Licensed to ABE

(b) In addition, a reconstruction scheme must treat all parties fairly, and not favour one group over others. The outcome of the scheme should be more beneficial to creditors than if the company went into liquidation, or they may press for the winding-up of the company. Often this is avoided by including in the scheme provision for paying off the company’s debts in full. The increased benefits to creditors (and investors) from the reconstruction scheme can be shown by comparing the liquidation value of the firm with the estimated future results arising from the reconstruction scheme.

(c) The company must also take account of fixed charges (e.g. a mortgage on a factory) on assets of the business used as security for loans. These charges mean that the creditor is “secured” and thus entitled to first claim in the process of liquidation (if there are insufficient funds the creditor becomes unsecured for the balance of the loan unpaid). The charges mean that such creditors may have less incentive to keep the business afloat. Second charges are when the lender has the prior claim on the surplus from the sale of a secured asset after the prior claim has been met.

(d) In addition, the company may have to consider the existence of floating charges over the assets (generally the current assets of the business which are continually turned over during trading) that crystallise on liquidation thus providing the charge owner with prior call on the funds realised from these assets. Similarly to above, such creditors may have less incentive to keep a business afloat, unless it can be shown that it would be financially beneficial to them to do so.

You may also find in practice that certain creditors can have a fixed charge on an asset, with a floating charge over a group of assets for the balance of their loan – this is known as a fixed and floater.

Page 69: libvolume5.xyzlibvolume5.xyz/law/ballb/3rdyear/semester5/corporatelaw/corporate... · ABE Advanced Diploma in Business Administration Study Manual CORPORATE FINANCE Contents Study

61

© Licensed to ABE

Study Unit 3

Acquisitions and Mergers

Contents Page

Introduction 63

A. Company Growth 63 Strategies for Growth 63 Economic Justification for Growth via Acquisition 65 The Development of “Mega-Mergers” 66

B. The Regulation of Takeovers 67 Extent of Coverage of the Code 67 General Principles 67 Rules of the Code 68 Companies Act 1985 70 Competition Commission 71 European Union 71

C. The Acquisition/Merger Process 71 Tactics for Acquisitions and Mergers 71 Defences Against an Unwelcome Takeover Bid 72 Consideration 73

D. Measuring the Success and Failure of Mergers and Takeovers 74 Company Performance 74 Shareholder Wealth and Power 75 Employment 76 Example 76

E. Disinvestment 78 Management Buy-Outs 78 Buy-Ins 80

(Continued over)

Page 70: libvolume5.xyzlibvolume5.xyz/law/ballb/3rdyear/semester5/corporatelaw/corporate... · ABE Advanced Diploma in Business Administration Study Manual CORPORATE FINANCE Contents Study

62 Acquisitions and Mergers

© Licensed to ABE

Spin-Offs 81 Sell-Offs 81 Demergers 81 Going Private 81

Answer to Practice Question 82

Page 71: libvolume5.xyzlibvolume5.xyz/law/ballb/3rdyear/semester5/corporatelaw/corporate... · ABE Advanced Diploma in Business Administration Study Manual CORPORATE FINANCE Contents Study

Acquisitions and Mergers 63

© Licensed to ABE

INTRODUCTION

In this study unit we will start to consider corporate structure and corporate growth. The first part of the unit investigates what happens when a company goes beyond being an individual, single entity and, by means of acquisition or merger, becomes a multiple concern or a group of companies.

When a firm is considering expanding whether internally (by expansion, integration or diversification) or externally (via a merger or acquisition) it must ensure that growth is economically justified, carefully planned and structured. Management must consider the impact on the company, its (and if appropriate the target’s) shareholders and employees, the environment it operates in, and the Stock Market’s views. In addition, the current regulatory framework should be borne in mind and it is very important that the firm allows for a period of transition for success to be achieved.

The converse of mergers and acquisitions is where a firm may decide to disinvest part of itself, and this is the subject of the second part of the unit.

Reasons for disinvestment include removing a part of the business which does not fit correctly into a group’s portfolio or its core business, selling an unprofitable subsidiary, or selling a profitable subsidiary to finance expansion elsewhere. There may also be a desire by the owners of a private concern to arrange their affairs to the best advantage to their heirs in the light of favourable political or tax regimes. Disinvestment includes demergers, management buy-outs, management buy-ins, spin-offs and sell-offs.

In common with the expansionary policies discussed in the first part of the unit, it is essential that such activity is carefully planned and monitored, and that the needs of the various parties involved (especially the shareholders) are considered.

We saw earlier in the course that there are several stakeholders in an organisation, and the majority of them will be concerned with its stability and long-term viability. In order to help them assess this several models of corporate failure have been developed using financial and other ratios, often based on past empirical evidence. Although these models can provide some useful guidance, as yet there is no one method that is capable of predicting corporate failure in advance.

Acquisitions and mergers are a fairly frequent occurrence, and you should read the financial press for details of any currently taking place – providing up-to-date examples is always useful to your arguments.

A. COMPANY GROWTH

Strategies for Growth The three main strategies a firm may adopt for growth are expansion, integration and diversification.

(a) Expansion

This is the growth of existing, or development of new, markets or products, which can be in response to changes in technology, customer taste or simply to exploit an opportunity in the market.

(b) Integration

Integration, of which there are two forms – horizontal and vertical – is a form of expansion.

� Horizontal integration is when a firm adds either new markets for its existing products, or introduces new products to its current markets. It may be done so the firm can benefit

Page 72: libvolume5.xyzlibvolume5.xyz/law/ballb/3rdyear/semester5/corporatelaw/corporate... · ABE Advanced Diploma in Business Administration Study Manual CORPORATE FINANCE Contents Study

64 Acquisitions and Mergers

© Licensed to ABE

from economies of scale, although this may cause difficulties for the firm if there are problems with the markets or the products.

� Vertical integration is expansion of the firm along the supply chain and can be either backward (supply of components or raw materials) or forward (being one step closer to the end customer). It allows a firm to have greater control over the industry including quality, quantity, price and share of the profits, although it becomes more prone to falls in demand within the industry as a whole.

(c) Diversification

This policy is also a form of expansion – indeed, integration is sometimes referred to as related diversification.

The diversification we are now going to consider is referred to as unrelated diversification and comprises concentric and conglomerate diversification.

� Concentric diversification is the development of products which are synergetic with current products.

� Conglomerate diversification is the development of products with no marketing, technology or product synergy with the business’s current products. The firm, however, expects to obtain management synergies from the conglomeration.

There are several other potential advantages of conglomerate diversification. Can you think what they might be?

Advantages include the following points:

� The firm can move quickly into high profit areas by acquiring a firm in that market.

� The resultant larger firm may have better access to funds.

� A larger firm may have greater influence in the market and in the political environment.

� A spreading of risk may occur from operating in different markets.

� Profitability may improve as a result of the diversification.

� The new firm may be more flexible, and the acquisition of new firms may allow withdrawal from existing markets.

� There may be synergies to be obtained from the merger utilising a surplus in one firm to satisfy a deficit in another (e.g. cash).

� Unlike the takeover of a similar firm which may lead to a referral to the Competition Commission, conglomerate (and concentric) diversification are unlikely to be regulated by the state.

However, there are several problems associated with conglomerate diversification.

� Profits in one part of the business may be used to help others making losses, which may lead to the failure of the whole organisation.

� Empirical evidence has shown that EPS are diluted when companies with high P/E ratios are acquired and that risk may be increased rather than reduced.

� Empirical evidence has also shown that management synergies are often not obtained in practice.

Page 73: libvolume5.xyzlibvolume5.xyz/law/ballb/3rdyear/semester5/corporatelaw/corporate... · ABE Advanced Diploma in Business Administration Study Manual CORPORATE FINANCE Contents Study

Acquisitions and Mergers 65

© Licensed to ABE

Economic Justification for Growth via Acquisition Internal growth is one method of growth; another method is by acquiring or merging with another company (known as external growth). The purchase of a controlling interest by one company in another is known as an acquisition or takeover. A merger or amalgamation is the combination of two separate companies into one single entity. It is often difficult to determine in practice whether a takeover or merger has occurred, especially when there is a difference in size between the organisations. Whilst many such joinings are called mergers, and the two terms are often used interchangeably, in reality there are very few true mergers, and those which are tend to occur in industries with histories of poor growth and returns.

Note that you should take care not to confuse acquisitions and mergers with joint ventures. In a joint venture the managers of two or more businesses decide to establish a new company under their common ownership and management for the purposes of exploiting an opportunity which neither of them has the resources to exploit individually.

There are several potential reasons for an acquisition or merger but it is important there should be a resulting synergy with, and that it helps in achieving the overall strategic objectives of, the firm.

A firm must also consider the cost and value of the merger or acquisition and the relationship between the two.

Common reasons for acquisitions and mergers are:

� To reduce competition, although this may be prevented in the UK by the Competition Commission.

� To purchase a new product range or move into a new market.

� To obtain tax advantages (discussed in greater detail in the b module).

� To spread risk by diversification into new markets and/or products, which should lead to more secure earnings.

� To obtain assets that are undervalued or can be sold off (“asset stripping”), cash (if the victim company is very liquid) and/or access to finance, expert staff, management expertise, technology, suppliers or production facilities. Whilst they might all be acquired internally they can be acquired a lot quicker by a takeover.

� To achieve economies of scale in production, purchasing or marketing.

� To act as a defence against being acquired itself, either by purchasing the predator company or by making itself bigger and thus harder to be taken over.

� Growth can be achieved via a share exchange rather than having to acquire the cash that an internal policy of growth would require. The policy may also be less expensive than internal growth if a premium has to be paid to attract assets and staff.

There are also major problems with acquisitions and mergers:

� Economies of scale, especially in head office functions, often do not materialise, or indeed become diseconomies of scale. There can also be transportation problems when the acquired sites are geographically separate from existing sites.

� There can be problems integrating the different work forces and there may be large-scale redundancies. Similarly problems in integrating new products, markets, customers, suppliers, management and systems can lead to management overload. It is sometimes known as “indigestion”.

Page 74: libvolume5.xyzlibvolume5.xyz/law/ballb/3rdyear/semester5/corporatelaw/corporate... · ABE Advanced Diploma in Business Administration Study Manual CORPORATE FINANCE Contents Study

66 Acquisitions and Mergers

© Licensed to ABE

� There may be public relations problems with customers and the general public boycotting the firm because they disagree with the takeover. Directors of the victim company may also do their best to impede the takeover.

� There may be regulatory intervention (see later).

� The cost of the acquisition may be too high.

� Shareholders of the target company may adopt defensive tactics to oppose the bid, and there may be problems in unifying dividend, reporting and other policies affecting the shareholders of both companies.

In addition to the reasons for, and potential problems with, a mooted acquisition or merger the firm has to consider the views of its shareholders, the shareholders of the target company and of the market. The company must also determine how it will pay for the company – will it be with cash, share exchange, via loan stock or some combination? Unless it is purely financed by cash the target company’s shareholders will have an interest in the new merged firm.

Reasons why a company’s shareholders and the market may not approve of a takeover or merger include the following:

(a) There may be social or moral disapproval of the target company, e.g. it may produce arms or deal with a country with an unpopular regime.

(b) The merger may result in a fall in the EPS or net asset backing per share.

(c) The merger may result in an increase in risk due to the nature of the target’s industry or financial profile.

A company may not need shareholder approval, but a lack of shareholder and market backing can lead to a fall in the market price of the company’s shares, which is not achieving the company’s primary objective of maximising shareholder wealth. Moreover, when a takeover is to be paid for by the issuing of a large number of new shares in the predator company, shareholder approval at an AGM or EGM will be required by Stock Market regulations.

The views of the target company’s shareholders are considered below.

The Development of “Mega-Mergers” You may be aware from your reading of the financial press that in recent years there have been several very large or “mega” mergers. The reasons for these mega mergers include the following:

� Globalisation and deregulation of financial markets makes it easier to arrange finance for such deals.

� Buoyant equity markets allow share exchange schemes to be successful and cash to be raised via rights issues.

� Investors are demanding growth in earnings and a merger is often the cheapest and most expedient way of doing this.

� The globalisation of operations and opportunities to achieve operating economies in some fields (e.g. oil).

� Market trends (e.g. in the car industry) may mean that the small company can no longer compete successfully – a merger may be a defence against being taken over.

Page 75: libvolume5.xyzlibvolume5.xyz/law/ballb/3rdyear/semester5/corporatelaw/corporate... · ABE Advanced Diploma in Business Administration Study Manual CORPORATE FINANCE Contents Study

Acquisitions and Mergers 67

© Licensed to ABE

� Mergers may lead to economies of scale in overhead costs. For example, the merger of Glaxo and Wellcome was undertaken with the aim (amongst others) of achieving economies of scale in research, development, testing and marketing of drugs.

B. THE REGULATION OF TAKEOVERS

The City Code administered by the Takeover Panel specifies the behaviour which companies are expected to adopt during a takeover or merger. The Code is issued by the Panel on Takeovers and Mergers (the Panel), and whilst it does not deal with the price to be offered in a merger or acquisition, both the Code and the Panel operate to see that shareholders receive fair and equal treatment during this process.

Extent of Coverage of the Code The Code applies to offers for all listed and unlisted public companies as well as, when appropriate, statutory and chartered companies considered by the Panel to be resident in the UK, the Channel Islands or the Isle of Man. It also applies to an offer in respect of a private company of the same residency, where at some time during the 10-year period prior to the announcement of the offer:

(a) Its equity capital has been listed on the Stock Exchange;

(b) Dealings in its shares have been advertised in a newspaper on a regular basis for a continuous period of at least six months; or

(c) It has filed a prospectus for the issue of equity share capital at Companies House.

The Code is concerned with takeovers and mergers for all companies defined above and includes partial offers and offers by a parent company wishing to acquire shares in its subsidiary. Generally, the Code excludes offers for non-voting, non-equity capital.

You should note that, unlike legislation, the Code is not enforceable in law. However, those who fail to conduct themselves in accordance with its rules may, by way of sanction, have the facilities of the securities markets withdrawn from them.

The Code is made up of a number of general principles, which are essentially statements of good commercial conduct, together with a set of 38 Rules supported by substantial notes. The Rules are not laid down in technical language, and they should therefore be interpreted by their underlying spirit and purpose rather than appertaining to some specific legal framework.

General Principles The ten general principles are listed below:

1. All shareholders of a specific class of an offeree company should be treated similarly by the offeror.

2. During the course of an offer, information provided by the offeror, the offeree or the advisors thereto must be made available to all shareholders. There is one exception which concerns information supplied in confidence by the offeree company to a prospective offeror (or vice versa).

3. The offeror should only announce an offer after careful consideration, and it, and its financial advisors, must have reason to believe that it can, and will, be able to implement the offer.

4. No information may be withheld from shareholders, who must be allowed sufficient time and advice to arrive at an informed decision.

Page 76: libvolume5.xyzlibvolume5.xyz/law/ballb/3rdyear/semester5/corporatelaw/corporate... · ABE Advanced Diploma in Business Administration Study Manual CORPORATE FINANCE Contents Study

68 Acquisitions and Mergers

© Licensed to ABE

5. The highest standards of care and accuracy must be maintained in the preparation of documents, information and advice supplied to shareholders.

6. All parties are obliged to try to prevent the creation of a false market in the shares of both the offeror and the offeree company, and must take care not to make statements which may mislead shareholders or the market.

7. The board of the offeree company may only take action to frustrate an offer (or an expected imminent offer) following consultation with the shareholders of the offeree company.

8. Rights of control must be properly exercised in good faith. Minority interests may not be suppressed.

9. Directors of both companies must disregard their personal interests and may only act in their capacity of directors in advising their shareholders.

10. Where control of a company is acquired or consolidated, a general offer to all other shareholders is usually required. The offeror must, prior to making the acquisition, ensure that he can and will continue to be able to implement such an obligation.

From time to time conflicts of interest may arise for the financial advisors involved. This may apply where material confidential information is available to them or where the advisor is a part of a multi-service organisation. Where the first situation arises, conflict may be removed by the advisor declining to act; in the second circumstances, a careful segregation of the business will be necessary to prevent conflict occurring within the rules – the latter is sometimes referred to as “building a Chinese wall”.

Rules of the Code Thirty-eight Rules are contained in the Code and they are supported by detailed notes. Whilst you do not have to memorise individual Rules, you should have a thorough understanding of their nature and the way they impact on the parties to a takeover. We will consider the Rules of the Code under the following headings:

� The approach, announcements and independent advice

� Dealings and restrictions on the acquisition of shares and rights over shares

� The mandatory offer and its terms

� Conduct during an offer

� Substantial acquisition of shares

(a) The Approach, Announcements and Independent Advice

An offer should be proposed to the board in the first instance. The identity of the offeror or, in an approach with a view to an offer being made, the potential offeror must be disclosed at the outset. The board must be satisfied that the offeror has the resources to implement the offer (Rule 1).

An announcement should be made as soon as sufficient details have been decided. Any announcement of a firm intention to make an offer must contain:

(i) The terms of the offer;

(ii) The identity of the offeror and details of any existing shareholding;

(iii) The conditions to which the offer is subject;

Page 77: libvolume5.xyzlibvolume5.xyz/law/ballb/3rdyear/semester5/corporatelaw/corporate... · ABE Advanced Diploma in Business Administration Study Manual CORPORATE FINANCE Contents Study

Acquisitions and Mergers 69

© Licensed to ABE

(iv) Details of any arrangements which may be an inducement either to deal, or not to deal, in the shares.

Promptly after the start of the offer period, the board of the offeree company must send a copy of the announcement to its shareholders and to the Panel.

Any person stating that he does not intend to formulate an offer for a company will normally be bound by the terms of that statement. All statements should be as clear and unambiguous as possible (Rule 2).

Rule 3 requires that the board of the offeree company must obtain competent independent advice on any offer and the substance of that advice must be communicated to the shareholders. This is especially important in cases such as management buy-outs which we shall look at later in the unit. The Panel consider that it is inappropriate for independent financial advice to be given by a person who is either in the same group as the financial advisor to the offeror or who has a substantial interest in either the offeror or the offeree.

(b) Dealings and Restrictions on the Acquisition of Shares and Rights over Shares

Whilst Rule 30 deals specifically with the criminal offence of insider dealing (the EC Directive on Insider Dealing was implemented through the Criminal Justice Act 1993), Rule 4.1 restricts dealing in the securities of the offeree company by any person other than the offeror, where such a person has access to confidential, price-sensitive information, from the time when there is reason to believe an offer is imminent to the time of its determination (or lapse). Additionally, dealing will not be permitted in the securities of the offeror company where the offer is price-sensitive in respect of the offeror’s securities.

Rule 4.2 restricts the sale of securities in the offeree company by the offeror during the period of the offer, unless the Panel has given its approval and at least 24 hours’ public notice has been given. After such consent and notice, the offeror may make no further purchases.

Rule 4.3 limits the opportunity for persons to contact private or small corporate shareholders with a view to seeking irrevocable commitments to accept (or to refrain from accepting) an offer, or a contemplated offer, without the prior approval of the Panel.

Generally, unless a person (including those acting in concert with him) holds less than 30%, or rights over less than 30%, of the voting shares in a company, he may not acquire a holding that would carry voting rights of more than 30%. Where a person similarly holds between 30% and 50% of shares, or rights to shares, of the voting kind within a company, he may not acquire more than a further 1% (2% before 3rd March 1993) of the voting rights in any 12-month period. Exempt from those described in this paragraph are those who make an offer for the company (Rule 5).

When an offer is contemplated and the offeror (or person acting in concert) acquires shares in the offeree in the three months prior to the offer, subsequent general offers must not be on less favourable terms without the consent of the Panel. If, while the offer is open, the offeror purchases shares at a higher price than the offer price, then the offer price must be increased to be not less than the highest price paid for the shares so acquired (Rule 6).

Immediate announcements may be required should the terms of the offer have to be amended under Rules 6, 9 or 11. Rule 7.1 requires immediate disclosure relating to the number of shares acquired and the price paid, if practical, as soon as an acquisition at a price higher than the offer price has been agreed.

Any dealings by the parties to a takeover or their associates must be disclosed daily by 12 noon on the business day following the transaction to the Stock Exchange, and it will then be made

Page 78: libvolume5.xyzlibvolume5.xyz/law/ballb/3rdyear/semester5/corporatelaw/corporate... · ABE Advanced Diploma in Business Administration Study Manual CORPORATE FINANCE Contents Study

70 Acquisitions and Mergers

© Licensed to ABE

available to the Panel and to the press. Additionally, disclosure (excluding the financial press) will be required where purchases or sales of relevant securities in the offeree or the offeror companies are made by associates for the account of non-discretionary clients, themselves not being associated. Intermediaries may be required to disclose the name(s) of their client(s) (Rule 8).

(c) The Mandatory Offer and its Terms

Various Rules lay down the requirements and mechanics of a formal offer, providing time limits in respect of acceptances, counter-offers, etc. The various options available to both the offeror and the offeree are also laid down and reflect the percentage of shareholders accepting or rejecting the offer. Whilst you do not have to remember detailed prescriptions, you should remember that they exist and must be adhered to by all parties concerned.

(d) Conduct During an Offer

The Rules lay down the requirements of a code relating to the conduct of the parties to an offer while it is progressing. They are summarised below:

(i) All shareholders must have an equality of information.

(ii) Advertisements must be cleared by the Panel before their publication.

(iii) Details of all documents and announcements must be lodged with the Panel.

(iv) Generally, no actions are to be taken that would mislead shareholders or the markets, including taking any action by the offeree that may frustrate the offer prior to a bid being under way.

(v) Transfers by the offeree must be promptly registered.

(vi) Special care must be exercised with all documents, and the terms of the bid must be covered carefully, including conflicting views, and so forth. Offer documents should always be available and on display.

(vii) Specific rules govern the way profit forecasts are stated and assets valued.

(viii) The offer document should normally be posted within 28 days of the announcement of a firm intention to make an offer. An offer must be open for at least 21 days after it is posted, and this period of time may be extended by further notice.

(e) Substantial Acquisition of Shares

The Rules regulate the speed at which a person, or persons acting in concert (a concert party), may increase shareholdings between 15% and 30% of the voting rights of the company. They also invoke the accelerated disclosure of acquisitions of shares or rights over shares relating to such holdings.

Companies Act 1985 Section 151 prevents financial assistance being given by a company for the purchase of its own shares. This is done in order to prevent the manipulation of share prices and ownership in several scenarios including takeover and merger discussions.

Under Sections 428-430 a company holding more than 90% of the shares in another company may compulsorily purchase the remainder on the same terms. The minority shareholders can also insist that their shares are purchased on these terms.

Page 79: libvolume5.xyzlibvolume5.xyz/law/ballb/3rdyear/semester5/corporatelaw/corporate... · ABE Advanced Diploma in Business Administration Study Manual CORPORATE FINANCE Contents Study

Acquisitions and Mergers 71

© Licensed to ABE

You should also note that the legislation dealing with insider dealing may also be invoked when considering a merger or takeover bid – it being illegal to act on unpublished information regarding such a bid (especially if with a view to financial gain).

Competition Commission The Monopolies and Mergers Act 1965 allows the Office of Fair Trading (OFT) to consider all acquisitions and mergers above a certain size, and refer them to the Competition Commission if it feels that they may be against the public interest. A referral normally takes place within 20 working days. The OFT can be informed in advance of a proposed bid in order to assess its reaction, and a proposal can be withdrawn if it is felt that the Commission will reject it. The Commission makes recommendations to the Department of Trade and Industry and/or the parties involved in the bid. The recommendations can include an outright acceptance or rejection of the bid, or acceptance on the basis of certain conditions, e.g. in the price levels of the new firm.

European Union A regulation introduced in 1990 gives the European Commission the power to block or authorise mergers with a world-wide turnover of over 5 billion ecu (approximately £3.5 billion). Mergers with EU-wide turnover of 250 million ecu (approximately £175 million) need to be agreed by the Commission. Reasons used to block mergers include incompatibility with the European Common Market.

The European Union’s 13th Company Law Directive dealing with takeover bids and procedures will have statutory power in EU member states when it is adopted. This will be a major change from the current UK approach of self-regulation.

You may find it interesting to note here that Britain has several more large industrial units resulting from past mergers than the rest of Europe.

C. THE ACQUISITION/MERGER PROCESS

This area is well known for its tactics both for acquisition and defence.

Tactics for Acquisitions and Mergers A common acquisition tactic is for the predator company to purchase shares in the target company in the market place quickly, to prevent the market interpreting its motives and increasing the share price – known as a dawn raid. It is illegal for a “concert party” acquisition of shares to take place (i.e. a number of connected or unconnected parties purchasing shares together).

When the predator has obtained 3% of the shares it must inform its target of its holding. The London Stock Exchange publishes market-makers’ holdings of 3% or more in UK-listed companies quoted either on the main Stock Market or the Alternative Investment Market (AIM).

When a company has acquired 30% it must make an offer to the remaining shareholders in the target company (we will discuss this in the section on the City Code later in this study unit). The company should then make an offer to the target’s board (either directly or through its merchant bank) in order to determine the board’s view of the bid. The price discussed should be below what the predator feels the company is worth but above its current share price. If the predator continues with the bid a formal offer document is then sent to shareholders with details of its offer price.

Page 80: libvolume5.xyzlibvolume5.xyz/law/ballb/3rdyear/semester5/corporatelaw/corporate... · ABE Advanced Diploma in Business Administration Study Manual CORPORATE FINANCE Contents Study

72 Acquisitions and Mergers

© Licensed to ABE

Defences Against an Unwelcome Takeover Bid The City Code requires directors to act in the best interests of the shareholders, employees and creditors and there may be circumstances when they consider that it is in the best interests of the above to contest the bid. The directors can contest an offer because they feel that the terms offered are too low, that there is no advantage to the merger, or because employees or founder members may be opposed to the bid. If the former is the case the predator may offer a higher price. The relative prices of the companies’ shares is a very important issue during a takeover or merger bid which uses shares as part of the consideration. A significant rise in the target’s shares or a fall in those of the predator can jeopardise the takeover by reducing the value of the consideration offered to the shareholders of the firm being taken over.

In order to defend against an unwelcome takeover bid, the directors have to plan and take action as early as possible. They should keep a careful watch on dealings in the company’s shares to spot whether an individual (or group of individuals) is building up a significant holding. They must also review the market price of their shares constantly in relation to their earnings and asset values, in order to determine whether the company is undervalued by the market and therefore prone to a takeover. In addition, directors should assess the company’s position within its industry as regards technology, size, etc., to see whether it is uncompetitive and likely to attract a takeover bid by a major player in the industry. A further tactic is to maintain contact with a range of stockbrokers, analysts and merchant bankers who are the most likely to hear of hints and rumours of any takeover strategies at an early stage.

The majority of mergers and bids are masterminded and engineered by the merchant banking firms, and a defending company will almost invariably have to appoint its own merchant bank to act in its defence.

The appointment of a merchant bank is just one of the substantial costs which may be incurred in contesting a takeover bid – others include advertising, public relations and underwriting costs. Similarly, the predator company will incur such costs. There is also the possibility of capital gains or losses on the sale and repurchase of shares in the victim.

For a takeover bid to succeed enough shareholders must be willing to sell; this will happen when they are attracted by the potential capital gain due to the high offer price, or when they are unhappy with the current performance and its shares.

The tactics which may be adopted by the directors to contest a bid may include any or all of the following:

� Convincing the shareholders that the shares are valued too low and they should therefore not sell them, usually by circulating profit and dividend forecasts. They can also suggest that forecasts may be at risk on a change of management, by the issue of “defence documents” and press releases.

� Revaluing the company’s assets (using independent expert valuers) to increase the asset-backing and encourage upward movement in the share price.

� Launching a strong publicity campaign, aimed at highlighting present strengths and potential, including promised improvements, e.g. in efficiency.

� Using additional shares either by issuing a block of shares to a friendly party, who will act in the directors’ interests making it almost impossible for the bidder to acquire 100% control; or by issuing “A” shares, normally non-voting, so as to maintain shareholder control but increasing the funds required by the predator to purchase the company.

Page 81: libvolume5.xyzlibvolume5.xyz/law/ballb/3rdyear/semester5/corporatelaw/corporate... · ABE Advanced Diploma in Business Administration Study Manual CORPORATE FINANCE Contents Study

Acquisitions and Mergers 73

© Licensed to ABE

� To invite a bid from another company (a white knight) which the directors believe would be friendlier than the initial offeror. This is called a defensive merger.

� To arrange a management buy-out.

� If the companies are of a similar size, then the target company could make a counter-bid for the predator.

� Launching an advertising campaign against the predator, its accounts and methods of operation.

� Try to have the bid referred to the Competition Commission.

� The target company could introduce a “poison pill” preventing a build up of shares by causing a change in structure and rights to be triggered by “abusive” takeover tactics.

It is important to remember that just as bids must follow the City Code on Takeovers and Mergers, so must the actions taken by directors defending a company against a takeover bid.

If a takeover of an unquoted company is resisted then the bid may simply fail. However, with a quoted company some or all shareholders may wish to sell and there is more chance that the takeover may succeed.

Consideration Acquisitions will be financed by cash, shares, debentures or a mixture of the three. The choice of payment will be determined by individual circumstances. When a merger takes place, a share-for-share exchange occurs.

The factors to be considered when deciding the form that the consideration for acquiring a firm will take are:

� A potential capital gains tax liability may arise when shareholders dispose of their shares for cash.

� An increase in the number of shares may lead to a fall in the EPS and, perhaps, a significant change in shareholder control.

� Increases in borrowing limits or authorised share capital may have to be formally approved by shareholders. Such increases will also have an effect on the gearing level of the firm.

� It may be cheaper to fund the takeover with debt rather than equity because interest is allowable against tax.

� The views of the shareholders in the target company should be considered; they may wish to maintain an investment in the firm and thus prefer shares, and they will want to ensure that they maintain their income levels – a fall in dividends will need to be matched by a capital gain. Each shareholder will want to ensure that he receives at least an equivalent return from his holding in the new company, assuming that there are no other pressing external factors which make aspects other than return on investment significant criteria for the shareholders.

As we discussed in the previous study unit there are several accepted methods of valuing a company. Prices will generally reflect market forces and will tend to be higher when there are several interested parties in competition. The package can be negotiated in such a way as may benefit both parties, perhaps by staggering the purchase over a period of time to aid the purchasers’ cash flow and to minimise capital gains tax liability.

The cost of the acquisition or merger will be the purchase price, plus any extra amounts to be invested in, less the sale proceeds of any surplus assets in, the target company. When considering the cost of

Page 82: libvolume5.xyzlibvolume5.xyz/law/ballb/3rdyear/semester5/corporatelaw/corporate... · ABE Advanced Diploma in Business Administration Study Manual CORPORATE FINANCE Contents Study

74 Acquisitions and Mergers

© Licensed to ABE

the investment the projected returns and profits must be considered alongside existing figures to ensure that the merger or acquisition is in the company’s best interests.

(a) Shares

A share (or paper) purchase involves the exchange of shares in the predator company for the shares in the target company. The shareholders of both companies are now shareholders in the predator company.

(b) Cash

A cash purchase simply involves the exchange of cash for the shares in the target company. The predator should offset the expected earnings on the cash (if no takeover were to take place) against the expected earnings from the acquired firm when considering the purchase. The cash for the purchase can be raised from a Stock Market issue of the predator company’s shares or loan stock.

In assessing the sources of funding available, the financial manager should take into account existing cash resources, and that a proportion of the capital can be generated by:

� Increasing working capital, e.g. by improving credit control. However, care should be taken not to fund long-term assets with short-term finance.

� Sale and leaseback of equipment or premises.

� Staff share purchase schemes, or a rights issue to existing shareholders.

� Disposal of surplus assets.

� Borrowing, e.g. from the bank or by issuing debentures.

(c) Vendor Placing

This is a mixture of the above two approaches. Shares are exchanged in the target company with those of the predator company – the predator shares are then “placed” by the predator’s stockbrokers with other buyers in order to raise the cash for the target’s shareholders.

You should remember that when shares are issued in the process of a takeover bid the predator company’s share capital will increase, and the effects on, for example, earnings per share should be allowed for when considering the impact of the merger on the company’s performance.

D. MEASURING THE SUCCESS AND FAILURE OF MERGERS AND TAKEOVERS

Company Performance Following a merger or takeover there has sometimes been a fragmentation process, with the decentralisation of individual companies and often the promotion of competition between them. There have also been times when the advantages planned for the merger have failed to materialise, with economies arising from increased buying power being lost completely by increased administration and duplication of effort.

There will always be a certain level of risk in such an investment, but adequate research and preparation should help to minimise such risk. Key features of the target company which will improve the chances of success include:

Page 83: libvolume5.xyzlibvolume5.xyz/law/ballb/3rdyear/semester5/corporatelaw/corporate... · ABE Advanced Diploma in Business Administration Study Manual CORPORATE FINANCE Contents Study

Acquisitions and Mergers 75

© Licensed to ABE

� A well-defined market niche

� A balanced customer portfolio

� A growth industry

� Seasonal, fashion and economic cycle stability

� A stable and motivated work force

� High added value

� Good technical know-how

� A short production cycle

� Located near the acquisitor’s business

� Matches the corporate strategic plan

� Provides something the firm does not have for itself.

The early months will be critical in settling down the enlarged business and gradually bringing about new working practices to improve the efficiency of the operation. It requires commitment from all levels of the organisation. Mergers and acquisitions have often been found to fail because senior management are more concerned with future expansion, especially via further acquisitions, rather than with integrating the organisation’s current units. There is a particular problem if the cultures of the merging organisations are very different.

A further point for you to note is that mergers have often been criticised as being implemented to reduce competition and thus to create a more comfortable environment for further business development, rather than due to a desire for increased operating efficiency.

Shareholder Wealth and Power We saw above that a major concern of the predator’s shareholders is changes in their earnings per share. The change in EPS depends on the relative price earnings ratios of the two companies – in general, if the target has the higher P/E ratio then the EPS will fall, but may rise if sufficient levels of synergy and profit growth are achieved. Often, in practice, companies who purchase shares with high P/E ratios do not achieve sufficient growth and experience falls in their EPS. A fall may be acceptable if the shareholders wish to, and do, obtain an increase in net assets per share or a reduction in the risk of its earnings.

In a takeover environment, it will be possible for companies to increase their earnings per share simply as a result of the acquisition, for example:

Company A Company B Total earnings £300 £300 No. of shares 1,000 1,000 EPS 30p 30p Share market price £4.50 £3.00 P/E ratio 15 10

Company A is seen by the market to be better managed, with better growth prospects, when compared with Company B, and therefore its P/E ratio is higher. In the event of a takeover, the position would be:

Page 84: libvolume5.xyzlibvolume5.xyz/law/ballb/3rdyear/semester5/corporatelaw/corporate... · ABE Advanced Diploma in Business Administration Study Manual CORPORATE FINANCE Contents Study

76 Acquisitions and Mergers

© Licensed to ABE

New A plc Total earnings £600 No. of shares* 1,666 EPS 36p

* Company B’s value on the market was (1,000 × 3) = £3,000.

Company A would need to issue 3,000 ÷ 4.50 = 666 (say) shares to acquire B.

The EPS of New A plc has increased because it was able to “save” 334 of its shares (1,000 − 666), because the market has set a higher value on its shares when compared to company B.

We are left with the question: “What P/E ratio should we set for the new company?”

Generally the market will tend to place a higher P/E ratio on the amalgamated companies than would be expected from the result of an averaging. In the scenario above, we might expect a P/E ratio of around 12½.

If the P/E ratio were 14, representing a partial fall, the market price of the shares in New A plc would be £5.04. A’s original shareholders would have seen their shares increase in value from £4.50 to £5.04, whilst B’s (who owned 1,000 shares at £3.00) now own 666 shares at £5.04 = £3,356.64. The somewhat illogical situation arises because of the expectations of the market that the assets of B will be managed similarly to those of A. This, of course, may not always be the case.

The make up of shareholder control will change after a merger, and often changes after an acquisition, especially in the case of a reverse takeover (when a smaller company acquires a much larger one and may require a more than doubling of its equity capital in order to fund the purchase).

Employment Staff will want to talk to the new owner and to be recognised for what they are able to contribute. Talk will be important in order to maintain staff morale which may fall when the sale is confirmed, especially if there is no more information forthcoming. The best staff will be the first to go, because they will readily find jobs even in a depressed market. Competitors may even tempt them away, with a resulting loss of goodwill. One method of keeping key personnel is to require, as part of the purchase offer, that they sign service contracts forbidding them to resign (and perhaps preventing them from selling their shares) for a period of time (often three years). This will be in exchange for an attractive employment contract.

In some cases new contracts of employment may be required for all staff to unify procedures between the two firms.

Example Strachan plc, a successful engineering company has made a bid for Atkinson plc, a large but declining competitor. The following information is available for both companies which are quoted on the Stock Exchange:

Atkinson Strachan Share price £5 £3.10 No. of shares 10 m 25 m

Page 85: libvolume5.xyzlibvolume5.xyz/law/ballb/3rdyear/semester5/corporatelaw/corporate... · ABE Advanced Diploma in Business Administration Study Manual CORPORATE FINANCE Contents Study

Acquisitions and Mergers 77

© Licensed to ABE

Both a cash and a share bid have been made. Strachan has offered Atkinson two shares in Strachan plc for every share in Atkinson plc. Alternatively a cash offer of £6 per share has been made. Strachan plc expects the takeover to generate savings of £5 m in present value terms.

(a) Advise the shareholders of Atkinson plc on which offer to accept. Include financial and other factors in your advice.

(b) How might Strachan plc expect to achieve the extra value of £5m? What uncertainties might face Strachan plc in achieving this figure?

Framework Answer

We will consider briefly some points you could make in answering.

(a) Cash Offer:

Cash received = 10 m × £6

= £60 m.

Share offer:

Shares in Strachan received = 10 m × 2

= 20 m shares.

At the current valuation of Strachan shares this would be worth 20 m × £3.10 = £62 m. However, Strachan would now have 45 m shares in issue, which would be likely to have an impact on earnings per share. In addition, the share price of the expanded company will change as a result of the acquisition.

Advice to shareholders of Atkinson plc

The cash offer of £60 m is £10 m greater than the current value of the company’s shares, i.e. 10 m × £5. We are not given any information about earnings per share of each company or about future growth in earnings, other than the projected savings of £5 m. The acquisition may result in a fall in earnings per share in Strachan plc. This in turn may reduce the value of shares from £3.10 per share.

Cash proceeds are certain, the return on shares in Strachan plc is less certain. However, receipt of cash means that shareholders of Atkinson plc face an immediate tax liability on the capital gain. Strachan plc is already a successful company and it is likely to have considered the acquisition of Atkinson plc very carefully. Strachan plc must consider that it will be able to increase future earnings because of the acquisition. With a competitor removed from the market, there is a real possibility of increased turnover for Strachan plc.

On balance the acceptance of the cash offer would seem to pose less risk to shareholders of Atkinson plc.

(b) Savings on acquisition may be achieved by:

� Economies of scale, e.g. one head office with less staff than exists in the two companies currently;

� Additional expertise from staff of Atkinson plc, reducing training and development costs;

� Lack of competition should result in operating economies, e.g. the advertising budget may be reduced;

� Tax advantages may occur on acquisition.

Page 86: libvolume5.xyzlibvolume5.xyz/law/ballb/3rdyear/semester5/corporatelaw/corporate... · ABE Advanced Diploma in Business Administration Study Manual CORPORATE FINANCE Contents Study

78 Acquisitions and Mergers

© Licensed to ABE

Problems likely to be faced by Strachan plc include:

� Economies of scale are often difficult to achieve in practice. Additional expenditure may be required, for example, to integrate the computer systems of the two companies.

� Staff redundancies may be necessary, incurring redundancy costs.

� Strachan plc may be keen to retain the skills of senior management of Atkinson plc. Additional costs may be incurred in drawing up new management service contracts.

� If the operational units of the two companies are geographically separate, additional transport costs will arise.

Practice Question

Blue plc and Yellow plc have entered into negotiations to merge and form Green plc. Details of the companies are as follows:

Blue plc Yellow plc £1 ordinary share capital £500,000 £300,000 Estimated maintainable future earnings £200,000 £92,280 Agreed P/E ratio for amalgamation 15 13

Suggest a suitable scheme for a merger between the two companies.

Now check your answer with the one given at the end of the unit.

E. DISINVESTMENT

Some groups specialise in buying companies, either stripping their assets or “turning” the business around, and then selling the companies, generally at a profit. Other groups may be forced to sell off parts of themselves due to financial problems, changes in the markets or alterations in their strategic plans.

It is not always easy to move out of a market – one reason may be a reluctance to admit to failure. Management may feel they are safer continuing in the market and may want (wrongly) to attempt to recover sunk costs. There may also be economic costs – it may be easier to sell a going concern; there may be large redundancy costs; and the withdrawal of a product may have a detrimental impact on the sales of the company’s other products. In certain countries government action may prevent withdrawal from a market.

Management Buy-Outs A management buy-out (MBO) is the acquisition by the management of all or part of a business from the owners. The owners can be the shareholders, an owner-proprietor or the parent company, although it is generally the directors who make the disinvestment decision. In general, MBOs have been of profitable subsidiaries which do not match the group’s strategic plans.

Page 87: libvolume5.xyzlibvolume5.xyz/law/ballb/3rdyear/semester5/corporatelaw/corporate... · ABE Advanced Diploma in Business Administration Study Manual CORPORATE FINANCE Contents Study

Acquisitions and Mergers 79

© Licensed to ABE

Management buy-outs have become more common since the late 1980s, the number increasing five fold in 10 years and the value increasing from less than £50 million to more than £2,800 million. It has been due partly to incentives provided by the UK Government and partly due to a belief that management functions better as part of an autonomous profit-seeking unit.

(a) Advantages of MBOs

The potential benefits of an MBO include:

� For the vendor an MBO provides an alternative to the closure of the business or part of it, and prevents the sale to a third party who could be a competitor. The management and employees are more likely to be cooperative to an MBO rather than in a sale to a third party. Moreover, the sale of the going concern may well achieve more money for the vendors.

� For the management team the buy-out allows them to purchase an operation of which they already have full operational knowledge. They must be certain, however, that they can turn the business around to obtain a better return once they are released from the constraints of the current ownership. An MBO will also allow the management team to be owners rather than employees and, if the business is threatened with closure, will prevent them losing their jobs.

� For the financiers who may be invited to participate there is a reduced risk compared to a new venture which has no track record to be evaluated.

(b) Disadvantages of MBOs

There are also potential problems with management buy-outs which include:

� Technical managers, who are good at managing processes, may not have the financial or legal knowledge required to conduct a management buy-out. They will have to use expert advisors, who can be costly, for the tax and legal complications that can arise from an MBO.

� There are often redundancies following an MBO, used as a means of reducing costs. In addition there may be problems convincing employees of the need to adjust working practices and the company may lose key employees. There may also be previous employment and pension rights to be maintained which may be a drain on resources.

� Individual managers will be required to be financially committed to the venture. This may include borrowing from the bank, which may create problems in the manager’s personal financial affairs.

� Problems may arise with regard to the continuity of relationships with suppliers and customers.

� It may be difficult to decide on a fair price to be paid for the business.

� Management may resent the board representation required by suppliers of finance.

� Cash flow problems can arise, especially if fixed assets need replacing.

(c) Financing MBOs

The management’s own financial resources which they are willing to invest will generally be insufficient for the purchase of the business, and they will have to find financial backers. In order to convince the backers of the viability of their idea the management team should prepare a business plan. The business plan should contain cash flow, sales and profit forecasts and

Page 88: libvolume5.xyzlibvolume5.xyz/law/ballb/3rdyear/semester5/corporatelaw/corporate... · ABE Advanced Diploma in Business Administration Study Manual CORPORATE FINANCE Contents Study

80 Acquisitions and Mergers

© Licensed to ABE

planned efficiency savings. This will probably be relatively straightforward – management should have access to a lot of this information, especially if the vendor is happy with the sale.

Financial backers will include banks, accepting houses and venture capitalists, and in general they view their investment as a long-term one. There will often be several financiers providing venture capital for an MBO, and they may require an equity stake in the business because of the risk they are taking – indeed, often the management have only a minority of the shares in the business. Some backers may insist that some of their capital be in the form of redeemable convertible preference shares with voting rights should dividends become in arrears. This form of security allows them to capitalise on the business if it is successful and to cover themselves if the business fails. The purchasing company will usually be financed by bank and subordinated debt, together with the management’s and financiers’ equity. The resultant business will often have a financial gearing level which is far higher than that accepted in general trading circumstances.

We noted above that the financiers’ money is at risk in such an investment, and should be assessed in relation to:

� The expertise, motivation and ability of the management team (including the mix and range of management skills) and the amount the management are willing to invest from their own funds.

� The relationships with its suppliers and customers.

� The projections contained in the business plan.

� Is the projected return from their investment sufficient to offset the risk they are taking?

� Why is the business being sold, and what is being bought? Is additional capital required to replace or improve the assets of the business?

� What is the price and has it been set at the correct level?

The risks will be considered against projected returns before the financiers agree to back the buy-out.

More capital will often be wanted to finance growth. Raising it may be difficult because of the high existing debt/equity ratio. Financiers may commit to the provision of further capital at the start, but will apply stringent performance conditions before allowing a further release of funding.

An alternative form of funding for expansion would be for the management to float the company, perhaps on the Alternative Investment Market. However, this, too, will require careful thought as such action will involve relinquishing at least some control over the business, but it does allow financiers a means to realise their investment.

The long-term viability of a typical MBO in its initial form is a question of very careful risk assessment. It is the ability of the management to plan and develop the business which determines its success. There have been a number of great successes in the past, e.g. National Freight. Empirical evidence has found a number of possible reasons, including reduced overheads; higher levels of managerial motivation; quicker and more flexible decision-making; more austere action on pricing and debt collection; and the MBO having acquired the business for a good price.

Buy-Ins A management buy-in is similar to an MBO except that it is a team of outside managers who mount a takeover bid to run the company.

Page 89: libvolume5.xyzlibvolume5.xyz/law/ballb/3rdyear/semester5/corporatelaw/corporate... · ABE Advanced Diploma in Business Administration Study Manual CORPORATE FINANCE Contents Study

Acquisitions and Mergers 81

© Licensed to ABE

Spin-Offs A spin-off is the creation of one or more new companies with shares being held by the shareholders of the “old” company in the same proportion as before. The assets of the business, having been separated, will be transferred to the new company which will usually be under different management from the “old” company.

Sell-Offs A sell-off is the sale of part of the company, generally for cash, to a third party. An extreme form of sell-off is the liquidation of the entire company.

Demergers A demerger is simply the opposite of a merger. A demerger can either be the selling of part or parts of the business to a third party, or offering shareholders shares in the demerged parts of the business in place of their current shares in the “merged” company. A famous example of a demerger is the split of ICI plc.

There are both advantages and disadvantages of demergers, mainly arising from their smaller size, e.g. if there were diseconomies of scale before the demerger these should be reduced. Similarly any economies of scale would also be reduced. There may also be an increased risk of takeover or, if the attractive parts of the company have been disinvested, there may be a reduced takeover risk. Other advantages include an ability to concentrate on fewer areas, thus hopefully improving efficiency and removing control problems, and an increase in cash and earnings. Disadvantages include the reduction in the ability to raise finance, lower turnover, status and profits.

The Stock Market’s reaction to the demerger will depend on the reasons for it and the view on the future profitability of the demerged components, but generally demergers are viewed favourably. This may be because visibility of individual parts of the business is improved, and there is a greater choice of shares for investors.

Going Private A listed company may decide to go private. This occurs when a small group purchases all a company’s shares and the company is no longer quoted on the Stock Exchange – a well-known example being the repurchase of shares in Virgin by Richard Branson.

The reasons for such a move may be to prevent takeover bids, reduce the costs of meeting listing requirements and limit the agency problem. In addition, because the firm is not subject to volatility in share prices, it can concentrate less on the short-term needs of the Stock Market and more on its own medium- and long-term requirements.

Page 90: libvolume5.xyzlibvolume5.xyz/law/ballb/3rdyear/semester5/corporatelaw/corporate... · ABE Advanced Diploma in Business Administration Study Manual CORPORATE FINANCE Contents Study

82 Acquisitions and Mergers

© Licensed to ABE

ANSWER TO PRACTICE QUESTION

Value of shares for merger:

Blue plc Yellow plc

EPS = 40p500200 = 30.76p

30092.28 =

EPS × P/E ratio = 40p × 15 30.76p × 13

∴ Market value = £6.00 £4.00

If Green plc is to have ordinary share capital of £500,000 + £300,000 (£800,000), the holdings of ordinary shares in the new company could be divided between the former companies’ shareholders in the following proportions:

Blue: 500,000 × £6 = £3,000,000

Yellow: 300,000 × £4 = £1,200,000 (i.e. in the proportion of 3 : 1.2)

or:

Blue: 571,429 shares in Green (approx. 114 for 100)

Yellow: 228,571 shares in Green (approx. 76 for 100)

Total 800,000 This would mean that Green would have an EPS of:

shareper 36.535p800,000292,280 =

and that the proportions attributable to Blue and Yellow are:

Blue: 571,429 × 36.535p = £208,772

Yellow: 228,571 × 36.535p = £83,508

Total £292,280 In this scheme Blue gains some earnings contributed by Yellow, and Yellow loses earnings to Blue. The critical point to both sets of shareholders will be the rating placed on the shares of Green by the market. Gains and losses in EPS will be mitigated by movements in the new share prices.

As an alternative to the above, the shares in Green could be apportioned on the basis of input of earnings without heeding the market price situation. This would penalise Blue in the sense that the market appears to favour its development and growth potential more highly than it does Yellow’s. A straight proportional split would not acknowledge the apparent difference in market rating.

Page 91: libvolume5.xyzlibvolume5.xyz/law/ballb/3rdyear/semester5/corporatelaw/corporate... · ABE Advanced Diploma in Business Administration Study Manual CORPORATE FINANCE Contents Study

83

© Licensed to ABE

Study Unit 4

Financial Markets

Contents Page

Introduction 84

A. Stock Markets 84 Why do we have Stock Markets? 84 The London Stock Exchange 84 Over-The-Counter 87 “Big Bang” 87 Market Participants 88 Advisors to Share Issues 88 Some Other Common Market Terms 89

B. Other Sources of Finance 90 Banks 90 Merchant Banks 90 Institutional Investors 90 Government and European Union Assistance 91

C. Other Financial Markets 92 The London Money Market 92 Parallel Markets 92 Option Markets 93

D. Recent Changes in Capital Markets 93

E. Impact of the Markets on Market Decisions 94

Page 92: libvolume5.xyzlibvolume5.xyz/law/ballb/3rdyear/semester5/corporatelaw/corporate... · ABE Advanced Diploma in Business Administration Study Manual CORPORATE FINANCE Contents Study

84 Financial Markets

© Licensed to ABE

INTRODUCTION

Financial managers have the responsibility of ensuring that sufficient short- and long-term capital is available to organisations, at the time it is needed, and that surplus funds are placed at suitably high rates of return. The financial manager must therefore have a good all round working knowledge of the financial markets that are available to the firm.

This study unit considers the financial markets, the institutions which operate upon them and some of the terminology used. We shall pay particular attention to the markets for share capital – highlighting the importance of the Stock Market and its players in company and market decisions. (However, share capital itself, and alternatives to equity, are dealt with in more detail later in the course, as are options and other financial instruments.

A. STOCK MARKETS

There are several different stock markets in the world, of differing levels of sophistication. Indeed, most major industrial countries have stock markets of some form. However, the three major ones (or the “golden triangle”) are those in London, New York and Tokyo. Here we are mainly concerned with the London Stock Exchange.

Why do we have Stock Markets? The main reasons for having stock markets are:

(a) To provide an efficient mechanism for the bringing together of organisations wishing to raise capital to invest in new projects and investors wishing to place capital in such companies and institutions. A common way of doing so is for a company to issue shares.

There are a number of reasons why a company may want to issue more shares. Before reading further, think of as many different reasons as you can.

The reasons include:

� The company may need to raise more cash to fund investment in profitable projects.

� The company may wish to be “floated” on a stock market. In the UK, when a company is floated on the Stock Exchange there is a minimum proportion of shares which must be made available to the general (investing) public unless the shares are already widely held. This also allows the owners of the company to realise some of their investment in the company by offering their shares for sale.

� Shares may be issued to shareholders in another company in the process of a takeover bid. This is really only feasible when shares that are offered have an identifiable market value and can be easily traded on a recognised stock exchange.

(b) Another reason why stock markets have developed is to allow a ready “second-hand” market in stocks and shares allowing investors to realise their investment if they wish to, either for investing in other securities or for consumption.

The London Stock Exchange The London Stock Exchange functions both as a primary and a secondary market:

� A primary market is one in which organisations can raise “new” funds by issuing shares or loan stock.

Page 93: libvolume5.xyzlibvolume5.xyz/law/ballb/3rdyear/semester5/corporatelaw/corporate... · ABE Advanced Diploma in Business Administration Study Manual CORPORATE FINANCE Contents Study

Financial Markets 85

© Licensed to ABE

� A secondary market is one for dealing in existing securities.

The primary market is known as the New Issue Market. The more important function, especially in terms of volume, of the Stock Exchange is the latter. In addition to acting as primary and secondary markets for corporate shares and loan stock, the London Stock Exchange also functions as the market for dealing in gilts (government securities).

The London Stock Exchange administers and regulates the main market which deals with companies which are “fully listed” (see below) and the second-tier market for smaller companies. Until recently the second-tier was the Unlisted Securities Market (USM), but this was phased out and replaced by the Alternative Investment Market (AIM).

(a) The Main Market

Any company wishing to be floated on the main market must comply with Stock Exchange rules and regulations in addition to the legislation in this area.

Part IV of the Financial Services Act 1986 contains the statutory requirements relating to the listing of securities on the Official List of the Stock Exchange. An application for listing must be made, in accordance with Section 143 of the Act, to the competent authority and must comply fully with the listing rules.

The listing rules are contained in the Stock Exchange’s Admission of Securities to Listing (The Yellow Book). The requirements of the Yellow Book seek to secure the confidence of investors in the conduct of the market. This is done by:

� Ensuring that applications for listing are of at least a minimum size. The size rules relate to the market capitalisation and annual profit figures.

� Requiring companies to have a successful track record of at least three years.

� Ensuring that companies requesting listing at least appear to be financially stable.

� Insisting that a sufficient number of shares of a tradeable value (i.e. several shares of a smaller value rather than fewer shares of a higher value are preferred), are made available to the general investing public to allow a free market to exist in the company’s shares.

� Requiring the directors of a company to make a company resolution to adopt the terms and conditions of the Stock Exchange Listing Agreement. This includes the provision of sufficient information (e.g. interim results) to form a reliable basis for market evaluation.

Note that you may also see flotation on the Stock Exchange referred to as “going public”, or “getting a listing on the Stock Exchange”.

From a company’s viewpoint there are both advantages and disadvantages in obtaining a full listing. Table 4.1 summarises the principal advantages and disadvantages of a full listing to a company.

Page 94: libvolume5.xyzlibvolume5.xyz/law/ballb/3rdyear/semester5/corporatelaw/corporate... · ABE Advanced Diploma in Business Administration Study Manual CORPORATE FINANCE Contents Study

86 Financial Markets

© Licensed to ABE

Table 4.1: Advantages and disadvantages of full listing

Advantages Disadvantages

Once a listing is obtained, a company will generally find borrowing funds easier because its credit rating will be enhanced.

Publicity will not always be of advantage. An unquoted company will be able to conceal its activity because of its lower profile in the community at large.

Additional long term funding can be raised by a new issue of securities.

The costs of entry to the Stock Exchange are high.

Shares can be easily traded, facilitating expansion of the capital base.

At least some control will be lost when shares are available to the general public.

Future acquisitions will generally be easier because the company will have the ability to issue equity as a consideration for the transaction.

The requirements of the Stock Exchange are onerous and compliance is enforceable.

Share option schemes can be arranged potentially attracting the highest calibre employees.

The company can (potentially) be more exposed to a hostile takeover bid.

The profile of the business and its management team will be raised considerably.

(b) Alternative Investment Market

The Unlisted Securities Market (USM) of the London Stock Exchange was a second-tier market for shares in companies not floated on the main market. For some companies the USM was seen as being a stepping stone to full listing – a major purpose in its development. The USM closed to new members at the end of 1994 and completely at the end of 1996.

To replace the USM, which was being gradually phased out, a new market – the Alternative Investment Market (AIM) – was launched by the London Stock Exchange in June 1995. Like the USM it is a market for smaller growth companies that either do not wish to, or fail to qualify to, join the official list. It has, however, less stringent entry requirements and regulations than the USM (or the Official List, as the main market is sometimes referred to) and as such should not be seen as a direct replacement for the USM.

Companies wishing to enter the AIM must, with certain exemptions, issue a prospectus. This prospectus must follow the Public Offer of Securities Regulations (POS) 1995. The company’s directors are responsible for the document as it is not examined by the Stock Exchange.

The exemptions from the POS regulations include:

� Issues by charities

� Issues by housing associations

Page 95: libvolume5.xyzlibvolume5.xyz/law/ballb/3rdyear/semester5/corporatelaw/corporate... · ABE Advanced Diploma in Business Administration Study Manual CORPORATE FINANCE Contents Study

Financial Markets 87

© Licensed to ABE

� Small offers to raise less than £25,000

� Subsequent offers when a POS prospectus has already been issued

� Restricted offers to a small number of knowledgeable people

� Larger offers with a minimum subscription of £25,000 per subscriber.

Companies on the AIM have to publish details of directors’ dealings and interim reports and release any price-sensitive information promptly for which public announcements will generally suffice.

Companies trading on the AIM must choose a Nominated Advisor from an official list given; this is a permanent post which involves advising the company’s directors of their obligations under the AIM rules. Each company must also select from the member firms of the Stock Exchange a Nominated Broker who will support trading when there is no market-maker, and deal with investors.

In common with all quoted companies, AIM companies are subject to the Exchange’s Market Supervision and Surveillance Department and can be fined for breaches of AIM rules.

Unlike the Official List, however, there are no requirements as to size, profitability, track record, number of shareholders or ratio of shares in public hands for companies wishing to join the AIM. The only restriction on type of security which can be traded is that it must be fully transferable.

The Inland Revenue will treat AIM shares as unquoted, thus providing a number of reliefs for investors, which should help provide finance at critical stages for firms (e.g. start-ups). We shall deal with these reliefs in more detail later in the course.

A recent survey by the accountancy firm Kidsons Impey found that small and medium-sized companies consider that the high costs of entry to the AIM make it less accessible. Only 4% of potential users felt that the market was very accessible.

Several companies crashed in 1997–1998 soon after they were listed on the AIM; they included Crown Products Group and Firecrest.

However, the survey also revealed that fast-growing businesses, with turnovers between £4 million and £20 million are still keen to list on the market. In particular, computer and hi-tech companies feel that an AIM listing would give them a higher City profile and access to a wider source of funds.

Over-The-Counter It is also possible to buy shares and other financial instruments in the “over-the-counter” (OTC) markets. These are not regulated or supervised by the Stock Market so there is less protection for the investor, although the costs of dealing on the OTC are less than those of dealing on the main or secondary markets.

“Big Bang” “Big Bang” occurred on 27 October 1986 and was considered necessary because of restrictive practices on the London Stock Exchange, including fixed commission scales and the limited number of firms permitted to participate in dealing.

The effects of “Big Bang” included the abolition of the fixed commission scales and the opening of the Stock Exchange to a wider range of participants. The changes resulted in an increase in competition due to large institutional investors being able to negotiate commission levels. There has

Page 96: libvolume5.xyzlibvolume5.xyz/law/ballb/3rdyear/semester5/corporatelaw/corporate... · ABE Advanced Diploma in Business Administration Study Manual CORPORATE FINANCE Contents Study

88 Financial Markets

© Licensed to ABE

also been a change in the constitution of the membership, with foreign companies moving into the Stock Exchange (a well-known case being Merrill Lynch) and some other overseas organisations acquiring hitherto private dealers, e.g. Citicorp purchased Scrimgeours. In addition, there is now 24 hour trading in leading international shares – starting in the Far East, moving to London and then to New York and back to the Far East for the following day.

Market Participants Prior to “Big Bang”, there was a rigid distinction in the membership of the Stock Exchange with brokers buying and selling financial instruments on behalf of clients for which they charged fixed commissions, and jobbers acting on their own behalf in buying and selling securities. Jobbers would trade with the brokers, but never actually came into contact with the original investors. This was known as single capacity trading.

Single capacity trading has now been replaced by dual capacity trading as both activities can now be conducted by a single firm. Firms can act as agents of the investor (previously the responsibility of the broker) whilst also acting as principal in their own right in buying and selling shares (previously the role of the jobber). Some firms only act as agency brokers, working solely as agents on the behalf of clients in return for commission, and do not deal as principals for themselves.

The new traders acting in a dual capacity are called market-makers. Market-makers must be members of the Stock Exchange. They must register to deal in certain types of security, and must agree to quote two-way prices (for buying and selling) in the shares they trade in.

“Chinese walls” are required to be set up in firms which deal as market-makers in order to separate the broker and dealer functions. This is to prevent the investor (and the firm) from unscrupulous market-makers making profits at their expense, and will also help provide some protection for market-makers and firms against allegations of this type.

Only market-makers are permitted to use the Stock Exchange Automated Quotations System (SEAQ) which is a computerised price display regularly updated by the market-makers themselves. Dealers and investors may view the system on computer terminals via dedicated networks.

Two other terms you may come across in this area are GEMMs and SEMBs. GEMMs are Gilt-Edged Market-Makers and SEMBs are Stock Exchange Money Brokers. SEMBs help to ensure liquidity in the share markets by dealing with the lending and borrowing of money and stocks to those who are experiencing difficulty in fulfilling share-trading deals.

For larger institutional investors there is a growing alternative to this system of market-makers known as crossing networks. Crossing networks, of which examples are Instinet and Global Posit, electronically link buyers and sellers of shares and other financial instruments. Another example of a crossing network is Tradepoint, which operates a screen-based matching of orders for certain UK equities. Whilst the use of crossing networks is still rare in the UK it is common practice in the US.

Advisors to Share Issues We shall discuss the different forms a share or debenture issue can take in detail in the next study unit. The process of obtaining a quotation is a specialised area and one in which a company will generally call on a number of professional advisors. They include:

� Issuing houses, whose functions include giving advice on, and organising, the most appropriate capital structure and time to make an issue of shares or debentures; providing publicity, marketing facilities and backing for the issue; and giving advice to a private company with regard to a decision to become a public company. Issuing houses thus play a vital role in the

Page 97: libvolume5.xyzlibvolume5.xyz/law/ballb/3rdyear/semester5/corporatelaw/corporate... · ABE Advanced Diploma in Business Administration Study Manual CORPORATE FINANCE Contents Study

Financial Markets 89

© Licensed to ABE

issuing of shares and debentures. Issuing houses, including merchant banks acting as such, are regulated by the Issuing Houses Association.

� Underwriters, who are usually issuing houses or merchant banks, generally acting in a syndicate by agreeing to purchase any securities not taken up at the issue price (underwriting). The underwriters will charge a fee which will be payable whether or not they are called upon to take up surplus securities. Whilst not used in placings, they are often used in rights issues.

� Merchant banks, who are sometimes employed to give financial advice, but may also act in the role of issuing house or sponsor.

� Sponsors – the Stock Exchange requires that a company seeking a listing has a firm of stockbrokers to act for it and to ensure that all regulations applying to the process are complied with. The brokers may also act as sponsors to the issue.

� Solicitors – generally two firms of solicitors will be employed, the first supervising the issue itself and the second acting for the company, ensuring the accuracy of legal documentation, e.g. the Memorandum and Articles.

� Accountants, who are employed to give advice on the issue, and to ensure that the legal requirements regarding financial information for share issues are complied with.

Some Other Common Market Terms The Stock Market, and its related activities, is full of unusual terms, e.g. the “Chinese walls” noted above. Below we mention a few of the more common terms, which you may have heard of but be unsure as to their meanings. A useful exercise for you to undertake is to see if you can relate the terms to your wider reading of the financial press.

Bull markets are those with rising prices, and bear markets are ones whose prices fall. Similarly, optimistic people are bullish, expecting increases in share prices, whereas bearish people are more pessimistic about market prospects.

If an investor goes long in a sector it means he is building up a holding of shares in the bullish hope of them increasing in value, whereas someone who goes short (generally a bear) sells shares he does not own, again with the hope of making a profit. A short bear is someone who takes a gamble that he may be able to purchase the shares cheaper (a bear covering) before he has to deliver them (at a pre-arranged price) and thus make a profit. An uncovered bear (one who sells shares he does not have) can face unlimited losses if the price rises quickly and he has to purchase shares to sell at a loss.

A bear-raid is when a number of dealers sell ostentatiously in an attempt to drive down the market. In order to prevent this market-makers can mark up the prices near the end of the account period – a so-called bear squeeze.

A bull may buy shares and intend to sell them at a profit before he receives them (he may need the sale proceeds to pay for the shares); if this goes wrong and a loss is made he is referred to as a stale bull.

A stag is someone who applies for shares at the start of a new issue of shares intending to sell them straightaway. This is because new issues are usually priced low in order to ensure that they are sold, and hence start trading at a substantially higher price. The difference between the issue price and the initial trading price (the premium) is the stag’s profit.

Page 98: libvolume5.xyzlibvolume5.xyz/law/ballb/3rdyear/semester5/corporatelaw/corporate... · ABE Advanced Diploma in Business Administration Study Manual CORPORATE FINANCE Contents Study

90 Financial Markets

© Licensed to ABE

B. OTHER SOURCES OF FINANCE

The markets we discussed earlier are not the only sources of finance. Other major sources include the following:

Banks Banks are a major source of short-term (and recently medium-term) corporate financing. Long-term loans from banks are in the form of mortgages. The banks will expect the customer to provide a reasonable proportion of the required funding from its own resources (e.g. from equity). Clearing banks like to lend against security (i.e. assets).

The rate of interest charged to larger companies on medium-term bank loans will be set at LIBOR (London Inter-Bank Offer Rate) plus a margin which depends on the credit rating and riskiness of the borrowing. For smaller companies the interest rate will be the bank’s base rate plus a margin, again depending on the riskiness and credit rating of the firm. The rate of interest in both cases can be either fixed or variable. Often in the former cases it is adjusted every quarter, half-yearly, three-quarterly, or yearly in line with changes in LIBOR.

Merchant Banks These are banks dealing as investment banks, generally for corporate clients. They have a range of activities, including:

� Involvement in the business of the Stock Exchange for their clients, helping in the issuing and underwriting of share issues and share registration.

� The provision of venture capital and large scale medium-term loans to companies.

� The taking of wholesale deposits in all currencies.

� Dealing in foreign exchange and the bullion markets, and through subsidiaries in stocks and shares.

� The provision of advice during takeovers and mergers.

� Granting acceptance credits.

� The management of client investments and acting as trustees.

� The provision of general investment and deposit advice to corporate clients.

Institutional Investors These are institutions with large funds to invest in assets which provide sufficient returns and security, including investments in company stocks and shares. The UK’s main institutional investors are:

(a) Unit Trusts

Unit trusts were developed in order to allow small investors to hold a diversified portfolio of investments. The portfolio is managed on behalf of investors by a unit trust company which deducts management expenses from the income of the portfolio before paying the investors their share of the income. Each investor holds a sub-unit or stake in the portfolio. The sub-units are sellable, their price being determined by the underlying value of the securities included within them.

(We shall examine portfolios in detail later in the course.)

Page 99: libvolume5.xyzlibvolume5.xyz/law/ballb/3rdyear/semester5/corporatelaw/corporate... · ABE Advanced Diploma in Business Administration Study Manual CORPORATE FINANCE Contents Study

Financial Markets 91

© Licensed to ABE

(b) Investment Trusts

Investment trusts invest in a wide range of securities though they are generally concerned with quoted and larger unquoted companies. They are interested in both returns and securities, wishing to maintain a steady growth in income in order to pay their shareholders’ dividends.

(c) Venture Capital Organisations

Venture capital is the investment of money in a new or expanding business, or in a management buy-out. It is generally done in exchange for an equity stake in the business and a seat on the board.

There are several organisations offering venture capital, including venture capital funds acting as agents for the venture capital industry, the clearing banks. Those which have joined are regulated by the British Venture Capital Association. The biggest and oldest venture capital provider is the 3i group (Investors in Industry plc).

Venture capital is a high risk, high return investment, which is generally short term, being between five and seven years in duration. Often the venture capitalist will be able to realise his investment (and hopefully profits) when the business is floated on the AIM and the venture capitalist can sell his shares.

In order to qualify for venture capital a company must be able to show the venture capitalist that it has the ability to succeed; that the product or service is viable and meets a need in the market; that sufficient levels of the correct form of finance are available; and there is, or will be, a good management team.

Venture Capital Trusts (VCTs) are investment trusts investing a significant proportion of assets in unquoted companies to a maximum in a company of £1m, and per investor of £100,000. There are general tax reliefs of 20% of dividends provided shares are held in the VCT for five years, and capital gains rollover relief provided the capital gains are reinvested in a VCT. VCTs help to spread risk by holding portfolios of new companies.

(d) Pension Funds

Pension funds hold large amounts of money which will provide retirement benefits for their members. In most pension funds there is a surplus of incoming funds from contributions over outgoings as pension payments. The surplus must be invested to achieve the best possible return whilst maintaining the security of funds. The pension fund manager often spreads the investment between high yield securities, e.g. gilts, and equity or property.

Pension funds may provide substantial finance for major investment schemes. For example, funds from the Coal Industry Pension Scheme helped fund the new “Galleries” in Wigan town centre. A new multi-million pound expansion of Milton Keynes shopping centre is being funded by the Universities Superannuation Fund.

(e) Insurance Companies

These, along with pension funds, provide the most funds for investment in the UK. They have similar investment policies, seeking secure returns and steady growth in order to meet their commitments.

Government and European Union Assistance In common with the majority of governments, the UK Government provides financial assistance to companies, especially those operating in high technology industries or in areas of high unemployment.

Page 100: libvolume5.xyzlibvolume5.xyz/law/ballb/3rdyear/semester5/corporatelaw/corporate... · ABE Advanced Diploma in Business Administration Study Manual CORPORATE FINANCE Contents Study

92 Financial Markets

© Licensed to ABE

� Enterprise Initiative Scheme

This is a series of measures including local business advice centres, “Business Links”, and grants provided by the Department of Trade and Industry (DTI).

� Regional Enterprise Grants

These are available for companies in Development Areas employing fewer than 25 staff, and are used to help finance investment in fixed assets or to aid innovative projects.

� Regional Selective Assistance

This is considered by the local office of the DTI and is only available in areas defined geographically for the purpose of the availability of financial assistance. The amount of grant will be the minimum required to commence the project. To be eligible the project must need assistance, be commercially viable, safeguard or create jobs and offer a distinct advantage to the region or country as a whole.

The EU funds the European Regional Development Fund (ERDF) which gives money to member governments – the majority going to the poorest member nations. The UK received over £3.5 billion during the first 15 years of the ERDF’s life.

Note that the sources of finance available to the company, especially from governmental bodies, are an ever-changing area, and you should follow developments in the financial press.

C. OTHER FINANCIAL MARKETS

The London Money Market The London money market covers a wide range of UK institutions.

� The Bank of England

� Merchant banks

� Discount houses

� Finance houses

� Pension funds

� Unit trusts

� Parallel markets

� Clearing banks

� Other banks

� Insurance companies

� Investment companies

� Building societies

� The Stock Exchange.

Parallel Markets These consist of the Foreign Exchange Market and the Eurocurrency Market, both of which we shall consider later in the course. They also include:

Page 101: libvolume5.xyzlibvolume5.xyz/law/ballb/3rdyear/semester5/corporatelaw/corporate... · ABE Advanced Diploma in Business Administration Study Manual CORPORATE FINANCE Contents Study

Financial Markets 93

© Licensed to ABE

� The Local Authority Market

� The Interbank Market

� The Finance House Market

� The Intercompany Market

� The Certificate of Deposit (CD) Market.

Option Markets Options are a form of dealing which can be used as an insurance against, or for speculation on, a rise or fall in a particular share, and act as a way to limit the cost of taking a view on the future performance of the share.

An option is the right to buy (a call option) or sell (a put option) shares at a future date (generally three months forward) at a fixed price agreed now. There is also a double option which gives the right to buy or sell and this is called a put and call option or a straddle.

Bargains are agreed at a striking price (the figure at which shares can be called or put at the end of the option period), which is the current share price, plus interest to meet the carry-over facilities during the option period. It is not necessary to wait until the end of the option period before taking action, and it is possible to deal in those shares during the period. A fee per share will be charged by the broker whether or not the option is exercised.

Options have a maximum life of nine months with expiry dates arranged on a three-month cycle, so that a maximum of three extensions can be arranged on shares of any one company. They are popular because they provide investors with price and time variety not possible with traditional call and put options and the opportunity to profit from price changes which involve a smaller outlay than direct involvement in the securities themselves.

An extension of the options market is the Traded Options Market. This gives the holder of the option the right to buy or sell the option itself before its expiry date.

(We shall examine options in detail later in the course.)

D. RECENT CHANGES IN CAPITAL MARKETS

There have been vast changes in the capital markets over recent years including:

� Increased competition with the impact of Big Bang, building societies becoming banks, and banks operating in non-domestic markets have, amongst other things, increased the levels of competition in the financial services industry.

� Securitisation of debt, i.e. firms are increasingly issuing securities rather than borrowing from the banks.

� The deregulation of the financial and other service markets.

� The abolition of exchange controls allowing companies to use overseas markets to obtain funds.

� Disintermediation – firms using securitisation to avoid using banks.

� Globalisation – shares issued in one country are traded around the world.

� The development of, and greater use of, risk management tools such as the use of swaps and options which help in the borrowing of money and also reduce the financial risk apparent in the balance sheet.

Page 102: libvolume5.xyzlibvolume5.xyz/law/ballb/3rdyear/semester5/corporatelaw/corporate... · ABE Advanced Diploma in Business Administration Study Manual CORPORATE FINANCE Contents Study

94 Financial Markets

© Licensed to ABE

We shall consider these points in greater detail elsewhere in this course.

E. IMPACT OF THE MARKETS ON MARKET DECISIONS

The Stock Market and its players have a major impact on a company’s, and on the market’s, decisions. We will refer to them throughout this course, and you should take time to consider the point when you have completed each study unit.

One example we can see from the last study unit is in the field of MBOs. Many firms are adopting the policy of focusing management and financial resources on core activities, and to a large extent it has been supported by the stock markets and institutional investors. With diversification often less attractive, opportunities for managers to buy-out their operations have become more common.

Other examples from earlier study units include the existence of short-termism (see Study Unit 1); the market’s views on the relative strengths of two companies will affect the valuation placed on them and the resulting terms of any acquisition or merger (Study Unit 2); and the three financial management decisions discussed in Study Unit 1 will be affected by the market and its workings.

Page 103: libvolume5.xyzlibvolume5.xyz/law/ballb/3rdyear/semester5/corporatelaw/corporate... · ABE Advanced Diploma in Business Administration Study Manual CORPORATE FINANCE Contents Study

95

© Licensed to ABE

Study Unit 5

Sources of Company Finance

Contents Page

Introduction 97

A. Share Capital 97 Value 97 Authorised and Issued Capital 97 Types of Share Capital 98 Retained Profits 99 Dividend Payable 99 Newspaper Information on Shares 99 Share Categories 99 Penny Shares 100

B. Methods of Issuing Shares 100 Placing or Selective Marketings 100 Offers for Sale 100 Sale by Tender 101 Rights Issue 102 Other Methods 104 Pricing Shares for a Stock Market Quotation 105 Costs of Share Issues 106 Issuing New Shares Without Raising Capital 106

C. Share Repurchases 107

D. Debt and Other Forms of Loan Capital 108 Debentures and Loan Stocks 108 Mortgages 111 Convertible Loan Stock 112 Warrants 114 Leasing 115

(Continued over)

Page 104: libvolume5.xyzlibvolume5.xyz/law/ballb/3rdyear/semester5/corporatelaw/corporate... · ABE Advanced Diploma in Business Administration Study Manual CORPORATE FINANCE Contents Study

96 Sources of Company Finance

© Licensed to ABE

Hire Purchase 116 Licensing and Franchising 117

E. Short-Term Finance 117 Securitisation 117 Note Issuance Facilities (NIFs) and Revolving Underwriting Facilities (RUFs) 117 Commercial Paper (CP) 117 Syndicated Credits 118 Banks 118

F. International Capital Markets 118 Eurocurrency 119 Eurobonds 119 Euroequity 120 Choice of Currency for Borrowing 121 Advantages of Raising Funds in International Markets 121

G. Finance and the Smaller Business 121 Government Measures 121 Grants 122 Banks 122 Venture Capital Providers 122 Finance Companies and Lessors 122

Answers to Practice Questions 123

Page 105: libvolume5.xyzlibvolume5.xyz/law/ballb/3rdyear/semester5/corporatelaw/corporate... · ABE Advanced Diploma in Business Administration Study Manual CORPORATE FINANCE Contents Study

Sources of Company Finance 97

© Licensed to ABE

INTRODUCTION

In this study unit we will look at the main sources of finance available to companies.

We start by considering the primary source of funding for limited companies – equity finance – including how it is raised. Whilst under UK company law all limited companies must have equity, many also utilise additional types of finance. We go on, then, to examine the different forms available, including those from international capital markets. This is an area in which there are always new arrangements developing, and you will almost certainly learn of more from your reading of the financial press as new ideas come to market.

A. SHARE CAPITAL

Shares are described as permanent capital because the funds supplied for their acquisition are non-returnable (to the investor who provided them) in most circumstances other than in the event of a liquidation. The ordinary shareholders collectively own the company, but stand last in line for rewards on investment and in the event of liquidation, although they do have ownership of the remaining funds either in a trading period or in the event of liquidation.

Ownership means that the ordinary shareholders bear the greatest risk. In return for their acceptance of the risk they are equity shareholders, each share carrying a vote in the management of the business, although as we saw in Study Unit 1, managerial control may be limited.

In the case of a limited company, the Articles of Association will include details of each class of shares which comprise the capital structure of the company.

Value We have already determined that the owners of the company are the shareholders – and to show their ownership they have shares issued to them. Shares are issued in the UK at a nominal, authorised or face value generally of £1, 50p, 25p, 10p or 5p.

Be careful not to confuse nominal value with market price, to which it bears no relationship. The market price is the price that shares are sold for. (If the market value and the nominal value are the same, the shares are said to be at par value.)

The only exception to this rule of no relationship is when the shares are first issued, and the nominal value is the minimum at which the price would be set. Established companies issuing new shares to the market, with existing shares that have a market value in excess of the nominal value, may issue the new shares at a premium (i.e. at a value greater than their nominal value).

(Not all countries have company shares with nominal value. It would be a useful exercise for you to find out if countries which you deal with at work have shares with nominal values.)

Authorised and Issued Capital The maximum amount of capital (detailed in the Memorandum of Association) that a company may issue is known as the authorised capital. Do not confuse this with a company’s issued capital.

The issued capital is the nominal value of share capital allotted (also referred to as issued) to shareholders. The maximum amount of capital that can be issued is an amount equal to the authorised share capital.

Page 106: libvolume5.xyzlibvolume5.xyz/law/ballb/3rdyear/semester5/corporatelaw/corporate... · ABE Advanced Diploma in Business Administration Study Manual CORPORATE FINANCE Contents Study

98 Sources of Company Finance

© Licensed to ABE

Types of Share Capital The rights and voting powers of shares, and the differentials between the different classes of shares, are listed in the Articles of Association. The issue of different classes of ordinary (equity) shares is strongly discouraged by the Stock Market and only allowed in exceptional cases. We will now look at some of the different types of share capital, but unless stated otherwise we will generally be considering ordinary share capital.

(a) Redeemable Shares

A limited company may, if authorised by its Articles, issue shares which are to be redeemed at the option of the shareholder, provided it has issued other shares before the redeemable ones. There are various rules relating to the redemption of redeemable shares, varying between public and private companies, developed in order to protect shareholders and the general investing public.

(b) Preference Shares

Unlike ordinary share capital, preference shares are entitled to a fixed percentage dividend, which is paid before any profits are distributed to ordinary shareholders. However, like ordinary share dividend it can only be paid if there are sufficient distributable profits available. However, unpaid dividends on cumulative preference shares are carried forward and must be paid before any dividends can be paid to ordinary shareholders.

Preference shareholders may be entitled to receive the nominal value of their shares in the event of a company winding-up, before any distribution is made to ordinary shareholders – in which case the rules underlying it will be stated in the company’s Articles of Association.

There are three further types of preference shares.

� Participating preference shares may be entitled to some extra dividend, over and above their fixed dividend entitlement, but it may only be paid after the ordinary shareholders have received an agreed amount in dividend.

� Convertible preference shares are preference shares that can be converted into equity shares. A company may issue them to finance major acquisitions without increasing the company’s gearing or diluting the EPS of the ordinary shares. The shares potentially offer the investor a reasonable degree of safety with the chance to make profits as a result of conversion to ordinary shares if the company prospers.

� Redeemable preference shares may be issued provided the Articles permit the company to do so. The shares will be redeemable, either at some future date or within a range of dates, and at that time the shareholder will have his investment returned to him.

Preference shares constitute a small proportion of companies’ finance, especially in recent times – investors prefer loan stock where the returns and security are greater and, from the companies’ point of view, the dividend on preference shares, unlike interest on loan stock, is not allowable against corporation tax.

(c) Deferred Shares

These are the last in line for dividends and the proceeds of liquidation. In rare circumstances, a founder’s ordinary share may be authorised and held by the founders of the company. Where it exists, it will rank behind all other shares for dividend.

Page 107: libvolume5.xyzlibvolume5.xyz/law/ballb/3rdyear/semester5/corporatelaw/corporate... · ABE Advanced Diploma in Business Administration Study Manual CORPORATE FINANCE Contents Study

Sources of Company Finance 99

© Licensed to ABE

(d) Non-voting Shares

A company may issue non-voting shares which still carry the risk of equity shares but do not allow the shareholder any say in the running of the company – for this reason they are not very popular except in takeover bids.

Retained Profits You will recall from your earlier accounting studies that part of shareholder funds is retained profit. Indeed the major source of new funding for companies in the UK is retained earnings.

Dividend Payable Dividends are paid on ordinary shares as a percentage return on their nominal value. For example, the holders of a £1 nominal ordinary share in a company which declares a dividend of 20% will receive 20p per share regardless of the price paid for the share.

You will recall from an earlier unit that the dividend yield relates the market price of a share and the dividend received. If the dividend yield falls, then the share becomes less attractive compared to other investments, demand for it will fall, and supply will increase as investors wish to sell. Its market price will then reduce until the yield on its new market price is in equilibrium with returns received elsewhere for similar levels of risk. Although this is a highly simplified explanation, it does underpin the basic rationale behind the workings of the stock markets.

We shall discuss the level of dividends paid by the company in greater detail later in the course, but dividends are generally dependent on the results of the company. This benefits the company, since ordinary shares have a fluctuating service cost which only increases (in theory at least) during periods in which the company is successful.

Newspaper Information on Shares Many of the broadsheet papers include a section showing security prices and related information including: the highest and lowest prices during the year; the closing price the previous day; the change in price over the previous trading day; dividends net of tax; dividend cover; gross yield; and the P/E ratio. You should try to “read” such information on a regular basis and ensure that you understand the information contained in the different columns.

Share Categories Shares are categorised according to the company type and trading frequency:

(a) Alphas

These are the shares of the most prestigious companies which are generally heavily traded and dealt in by a large number of market-makers. Prices are posted immediately on SEAQ.

(b) Betas

These are also large company securities, but they are not as heavily traded as alphas. They must have at least four market-makers dealing in them and prices quoted on SEAQ are those at which firm deals can be made.

(c) Gamma

These securities are traded less frequently than betas and the prices quoted for them are merely indicative.

Page 108: libvolume5.xyzlibvolume5.xyz/law/ballb/3rdyear/semester5/corporatelaw/corporate... · ABE Advanced Diploma in Business Administration Study Manual CORPORATE FINANCE Contents Study

100 Sources of Company Finance

© Licensed to ABE

(d) Deltas

These are the least traded of all and no prices are actually shown, simply indications of a dealer’s interest.

Penny Shares Penny shares are shares which have a very low value (often because the company is experiencing difficulties) with the bid or offer spread of such shares exceeding 10% of their market value. Investors buy such shares in the hope that the market has undervalued their prospects and that they will make a substantial profit when the price recovers. In practice, however, the market is rarely wrong and the price often falls further or the company goes into liquidation. The low valuation of such shares reflects the high risk attached to them and the efficiency with which the market operates.

B. METHODS OF ISSUING SHARES

One way in which a firm can expand is to issue additional equity – usually on the main Stock Exchange or second-tier market, either by a quoted company issuing additional shares or by an unquoted company obtaining a quotation. An unquoted company may also wish to issue shares without being floated.

We discuss below various methods of issuing shares. The factors which help to determine the best approach to adopt are the amount to be raised, the cost of raising it, and the state of the market and economic conditions generally.

Placing or Selective Marketings This method is often used for a company wishing to be floated for the first time and to issue a small issue, or by a quoted company wishing to raise additional finance. It involves securities’ acquisition by a market-maker so that they can be purchased by a small number of investors.

The method is widespread because it is relatively cheap, with the sponsoring firm approaching selected institutional investors privately. However, it does limit the number of shares available for trading on the market.

Offers for Sale This method is popular with companies being floated for the first time and undertaking a large issue, e.g. an unquoted company may sell some of its existing shares on the market. An unquoted company can also issue new shares and market all (new and existing) shares. The selling of existing shares provides a wider market for finance (and shares) in a company and allows shareholders a chance to sell their shares. A company will only receive additional funds if new shares are issued.

An unquoted company applying for quotation has to be sponsored by a Stock Exchange member firm and an issuing firm. The responsibility of the former is to ensure that the company fulfils the requirements for listing; the issuing house has to ensure that the issue is successful both in price set and take up of shares.

In an offer for sale the issue price (which must be advertised a short time in advance) is usually given as based on a P/E ratio allowing for easy comparison by investors. The price of shares is generally set at a discount to what could be obtained because of uncertainties regarding the market, and to prevent an undersubscription (when shares have to be bought by the underwriters). Whilst the issuing house attempts to set the price so that share prices rise briskly once trading commences, they should prevent

Page 109: libvolume5.xyzlibvolume5.xyz/law/ballb/3rdyear/semester5/corporatelaw/corporate... · ABE Advanced Diploma in Business Administration Study Manual CORPORATE FINANCE Contents Study

Sources of Company Finance 101

© Licensed to ABE

the price being too low and causing the company to issue more shares than necessary. A rise in share prices of approximately 20% is usually aimed for.

Sale by Tender This method is occasionally used for large first time issues but it is less common than offers for sale because of uncertainties as to the amount of finance which will be raised and it may give the impression that the issuing house is unable to set a price for the shares. An issue by tender has become a more common method of share issue in recent years.

The issuing house (or the company) invites members of the public to subscribe by tender, i.e. to state at what price they will take the security offered; sometimes there is a minimum price below which tenders will not be accepted. Applications may be dealt with in a number of ways and the most common methods are explained in paragraphs (1) to (4) in the following example:

Example

ABC plc is to make an issue of £2m ordinary shares by tender. The minimum tender price has been fixed at 50p. Applications were received as shown in the table below:

Tender Price Number of Shares Applied For

Cumulative Applications

Value of Application

Cumulative Value

£ £ £ 2.00 100,000 100,000 200,000 200,000 1.75 200,000 300,000 350,000 550,000 1.50 300,000 600,000 450,000 1,000,000 1.25 400,000 1,000,000 500,000 1,500,000 1.00 450,000 1,450,000 450,000 1,950,000 0.75 550,000 2,000,000 412,500 2,362,500 0.50 1,000,000 3,000,000 500,000 2,862,500

Different methods are:

� Applications are accepted in full at the prices tendered, starting with those offering the highest prices and working downwards until the new issue has been allotted in full. Therefore in the example above, the full issue is fully subscribed at a price of £0.75, which yields an overall value of £2,362,500 for the 2m shares. This method is unsuitable for most public issues as it tends to frighten away the private investor and the issue is not spread amongst many applications.

� Shares are allotted in full, starting with the applications offering the highest prices and working downwards until all the issue has been fully allotted. However, here, the price fixed is that of the lowest tender to be accepted and all applications pay at this price. In our example, the issue price is the point where the full 2m shares are allotted, namely at 0.75p. As everyone will pay this price, the issue will raise £0.75 × 2m = £1,500,000. Whilst the approach is fair for the small shareholder, the issue is not usually spread amongst many applicants.

� The applications accepted are scaled down so that each applicant only receives a proportion of the shares for which he asked. The issue price is again the lowest successfully tendered. In our example and assuming that 2.5m shares are to be issued, five would be issued for every six applied for by each applicant – the issue price being 50p. If therefore an investor had requested

Page 110: libvolume5.xyzlibvolume5.xyz/law/ballb/3rdyear/semester5/corporatelaw/corporate... · ABE Advanced Diploma in Business Administration Study Manual CORPORATE FINANCE Contents Study

102 Sources of Company Finance

© Licensed to ABE

600 shares and tendered at £1 per share, he would have sent in a cheque with his application for £600. Under the rules, he would be allowed 500 shares at 50p per share (£250) and receive a refund of £350. This method is the fairest so far for the small investor and generally means that more applications are successful.

� A further improvement results from a method whereby two prices are fixed. Applications at prices between them are scaled down. Applications at lower prices are rejected, and those above the higher price are accepted in full, up to a certain maximum number, or scaled down slightly.

There are, of course, a number of variations to the above and issuing houses are continually researching to make improvements. Although small investors tend to avoid tenders as they have difficulty in deciding on a fair price, recent experiences suggest that the fixing price is usually not much different from the ensuing market price, reflecting the influence of the institutional investors.

The two main advantages for the company issuing shares are that:

(a) Paperwork is reduced as the number of applications is reduced.

(b) The fixing price in recent years has usually been higher than would be necessary in an offer for sale. In an offer for sale, the price is kept low in order to ensure that the offer is accepted, but in a tender the forces of supply and demand have improved the price.

Rights Issue The term rights issue is applied to the system of issuing shares to existing shareholders, usually at a discount from the market price, in order to raise further capital from existing shareholders. (A discount makes rights issues impractical when current market price is at or below nominal value.)

When offering shares in a rights issue, the company sends an explanatory letter to each shareholder detailing the price, accompanied by a provisional allotment letter indicating the number of shares to which the member is entitled to subscribe, e.g. a rights issue on a 1 to 5 basis at 120p per share means that for every 5 shares a shareholder holds he can purchase an additional 1 at 120p. The offer must be on a “rights basis” in proportion to the member’s existing holding as a fraction of the holdings of all members eligible to receive the offer. Forms of acceptance or renunciation accompany the offer, and should a member choose not to accept he may renounce his “rights” in favour of someone else. In due course, if the member accepts the offer, or exercises his rights, he will lodge his acceptance and cheque with the company.

Should the member choose to renounce the offer he can sell the rights to his shares, a sale being attractive because the price of the shares is below their current market value. The rights of shareholders to buy the rights are known as pre-emptive rights.

If shareholders do not take up the rights or sell them, the company must, under Stock Exchange rules, sell them for the shareholders who get the rights value. If small amounts are involved the benefits can go to the company. In order to maintain his share of control the shareholder must take up all his rights. The shares may be issued to outsiders or in different proportions between members. Members may, if permitted under the company’s Articles of Association, be offered the shares which their peer group have rejected.

The key to a successful rights issue is to achieve a balance between the ratio of the proposed new issue to those already in issue, and the price at which rights may be taken up, considering that the rights issue will generally be at a discount.

Page 111: libvolume5.xyzlibvolume5.xyz/law/ballb/3rdyear/semester5/corporatelaw/corporate... · ABE Advanced Diploma in Business Administration Study Manual CORPORATE FINANCE Contents Study

Sources of Company Finance 103

© Licensed to ABE

Rights issues offer advantages to both the company and existing shareholders – for the company there is less administration and no prospectus is required, both factors reducing costs. The cost of underwriters, if used, will also be lower.

Shareholders obtain shares at a discount or can sell their rights allowing the purchaser a chance to buy shares at a discount. The share price before dealing in the newly-issued shares will be “cum” rights, and thus the shares will trade at a higher price. Shareholder control can be maintained provided the shareholder exercises his full rights.

Calculation of Issue Price

This is the amount that the company wishes to raise from the issue divided by the number of new shares. It is important to issue a sufficient number of shares so that the issue price is below current market price.

Alternatively, if the number of shares as well as the price is to be determined, the factors to be considered include:

� the ease of issue

� the current market price – too large a discount is to be avoided, and a discount of approximately 25% is usual; and

� the number of shares currently in issue – too many new shares might dilute the EPS too much.

Calculation of Price after Issue

(a) Ex-rights price

This is the theoretical price and is essentially a weighted average. If, for example, a company is making a 1 for 3 rights issue, with each new share costing £2 and the current market price being £3, then the ex-rights price will be:

Cum rights value of 3 shares (× £3) £9New share £2

£11 Therefore the theoretical ex-rights price is 11/4 = £2.75.

This could also be shown in terms of total value of the issue divided by the total number of shares.

(b) Actual Price After Issue

The actual price after issue often differs from the theoretical price, which is due to differences in expected earnings between the existing and the new shares.

Here the shares can be valued on a P/E basis. Using the figures in (a) above, we will first consider the situation where the earnings from the new shares are expected to yield the same return as the existing shares. We will then look at what happens if expected earnings are relatively higher or lower.

� At current earnings:

Assume a P/E ratio of 10, which means that the earnings are currently 10% of the market value (i.e. £3/10 = 30p) or if, for example, 3m shares are currently in issue, £900,000 in total.

Page 112: libvolume5.xyzlibvolume5.xyz/law/ballb/3rdyear/semester5/corporatelaw/corporate... · ABE Advanced Diploma in Business Administration Study Manual CORPORATE FINANCE Contents Study

104 Sources of Company Finance

© Licensed to ABE

If expected earnings from the new shares are at the same level, they will yield 1m × £2 × 10% = £200,000. Total earnings will, therefore, be £1,100,000.

Monetary value of the shares (× P/E of 10) = £11,000,000.

The share value is therefore 11,000,000/4,000,000 or £2.75, i.e. the same as the theoretical ex-rights price.

� At lower earnings: if, say, the new shares are expected to yield 5% earnings:

Total earnings will be (£900,000 + (1m × £2 × 5%)) = £1,000,000.

Monetary value of shares = £10,000,000.

Value per share £10,000,000/4,000,000 = £2.50 per share.

� At higher earnings: if, say, the new shares are expected to yield 15% earnings:

Total earnings (£900,000 + (1m × £2 × 15%)) = £1,200,000.

Monetary value of shares = £12,000,000.

Value per share = £12,000,000/4,000,000 = £3.00 per share.

The break-even point (i.e. no dilution of earnings per share) is when the rights price equals the current capital employed per share, assuming that current trends continue. The actual impact on shareholder wealth will depend on the earnings generated compared to the existing earning rates – if lower the market value will fall, if higher it will rise and so forth.

Empirical evidence has found that the market price of shares tends to fall after a rights issue due mainly to uncertainty as to the company and its prospects – the level depends on the company and the rights issue.

Value of Rights

We noted earlier that shareholders can sell their rights. How do we determine what price they would receive.

The value is the difference between the price of the rights shares and the ex-rights shares. In the above example, the price of the rights shares was £2; if the value of ex-rights shares was £2.75, then the value of the rights is £2.75 − £2 or 75p. The new shareholder in effect pays the ex-rights price purchasing the rights for 75p and the share for £2. The value of rights attached to each existing share is the value of rights divided by the number of shares required to issue. In the above example this would be 75p/3 shares or 25p per share. Shareholders can take up part of their rights and sell the others.

Other Methods

(a) Subscription offers

An offer for subscription is an invitation to the public by, or on the behalf of, an issuer to subscribe for securities not yet in issue or allotted.

(b) Prospectus issue

If reasonably substantial, the company might make an issue direct to the public with the absolute minimum of assistance from the outside sources we discussed earlier. It is rather unusual because of the complexity of the nature of the capital issue.

Page 113: libvolume5.xyzlibvolume5.xyz/law/ballb/3rdyear/semester5/corporatelaw/corporate... · ABE Advanced Diploma in Business Administration Study Manual CORPORATE FINANCE Contents Study

Sources of Company Finance 105

© Licensed to ABE

(c) Stock Exchange introduction

This is the introduction of the shares on the Stock Exchange so that a quotation (i.e. a price) can be fixed, rather than the issue of shares. An introduction cannot be made unless sufficient shares are available to establish a market and thus a price, and this method tends to be restricted to larger firms.

(d) Role of underwriters

As we saw earlier, underwriters agree to purchase any securities not taken up at the issue price and will charge a fixed fee for their service. Use of underwriters removes the risk of a share issue being under-subscribed.

However, the costs of underwriting are high and companies with marketable securities may choose to use another method known as the bought deal. Here a major investment bank will buy all the shares in the new issue at a slightly discounted price.

(e) Open offers

This is an offer to existing shareholders to subscribe for securities, whether or not pro rata to their existing shareholdings. They are not allotted through the use of renounceable documents.

An open offer is a concessionary method of listing, approval being sought in principle early on. Once approval has been granted, the sponsor firm has to inform the market ten business days prior to the “on-register” date. The announcement will state that the proposals are subject to shareholder approval at general meeting.

(f) Vendor share scheme or placing

Where a vendor prefers cash to shares issued to finance an acquisition by a purchaser, an issuing house can place the securities with clients for cash.

(g) Exchanges and conversions

This is the process used to replace one security with another, e.g. a vendor consideration issue or paper issue used in a merger or takeover bid.

(h) Employee share schemes

Such schemes are often used as incentives, e.g. share option schemes which give certain employees (usually directors) the chance to purchase shares in the company at a price determined in advance, hopefully, for a financial benefit.

Pricing Shares for a Stock Market Quotation We have seen that the setting of a price is the job for the sponsor, and we dealt with the valuation of companies in Study Unit 3. However, factors a firm should take into consideration include:

� The current and future market conditions and the firm’s results.

� P/Es of similar quoted companies.

� Whether the quotation is to be on the main market or the AIM.

� An initial premium on launch, followed by steady growth in the share price is desirable.

� The amount of finance required.

� Future dividends and earnings forecasts (the more shares the more dividends need to be paid out).

� Underwriting costs or deep discount required to avoid undersubscription of shares.

Page 114: libvolume5.xyzlibvolume5.xyz/law/ballb/3rdyear/semester5/corporatelaw/corporate... · ABE Advanced Diploma in Business Administration Study Manual CORPORATE FINANCE Contents Study

106 Sources of Company Finance

© Licensed to ABE

New issues are generally made when share price and expectations are high – investors are more likely to buy shares, dilution of the EPS will be limited and the increases in the dividends required to be paid will occur.

Costs of Share Issues Share issues (and debenture issues) can be expensive and depend on the method used. Costs involved in share issues include:

� The Stock Exchange listing fee for the new securities.

� Fees of advisors, including those of the issuing house.

� Underwriting costs.

� The compulsory advertising in national newspapers.

� Printing and distribution costs of details and prospectus.

Whilst some costs are variable, such as the commission payable to issuing houses, most are fixed and thus it is more economical to issue large amounts of shares, which discourages small firms.

Compliance with Stock Exchange rules (which vary according to the size of the issue) is also a cost to be borne by the company (the rules include the number of market-makers to use and how to issue the shares).

Issuing New Shares Without Raising Capital There are also three methods by which new shares may be issued without bringing additional funds into the company.

(a) Scrip issues

A scrip issue, also referred to as a capitalisation, or a bonus issue, involves the conversion of reserves into capital, causing a fall in the reserves. Shareholders receive additional shares in proportion to their holding. Unlike a rights issue no additional funds are brought into the company – the shareholders do not “pay” for the shares. This results in more equity in circulation with the result that the market value will generally fall in the short term, thus making it more attractive to potential investors. The reserves used are either from a credit balance in the profit and loss account or from reserves specifically marked for the payment of shares, and requires authority from the Articles of Association and the AGM.

(b) Scrip dividends

Scrip dividends are a conversion of profit reserves into issued share capital offered to shareholders in lieu of a cash dividend. Enhanced scrip dividends are those where the value of shares is greater than the cash dividend offered as an alternative. Such dividends are of benefit to the company as they maintain cash within the business and are not liable to advance corporation tax. There may, however, be tax complications arising for individual investors.

(c) Stock split

A stock split is the splitting of existing shares into smaller shares, e.g. each ordinary share of 50p is split into two of 25p, in order to improve the marketability of the company’s shares. It can also be used to send signals that the company is expecting significant growth in EPS and dividends per share, and for this reason the resulting market price of the split shares is higher than the simple split price would be. For example, if a share with a market value of £10 was split into two shares their price would be higher than £5. Reserves are not affected.

Page 115: libvolume5.xyzlibvolume5.xyz/law/ballb/3rdyear/semester5/corporatelaw/corporate... · ABE Advanced Diploma in Business Administration Study Manual CORPORATE FINANCE Contents Study

Sources of Company Finance 107

© Licensed to ABE

C. SHARE REPURCHASES

Repurchases, or buy-ins, of shares may be made by companies out of their “distributable profits”, or out of the proceeds of a new issue of shares made especially for the purpose, provided they are authorised to do so in the company’s Articles of Association.

A company may not, however, purchase its own shares:

� Where, as a result of the transaction, there would no longer be any member of the company holding other than redeemable shares.

� Unless they are fully paid up, and the terms of the purchase provide for payment on repurchase.

Purchases may be in the market or off-market. An off-market purchase is said to occur when the shares are purchased not subject to the marketing arrangements of the Stock Exchange, or other than on a recognised stock exchange. A buy-in of shares by a public company will be subject to the rules of the Stock Exchange and to the provisions of company law.

The change in the capital base will cause management to rethink its investment decisions, gearing, interest cover, earnings, etc. This is particularly important as the financial institutions focus their attention more towards income and gearing as an indicator of financial risk.

It is important to be aware of the various advantages and disadvantages of share repurchases. The advantages of share repurchases include the following:

� It may allow a company to prevent a takeover bid. The control by the existing shareholder group will be increased.

� A quoted company may purchase its shares in order to withdraw from the Stock Market (see below).

� It can be a useful way of using surplus cash.

� Repurchasing shares will reduce the number in circulation which should allow an increase in earnings and dividends per share, and should lead to a higher share price. It will increase future EPS as future profits will be earned by fewer shares.

� Reducing the level of equity will increase the gearing level for a company with debt which may be considered beneficial by the company.

� If the business is in decline a share repurchase may give the firm’s equity a more appropriate level.

The disadvantages of share repurchases are that:

� Repurchasing of shares may be viewed as a failure by the company to manage the funds profitably for shareholders.

� The company requires cash for the repurchase.

� It may be difficult to fix a price which is beneficial to all involved.

� It requires existing shareholder approval.

� Capital gains tax may be payable by those shareholders from whom the shares are purchased.

� It increases gearing.

We mentioned above that a firm may decide to go private, which happens when a small group purchases all a company’s shares and the company is no longer quoted on the Stock Exchange – a well-known example was the repurchase of shares in Virgin by Richard Branson.

Page 116: libvolume5.xyzlibvolume5.xyz/law/ballb/3rdyear/semester5/corporatelaw/corporate... · ABE Advanced Diploma in Business Administration Study Manual CORPORATE FINANCE Contents Study

108 Sources of Company Finance

© Licensed to ABE

The reasons for such a move may be to prevent takeover bids, reduce the costs of meeting listing requirements and to limit the agency problem. In addition, because the firm is not subject to volatility in share prices it can concentrate less on the short-term demands of the Stock Market and more on its own medium- and long-term requirements. Another key reason for a company wanting to buy its own shares stems from the desire of management to improve earnings per share, a financial ratio in which investors are becoming increasingly interested. Such opportunities will be considered against financial performance, share price and capital structure.

Practice Question 1

TT is a public limited company with a paid up share capital of 1.2m £1 shares and no long-term debt. The dividend has been paid at a rate of 45 pence per share in each of the last two years and the market price per share has not varied significantly from its current market price of £3.60.

The company now wishes to finance the expansion of its existing premises and the board has announced a rights issue of 1 new ordinary share at a price of £3.00 for every 4 shares currently held. The company forecasts that future dividends will be at a rate of 43.5 pence per share on the enlarged share capital and that its existing ratio of dividend cover can be maintained.

You are required to comment on the theoretical and the practical validity of the following three reactions to the announcement from various shareholders.

(a) I hold 5% of the equity. Whether I take up or surrender my rights, I should be getting a bargain at the offer price of £3.00.

(b) As I understand the position, I should not have been out of pocket in the long run if the company had priced the rights issue at £5.00 instead of £3.00. I think they should have done that to raise a bit more money.

(c) The company could have fixed a lower price for the issue if the underwriters hadn’t been looking for a high amount of commission, 1¼% being the amount charged.

Now check your answer with the one given at the end of the unit.

D. DEBT AND OTHER FORMS OF LOAN CAPITAL

Debentures and Loan Stocks Companies often raise long-term interest-paying debt, which is known as loan stock, and its holders are long-term creditors of the company. Debt capital is attractive to companies because the interest charges on it are allowable against corporation tax, and the status quo of shareholder control is also maintained. In addition, we will see later in the course that an increase in the gearing ratio may be beneficial to shareholders by improving their EPS.

There are, however, limits to the amount a firm will borrow, including restrictions in the Articles of Association, debenture trust deeds (see below) and market attitudes. In addition, high interest rates may make high levels of borrowing impractical and there may be insufficient security to cover new

Page 117: libvolume5.xyzlibvolume5.xyz/law/ballb/3rdyear/semester5/corporatelaw/corporate... · ABE Advanced Diploma in Business Administration Study Manual CORPORATE FINANCE Contents Study

Sources of Company Finance 109

© Licensed to ABE

loans. We shall discuss the optimal amount of gearing a company should have in detail in a later study unit.

Loan stock is often issued at its nominal value or face value (i.e. at par). The nominal value represents the amount the company owes and the “coupon” (or interest rate) is based on the nominal value. The coupon rate set depends upon the company, its credit rating and the market conditions when the debt is issued. The market value of the stock will, however, fluctuate with changes in interest rates and in the company’s results and prospects.

(a) Features of debentures

A debenture is a multiple loan to the company in the sense that it is contributed by several people as opposed to just one individual. Debentures attract a fixed rate of interest, and debenture-holders are creditors, not members, of the company. Therefore their interest ranks for payment prior to shareholders’ dividends, and must be paid even if the company has made a loss.

Debentures may be either redeemable or permanent; a holder of permanent debentures can obtain a return of his original investment by disposing of his holding to a third party, and companies may repurchase permanent debt.

Most debentures are redeemable, typical issue periods being from 10 to 30 years, often with two redemption dates, e.g. 2008/09, the choice of redeeming in 2008 or 2009 is the company’s and will depend on:

� Whether the company has sufficient liquid funds to redeem the debentures, and

� The current levels of interest rates compared to the rate being paid on the debentures.

Companies frequently find that interest rates are higher as the redemption date approaches and will delay their redemption to the later date.

Debentures tend to be issued in times of low inflation and low interest rates. The redemption will be financed either by cash reserves or by the raising of fresh debt or equity capital. A company, and potential investors, will compare the finance a company has available with the planned repayment of its debt shown by the repayment dates of its loan stock and debentures.

Debentures may be issued at par, at a premium or at a discount. Debentures issued at large discounts and redeemable at par or above are known as deep discount bonds. They are generally issued at low rates of interest which can be attractive to companies with cash flow problems. However, there is a high cost of redemption. The investor may be attracted by the capital gain at maturity, but you should note that it is taxed as income (less notional interest not paid).

(b) Security and debentures

Debentures are generally secured by a trust deed setting out the terms of the contract between the company and the debenture-holders. The deed may include security given in the form of:

(i) A specific or fixed charge over particular asset(s) in the form of a mortgage debenture, restricting the alteration and disposal of the asset by the company.

(ii) A general or floating charge on assets, giving a general lien to the debenture-holders, but not restricting the company in its utilisation of assets.

The trust deed may also contain provisions for a trustee (e.g. a bank) acting on the behalf of debenture-holders to intercede if the terms of the trust deed or Articles of Association in relation to the debentures were breached, e.g. failing to pay the correct amount of interest, or

Page 118: libvolume5.xyzlibvolume5.xyz/law/ballb/3rdyear/semester5/corporatelaw/corporate... · ABE Advanced Diploma in Business Administration Study Manual CORPORATE FINANCE Contents Study

110 Sources of Company Finance

© Licensed to ABE

exceeding prearranged borrowing limits. A receiver may be appointed if the company is unable to honour its debts. However, as we saw in Study Unit 5 a reconstruction scheme may be used to avert liquidation.

A well-established company may occasionally issue an unsecured or naked debenture. Naked debentures generally have interest rates at least 1% higher than secured debt in order to compensate investors for the additional risk they are bearing.

(c) Registration

Mortgages and debentures must be registered in the company’s own register of charges and with the Registrar of Companies to record their existence.

(d) Issue price and conditions for purchase

Debentures can be issued at a discount and a company may also enter the market and buy up its own debentures without formality.

(e) Types of interest

The great majority of debentures are issued at fixed rates of interest. There are two possible variations and these are:

(i) Floating rates

The issuer will be able to vary the interest paid. For the issuing company, floating rates afford protection in periods of volatile interest rates since it will benefit when rates fall. Investors benefit, since they should obtain a fair return whatever happens to interest rates generally.

The market value of the debentures depends on the coupon rate of interest compared to general market rates. The value should remain stable since the interest payable on the debentures will follow that of the market.

(ii) Zero coupon

These are debentures which are issued with no rate of interest attached. Instead, they are issued at a discount. Thus there is an implied rate of interest in the level of the discount. The advantage to the borrower is that there is no cash outlay until redemption. For the lender, there may be tax advantages in not receiving income in the short term.

(f) Return for investors

To determine whether a potential investor will receive a certain rate of return by investing in a particular debenture, we need to calculate the NPV (net present value) of all the cash flows involved. Whilst we will cover this topic in more depth later in your course, it is worth spending a few minutes considering an example.

Example

An investor requires a return of 15% on his investments and is considering investing £200 in 12% debentures redeemable in exactly three years’ time. Their current price is £80.

First calculate the amount of stock he can purchase:

£(200 × 100/80) = £250.

The yield will be (£250 × 12%) = £30 per year.

The calculate the return (assume interest is paid at the end of the year and personal taxation is ignored).

Page 119: libvolume5.xyzlibvolume5.xyz/law/ballb/3rdyear/semester5/corporatelaw/corporate... · ABE Advanced Diploma in Business Administration Study Manual CORPORATE FINANCE Contents Study

Sources of Company Finance 111

© Licensed to ABE

Year Cash Flow Discount Factor 15% Present Value 0 Purchase 200 1.00 (200.00) 1 Interest 30 0.87 26.10 2 Interest 30 0.76 22.80 3 Interest 30 0.66 19.80 3 Redemption 250 0.66 165.00

NPV +33.70 As the NPV is positive, the investor will achieve more than his required return of 15%.

To find the market price given the cash flows and the anticipated market rates of interest is simply a matter of excluding the purchase price from the calculation. In other words, the NPV would be 233.70. This is, of course, the value of £250 worth of stock. For £100 worth of stock, the calculation is: 233.70/2.50 = £93.48.

If market interest rates were 20%, the value would be as follows:

Year Cash Flow Discount Factor 15% Present Value 1 30 0.83 24.90 2 30 0.69 20.70 3 Capital & interest 280 0.58 162.40

NPV 208.00 £100 worth of stock would be valued at 208/2.50 = £83.20

Because market rates have increased, the price of debt capital has fallen, so there will be a greater capital gain on redemption to make up for the interest shortfall when compared with what is available in the market.

(g) Reverse yield gap

Generally, the investor will expect to be compensated for increased risk of loss by an enhanced return and vice versa. Consequently, ordinary shareholders expect a higher return than debenture-holders.

However, in times of inflation this may be an oversimplification. Investors may be prepared to bear the risk and, at the same time, accept modest returns in the anticipation of future capital growth which will maintain the value of their investment in monetary terms.

When the return from equities exceeds that available from fixed interest stocks, there is said to be a yield gap. When fixed interest returns are higher than those from equities, there is said to be a reverse yield gap.

Mortgages A mortgage is a form of secured loan placing the title deeds of freehold or long leasehold property with a lender as security for a cash loan, usually up to two-thirds the value of the property. The cash is generally repayable over a prearranged period, and interest (at either a fixed or floating rate) is payable on the amount borrowed.

Page 120: libvolume5.xyzlibvolume5.xyz/law/ballb/3rdyear/semester5/corporatelaw/corporate... · ABE Advanced Diploma in Business Administration Study Manual CORPORATE FINANCE Contents Study

112 Sources of Company Finance

© Licensed to ABE

Convertible Loan Stock Convertible loan stocks have proved to be a particularly attractive form of capital instrument during recent years. Usually this class of stock is sold as fixed interest loan stock initially, but there will be an option to convert the loan into equity shares at a given price and during a specified period. The terms of conversion often increase over time with increased expectations as to the share price and returns from the shares. The conversion value (the current market value of a unit of stock converted into shares) will be below the loan stock value on issue but is expected to rise as conversion approaches.

From the investor’s point of view, he will stand to gain a stake in the company, whilst maintaining the status of a creditor and the security of fixed interest during the potentially risky period when his funds are being used. At a later date and without extra outlay, he may expect the right to equity by which time it will be his hope that his money will have begun successfully to contribute to the company’s profits.

The company benefits by securing funds at fixed interest rates, supplemented by tax relief on its interest payments, at a time when it may not be able to support the burden of dividend payments. As the intention is to convert into equities funds do not have to be repaid, and there should be a realistic chance that the future increased dividend payments can be met from profits generated by successful utilisation of the invested moneys.

There is no specific requirement for loans to be converted into shares on every occasion. There may even be an option not to convert but simply to redeem the original investment.

You should understand and be able to calculate the following:

(a) Conversion Ratio

This is the number of ordinary shares that will be obtained from each unit of loan stock. For example, if a company has 8% convertible loan stock standing at par and £10 of loan stock can be converted into three £1 ordinary shares, then the conversion ratio is:

£1£10 3 ÷ = 0.30

∴ 0.3 shares will be obtained for every £1 of loan stock converted.

(b) Conversion Price

This is the amount of stock necessary to obtain each ordinary share. In the example above this would be:

3

£10 = £3.33

∴ £3.33 of stock is required for each share.

(c) Conversion Premium

This is the difference between the value of the stock and the conversion value of that stock on the date of issue. Therefore, again using the figures above, £100 of stock was worth £120 on issue and we know that £100 of stock can be converted into 30 shares. (We ascertained 0.30 shares can be obtained for every £1 of loan stock in (a) above.) Supposing that on the day of issue, the market value of each share was £3, then the conversion premium would be:

3)(303) (30 120

××− × 100 = 33.33%

Page 121: libvolume5.xyzlibvolume5.xyz/law/ballb/3rdyear/semester5/corporatelaw/corporate... · ABE Advanced Diploma in Business Administration Study Manual CORPORATE FINANCE Contents Study

Sources of Company Finance 113

© Licensed to ABE

The premium per share is calculated as:

date conversionat shares of No.stock loan econvertibl of ueMarket val −−−− Current price

In our example we have:

(120 ÷ 30) − 3 = 4 − 3 = £1 per share.

Looking at it another way, 30 shares, currently worth £90 (£3 per share) will be worth £120 on their conversion (i.e. £4 per share).

From the point of view of the issuing company, the greater the amount of conversion premium the better, because they will have to issue fewer shares for the amount of original loan stock. For the investor, the level of conversion premium which is acceptable will be weighed up against expectations for the company. If the investor considers the premium reasonable, he will invest.

He may, for instance, expect that the value of each share will rise between the date of purchase and the date of conversion. In our example, if the price has risen to £5, the value of the conversion will then be (30 × £5) = £150 for an original investment of £120.

The conversion premium is often stated as a percentage of the conversion value.

The attractiveness of convertible loan stock will depend on a combination of factors such as:

� The cost of the stock at the time of purchase.

� The period of time to conversion.

� Stockholders’ future expectations for the company.

As a rule of thumb, convertible loan stock issued at par has a lower coupon rate than normal stock, the lower returns effectively being the price the investor is prepared to pay for his conversion rights.

The market value of the convertible stock cannot go lower than the market value of normal stock of the same coupon rate. Should this occur, it will signify that the market does not attach any value to the conversion rights.

The advantages of convertible loan stock to the respective parties are set out in Table 5.2.

Page 122: libvolume5.xyzlibvolume5.xyz/law/ballb/3rdyear/semester5/corporatelaw/corporate... · ABE Advanced Diploma in Business Administration Study Manual CORPORATE FINANCE Contents Study

114 Sources of Company Finance

© Licensed to ABE

Table 5.2: Advantages of convertible loan stock

The Issuing Company The Investor

Stock can be issued at a lower coupon rate – useful in times of high interest rates.

The market value of the stock cannot fall below that for similar ordinary stock of the same coupon rate.

Interest on loan stock should be tax-deductible unlike dividends on equity.

Increases in share prices will cause the value of the conversion to rise because this is the amount the investor will eventually receive.

As it is a form of deferred equity, there will be no cash outlay on redemption.

Stockholders will be paid before shareholders in the event of a liquidation.

Convertible loan stock may be counted as equity for gearing calculations, unlike ordinary loan stock.

If share prices are depressed, it may be easier to issue loan stock instead of equity.

Warrants Warrants are rights given to investors allowing them to buy new shares in a company at a future date at a fixed, given price. This price is known as the exercise price, and the time at which they can be used to obtain shares in is known as the exercise period.

Whilst warrants are generally issued alongside unsecured debt as a “bribe” to potential investors, they are detachable from the debt and can be traded in any time up to the end of the exercise period.

The value of the warrant is dependent on the market’s view of the likely price of the shares it can be traded for in the future. Its “theoretical value” is the difference between the current share price of the company and the exercise price multiplied by the number of shares which each warrant can be used to obtain. During the exercise period the value of the warrant will not fall below this price; if this theoretical price is zero, then the value of the warrant will also be zero (the holder would be better not exercising his rights and obtaining £0, than obtaining something worth less than he paid for it).

The price of warrants and their attached premiums depend on the length of time until the end of the exercise period, the exercise price, the current share price and the future prospects of the company.

Generally, if the company has good prospects then the warrants will be quoted at the warrant conversion premium which is calculated by comparing the cost of purchasing a share using the warrant, and the current share price. The premium will reduce the closer it is to the exercise price because if there was a premium during the exercise period then it would be cheaper to purchase the shares directly rather than via a warrant.

Page 123: libvolume5.xyzlibvolume5.xyz/law/ballb/3rdyear/semester5/corporatelaw/corporate... · ABE Advanced Diploma in Business Administration Study Manual CORPORATE FINANCE Contents Study

Sources of Company Finance 115

© Licensed to ABE

Example

Ella plc issued 50p warrants which entitle the holder to purchase one share at £1.75 at a specified time in the future. If the current share price of Ella is £1.50 calculate the conversion premium.

The conversion premium is:

£ Cost of the warrant 0.50Exercise price 1.75

2.25Current share price 1.50

Premium 0.75 The premium would be quoted as a percentage of the current share price – in Ella plc’s case 50%.

There are several advantages to investors of purchasing warrants. Initially they have to spend a smaller amount than if they were purchasing shares and, because of this lower outlay, the potential loss is much less (the value of the warrant compared to the value of the share). Conversely, because the initial outlay is less than for shares any increase in the share price (and thus in the warrant’s price) will result in a greater percentage increase in the wealth of the warrant-holder than of the shareholder. For example, a 25p increase in the value of Ella plc’s shares (see above) would give shareholders an increase in their wealth of 25/150 or 16.67%, but warrant-holders would have an increase in wealth of 25/50 or 50%, – this is known as the gearing effect of warrants. A final advantage for taxpayers is that profits from warrants are classed as capital gains rather than income.

Companies often issue warrants to make debt issues more attractive, and even sometimes to make them viable. In addition, they may be able to offer a lower rate of interest on the debt if warrants are attached to the issue. Warrants are a potential future source of equity, which do not require dividends immediately or cause a dilution of earnings per share or current shareholder control.

Leasing Leasing is a common method of financing the use of an asset – commonly equipment such as office equipment and cars.

A leasing agreement is between two parties – a lessor and a lessee. A financier (the lessor), generally finance house subsidiaries of banks, purchases the asset and provides it for use by the company (the lessee). The lessor is considered to be the legal owner and can claim capital allowances for the asset. The lessee makes payments to the lessor for the use of the asset.

There are two forms of leases – finance leases and operating leases. In addition, you need to be aware of the use of sale and leaseback methods.

(a) Finance leases

Finance leases are leases in which the lessor (the owner) will expect to recoup the whole (or most) of his cost of performing the contract during the initial period of rental, referred to as the basic lease period (or primary term). At the end of the period the asset is generally either leased for a further period for a peppercorn rent, sold by the lessor, or sold by the lessee for the lessor for a large amount of the proceeds. The servicing and maintenance of the asset is the lessee’s responsibility. Finance leases are reported on the face of the balance sheet.

Page 124: libvolume5.xyzlibvolume5.xyz/law/ballb/3rdyear/semester5/corporatelaw/corporate... · ABE Advanced Diploma in Business Administration Study Manual CORPORATE FINANCE Contents Study

116 Sources of Company Finance

© Licensed to ABE

(b) Operating leases

Operating leases are leases other than finance leases, and do not have to be reported on the face of the balance sheet. Common examples of operating leases include short-term rental contracts for office equipment, and contract hire agreements for the provision of vehicles. Operating leases do not cover the economic life of the asset, and at the end of one contract the equipment is leased to someone else. This is especially useful for the lessee in the case of high-technology products which are quickly obsolete, the risk being borne by the lessor. The servicing and maintenance of the asset is the lessor’s responsibility.

As operating leases are not reported as balance sheet items, they will not be included in gearing calculations. However, liability for payment of future rentals under the terms of contract will be reported as a note to the accounts, and will thus be considered when the company’s accounts are analysed, e.g. by potential lenders.

Leases are popular with lessors, lessees and the suppliers of equipment. The latter like them because they are paid fully for the asset at the start of the contract. Lessors are able to make profits from leasing equipment, and obtain capital allowances on the purchase. A lessee may find this option cheaper and easier than taking a bank loan out to purchase the asset. A lessee may also have insufficient cash to purchase the asset outright.

(c) Sale and leaseback

A financing arrangement, whereby a company will sell its building to an investment company or other specialist in the field. The purchasing company (lessor) takes an interest in the freehold land on which the property stands, and the selling company becomes the lessee who then rents the building which it previously owned, generally for a minimum of 50 years. The rent is reviewed every few years.

The lessor will wish to obtain a good investment which could be rented to another company if for some reason the original lessee no longer wishes, or is unable, to rent the property. Thus they would prefer the building to be in good repair, non-specialised and in an area which is, or is likely to be experiencing, rises in commercial property prices.

The main advantage from this method of financing is that the company can raise more money than would be the case if the property was used as security for a mortgage.

The main disadvantage is that fewer assets remain to support future borrowing, and the effect of the removal of a significant asset from the balance sheet may cause an adverse reaction by financial commentators and the market in general.

Other disadvantages are that the company loses the flexibility to move and sell the property, the real cost can often be very high especially if rents are increasing rapidly, and the company loses future capital gains on the property.

Although this method is complex, you do not a detailed working knowledge – merely be aware of it as a financing arrangement, whereby the company is able to obtain additional capital without recourse to further borrowing.

Hire Purchase Hire purchase is in many respects a hybrid between lending and renting. The facility may be simply defined as “hiring with the option to purchase”. The HP payments consist partly of capital and partly of interest payments. On payment of the final instalment ownership of the asset passes to the customer. By concession the Inland Revenue will generally permit the customer to claim and retain

Page 125: libvolume5.xyzlibvolume5.xyz/law/ballb/3rdyear/semester5/corporatelaw/corporate... · ABE Advanced Diploma in Business Administration Study Manual CORPORATE FINANCE Contents Study

Sources of Company Finance 117

© Licensed to ABE

capital allowances provided that the option to purchase fee is less than the market value at the end of the contract term.

The common procedure is for the finance house to buy the good from the supplier who delivers it to the customer. The finance house and the customer set up a hire-purchase agreement, which includes the payment of an initial deposit by the customer, the size of which depends upon the finance company’s policy and the credit worthiness of the customer.

The advantage for the customer is that the interest part of the payments are allowable against tax, and capital allowances can be claimed on the asset.

Licensing and Franchising These allow for the acquisition of a product, service or business concept of another organisation, without the purchase of the other entity as a whole. Examples of the former are most forms of computer software, where the user pays a licensing fee for the right to use the software, and examples of the latter are McDonalds and Kentucky Fried Chicken. Generally for a franchise an initial payment is required by the provider, and royalties will be due on subsequent sales, in return for marketing the provider’s goods and services in an exclusive territory. It is usual for the franchisee to have to acquire all goods and consumables from the franchisor under the terms of the agreement.

E. SHORT-TERM FINANCE

The company will not always wish to raise long-term finance. In recent years, the capital markets have recognised this need in the growing company, and there has been an increased concentration on the short- or medium-term floating rate sector.

Securitisation This is the practice whereby instead of lending money to customers, banks raise finance for them by arranging and selling to customers their securities (e.g. commercial paper) often allowing lower interest rates. Similarly companies can invest short-term securities in commercial paper for better rates.

Note Issuance Facilities (NIFs) and Revolving Underwriting Facilities (RUFs) A major development in capital provision is the arrival of the Note Issuance Facilities (NIFs) and Revolving Underwriting Facilities (RUFs). They are examples of Multiple Option Facilities (MOFs) which are arranged by a company’s bank, and involve a package of medium-term back-up facilities provided by a group of banks. The banks will underwrite the facility to ensure that the borrower obtains the required funds, usually over a period of three to ten years. The bank will also arrange a panel of banks tendering loans in a variety of forms and currencies to the company, and a panel to provide “standby” loans. The firm can then, at competitive interest rates, have a series of short-term loans and in effect obtain medium-term financing, or have standby loans for use as and when required.

Commercial Paper (CP) The company will usually be afforded a number of other mechanisms so that it can raise short-term funds, not just from underwriting banks. One example is where the company issues six month dollar notes in the European Commercial Paper Market (ECP). Other examples of commercial paper include ECP in other currencies and SCP (Sterling Commercial Paper). These short-term securities issued in bearer form are a written promise to pay (promissory note), within a fixed period of less than

Page 126: libvolume5.xyzlibvolume5.xyz/law/ballb/3rdyear/semester5/corporatelaw/corporate... · ABE Advanced Diploma in Business Administration Study Manual CORPORATE FINANCE Contents Study

118 Sources of Company Finance

© Licensed to ABE

one year, the amount on the note. The rate of interest is the difference between its face price and the issue price which is at a discount.

Medium-term notes (MTNs) are similar to commercial paper but have a stated rate of interest and are issued for between one and five years. The use of CPs and MTNs is limited in that a minimum size of company and issue restricts it to large companies, multinationals and certain public sector bodies, and they are driven by the demand of investors (investor-driven). However, where they can be used they are very flexible, allowing the company to schedule borrowing to match its requirements and make the best use of interest rate fluctuations. The company does not have to be finely rated.

The company may also be able to call for advances of a multi-currency nature, perhaps in dollars or sterling.

Syndicated Credits This is a facility whereby a borrower can borrow money when necessary up to the limit of the facility – thus the company only has to borrow as and when required. The interest rates are high compared to those offered elsewhere and are generally used by highly-geared companies (who may find it difficult to find other sources of finance), and government-related bodies (e.g. Eurotunnel). Another common use is in takeover bids. Syndicated credits are most common in the US and are generally denominated in US dollars.

Banks Bank lending to companies is predominantly short term, although it is also now a valuable source of medium term finance.

Short term finance from banks comprises:

� Short term loans: Fixed rates of interest are available for up to five years;

� Overdrafts negotiated with the bank. Interest is charged on a daily basis depending on how much the company is overdrawn each day. Interest rates of overdrafts are usually higher but it is a very flexible method of finance, particularly when a company has to provide for seasonal variations in cash flow which will require facilities for short periods of time only. There is no penalty for repayment of an overdraft, unlike the early repayment of a medium-term loan which is generally for a fixed term period.

The interest rate for small companies on medium-term loans may either be at a fixed rate or at a margin above the bank’s base rate. For larger companies the interest rate on medium-term loans may again be fixed for up to five years but is usually at a margin above the London Inter-Bank Offered Rate (LIBOR) adjusted every three, six, nine or 12 months in line with LIBOR movements (the size of the margin being determined by risk and the credit standing of the company.

F. INTERNATIONAL CAPITAL MARKETS

There is an increasing internationalisation of capital markets, especially for the larger companies, e.g. when BT was floated it was on both the domestic and international markets.

Note that many of the terms discussed below have the prefix “euro” but it does not limit them to European currencies, the term being an historical one. The transactions are any undertaken in a currency outside the country of origin of the particular currency concerned.

Page 127: libvolume5.xyzlibvolume5.xyz/law/ballb/3rdyear/semester5/corporatelaw/corporate... · ABE Advanced Diploma in Business Administration Study Manual CORPORATE FINANCE Contents Study

Sources of Company Finance 119

© Licensed to ABE

Eurocurrency The Eurocurrency market is the borrowing and lending of Eurocurrency loans via banks based outside the currency’s country, e.g. a UK company could borrow Yen for trade with Japan from a British bank (Euroyen). The loans are generally short term (three months), often between banks.

The market generally offers high rates of interest, flexibility of maturities and a wide range of investment qualities in comparison with other capital markets. The dealings for substantial funds (generally in excess of $1m) are highly competitive. The unique feature is that the transactions in each currency take place outside the country from which that currency originates. The main function of the market is the financing of international trade, e.g. to fund down payments of goods sold on export credit terms.

On the short-term, inter-bank Eurocurrency market, transactions may take place between banks on an unsecured basis from overnight to five years’ duration. Most transactions are for six months or less and transactions of over £1m are common.

Eurobonds The Eurobond market is an international capital market which has developed alongside the Eurodollar market since the 1960s. It deals in the lending of currencies for longer terms under bonds which are usually denominated in dollars or Deutschmarks.

The following advantages are claimed for the Euromarkets in various currencies:

� Extremely large sums can be raised or deposited.

� Money will often be cheaper than the domestic markets.

� Controls tend to be less restrictive.

� Some protection against exchange rate movements is offered, but here the user will require a high level of skill in selecting his or her alternatives.

� Any period of surplus or shortage from one day to five years can be accommodated.

� The markets offer a useful alternative to other sources of capital which the company may not wish to use at the time.

� US involvement in world trade makes a pool of dollars outside the home country most useful when:

(i) It is impossible or undesirable to transfer funds from the home country.

(ii) Projected currency movements suggest that it is advantageous to rearrange dollar commitments to parent companies, etc.

Eurobonds Available

There are four principal types of Eurobond and they are typically unsecured bearer securities in units of 5,000 and 10,000 US $, DM, etc. Each is described briefly below:

� Fixed rate (straight) bonds

The majority of Eurobonds with a fixed coupon can be described thus. Interest is generally paid out once each year and is calculated using a 360-day year. Some older issues do pay half-annually.

Page 128: libvolume5.xyzlibvolume5.xyz/law/ballb/3rdyear/semester5/corporatelaw/corporate... · ABE Advanced Diploma in Business Administration Study Manual CORPORATE FINANCE Contents Study

120 Sources of Company Finance

© Licensed to ABE

Whilst a 15-year term is typically the norm, maturity may range from three to 25 years. Shorter maturity bonds (up to five years) are usually referred to as notes and issues from the Netherlands are often within this period.

� Floating rate bonds (notes)

The benefit of the floating rate is that there will be a greater allowance for inflation and changing interest rates. Most bonds in this category have maturities between five and seven years, although there is no reason why longer periods cannot be taken.

� Option Eurobonds

These are fixed rate bonds with an option for the holder to receive principal and interest in a different currency from that of the bond.

� Convertible Eurobonds

These contain the right to convert a fixed interest bond into equity capital and are regularly used by the Japanese.

Although these are generally the most common instruments, other bonds we should mention here include:

� Deep discount and zero coupon bonds

These produce either a low or zero rate of interest to investors. To be attractive to investors, they are offered at a discount to their redemption value. The value is therefore in the future capital gain arising during the period between issue and redemption. Deep gain securities are another form of this type of security. The deep discount accrues over the life of the security at compound interest.

� Eurobonds with warrants

These are similar to convertible bonds in that warrants are attached to the Eurobond enabling the holder to purchase ordinary shares represented by the warrants at, or between, specified dates. The exercise price will be set in a similar fashion to that of convertible Eurobonds.

As well as notes and bonds denominated in the usual currencies, there are those with Special Drawing Rights (SDRs) and in European Currency Units (ECUs).

Borrowing in the Euromarket

Borrowers in the Euromarket include:

� Companies needing dollars for investment in the USA.

� Unit trusts and investment trusts investing in foreign securities.

� The United States banks, which find that it is expedient to take up loans through the European market rather than to borrow in the USA.

� Multinational companies wishing to invest in a particular country without wishing, or being able, to transfer capital from their base country.

� National governments and bodies associated with national and international agencies.

Euroequity This is the issue of equity in a stock market outside the company’s domestic stock market. A lack of sufficiently active secondary markets limits Euroequity markets (as you can see by the poor showing of the attempted issues by US and Japanese companies in European markets). Hence they are not as

Page 129: libvolume5.xyzlibvolume5.xyz/law/ballb/3rdyear/semester5/corporatelaw/corporate... · ABE Advanced Diploma in Business Administration Study Manual CORPORATE FINANCE Contents Study

Sources of Company Finance 121

© Licensed to ABE

popular as Eurobonds. This has led to the use of sweeteners in an attempt to sell Euroequity, including the issuing of bonds with warrants attached.

Choice of Currency for Borrowing The factors affecting the choice of currency used for company borrowing include:

� The ease and speed of raising finance, which is often easier outside the domestic markets.

� The size of the debt – larger loans tend to be borrowed in the Euromarkets.

� The cost of issues – small changes in interest rates can have a significant impact if the loan is large.

� Whether the currency is required immediately (including coverage in preventing exchange exposure) and in the long run.

� The security the company has available – Euroborrowings are generally unsecured.

Advantages of Raising Funds in International Markets You will realise from the above that there are several advantages to the company in raising funds in international markets, including:

� It is useful if the company’s own capital market is too small, or too complex and/or regulated to raise the required finance quickly and easily.

� It improves the liquidity and reputation of the company.

� It may improve trade if the company trades in the country of the currency/capital market.

� It can help in preventing takeover bids.

G. FINANCE AND THE SMALLER BUSINESS

Small businesses generally rely on retained earnings, bank borrowings and (limited) issuing of shares to private shareholders. They are often criticised as being too dependent on short-term finance, but only because they often experience difficulty in raising finance due to a perceived higher risk of failure.

Several initiatives have been taken to try and solve the funding gap, a sample of which are discussed below. Whilst your examiner will not expect you to be able to demonstrate intimate knowledge of every source of assistance, those described below are given to give you some insight into the variety of assistance available.

Note that the topic is continually changing and you should supplement your studies by your reading of the financial press.

Government Measures

� Enterprise Investment Scheme (EIS)

This includes measures to allow qualifying (i.e. those unconnected with the company two years prior to, and five years after) individual subscribers, in qualifying companies, to subscribe for eligible shares to a limit of £1m per company and obtain tax relief at 20% on the subscribed amount. The scheme is limited to £150,000 per year for each individual. Individuals are exempt from capital gains tax if they hold the shares for five or more years. The scheme also

Page 130: libvolume5.xyzlibvolume5.xyz/law/ballb/3rdyear/semester5/corporatelaw/corporate... · ABE Advanced Diploma in Business Administration Study Manual CORPORATE FINANCE Contents Study

122 Sources of Company Finance

© Licensed to ABE

permits “business angels” to become paid directors of companies they invest in and to obtain tax relief.

� Small Loans Guarantee Scheme

The scheme is a joint venture between the DTI and several of Britain’s high street banks. The maximum loan available for viable projects is £250,000 with 85% guaranteed by the Government, but it is not available to small local services, e.g. grocery shops.

There is a premium on the cost of borrowing and business assets may be used as security.

Other government aid includes the Scottish and Welsh Development Agencies, European Union assistance, local authority grants and the Enterprise Investment Scheme discussed above.

Grants Grants are available to all businesses, most of which carry a test relating to the number of new jobs created from the project or development requiring assistance. The second test for grant assistance will usually be that the project cannot proceed without financial assistance. Grants are available from a range of institutions including governmental bodies and the Prince’s Youth Business Trust.

Banks Whilst banks will usually be willing to lend a degree of support, without the ability to offer some tangible security the business may have to seek finance from elsewhere.

Venture Capital Providers Venture capital specialists may be willing to participate in smaller businesses, but generally only show an interest in projects which require fairly substantial working capital and where there is a planned exit route for them to realise their investment at some agreed future date.

Finance Companies and Lessors Finance companies typically specialise in providing financial accommodation in respect of fixed assets. Since they generally retain title to the assets throughout the term of the contract they do not usually seek additional security. They may, however, seek directors’ personal guarantees when the directors of a small business are also the principal shareholders.

Practice Question 2

Redbrick plc’s 10% convertible loan stock is quoted at £138 per £100 nominal. The earliest date for conversion is in three years time at the rate of 25 ordinary shares per £100 nominal loan stock. The current share price is £5.20.

(a) Calculate the conversion price;

(b) Calculate the conversion premium.

Now check your answer with the one given at the end of the unit.

Page 131: libvolume5.xyzlibvolume5.xyz/law/ballb/3rdyear/semester5/corporatelaw/corporate... · ABE Advanced Diploma in Business Administration Study Manual CORPORATE FINANCE Contents Study

Sources of Company Finance 123

© Licensed to ABE

ANSWERS TO PRACTICE QUESTIONS

Practice Question 1

(a) Investor who holds 60,000 ordinary shares (5% equity)

Present position If shareholder takes up rights If shareholder sells rights

Value of investment: 60,000 O/S × £3.60 = £216,000 60,000 O/S × £3.60 = £216,000 60,000 O/S × £3.60 = £216,000 15,000 O/S × £3.00 = £45,000 less 60,000 × 12p* = £(7,200)

£261,000 £208,800 Dividend received: 60,000 × 45p = £27,000 75,000 × 43.5p = £32,625 60,000 × 43.5p = £26,100 % Return on investment (net):

12½% 12½% 12½%

* Theoretical ex-rights price would be 000,500,1000,220,5£ shares = £3.48.

Thus the value of one right = 4p48

= 12p

The total value of £5,220,000 derives from:

£ Value of shares on issue 4,320,000 (1,200,000 × £3.60) Value of rights issue 900,000 (300,000 × £3)

5,220,000

The present net dividend yield is 000,320,4000,200,1£ × 45p = 12.5p

The future net dividend yield is 000,220,5000,500,1£ × 43.5p = 12.5p

From these calculations, the shareholder will be no better or worse off after the rights issue and his or her action, therefore, can be determined on whether he or she wishes to continue to be involved in the company to the same extent or to increase or decrease his or her overall holding.

(b) The idea of a rights issue is to encourage existing shareholders to invest more funds in a company so as not to dilute further the membership interest. To do so, a discount on the market price is built into the price ruling on the market. To price the rights issue above market price would mean it would be cheaper to go to the market to purchase the shares, and the rights issue would fail.

Page 132: libvolume5.xyzlibvolume5.xyz/law/ballb/3rdyear/semester5/corporatelaw/corporate... · ABE Advanced Diploma in Business Administration Study Manual CORPORATE FINANCE Contents Study

124 Sources of Company Finance

© Licensed to ABE

(c) The total sum to be raised = £900,000, made up as follows:

4

1,200,000 × £3 = £900,000 (300,000 new shares)

Therefore, the underwriting commission will be £900,000 × 1¼% = £11,250.

It would have been possible to price the rights issue below that, at say £2.50, to raise the same sum on a 3 for 10 issue at £2.50 as follows:

10

1,200,000 × 3 × £2.50 = £900,000 (360,000 new shares).

The underwriting commission will still be:

£900,000 × 1¼% = £11,250

This will be payable whether the issue is fully subscribed or not.

An underwriter agrees to take up the remnants of an issue if it is not altogether successful, which ensures that the company raises the sum required, i.e. the, albeit small, element of risk associated with the failure of the issue is insured against.

It would be argued that a slightly lower price could be offered if underwriting commission were less or if the issue were not underwritten, but whether such a minimally lower price would influence the success of the issue enough to ignore the risk is unlikely. Underwriting commission is usually between 1¼% and 2%.

Practice Question 2

(a) The conversion price is the amount of stock required to obtain each ordinary share.

For Redbrick plc this is:

25100£ = £4

£4 of stock is necessary for each share.

(b) The conversion premium is:

25138 −−−− 5.20 = 5.52 −−−− 5.20

= £0.32 per share.

Page 133: libvolume5.xyzlibvolume5.xyz/law/ballb/3rdyear/semester5/corporatelaw/corporate... · ABE Advanced Diploma in Business Administration Study Manual CORPORATE FINANCE Contents Study

125

© Licensed to ABE

Study Unit 6

Cost of Finance

Contents Page

Introduction 127

A. Investors and the Cost of Capital 127 The Required Rate of Return 127 The Effect of Risk 127

B. Cost of Equity 128 Dividend Yield (or Dividend Valuation) Method 128 Cost of Preference Shares 130 Retained Earnings 130 Capital Asset Pricing Model 131

C. Cost of Debt Capital 131 Irredeemable Debt 131 Redeemable Capital 131 Cost of Floating Rate Debt 133 Cost of Convertibles 133 Cost of Fixed Rate Bank Loans 134 Cost of Short-term Funds and Overdrafts 134

D. Cost of Internally Generated Funds 135

E. Weighted Average Cost of Capital 136 Methodologies 136 Assumptions When Using WACC 138 Arguments Against Using the WACC 138

(Continued over)

Page 134: libvolume5.xyzlibvolume5.xyz/law/ballb/3rdyear/semester5/corporatelaw/corporate... · ABE Advanced Diploma in Business Administration Study Manual CORPORATE FINANCE Contents Study

126 Cost of Finance

© Licensed to ABE

F. Assessment of Risk in the Debt Versus Equity Decision 138 Effect on Market Value 138 Break-even Profit Before Interest and Tax 139

G. Cost of Capital for Other Organisations 140 Unquoted Companies 140 Not-For-Profit Organisations 140

Answers to Practice Questions 142

Page 135: libvolume5.xyzlibvolume5.xyz/law/ballb/3rdyear/semester5/corporatelaw/corporate... · ABE Advanced Diploma in Business Administration Study Manual CORPORATE FINANCE Contents Study

Cost of Finance 127

© Licensed to ABE

INTRODUCTION

It is essential for a company to know the cost of the various types of funds included in its capital structure in order to satisfy the terms of the providers of that capital – the investors. If the investors are not satisfied with their returns they may remove their money from the firm. The required rate of return to investors is how much the company has to pay to obtain its finance – i.e. the cost of capital to the firm.

In addition, it is important for companies to know their cost of capital in order to ensure that projects they invest in achieve the level of return required to satisfy those who provide funds which finance the project.

We will consider initially the costs of the different types of capital individually, before looking at the cost of the capital structure of the company as a whole.

It is important when you are revising this area to also consider the capital asset pricing model (CAPM) that we will discuss later in the course.

A. INVESTORS AND THE COST OF CAPITAL

It is important to remember that the cost of finance to the firm is the return required by investors in the firm.

The Required Rate of Return The required return of investors will be determined by the opportunity cost foregone of the investors, and the risk they are required to bear. For example, if the return on a company’s ordinary shares is 8% with no real prospect of capital growth in the short term, and a building society deposit will yield 10%, it is unlikely that the shares will be attractive to investors – the opportunity cost (the building society rate) is greater than the projected return, and the risk of investing in a company is generally considered to be higher than that of depositing the money in a building society account.

The Effect of Risk Investing in companies involves risk – investors may lose some or all of their funds placed in the company. The risk comprises two elements:

� Business risk due to a lack of certainty about the firm’s prospects and projects, and

� Financial risk – the higher the level of gearing the higher the risk of bankruptcy.

The return investors require depends on the level of risk – the higher the risk the higher the expected return, because people expect to be compensated for bearing additional risk. This “risk premium” (made up of premiums for business and financial risks) will be required in addition to the risk-free rate of return which is the basic return which would be required if there was no risk. (The risk-free rate is typically taken to be the return on government bonds.)

Thus the return required from investing in different companies will vary with the differing levels of risk involved in the firms.

Page 136: libvolume5.xyzlibvolume5.xyz/law/ballb/3rdyear/semester5/corporatelaw/corporate... · ABE Advanced Diploma in Business Administration Study Manual CORPORATE FINANCE Contents Study

128 Cost of Finance

© Licensed to ABE

B. COST OF EQUITY

The requirements and expectations of shareholders must be considered when looking at the cost of equity. The effect that changes in earnings and dividends may have on the share price must also be considered.

Dividend Yield (or Dividend Valuation) Method The main method of calculating the cost of equity is the dividend valuation or dividend yield model. The precise form of model used varies with the assumptions used. The simplest model to use assumes that dividends will remain at a constant level in the future.

The value of a company’s shares can be calculated using the formula we discussed earlier:

Market value = shares on the yield)(or return Expected

pencein payable dividendCurrent

The formula can be rearranged for use in calculating the cost of equity as follows:

Expected return (or yield) on the shares = ueMarket val

pencein payable dividendCurrent

The expected return (or yield) on the shares is the cost of equity, and the market value of the shares is the current ex div share price (i.e. the share price after the dividend has been paid).

This is often written as:

Ke = SeDe

where: Ke = cost of equity

De = current dividend payable

Se = current share price (ex div).

Example 1

Tigger plc’s current dividend is 25p and the market value of each share is £2. What is the cost of Tigger’s equity?

Using the above formula Ke = SeDe :

Ke = 20025 = 0.125 = 12.5%

However, shareholders prefer a constant growth in their dividends. In order to reflect this in the dividend valuation method, we have to predict future growth in dividends. Growth generally reflects predicted changes in a company’s earnings, although it is difficult to decide the level of growth that will be sustained in future years. The most usual approach is to take several years’ historical data and then attempt to extrapolate forward.

Page 137: libvolume5.xyzlibvolume5.xyz/law/ballb/3rdyear/semester5/corporatelaw/corporate... · ABE Advanced Diploma in Business Administration Study Manual CORPORATE FINANCE Contents Study

Cost of Finance 129

© Licensed to ABE

Example 2

Assume Pooh’s past dividends have been:

Dividend per Share

Year 1 0.26

Year 2 0.27

Year 3 0.28

Year 4 0.32

We can now find the average rate of growth by using the following formula:

Growth rate (g) ==== ndividendEarliest

dividendLatest − 1

where: n = number of years growth.

Applying this to the above figures, we get:

g = 326.032.0 − 1 = 0.0717 or approximately 7%.

Note that here we are using the cube root because there are three years of growth. If we had been given five years’ data (from which we could project four years’ growth) the fourth root would have been used.

When the expected growth figure has been determined we can calculate the value of the company’s shares using the Dividend Growth Model or Gordon’s Model of Dividend Growth.

This model states:

Po = dog) r(g) (1

−+ which is also written as Se = g) (Ke

g) + (1 De−

where: Po or Se = the current ex dividend market price

do or De = the current dividend

g = the expected annual growth in dividends

r or Ke = the shareholder’s expected return on the shares

and can be rewritten as:

Ke = Seg) + (1 De + g

Note that growth, if given, is usually be expressed as a percentage.

Example 3

Using the example of Pooh above, and assuming its share price is £2.50, then:

Ke = 2.50

)0.32(1.072 + 0.072 = 0.137 + 0.072 = 20.9%.

The dividend valuation and dividend growth model are based on the following assumptions:

� Taxation rates are assumed to be constant across all investors, and as such the existence of higher rates of tax are ignored. The dividends used are the gross dividends paid out from the company’s point of view.

Page 138: libvolume5.xyzlibvolume5.xyz/law/ballb/3rdyear/semester5/corporatelaw/corporate... · ABE Advanced Diploma in Business Administration Study Manual CORPORATE FINANCE Contents Study

130 Cost of Finance

© Licensed to ABE

� The costs of any share issue are ignored.

� All investors receive the same, perfect level of information.

� The cost of capital to the company remains unaltered by any new issue of shares.

� All projects undertaken as a result of new share issues are of equal risk to that existing in the company.

� The dividends paid must be from after-tax profits – there must be sufficient funds to pay the shareholders from profits after tax.

Share Issue Costs

Share issue costs can be incorporated in the formula, especially if they are considered to be high. The formula then becomes:

Ke = I) (SeDe

where: Ke = cost of equity

De = current dividend payable

Se = current share price (ex div)

I = cost of issue per share.

Thus, for the example of Tigger above, if issue costs divided by the number of shares is 5p, then the cost of equity becomes:

Ke = 5 20025

− = 0.128 = 12.8%

If you are given issue costs you should, unless told otherwise, incorporate them in the formula as shown above.

Cost of Preference Shares The formula for calculating the cost of preference shares is:

Kp = SpDp

where: Kp = cost of preference shares

Dp = fixed dividend based on the nominal value of the shares

Sp = market price of preference shares

Example

Anorak plc has 8% preference shares which have a nominal value of £1 and a market price of 80p. What is the cost of preference shares?

Using the above formula, dividends are 8% of the nominal value of £1 and as such are 8p. Therefore:

Kp = 808 = 10%.

Retained Earnings Retained earnings will also have an effect because, when left in the business rather than being distributed, they should achieve higher returns in the future to offset the lack of current dividends. The

Page 139: libvolume5.xyzlibvolume5.xyz/law/ballb/3rdyear/semester5/corporatelaw/corporate... · ABE Advanced Diploma in Business Administration Study Manual CORPORATE FINANCE Contents Study

Cost of Finance 131

© Licensed to ABE

shareholders’ expectations of increasing future dividends rather than constant payments may, however, persuade them to accept initial lower dividends.

Capital Asset Pricing Model The Capital Asset Pricing Model, which we shall discuss later in the course, can also be used to value the cost of equity. You should revise this topic when you have worked through the relevant unit.

C. COST OF DEBT CAPITAL

We saw earlier that debentures can be either redeemable or irredeemable. It is important that you know the type of debenture a firm has in issue when calculating its cost of capital because, as you will see, the approach used varies with the form of debentures being considered.

Irredeemable Debt The formula for calculating the cost of irredeemable debt is:

Kd = Sd t) I(1−

where: Kd = cost of debt capital

I = annual interest payment

Sd = current market price of debt capital

t = the rate of corporation tax applicable.

Taxation is considered because the interest can be offset against taxation, which will lower its nominal rate, and thus its cost. The higher the rate of corporation tax payable by the company, the lower will be the after-tax cost of debt capital. Thus the cost of debt capital is much lower than the cost of preference shares with the same coupon rate and market value as the debentures because of the availability of tax relief on the debt. Naturally this only applies if the business has taxable profits from which to deduct its interest payments. When the business has generated a taxable loss, the interest will increase that loss for carry forward (to be offset against future taxable profits in later years), and the immediate benefit of tax relief will be lost.

Example

Clown plc has £10,000 of 8% irredeemable debentures in issue which have a current market price of £92 per £100 of nominal value. If the corporation tax rate is 33% what is the cost of the debt capital?

The annual interest payment will be based on the nominal value, i.e. 8% of £10,000 or £800, so using the above formula:

Kd = 10,000 92/1000.33) 800(1

×− = 0.0583 = 5.83%.

Redeemable Capital In order to determine the cost of such capital to the date of redemption we must find the internal rate of return (IRR). IRR is discussed in greater detail later, but basically involves calculating all the necessary cash flows (generally the assumption will be made that all payments and receipts are made at the end of the year) and determining at what cost of capital the value of future cash flows are equal to zero. The IRR is calculated using discount factors for the appropriate cost of capital and the following formula. Wherever possible the ex-interest values should be used, so if the cum-interest

Page 140: libvolume5.xyzlibvolume5.xyz/law/ballb/3rdyear/semester5/corporatelaw/corporate... · ABE Advanced Diploma in Business Administration Study Manual CORPORATE FINANCE Contents Study

132 Cost of Finance

© Licensed to ABE

value is quoted (i.e. if the interest is due to be paid soon and thus is reflected in the market price of the debt) we should deduct the interest payment from the market price. The longer the period to maturity, the lower will be the overall cost of capital. This is to be expected because the real cost of redemption will be lower in the future because of the effects of the time value of money. (Do not worry if this appears complicated at this stage since we shall explain the IRR fully later in the course.)

Example

In this example, Clown’s rate of corporation tax is assumed to be 33% throughout and redemption is in 20x5. The following table sets out the workings on which the calculation is based.

Note: df = discount factor

Year Amount df 6% df 12% a b c b × c d b × d

Current market price 20x1 (94.0) 1.00 (94.00) 1.00 (94.00) Interest 20x2 10.0 0.94 9.40 0.89 8.90 Tax saved 20x3 (3.3) 0.89 (2.94) 0.80 (2.64) Interest 20x3 10.0 0.89 8.90 0.80 8.00 Tax saved 20x4 (3.3) 0.84 (2.77) 0.71 (2.34) Interest 20x4 10.0 0.84 8.40 0.71 7.10 Redemption (1.1.x5) 20x4 100.0 0.84 84.00 0.71 71.00 Tax saved 20x5 (3.3) 0.79 (2.61) 0.64 (2.11 )

8.38 (6.09 ) The cost of capital is therefore:

6 +

−×+

)612(6.09) (8.388.38 by interpolation

= 9.47% (that is the IRR)

If redemption occurs three years later in 20x7, the cost of capital changes as shown on the next page.

Page 141: libvolume5.xyzlibvolume5.xyz/law/ballb/3rdyear/semester5/corporatelaw/corporate... · ABE Advanced Diploma in Business Administration Study Manual CORPORATE FINANCE Contents Study

Cost of Finance 133

© Licensed to ABE

Year Amount df 6% df 12% a b c b × c d b × d

Current market price 20x1 (94.0) 1.00 (94.00) 1.00 (94.00) Interest 20x2 10.0 0.94 9.40 0.89 8.90 Tax saved 20x3 (3.3) 0.89 (2.94) 0.80 (2.64) Interest 20x3 10.0 0.89 8.90 0.80 8.00 Tax saved 20x4 (3.3) 0.84 (2.77) 0.71 (2.34) Interest 20x4 10.0 0.84 8.40 0.71 7.10 Tax saved 20x5 (3.3) 0.79 (2.61) 0.64 (2.11 ) Interest 20x5 10.0 0.79 7.90 0.64 6.4 Tax saved 20x6 (3.3) 0.75 (2.47) 0.57 (1.88) Interest 20x6 10.0 0.75 7.50 0.57 5.7 Redemption (1.1.x7) 20x6 100 0.75 75.00 0.57 57.0 Tax saved 20x7 (3.3) 0.70 (2.31) 0.51 (1.68)

10.00 (11.55) The cost of capital is:

6 +

−×+

)612(11.55) (10.0010.00

= 8.78% (i.e. the IRR).

Cost of Floating Rate Debt If a company has floating rate debt in its capital structure, then an estimated fixed rate of interest should be used to calculate its cost of debt in a way similar to the above. The “equivalent” rate will be that of a similar company for a similar maturity.

Cost of Convertibles To determine the cost of convertibles we have to find the internal rate of return (IRR) of the following equation:

P0 = r)(1t)K(1

+− + 2r)(1

t)K(1+

− + 3r)(1t)K(1

+− + . . . . nr)(1

t)K(1+

− + nn

r)(1CRV

+

where: P0 = current market price of the convertible ex interest (i.e. after paying the current year’s interest)

K = annual interest payment

t = rate of corporation tax

r = cost of capital

Vn = market value of the shares at year n

CR = conversion ratio.

It is also a useful exercise to calculate the cost of convertibles as though they were not converted – if the cost is higher the holders will choose not to convert, because not converting produces a higher

Page 142: libvolume5.xyzlibvolume5.xyz/law/ballb/3rdyear/semester5/corporatelaw/corporate... · ABE Advanced Diploma in Business Administration Study Manual CORPORATE FINANCE Contents Study

134 Cost of Finance

© Licensed to ABE

return to the investor. The higher cost of capital should therefore be the one used in the calculation of the company’s cost of capital.

Example

Quality plc has 10% convertible debentures due for conversion in two years’ time. They have a current market value of £108 per cent. The conversion terms are 5 shares per £10 of debentures. All the debenture-holders are expected to convert, and the shares are expected to have a price of £4 at this time. What is the cost of capital?

For ease of calculation, we shall assume the rate of corporation tax is 50% and is payable in the same year.

The market value of £108 per cent means that it is £108. Interest on convertibles is offsettable against tax, and thus is shown as a saving in the following calculation:

Year 0 1 2 £ £ £

Current market value of convertibles (108) Interest 10 10 Tax relief (5) (5) Value of shares on conversion: ((5 × 100/10) × £4)

200

Total yearly cash flow (108) 5 205 Now apply an estimate of the cost of capital – say 40%:

Year 0 1 2 £ £ £

Total yearly cash flow (108) 5.000 205.000 Discount factor at 40% 1 0.714 0.510 Present value (108) 3.570 104.550

Net present value = (108) + 3.57 + 104.55 = 0.12

Thus the cost of capital is just over 40% – the precise level could be found using the interpolation formula given above.

Again do not worry if the mechanics seem a little complicated since they will be covered in greater detail later in the course.

Cost of Fixed Rate Bank Loans The cost of this major source of finance is given by:

Cost = Interest rate = (1 − t)

Cost of Short-term Funds and Overdrafts The cost of short-term bank loans and overdrafts is the current interest rate being charged on the capital lent.

Page 143: libvolume5.xyzlibvolume5.xyz/law/ballb/3rdyear/semester5/corporatelaw/corporate... · ABE Advanced Diploma in Business Administration Study Manual CORPORATE FINANCE Contents Study

Cost of Finance 135

© Licensed to ABE

D. COST OF INTERNALLY GENERATED FUNDS

Internally generated funds typically represent around 60% of all sources of capital available to a business. The principal benefit of using internal funds is derived from the fact that there are no formalities to their acquisition, but it will often be difficult to generate the optimum amount at exactly the time the business needs additional funding.

Whilst internally generated funds avoid the formal costs of issue, e.g. issuing house fees, they are not free of cost to the company.

Retained earnings (e.g. provisions, retentions) belong to the shareholders and, in order to justify their retention, the company must be able to earn a return in excess of that which the shareholders could earn before tax had they been distributed to them. Thus there is an opportunity cost related to the cost of retentions. When the company is unable to meet that rate, it has an obligation to distribute its retentions to its shareholders, allowing them to obtain better returns on their investments.

We can show an example, using a comparison between two companies:

Example

Company X pays out most of its earnings, whereas Company Y retains a high percentage.

Company X £ Company Y £ Year 1 Profits 200,000 Profits 200,000 less Dividend 160,000 less Dividend 20,000

Balance c/f 40,000 Balance c/f 180,000 In Year 2, the capital needs of both companies are an additional £200,000. X obtains equity of £160,000 and Y equity of £20,000. Assume dividends of 10% on new capital.

Company X £ Company Y £ Year 2 Profits (year 2) 200,000 Profits (year 2) 200,000 less Dividends: £ less Dividends: £ On existing capital 160,000 On existing capital 20,000 On new capital 16,000 176,000 On new capital 2,000 22,000

Balance c/f 24,000 Balance c/f 178,000 Suppose in Year 3 profits fell sharply to £100,000 for each company. The following would be the result.

Company X £ Company Y £ Year 3 Profits 100,000 Profits 100,000 less Dividend (halved) 88,000 less Dividend (doubled) 44,000

Balance c/f 12,000 Balance c/f 56,000 What do these figures mean? That Y is more efficient than X? No, because profits each year have been the same, the only difference being that Y obtains large amounts of “cost-free” capital, whereas X is paying out most of its profits as it has to pay for its capital in the form of a dividend.

Page 144: libvolume5.xyzlibvolume5.xyz/law/ballb/3rdyear/semester5/corporatelaw/corporate... · ABE Advanced Diploma in Business Administration Study Manual CORPORATE FINANCE Contents Study

136 Cost of Finance

© Licensed to ABE

Is Y able to weather the storm better than X? Yes, because it has a large balance, made possible by its low pay-out ratio. It has been able to double dividends to shareholders despite reduced profits in Year 3.

Sooner or later the shareholders of Company Y will realise they are losing out, to the benefit of the company itself.

From this two important principles emerge:

� All capital has a cost.

� Even retained profits should carry a cost (an implied or imputed cost).

Here the opportunity cost is concept related to the cost of retentions that we noted earlier.

An alternative approach is offered by G.D. Quirin in “The Capital Investment Decision”, where he suggests that the change in share price following the retention of profits must equal the capitalised value of the potential dividend increase which the shareholder has forgone in order for the retentions to have been made. By observing share price movements following the retention of profits, the rate of share price change can be used to calculate the capitalisation (i.e. the cost) rate attached to the retention by the market.

The underlying problem of quantifying human behaviour is again present in this hypothesis and therefore limits this method. For this reason, perhaps the opportunity cost method is preferable. Shareholder behaviour continues to be an area for future research.

E. WEIGHTED AVERAGE COST OF CAPITAL

Companies tend to have a mixture of the different types of capital in their structure, and when considering the cost of capital used to finance a project it is common to use the cost of the mix of capital held by the company – the weighted average cost of capital (WACC).

The cost of capital that should be used in evaluating projects is the marginal cost of the funds raised in order to finance the project, and WACC is considered to be the best estimate of marginal cost (the capital structure of a company changes slowly over time). Note, however, that it is only the most reliable if the company is assumed to continue investment in projects of a normal level of business risk and funds are raised in similar proportions to its existing capital structure.

The weighted average cost of capital is the average of costs of the different types of finance in a company’s structure weighted by the proportion of the different forms of capital employed within the business. The financial manager will therefore need to ensure that any project which is under consideration will produce a return that is positive in terms of the business as a whole and not just in terms of an issue of capital made to finance it. Investments which offer a return in excess of the WACC will increase the market value of the company’s equity, reflecting the increase in expected future earnings and dividends arising as a result of the project.

Methodologies There is no one accepted method of calculating a company’s WACC – some use book values in the proportions that they appear in the company’s accounts and some use market values. For unquoted companies book values may have to be used because of the problems we discussed in earlier study units of estimating market values for their securities. Book values are generally easier to obtain than market values. However, many would argue that for quoted companies market values are more realistic and, indeed, may be easier because only one cost of equity is required – it being impossible to split the value of equity between the shares and the retained earnings.

Page 145: libvolume5.xyzlibvolume5.xyz/law/ballb/3rdyear/semester5/corporatelaw/corporate... · ABE Advanced Diploma in Business Administration Study Manual CORPORATE FINANCE Contents Study

Cost of Finance 137

© Licensed to ABE

Example

(a) Using book values in the proportions that they appear in the company’s accounts:

Weighting Cost Weighted Cost

Ordinary shares 60% 12% 7.2% Debentures 40% 8% 3.2%

WACC 10.4%

(b) Using market values:

Number Price Market Value Cost Weighted Market Value

Ordinary shares 6,000 2.50 15,000 12% 1,800 Debentures 4,000 1.50 6,000 8% 480

21,000 2,280 The WACC is then calculated as:

21,0002,280 = 10.86%

Both methods produce the historic WACC and you should remember that raising fresh capital could well alter the weighting and therefore the cost of capital, the model assuming that new investments are financed by new funds. A change in the risk level of the company will also affect the cost of a company’s capital.

You may also come across the following formula for use in calculating a company’s WACC:

WACC = Keg

+ D) (E

(E) + Kd (1 −−−− t)

+ D) (E

(D)

where: Keg = is the cost of equity

Kd = the cost of debt

E = the market value of the company’s equity

D = the market value of the company’s debt

t = the rate of corporation tax applicable to the company.

The model assumes that debt is irredeemable.

The model is simply another approach to calculating a company’s cost of capital. It is often considered easier by students, but you should use whichever method you feel the most comfortable with. If you wish to calculate WACC using the above formula when the debt in the company’s structure is redeemable then you should calculate the cost of debt using the methods above and replace Kd(1 − t) in the formula with the answer.

Page 146: libvolume5.xyzlibvolume5.xyz/law/ballb/3rdyear/semester5/corporatelaw/corporate... · ABE Advanced Diploma in Business Administration Study Manual CORPORATE FINANCE Contents Study

138 Cost of Finance

© Licensed to ABE

Assumptions When Using WACC To use WACC in capital investment appraisal the following assumptions have to be made:

� The cost of capital used in project evaluation is the marginal cost of funds raised in order to finance the project.

� New investments must be financed from new sources of funds, including new share issues, new debentures or loans.

� The weighted average cost of capital must reflect the long term future capital structure of the company.

Arguments Against Using the WACC There are arguments against using WACC for investment appraisal based mainly on the assumptions underlying WACC.

� Businesses may have floating rate debt whose cost changes frequently, and as we have seen only an estimate is used to calculate the cost of this type of finance. Thus the company’s cost of capital will not be accurate and will need frequent updating.

� The business risk of individual projects may be different to that of the company and will thus require a different premium included in the cost of capital.

� The finance used for the project may alter the company’s gearing and thus its financial risk.

F. ASSESSMENT OF RISK IN THE DEBT VERSUS EQUITY DECISION

Effect on Market Value The direct cost of borrowing is represented by the interest charges and fees which are applied by the lender. In common with debenture interest, such charges will generally be deductible for tax purposes, and therefore the after-tax cost of borrowing will usually be less than the gross cost. Although the cost of borrowing will by and large appear cheaper than equity, there is a risk to the company and the financial manager should take this into account when comparing the costs of borrowing.

Study the following example.

Example

A company has a current profit before interest and tax (PBIT) of £5m pa and current interest payable of £1.7m. The company’s issued share capital comprises £10m in ordinary shares and the earnings per share (EPS) are 5p.

The firm wishes to invest £7.5m of new capital and it expects to increase its PBIT by £1.25m pa as a result. The alternatives under consideration by the directors are as follows:

(a) To issue 3.75 million shares at 200p, representing a discount on the current market price of 240p.

(b) To borrow £7.7 million on 10-year debentures at 12% annual interest.

Assume a corporation tax rate of 33% although note that current rates will vary from this rate.

Page 147: libvolume5.xyzlibvolume5.xyz/law/ballb/3rdyear/semester5/corporatelaw/corporate... · ABE Advanced Diploma in Business Administration Study Manual CORPORATE FINANCE Contents Study

Cost of Finance 139

© Licensed to ABE

One approach to decide on the better route would be to attempt to predict the effect on the market value of the ordinary shares. The company would then elect for the opportunity which gives the best return to shareholders (remember the dominant objective of financial management).

Table 6.1 below shows the effect on the earnings per share:

Table 6.1

Current Projected Equity

Projected Debt

(£m) (£m) (£m)

PBIT 5.00 6.25 6.25 Interest payable (1.70) (1.70) (2.62) Profit before tax 3.30 4.55 3.63 Tax at 33% (1.09) (1.50) (1.20)

Profit after tax 2.21 3.05 2.43

Issued ordinary shares 10m 13.75m 10m

Earnings per share 22.10p 22.18p 24.30p

From this we can see that the market value of the shares will be improved by choosing to raise the debt capital, on the assumption that the PBIT really does increase by £1.25m.

However, the financial manager should always remember that debt is a riskier route than equity, because:

� Debt payments cannot be deferred whereas dividends to shareholders can, should trading estimates fail to materialise.

� Use of debt capital could result in a lower price/earnings ratio than an equity issue.

In our example the financial gearing ratio would increase and the interest cover will fall from the present 2.94 to 2.4. (We shall consider gearing in detail later in the course.)

Interest cover should be calculated as the number of times the interest payable can be divided into the PBIT figure. Unequivocally, the higher the number of times, the better the result and the less risk will be attached to the decision.

A low figure, generally less than three times cover (when interest rates themselves are low), indicates that the company should be cautious regarding further borrowings if these are likely to be sensitive to adverse (upward) movements in interest rates, because the company’s ability to service the necessary payments may be in doubt.

Break-even Profit Before Interest and Tax The financial manager may choose to compute the break-even PBIT at which the earnings per share will be the same for the use of either equity or debt.

Using the same information as in the above example, this is done as follows:

EPS under debt = 10

2.62) 67%(y − = EPS under equity = 13.75

1.70) 67%(y −

(67% is used to represent the position net of tax at 33%, and y represents the break-even PBIT.)

Page 148: libvolume5.xyzlibvolume5.xyz/law/ballb/3rdyear/semester5/corporatelaw/corporate... · ABE Advanced Diploma in Business Administration Study Manual CORPORATE FINANCE Contents Study

140 Cost of Finance

© Licensed to ABE

13.75 (y − 2.62) = 10(y − 1.70)

13.75y − 36.02 = 10y − 17

∴ 3.75 y = 19.02

y = 5

This shows us that the break-even PBIT in our example is £5m. Earnings per share will be greater using debt above this level, but below it equity should be favoured. In practice, more than one source of financing may be used, and it will be important for the financial manager to consider the risks and rewards of the alternatives.

It is quite common for a company to lease a large part of its expenditure on capital items and to use equity for its increased working capital needs although, due to the costs involved, a quoted company will be unlikely to consider issuing less than £250,000 in new shares to be worthwhile. Whilst the calculations demonstrated in this study unit will be simpler to apply to quoted companies (because of the ease with which share prices can be determined) the underlying principles will be appropriate to all businesses seeking to increase the capital available for investment.

G. COST OF CAPITAL FOR OTHER ORGANISATIONS

Unquoted Companies Unquoted companies do not have market values for their shares and thus calculating the cost of equity can be difficult. To estimate an approximate cost of capital the firm can either use the cost of equity of a similar quoted company and adjust it for difference in gearing and business risk, or it could add estimated premiums for its financial and business risk to the risk-free rate given by government bonds.

Not-For-Profit Organisations Government departments do not have a market value, nor do they have business or financial risk, and thus cannot calculate the cost of capital. To evaluate projects they use a targeted “real rate of return” set by the Treasury as a cost of capital.

Non-profit making firms do not generally have market values, and will thus have to determine a cost of capital to use to assess projects – many use the cost of any borrowing they have in their balance sheets, but you will appreciate from this study unit that it is not ideal.

Page 149: libvolume5.xyzlibvolume5.xyz/law/ballb/3rdyear/semester5/corporatelaw/corporate... · ABE Advanced Diploma in Business Administration Study Manual CORPORATE FINANCE Contents Study

Cost of Finance 141

© Licensed to ABE

Practice Questions

1. A company has a share value of £1.27 (ex-div) and has recently paid a dividend of 8p per share. If dividend growth is expected to be approximately 3% per annum into the foreseeable future, calculate the cost of equity.

2. Calculate the WACC from the following information:

Balance Sheet Extract from CD plc

Capital Balance Sheet Value Market Value

Ordinary shares (20,000 – 50p ordinary)

£10,000

£1.72 per share

8% Preference shares (£1 nominal value)

£5,000

£0.98 per £1

Long-term liabilities 10% debentures

£7,500

£1.04 per £1

The cost of equity has been calculated at 9.5%.

Now check your answer with the one given at the end of the unit.

Page 150: libvolume5.xyzlibvolume5.xyz/law/ballb/3rdyear/semester5/corporatelaw/corporate... · ABE Advanced Diploma in Business Administration Study Manual CORPORATE FINANCE Contents Study

142 Cost of Finance

© Licensed to ABE

ANSWERS TO PRACTICE QUESTIONS

1. Ke = Se)g1(De + + g

= 27.1

)03.01(08.0 + + 0.03

= 0.065 + 0.03

= 0.095 or 9.5%

2. (a) WACC at Balance Sheet Values

Value Weighting Return Weighted Average Return

Equity 10,000 0.45 0.095 0.0428 Preference Shares 5,000 0.22 0.08 0.0176 Debentures 7,500 0.33 0.10 0.0330

22,500 0.0934

WACC ==== 9.34%

(b) WACC at Market Values

Value Weighting Return Weighted Average Return

Equity 34,400 0.73 0.095 0.069 Preference Shares 4,900 0.10 0.08 0.008 Debentures 7,800 0.17 0.10 0.017

47,100 0.094

WACC ==== 9.40%

Note that strictly speaking the return should be recalculated in line with the market values if the information is available.

Page 151: libvolume5.xyzlibvolume5.xyz/law/ballb/3rdyear/semester5/corporatelaw/corporate... · ABE Advanced Diploma in Business Administration Study Manual CORPORATE FINANCE Contents Study

143

© Licensed to ABE

Study Unit 7

Portfolio Theory and Market Efficiency

Contents Page

Introduction 144

A. Risk and Return 144 Expected Return 144 Measuring Risk in a Portfolio 145

B. The Impact of Diversification 146 Correlation 146 Assessing Risk under Different Forms of Correlation 147 The Importance of Correlation 149

C. Portfolio Composition 150 Investment Indifference Curves 150 Efficiency Frontier 152 The Capital Market Line 153 Efficient and Inefficient Portfolios 155 Securities Market Line 157

D. The Application of Portfolio Theory 157 Planning Diversification 157 Selected versus Random Portfolios 158 Practical Difficulties 158 Limitations of Portfolio Theory 159

E. Market Efficiency 159 Fundamental Analysis Theory of Share Values 160 Technical Analysis 161 Random Walk Theory 164 Efficient Markets Hypothesis 164 Alternatives to the EMH 166

Answers to Practice Questions 169

Page 152: libvolume5.xyzlibvolume5.xyz/law/ballb/3rdyear/semester5/corporatelaw/corporate... · ABE Advanced Diploma in Business Administration Study Manual CORPORATE FINANCE Contents Study

144 Portfolio Theory and Market Efficiency

© Licensed to ABE

INTRODUCTION

Portfolio theory is a model which provides investors with an insight into the required rates of return for a given level of risk.

A portfolio is the collection of investments or projects which an individual holds. A company will generally also have a series of projects in its portfolio, and may make external investments (in the financial sense) in some other investment market, when it has surplus funds available. In order to do so effectively, and to understand investor behaviour in the management of his firm’s own share price, the financial manager requires a good working knowledge of the “City”, its efficiency and of portfolio theory.

In order to simplify the model, finance theorists have assumed perfect capital markets and, in particular, that there are no transaction costs, and that borrowing and lending rates are equal. Whilst they clearly do not exist in reality, the model still provides us with a basic understanding of the effect that levels of risk have on expected returns from investment.

In determining the composition of the portfolio the investor or company will consider the following points:

� The return received from the investment – obviously higher returns are more desirable than low or negative returns; in general the better the growth prospects of the firm the better the expected returns.

� Risk and security – investors will wish to minimise their risk of the level of returns obtained and maintain at least their initial investment; often by diversifying (or spreading the risk).

� The liquidity of the investments may be important if they are only to be held for a short term.

A. RISK AND RETURN

Different investors have different attitudes to risk – some are risk-seekers and others are risk-averse. Obviously an individual’s attitude to risk will affect his/her choice of portfolio. However, risk aversion is extremely difficult to quantify in tangible terms with many factors affecting it including age, personal wealth, family circumstances, taxation, time restrictions on the availability of investment capital and the requirements of trust deeds.

Traditional economic theory, upon which much of this course, and portfolio theory in particular, is based assumes that individuals are “rational risk-avoiders” – i.e. they are risk averse.

Expected Return An investor will expect his individual investments and portfolio as a whole to yield a certain return. The expected return of a portfolio is the weighted average of the expected returns of its constituent investments weighted by their proportion in the portfolio. This is best explained by an example.

Page 153: libvolume5.xyzlibvolume5.xyz/law/ballb/3rdyear/semester5/corporatelaw/corporate... · ABE Advanced Diploma in Business Administration Study Manual CORPORATE FINANCE Contents Study

Portfolio Theory and Market Efficiency 145

© Licensed to ABE

Mr Trollope holds the following shares in his portfolio which have these expected returns:

Share Proportion Expected Return % %

A 25 20 B 15 10 C 30 15 D 30 25

What is the expected return on his portfolio? It is the weighted average of returns:

Share Proportion Expected Return Prop. ×××× Expected Return % %

A 25 20 0.25 × 20 = 5 B 15 10 0.15 × 10 = 1.5 C 30 15 0.30 × 15 = 4.5 D 30 25 0.30 × 25 = 7.5

Total expected return from portfolio = 5 + 1.5 + 4.5 + 7.5 = 18.5%

Measuring Risk in a Portfolio The return the investor will expect to receive from his portfolio is dependent principally on the risk of the portfolio. In the portfolio the greatest risk is that the investments will fail to achieve the required return. The risk can be measured as the standard deviation of expected returns. These expected returns are calculated using probabilities.

Let’s consider an example of an investment with the following expected returns.

Probability Factor (P) Return (r) Expected Value 0.3 10% 3% 0.4 15% 6% 0.3 20% 6%

15% Here the expected (the most likely) return ( r ) is 15% – it is the most likely estimate from the information available to us. However, it is only an average figure and there will be variations around this point. To calculate the likely variation we need to work out the variance and, from this, the standard deviation:

P Return (r) (r −−−− r ) P(r −−−− r )2 0.3 10% (5%) 7.5 0.4 15% 0% 0 0.3 20% 5% 7.5

Variance: 15

Page 154: libvolume5.xyzlibvolume5.xyz/law/ballb/3rdyear/semester5/corporatelaw/corporate... · ABE Advanced Diploma in Business Administration Study Manual CORPORATE FINANCE Contents Study

146 Portfolio Theory and Market Efficiency

© Licensed to ABE

The standard deviation is the square root of the variance ( 15 ), which is 3.87%. In other words, we have found that our likely return could fluctuate by up to 3.87% in either direction.

(You should be familiar with the concept of standard deviation from your earlier studies in Quantitative Methods. If you have forgotten the calculations involved, we deal with some more detailed examples a bit further on in this study unit.)

The higher the standard deviation the higher the risk. Since the standard deviation shows the range of expected returns (in the above example these are 15% ± 3.87%), we can see that the higher the risk, the higher the expected returns or expected losses.

In general, the higher the risk of the investment the higher the expected return. Investors will avoid as much risk as possible whilst aiming for the highest return.

B. THE IMPACT OF DIVERSIFICATION

Investors and companies generally have more than one investment in order to minimise their exposure to risk. A good example is the unit trust, which comprises a large number of investments thereby spreading the risk incurred by the investor and generally reducing it.

Portfolio theory states that it is the relationship between the returns from the individual investments which is important, rather than the return on individual investments. The relationship between the returns is known as correlation.

Correlation This can take three forms.

(a) Positive Correlation

This concerns the situation which may arise when two or more investments in the portfolio are in connected industries. It is assumed that, if one investment is successful and rises in value, then the other in a related industry will also do well. Similarly, if the first does badly, then the second will follow. For example, if there is a slump in the property market, the shares in a house-building concern will fall and be followed by a decline in the share price of related firms, such as those supplying building materials.

(b) Negative Correlation

This concerns the theory that, if one investment performs poorly, the other will do well. In the real world negative correlation is almost impossible to achieve. One example could be a steep rise in the price of oil. Initially this will usually benefit oil companies, whilst the rest of the market will be depressed. You may like to try to think of similar circumstances.

(c) Nil Correlation

Here, the performance of an investment is unrelated to that of another. This situation would typically arise when investments include an engineering firm and a clothing manufacturer. Barring a total market collapse, both investments will be expected to perform entirely independently of each other.

From the above, we can see that, to reduce the risk, the finance manager in his or her role of investor should choose investments which are perfectly negatively correlated, and where this cannot be achieved, he or she should seek nil correlation as the next best alternative.

Page 155: libvolume5.xyzlibvolume5.xyz/law/ballb/3rdyear/semester5/corporatelaw/corporate... · ABE Advanced Diploma in Business Administration Study Manual CORPORATE FINANCE Contents Study

Portfolio Theory and Market Efficiency 147

© Licensed to ABE

Assessing Risk under Different Forms of Correlation We shall consider this through an example.

Two investment opportunities have the following potential outcomes.

Investment 1:

Probability (P) Return (r) Worst outcome 0.3 10% Most likely outcome 0.4 15% Best outcome 0.3 20%

Investment 2:

Probability (P) Return (r) Worst outcome 0.3 9% Most likely outcome 0.4 16% Best outcome 0.3 21%

Where the investments are perfectly correlated, we can assume that if investment 1 yields 20%, then investment 2 will yield 21%, and so on. If they are negatively correlated, then investment 1 will yield 20% whilst investment 2 will yield 9%, etc.

Having accepted the theory, we can move forward and calculate the return of a portfolio which is composed of exactly half of each type of the above securities under conditions of positive, negative and nil correlation.

(a) Positive Correlation

Where the two investments are positively correlated, the worst outcome will occur at the same time for both.

Firstly, we work out the expected return of the portfolio by combining the expected returns of each investment. The expected return for investment 1 is 15% (from the example considered previously). For investment 2 it is:

Probability Factor (P) Return (r) Expected Value 0.3 9% 2.7% 0.4 16% 6.4% 0.3 21% 6.3%

Expected return ( r ): 15.4% The expected return on the portfolio is therefore:

(50% × 15%) + (50% × 15.4%) = 15.2%.

Having determined the expected return from the investment, we can now go on to calculate the likely variation in it. From the example above, we know that the standard deviation for investment 1 is 3.87%. The standard deviation for investment 2 is calculated as follows:

Page 156: libvolume5.xyzlibvolume5.xyz/law/ballb/3rdyear/semester5/corporatelaw/corporate... · ABE Advanced Diploma in Business Administration Study Manual CORPORATE FINANCE Contents Study

148 Portfolio Theory and Market Efficiency

© Licensed to ABE

P Return (r) (r −−−− r ) P(r −−−− r )2 0.3 9% (6.4%) 12.29 0.4 16% 0.6% 0.14 0.3 21% 5.6% 9.41

Variance: 21.84 The standard deviation of investment 2 = 84.21 = 4.67%.

We can now consider the standard deviation of the portfolio under perfect correlation:

Return on 1

(50%)

Return on 2

(50%)

Combined Return

Probability Expected Value

(r −−−− r ) P(r −−−− r )2

Worst outcome 5% 4.5% 9.5% 0.3

2.85

(5.7)

9.75

Most likely outcome 7.5% 8% 15.5% 0.4

6.20

0.3

0.04

Best outcome 10% 10.5% 20.5% 0.3

6.15

5.3

8.43

r = 15.20 Variance: 18.22 Standard deviation = 22.18 = 4.27%.

(b) Negative Correlation

If the two investments in our portfolio are negatively correlated, then the worst outcome for investment 1 should coincide with the best outcome for investment 2 and vice versa. Our calculations will look like the following:

Return on 1

(50%)

Return on 2

(50%)

Combined Return

Probability Expected Value

(r −−−− r ) P(r −−−− r )2

Worst outcome 5% 10.5% 15.5% 0.3

4.65

0.3

0.027

Most likely outcome 7.5% 8% 15.5% 0.4

6.20

0.3

0.036

Best outcome 10% 4.5% 14.5% 0.3

4.35

(0.7)

0.147

r = 15.20 Variance: 0.210 Standard deviation = 21.0 = 0.46%.

The figures demonstrate the difference which occurs between perfect positive and perfect negative correlation. With positive correlation, the standard deviation (the risk) is 4.27% on our return of 15.2%, whereas with negative correlation it is only 0.46%.

Page 157: libvolume5.xyzlibvolume5.xyz/law/ballb/3rdyear/semester5/corporatelaw/corporate... · ABE Advanced Diploma in Business Administration Study Manual CORPORATE FINANCE Contents Study

Portfolio Theory and Market Efficiency 149

© Licensed to ABE

There is also a formula which you may encounter that can also be used to calculate the standard deviation of a portfolio which contains two investments:

S = ))()(c)()(2( + )( + )( 21212

22

22

12

1 σσ××σ×σ×

where: S = the standard deviation of the portfolio

×1 = the weighting applying to the first investment

×2 = the weighting applying to the second investment

σ1 = the standard deviation of the first investment

σ2 = the standard deviation of the second investment

c = the correlation coefficient between the investments

Using the two investments considered in the above example, we can use the formula to calculate the standard deviation, and hence the risk, of the portfolio.

In a situation of perfect positive correlation, the correlation is expressed as + 1 and the formula becomes:

S = (4.67))(1)(3.87)2(0.5)(0.5 + 21.84 (0.5) + 15 (0.5) 22

= 04.946.575.3 ++

= 25.18

S = 4.27%.

Under conditions of perfect negative correlation, the correlation coefficient is −−−−1 and the formula becomes:

S = .67)1)(3.87)(4)(2(0.5)(0.5 + 21.84 (0.5)+ 15(0.5) 22 −

= 04.946.575.3 −+

= 017.

S = 0.41%.

The answers, as you can see, approximate very closely to the answers we calculated previously. The negative calculation shows a slightly higher variation because of its relative size, i.e. a standard deviation of less than half a percent.

Note that if no correlation existed at all, the correlation coefficient would be 0 and the second half of the formula would, therefore, equal zero.

S = 046.575.3 ++

= 9 21.

= 3.03%

The Importance of Correlation You may be wondering why the relationship between the returns from the individual investments is more important than the returns on individual investments.

Page 158: libvolume5.xyzlibvolume5.xyz/law/ballb/3rdyear/semester5/corporatelaw/corporate... · ABE Advanced Diploma in Business Administration Study Manual CORPORATE FINANCE Contents Study

150 Portfolio Theory and Market Efficiency

© Licensed to ABE

When considering the risk from holding securities, we saw above that we look at the variations in returns from individual securities and the correlation between the returns. If we call the variance of an individual security v, and the correlation between returns from securities c, we can see the factors affecting risk as the number of securities in the portfolio is increased:

Figure 10.1: Variance and correlation in a portfolio

Number of Securities 1 2 3 4 5 6 7 8 9 10

1 v c c c c c c c c c

2 c v c c c c c c c c

3 c c v c c c c c c c

4 c c c v c c c c c c

5 c c c c v c c c c c

6 c c c c c v c c c c

7 c c c c c c v c c c

8 c c c c c c c v c c

9 c c c c c c c c v c

10 c c c c c c c c c v

Thus, as the number of securities in the portfolio increases, the significance of individual variances becomes less important, and it is the correlation between returns that is important.

C. PORTFOLIO COMPOSITION

When choosing his portfolio the investor will wish to maximise expected returns and minimise risk. We can illustrate the relationship between the two using graphs and this can help us determine portfolio composition.

Investment Indifference Curves The basis for this is the indifference curve. The indifference curve for an investor represents the mixtures of risk and return which will be acceptable in terms of an investment. Consider Figure 7.2. At any point along the curve, the risk-return relationship is the same for the investor and he will be indifferent to whether the investment is at point A or point B.

(Indifference curves are curved because of the diminishing returns provided as the quantities of risk or return become disproportionate in the mixture towards the extremes of the curves.)

Page 159: libvolume5.xyzlibvolume5.xyz/law/ballb/3rdyear/semester5/corporatelaw/corporate... · ABE Advanced Diploma in Business Administration Study Manual CORPORATE FINANCE Contents Study

Portfolio Theory and Market Efficiency 151

© Licensed to ABE

Figure 7.2: An Investor’s Indifference Curve

A will be preferable to D because it offers the same expected return for a lower risk and to C because it offers a higher expected return for the same level of risk. A is said to dominate C and D. Whether an investor will choose A or B depends on their attitude to risk – and remember that an investor may be risk averse or risk-seeking.

Traditional economic theory states that individuals will seek to maximise their utility. The Markowitz model of investment analysis seeks to measure the investor’s utility function U as:

U = U( R σ)

where: R = the investor’s expected return from the shares; and

σ = the standard deviation or risk of the investment.

We can be plot this as series of indifference curves for investors as shown in Figure 7.3.

Figure 7.3: An Investor’s Indifference Curves

Thus shows the portfolios between which the investor will be indifferent. They all give equal utility – for example, A gives a lower return but has a lower risk than B.

Page 160: libvolume5.xyzlibvolume5.xyz/law/ballb/3rdyear/semester5/corporatelaw/corporate... · ABE Advanced Diploma in Business Administration Study Manual CORPORATE FINANCE Contents Study

152 Portfolio Theory and Market Efficiency

© Licensed to ABE

The further the indifference curves go to the left the greater will be the value of utility to the investor, because these portfolios provide higher returns for the same level of risk, or lower risk for the same level of returns. Portfolios to the left of the curves are preferable because those below are mean variance inefficient and those on or above the utility curve are mean variance efficient. Mean variance efficient portfolios are those which give the maximum return for a given level of risk, or have the minimum risk for a given level of return. Mean variance inefficient portfolios are those which are not efficient in the sense explained above.

Efficiency Frontier We can develop the above analysis into a model of expected risk and return for all possible investments.

If we plot the risk-return relationship for a number of investments, we get a scattergram as shown in Figure 7.4. The resultant plot does not follow a linear course, but is regarded as having an “umbrella” shape.

Figure 7.4: The Efficient Frontier

Those investments which maximise the expected return for a level of risk or minimise risk for a level of expected return can be seen to fall on the line AB – this is known as the efficient frontier. A combination of investments falling on the line AB represent the most efficient portfolios. Before reading any further think why they give the most efficient portfolios.

The reason is that it is impossible to create a portfolio which gives a higher rate of return for a given level of risk, or a lower level of risk for a given level of return. Remember – the traditional economic theory on which it is based assumes that investors are risk averse.

We can now combine the indifference curve for a particular investor with this concept of the efficiency frontier. Consider Figure 7.5.

Page 161: libvolume5.xyzlibvolume5.xyz/law/ballb/3rdyear/semester5/corporatelaw/corporate... · ABE Advanced Diploma in Business Administration Study Manual CORPORATE FINANCE Contents Study

Portfolio Theory and Market Efficiency 153

© Licensed to ABE

Figure 7.5: The Optimum Portfolio

The optimum portfolio is the one where the efficiency frontier touches the individual’s indifference curve at a tangent, shown as portfolio M. Indifference curves above the efficient frontier are not attainable there being no portfolios of securities offering both levels of expected return and risk.

The Capital Market Line The return on government stocks is deemed to be risk-free (governments can always print money to meet their obligations). This is a point on the y axis (return) of the risk-return graph where risk is zero and is shown in Figure 7.6 as point Rf – the risk-free rate of return. If we draw a line tangential from Rf to the efficient frontier we obtain the capital market line(CML).

Figure 7.6: The Capital Market Line

Any portfolio not on this line can be seen to be either mean variance inefficient (producing lower expected return or higher risk than those on the capital market line), or to be not obtainable (those above the line). Investors will be able to invest in a mixture of the market portfolio and the risk-free

Page 162: libvolume5.xyzlibvolume5.xyz/law/ballb/3rdyear/semester5/corporatelaw/corporate... · ABE Advanced Diploma in Business Administration Study Manual CORPORATE FINANCE Contents Study

154 Portfolio Theory and Market Efficiency

© Licensed to ABE

investment. They can either invest in the risk-free investment with or without investing in the market, or borrow at the risk-free rate and invest in the market portfolio, or invest solely in the market.

You may be wondering what portfolio M consists of since it is the only portfolio investors wish to hold. All shares quoted must be held and therefore M must consist of all shares on the Stock Market. In practice, no one shareholder will be able to hold every one, but a well diversified portfolio of 15 to 20 shares has been found to mirror M.

An efficient portfolio would be one that offers a better combination of risk and return than that offered by the CML (although as we saw above it is unobtainable) and an inefficient one is one which offers a worse combination than that available on the CML.

Let’s examine this in more detail. Consider the CML shown in Figure 7.7.

Figure 7.7: The Capital Market Line

We see the higher the risk the higher the expected return (this is in line with all the theory we have discussed thus far). Using the equation for the gradient of a straight line (y = mx + c) we can give the gradient at any point on the CML.

Consider y = m

fm RRσ−

This shows the level of expected return required to compensate investors for the risk they are bearing (including both business and financial risk).

The expected return over and above the risk-free rate is known as the risk premium. If you look at the CML you can see that the statement “the greater the risk the greater the expected return” does, indeed, hold, with riskier securities offering a higher level of risk premium.

We saw earlier that the expected rate of return comprises an element for the risk premium and return required on the risk-free securities. The return required on portfolio P (and the equation of the CML) is:

Rp = Rf +

σ

−m

f m R R σp

Capital Market Line

m

p

σp σm Risk (σ)

ExpectedReturn R

Rp

Rm

Rf

Page 163: libvolume5.xyzlibvolume5.xyz/law/ballb/3rdyear/semester5/corporatelaw/corporate... · ABE Advanced Diploma in Business Administration Study Manual CORPORATE FINANCE Contents Study

Portfolio Theory and Market Efficiency 155

© Licensed to ABE

The risk premium

σ

−m

f m R R σp can be rewritten as σp

σ

−m

f m R R.

The expression m

p

σσ

is commonly known as the beta factor and, thus, the equation can be written as:

Rp = Rf + β(Rm −−−− Rf)

where: Rp = the return required on a portfolio by an investor

β = the beta factor

Rm = the return required for holding the market portfolio

Rf = risk-free rate

The beta factor equation can also be used to determine the required return on an individual investment or security. Thus, the required return on a portfolio is determined by the relationship of its risk in relation to that of the market. This is because the greater the difference in risk of the portfolio to that of the market (measured by their respective variances), the greater the difference in required returns.

The beta factor can be used to measure the extent to which the return on a portfolio or security should exceed the risk free rate of return.

This point and the equation above is the basis of the capital asset pricing model which we shall discuss in the next study unit.

The capital asset pricing model can be used to calculate the market value of equity (see Study Unit 2) and the cost of equity (and thus the weighted average cost of capital) taking financial and business risk into consideration.

Efficient and Inefficient Portfolios Consider the following information:

Expected return on market portfolio = 16% Risk-free rate of return = 8% Measure of risk on market portfolio = 4%

If we plot these figures on a graph it would give us the CML as shown in Figure 7.8. Plotting the CML simply involves locating the position on the graph which corresponds to the market portfolio (M) – here, where the expected return is 16% and the risk is 4% – and joining it to the point on the return axis corresponding to the risk-free rate (8%).

Page 164: libvolume5.xyzlibvolume5.xyz/law/ballb/3rdyear/semester5/corporatelaw/corporate... · ABE Advanced Diploma in Business Administration Study Manual CORPORATE FINANCE Contents Study

156 Portfolio Theory and Market Efficiency

© Licensed to ABE

Figure 7.8

Now consider a number of portfolios with different risk-return relationships. If we plot these on the graph, we can establish whether they are efficient or inefficient.

Portfolio Expected Rate of Return Standard Deviation (Risk) P 15% 5% Q 15% 8% R 17% 4% S 23% 7.5%

These figures are shown plotted on Figure 7.9.

Figure 10.9

The efficiency of each of the portfolios is given by their relationship to the CML:

Capital Market Line (CML)

M 16

Rf 8

4 Risk %

Expected Return

%

2 4 Risk %

CML S

M Q P 16

Rf 8

12

Expected Return

% R

Page 165: libvolume5.xyzlibvolume5.xyz/law/ballb/3rdyear/semester5/corporatelaw/corporate... · ABE Advanced Diploma in Business Administration Study Manual CORPORATE FINANCE Contents Study

Portfolio Theory and Market Efficiency 157

© Licensed to ABE

� Portfolios P and Q are “inefficient” as they yield a lower risk/return combination than the market portfolio.

� Portfolio R is “super efficient” as it yields a higher risk/return combination.

� Portfolio S is “efficient” because it lies on the CML and therefore yields a risk/return combination equivalent to the market.

Securities Market Line The Securities Market Line (SML) is the same as the CML, apart from the fact that beta co-efficients are substituted for levels of risk.

In the above example, therefore, a risk level of 4% (the market risk) is equivalent to a beta co-efficient (β) of 1.0. Similarly, a risk level of 2% would be equivalent to β of 0.5 and so on. The difference between the two is that the CML measures total risk whilst the SML is related to market risk only.

The SML and beta co-efficients can be used in two ways.

� If an investor knows the rate of return he or she requires, he or she can use a risk measurement service to identify the beta co-efficient of suitable shares, i.e. if he or she required a return of 12%, he or she would obtain shares with a beta co-efficient of 0.5 (i.e. a risk level of 2%).

� If the beta co-efficient of a share is known, it is possible to calculate the return it should provide, i.e. a beta co-efficient of 1.0 in the above example should yield 16%, and so on.

D. THE APPLICATION OF PORTFOLIO THEORY

Portfolio theory is concerned with selecting and optimising a set of investments by considering the returns on those investments and the variability of such returns. A “portfolio” is usually a collection of shares but it could also comprise other investment opportunities.

Planning Diversification Portfolio theory demonstrates that risk can be diversified away with a carefully selected type and number of securities. However, if an investor such as a company is obliged to diversify (say by legislation or similar restrictions), portfolio theory can show that risk will actually increase. Such investment over larger than optimum numbers of securities is called naïve diversification.

We saw above that portfolio theory can be applied to investments other than stocks and shares, including its use by companies choosing a selection of projects and business ventures to invest in. Companies will be able to reduce risk and stabilise profits by investing in businesses with a negative or weak positive correlation between them – for example, a company which produces central heating systems would reduce its risk by a greater amount diversifying into paints than by diversifying into winter coats.

The advantages of diversification are:

� Reduced risk of corporate failure due to lower total company risk (and thus lower potential costs of redundancy).

� More stable internal cash flows, which should help increase debt capacity, thus reducing the cost of capital and, in turn, increasing shareholder wealth.

� Reduction in systematic risk may arise from investing in foreign markets generally protected by barriers to trade, thus increasing the risk/return combinations available to investors.

Page 166: libvolume5.xyzlibvolume5.xyz/law/ballb/3rdyear/semester5/corporatelaw/corporate... · ABE Advanced Diploma in Business Administration Study Manual CORPORATE FINANCE Contents Study

158 Portfolio Theory and Market Efficiency

© Licensed to ABE

A company may, however, over-diversify and may experience the following problems:

� Conglomerates often have indifferent returns leaving them vulnerable to takeover. The Stock Market often values the P/E ratios of individual companies within the group higher than that of the conglomerate, thus providing an incentive for the buyer of the group to “unbundle” and sell off parts of the group.

� There will be difficulties in becoming familiar with all the parts of the group and this lack of knowledge could lead to missed opportunities.

� Companies may lack the skills and expertise to manage all the elements of the group, and indeed may lack the skills to manage a diversified group.

� Empirical evidence has found that investors can diversify more efficiently than companies.

Companies considering diversification should assess the above points (tailored to their particular industry). However, in general, some diversification does help to protect against short-term profit fluctuations, but too much can create severe problems for the group.

Selected versus Random Portfolios Recent years have seen a market growth in investment trusts, unit trusts, and pension and insurance funds – all of which are, in effect, investors seeking a portfolio of investments. How, then, do these portfolio operators fare? How do they perform in relation to, say, a randomly-selected portfolio?

There have been numerous surveys along these lines. One survey concluded that, on average, unit trusts performed no better than randomly-selected portfolios in the industries in which the trusts had invested – i.e. the trust managers probably picked the right branch of industry to invest in, but not necessarily the right company in that industry.

There is no great condemnation of professional portfolio managers, however. They will naturally tend to err on the prudent side and go for minimum risk, even though most of their promotional advertising stresses performance. What is never likely to be clarified is the extent to which portfolio managers use (or are even aware of) the formal portfolio selection techniques. Certainly, these large investors tend to have an effect on the shares they purchase, for large-scale investments inevitably affect the market prices. The main theme of the findings seems to be that, during times of rising markets, unit trusts do worse than random investments; in falling markets they do better.

Practical Difficulties When considering the application of portfolio theory, all sorts of practical difficulties come to mind:

� Can we really measure risk in statistical terms?

� Is the standard deviation of future expected returns the only dimension involved?

� How many investments shall we consider out of the thousands available on the world’s stock exchanges if the sheer volume of computation is not to be prohibitive?

� What about the changing, dynamic nature of investment, taxation provisions and world events?

� What of transaction costs in amending the portfolio, the marginal value of money to an investor, inflation, and so on?

Clearly, a great deal of work remains to be done on this topic. Markowitz refined his original model to cut down the volume of computation. Other writers have built on his basic work, and this is undoubtedly one area in which future bodies of theory will emerge in a more refined state than at present.

Page 167: libvolume5.xyzlibvolume5.xyz/law/ballb/3rdyear/semester5/corporatelaw/corporate... · ABE Advanced Diploma in Business Administration Study Manual CORPORATE FINANCE Contents Study

Portfolio Theory and Market Efficiency 159

© Licensed to ABE

Limitations of Portfolio Theory Portfolio theory is:

(a) Concerned with a single time-period framework. It is not a dynamic model and therefore can only be revised as the anticipated performance of securities alters or new ones become available. In this respect, it has no predictive qualities.

(b) Concerned with guesswork to estimate the probabilities of different outcomes, which may be a particular problem when the model is being used to assess diversification by the firm into new uncharted markets or products.

(c) Subject to investor attitudes which may be difficult to determine and reflect on when making decisions, and investor perception or propensity for risk may well change over time.

(d) Not the most convenient method for considering physical investment in fixed assets or those generating irregular cash flows. A mix of “paper” and “physical” investments is best handled by other techniques centred on risk-reward probability theory.

(e) Based on simplifying assumptions:

(i) That investors behave rationally and are risk-averse.

(ii) The agency problem is ignored – managers may be more risk-averse than shareholders as they may be concerned about job security.

(iii) Legal and administrative constraints are also ignored.

(iv) Taxation, in particular, can give rise to very complex models – for this reason its effects are often ignored.

(f) Not able to cope with investment policies, such as “selling short” and other leverage devices.

(g) Assuming constant returns to scale and divisible projects, both of which may not occur in practice.

(h) Ignoring various other aspects of risk, e.g. the risk of bankruptcy.

The volume and type of information required can also be a disadvantage, although this is now more widely available than used to be the case (at a cost). The quality of information, however, is still somewhat variable, and in any case, portfolios have to be updated and revised regularly. (The principle “Garbage in; garbage out” applies here, too!) Each revision will carry administrative, transaction and switching costs.

A full understanding of the model requires detailed mathematical skills and is therefore in danger of being seen as too theoretical to be useful and so ignored if its principles are not put across to sceptical managers in a convincing way.

E. MARKET EFFICIENCY

Individuals and companies invest in securities in order to receive income, the price reflecting the expected returns; from shares this income comprises dividends and capital gains (increases in share prices), from loan stock income is received as interest, the redemption value and perhaps capital gains (arising from an increase in the market value of the stock).

In order to maximise income from such securities, the optimal time for buying and selling them must be determined in order to maximise capital gains (or minimise capital losses). The general rule is to purchase cheap and sell dear. To determine the optimal points many investors enlist the help of

Page 168: libvolume5.xyzlibvolume5.xyz/law/ballb/3rdyear/semester5/corporatelaw/corporate... · ABE Advanced Diploma in Business Administration Study Manual CORPORATE FINANCE Contents Study

160 Portfolio Theory and Market Efficiency

© Licensed to ABE

stockbrokers and investment advisors who predict share prices and movements within them, basing their ideas on the following underlying theories.

Fundamental Analysis Theory of Share Values This theory uses the dividend yield valuation model we discussed in Study Unit 2 and argues that the price of a share is the present value of all future expected dividends and capital gains received from the share. Thus provided all investors and their advisors have the same information about a company and its prospects, and have identical required returns from the company’s equity, the price of the shares can be predicted.

You will remember that the model used under this method varies with the assumptions used. The simplest model to use assumes that dividends will remain at a constant level in the future. The value of a company’s shares can be calculated using the following formula:

Market value = shares on the yield)(or return Expected

pencein Dividend

The market price given is the ex-dividend price – i.e. excluding any dividend that may be payable.

Example 1

Susie Ltd’s shareholders expect a dividend yield of 10% and have been told that dividends per share for the foreseeable future will be 40p. Calculate the market value of Susie’s shares.

Using the above formula to calculate the value of one share:

Value = 40p/10% = 400p or £4

However, we have seen that shareholders prefer a constant growth in their dividends. When the expected growth figure has been determined we can calculate the value of the company’s shares using the Dividend Growth Model or Gordon’s Model of Dividend Growth.

You will remember that this model states:

Po = do )gr()g1(

−+

where: Po = the current ex dividend market price

do = the current dividend

g = the expected annual growth in dividends

r = the shareholder’s expected return on the shares

(Dividends are generally paid as interim dividends part way through the year and as a final dividend after the year end. However, unless told otherwise, assume that only one dividend is paid each year.)

Example 2

Bunny Ltd is expecting to pay a dividend of 50p this year, increasing at a rate of 5% per annum. If its shareholders have a required return of 25%, calculate the current market price.

Using the above formula:

Po = 0.05)(0.250.05)50(1

−+ = 262.5p

Page 169: libvolume5.xyzlibvolume5.xyz/law/ballb/3rdyear/semester5/corporatelaw/corporate... · ABE Advanced Diploma in Business Administration Study Manual CORPORATE FINANCE Contents Study

Portfolio Theory and Market Efficiency 161

© Licensed to ABE

Note the following two points:

� If you have a market price cum dividend (or cum div), this simply means that there is a dividend due to be paid soon. In order to calculate the ex div price to use in the formula, deduct the dividend from the cum div price.

� You can also apply the same principle to the valuation of debt, using the formulae we discussed in the last unit.

As with a lot of corporate finance there is only restricted empirical evidence testing the validity of fundamental analysis, partially because of uncontrollable variations in share prices caused by day-to-day fluctuations in the economy, interest rates and investor expectations and confidence. It is therefore difficult for an analyst to predict exactly a company’s earnings and dividends and what the required returns of shareholders are. If the analyst can predict these things, and in advance of others, then he will be able to advise his clients on which shares to buy, sell and hold. Difficulties in predicting the uncontrollable variations using the fundamental theory have led to the development of a body of techniques known as technical analysis.

Technical Analysis Technical analysis, or charting (chartism) as it is also known, assumes that successive price changes of shares are dependent, and that information about a company’s future can be determined by studying past movements in its share price. It is felt by chartists that past movements will be repeated.

Models include the Dow Theory, which states that movements in the daily average of one stock herald similar movements in others. A. G. Ellinger developed similar theories to Dow which stated that share prices are determined by:

� The yield on fixed interest securities (as interest rates rise, prices of fixed interest securities fall and exert similar pressure on ordinary shares).

� Dividends on ordinary shares.

� The confidence factor.

Ellinger stated that fundamental analysis is extremely important in deciding which shares to buy; technical analysis charts (see below) should only be used to determine when, not what, to buy and sell.

In addition to these systems there is the classic chartist technique of using the statistical method of moving averages. This technique is used to determine trends and more importantly, changes in trends in share prices and to remove the day-to-day fluctuations we discussed above. A 20 day moving average will provide an indication of the underlying trend, whilst 60, 90 and 240 days will provide indications of longer-term movements. The trends in share prices will be plotted and the following features highlighted as being of importance.

(a) Head and Shoulders

This is illustrated in Figure 7.10.

The rising price trend is reversed slightly before increasing to a higher level; there is then a fall in share prices to around the level of the previous low before rising to around the first high point before falling again. Any falls below the neckline are an indication to sell; future trends upwards would indicate the reverse. Sometimes this pattern is seen in reverse, and the chartist would supply the opposite advice.

Page 170: libvolume5.xyzlibvolume5.xyz/law/ballb/3rdyear/semester5/corporatelaw/corporate... · ABE Advanced Diploma in Business Administration Study Manual CORPORATE FINANCE Contents Study

162 Portfolio Theory and Market Efficiency

© Licensed to ABE

Figure 7.10: Head and Shoulders

(b) A Double Bottom

Here, the falling price trend reverses for a while, before falling to the minimum point again. Following this second fall the price begins to rise. Once the minimum point has been reached for the second time, the chartist would use past experience to predict that the price trend was now upwards.

Figure 10.11: Double Bottom

(c) A Double Top

Figure 10.12: Double Top

Page 171: libvolume5.xyzlibvolume5.xyz/law/ballb/3rdyear/semester5/corporatelaw/corporate... · ABE Advanced Diploma in Business Administration Study Manual CORPORATE FINANCE Contents Study

Portfolio Theory and Market Efficiency 163

© Licensed to ABE

Here, the rising price trend reverses for a while before rising again to the maximum point. Following this second rise the price begins to fall. Once the maximum point has been reached for the second time, the chartist would use past experience to predict that the price trend was now downwards.

(d) Resistance Level

Share prices can be seen to fluctuate around a rising or falling trend; once the trend departs from the line then the chartist would say that the previous trend has passed the resistance level.

Figure 10.13: Resistance Level

(e) The Hatch System

This theory, an example of a “filter” system, states that as a chart of share price movement is built up, the potential investor can not know when an exact peak or trough point has been reached. Thus he will try to sell or buy as near these turning points as possible (within 10% is considered to be as good as possible).

Figure 10.14: The Hatch System

Page 172: libvolume5.xyzlibvolume5.xyz/law/ballb/3rdyear/semester5/corporatelaw/corporate... · ABE Advanced Diploma in Business Administration Study Manual CORPORATE FINANCE Contents Study

164 Portfolio Theory and Market Efficiency

© Licensed to ABE

The investor should average a suitable index, e.g. the Financial Times Share Index, for every month. When the index is rising he will hold on to a share until the average indicates a reading which is 10% below the highest recorded level in chronological sequence. At this point he should sell.

Conversely, when the average is falling, the investor should not enter the market until the average reading is 10% above the previously recorded low.

The biggest drawback of this system arises because activating signals may occur too often with the result that transaction charges will be excessive in relation to the rewards.

A major problem with all “systems” is the absence of a really comprehensive indicator of the state of the market. Conventional indices include only a limited number of securities and cannot truly reflect the whole market.

In general terms the chartist models are all based around historic data from which attempts are made to predict the future. Statistically most have been proved to be invalid and lacking theoretical underpinnings, but it has not limited the market for new ideas and charts and chartists apparently flourish. This may be because of some very successful chartists who have had more success than average. However, probably the principal benefit of the use of charts stems from their clarity and ease of comprehension to the lay person.

Random Walk Theory In the 1950s Kendall working in the UK, and Roberts working in the USA, found that successive share price changes are practically independent over time, and that share prices follow a “random walk”, reacting to new information regarding the company as it becomes available. Research into this “random walk” led to the formulation of the efficient market hypothesis (EMH).

The random walk theory relies on the stock markets being efficient and displaying perfect market characteristics. The principles underlying the theory are that:

� The market price of a security represents the market’s consensus as to the valuation of that security.

� Public information is widely available to investors – about the economy, financial markets, the individual company, its results and prospects.

� Market prices adjust readily and quickly to new situations, e.g. changes in interest rates or decisions (good or bad) taken by the company.

� No investor is large enough by himself to influence the market price of the share.

� Transaction costs are low or zero.

� There are negligible restrictions on investment.

Efficient Markets Hypothesis Efficient markets are capital markets which have the following features:

� No individual dominates the market;

� Costs of buying and selling are at a level that does not discourage trading to any significant extent;

� Share prices change quickly in response to all new information available to buyers and sellers.

Page 173: libvolume5.xyzlibvolume5.xyz/law/ballb/3rdyear/semester5/corporatelaw/corporate... · ABE Advanced Diploma in Business Administration Study Manual CORPORATE FINANCE Contents Study

Portfolio Theory and Market Efficiency 165

© Licensed to ABE

To fully understand the EMH we will look first at three interrelated measures of efficiency – allocative, operational and information processing efficiency.

� Allocative efficiency is the optimum allocation of funds within the financial markets, i.e. that which will maximise economic prosperity.

� Operational efficiency is the lowest level of transaction costs that are possible and is present when there is open and free competition within the market.

� Information processing efficiency is the quick and accurate pricing of securities, preventing speculation pushing prices to unrealistic levels.

In finance a distinction is made between three potential levels of information processing efficiency – the weak form, the semi-strong form and the strong form.

� The weak form

The information available to investors is historical information on past share prices and company results. Share prices change as and when this information becomes available.

� The semi-strong form

The investor has access to all publicly available information about a company including press releases. Share prices react not only to released information, but also to the expectation of changes in the company’s fortunes. Investors will be able to see beyond creative accounting or “window dressing” by companies in an attempt to overstate profits.

� The strong form

The share price reflects all available information, including inside information. Acting on insider information is illegal and so it is very difficult to get an accurate picture of its use.

There has been a large amount of empirical research on this theory in the UK and the USA, and the overwhelming evidence is that security markets are efficient in a semi-strong sense (and thus also in the weak sense) – i.e. that share prices follow a random walk, and react to all available information and the expectations of information, e.g. of a proposed merger. As with all theories there are times when it does not appear to hold, e.g. in the Stock Exchange crash of October 1987 when 40% was removed from the value of shares with no underlying expectations of falls in the economy or company prospects. (However, the expected fall of 1989 did not occur.)

You may be wondering how the 1987 crash can be explained. One theory proposed by Hill is that the EMH holds best when the market is stable but, during a bull phase (as in 1987) or a bear phase the market is driven by speculation and uncertainties. This irrationality can also be seen when there is an over-reaction in the short term to company and/or economic events, over-emphasis placed on large companies, and the fact that price movements can sometimes be related to the time of day, week, month or year.

The EMH and the empirical evidence noted above indicating its validity have implications for companies, investors and analysts.

(a) The current market price is the best available indicator of a share’s intrinsic value – therefore the fundamental analysts’ search for undervalued shares using publicly available information is a waste of time, and analysts should concentrate more on efficient diversification for clients.

(b) Individual investors should not worry about investment analysis but should instead choose a well diversified portfolio consistent with their risk preferences.

Page 174: libvolume5.xyzlibvolume5.xyz/law/ballb/3rdyear/semester5/corporatelaw/corporate... · ABE Advanced Diploma in Business Administration Study Manual CORPORATE FINANCE Contents Study

166 Portfolio Theory and Market Efficiency

© Licensed to ABE

(c) NPV techniques should be used for evaluating projects because this is how the market will evaluate the company.

(d) The market value of the firm will only be as good an estimate of intrinsic value as the quality of public information available concerning the company permits.

(e) Creative accounting is seen by the market as a sign of weakness rather than strength.

(f) The timing of new issues is deemed to be unimportant and there is no need to substantially discount them as they will be correctly priced by the market.

(g) Large premiums paid above the market price on takeover are difficult to justify.

The efficient market hypothesis does not imply perfect forecasting ability, but states that the market makes a correct evaluation of uncertain future events. Nor does it imply that portfolio managers are incompetent, even though on average the experts do no better than the general investor, or that share prices cannot represent fair value because they are consistently moving up and down – indeed, it is the working of the EMH that is reflected in fluctuating share prices and which prevents investors/portfolio managers consistently outperforming the market.

You may be wondering why, if the EMH shows that fundamental analysis does not produce excess returns, so many people are employed doing fundamental analysis. Firstly, some people disbelieve the EMH and believe they can outperform the bull market during a bull phase. Secondly, some investors will be more successful than average while others will not (but due to chance rather than consistently outperforming the market using publicly available information), and therefore investors regard “gambling” on the markets as a “fair game”. There is also evidence that unit trusts, whilst doing no better in bull markets, do tend to do better in falling markets.

Finally, and ironically, it is the continual search for company information by fundamental analysts which ensures there is an efficient flow of information in the market which is essential for the EMH to hold.

Alternatives to the EMH Over the past two decades there have been several financial scandals including the use of (illegal) insider information. This has led to some experts dismissing the EMH except in its weakest form and deriving alternative theories of market behaviour.

(a) Speculative Bubble Theory

This theory states that the price of securities moves above their true value creating a bull market because investors believe they will rise in future. However, eventually the “bubble” will burst when investors look at all previously available economic and company information and a crash will occur. Thus rises and falls in the market do not reflect economic conditions.

There is some empirical research supporting this, including evidence that investors are risk-seeking in a bear market in an attempt to minimise their losses.

(b) Catastrophe Theory

This developed from the Speculative Bubble Theory and argues that capital markets have the following characteristics:

� They are dynamic.

� They use “feedback” mechanisms with “critical” levels – when activity reaches particular levels the equilibrium prices of the market no longer exist.

Page 175: libvolume5.xyzlibvolume5.xyz/law/ballb/3rdyear/semester5/corporatelaw/corporate... · ABE Advanced Diploma in Business Administration Study Manual CORPORATE FINANCE Contents Study

Portfolio Theory and Market Efficiency 167

© Licensed to ABE

� Prices in such markets are not based on economic forecasts, and small changes in the events affecting a company can lead to disproportionately large changes in its security’s prices.

The latter point leads to large scale “chaos” or instability making predictions of prices impossible except in the short term. The chaos is made worse if there is a large number of speculators who amplify price movements in their attempts to maximise their own profit. This, it is argued, is one reason for the phenomenon of short-termism we discussed in Study Unit 1.

(c) Coherent Market Hypothesis

Vaga (1991) developed the Catastrophe Theory and stated that the market may be in one of the four following states:

(1) Coherence

(2) Chaos

These two states are both based on prices being determined by crowd behaviour – the former in a bull phase and the latter when the markets are more bearish.

(3) Unstable transition

(4) Random walk

These two states are underpinned by economic events, the former market being an inefficient market and the latter being an efficient market.

Vaga argues that the 1987 crash (see above) was an example of state (2).

Page 176: libvolume5.xyzlibvolume5.xyz/law/ballb/3rdyear/semester5/corporatelaw/corporate... · ABE Advanced Diploma in Business Administration Study Manual CORPORATE FINANCE Contents Study

168 Portfolio Theory and Market Efficiency

© Licensed to ABE

Practice Questions

1. Show the effect of perfect positive and perfect negative correlation between the following investments within a portfolio. (Assume a 40 : 60 ratio.)

Investment A Investment B Return Probability Return Probability Best outcome 5% 0.2 7% 0.2 Most likely outcome 15% 0.6 14% 0.6 Worst outcome 25% 0.2 21% 0.2

2. From the following data, draw the CML and show whether portfolios A and B are efficient or inefficient.

Expected return on the market portfolio = 14% Risk-free rate of return = 5% Measure of risk of market portfolio = 6%

Portfolio Expected Return Standard Deviation A 17% 7% B 10% 4%

Now check your answer with the one given at the end of the unit.

Page 177: libvolume5.xyzlibvolume5.xyz/law/ballb/3rdyear/semester5/corporatelaw/corporate... · ABE Advanced Diploma in Business Administration Study Manual CORPORATE FINANCE Contents Study

Portfolio Theory and Market Efficiency 169

© Licensed to ABE

ANSWERS TO PRACTICE QUESTIONS

1. Using the formula:

Investment A

Return Probability Expected Value

(r −−−− r ) P(r −−−− r )2

Best outcome 5% 0.2 1.00 (10) 20 Most likely outcome 15% 0.6 9.00 0 0 Worst outcome 25% 0.2 5.00 10 20

r = 15.00 Variance: 40 Standard deviation = 40 = 6.32

Investment B

Return Probability Expected Value

(r −−−− r ) P(r −−−− r )2

Best outcome 7% 0.2 1.40 (7) 9.8 Most likely outcome 14% 0.6 8.40 0 0.0 Worst outcome 21% 0.2 4.20 7 9.8

r = 14.00 Variance: 19.6 Standard deviation = 6.19 = 4.43

(a) Under Perfect Positive Correlation

SD = (4.43))(1)(6.32)2(0.4)(0.6 + 19.6 (0.6) + 40)4.0( 22

= 44.1306.74.6 ++

= 9.26

= 5.19%

(b) Under Perfect Negative Correlation

SD = .43)1)(6.32)(4)(2(0.4)(0.6 + 19.6(0.6)+ 40)4.0( 22 −

= 44.1306.74.6 −+

= 02.0

= 0.14%

Page 178: libvolume5.xyzlibvolume5.xyz/law/ballb/3rdyear/semester5/corporatelaw/corporate... · ABE Advanced Diploma in Business Administration Study Manual CORPORATE FINANCE Contents Study

170 Portfolio Theory and Market Efficiency

© Licensed to ABE

2. The CML, market portfolio (M) and portfolios A and B are shown on the following graph.

Portfolio A is superefficient.

Portfolio B is inefficient.

2 8 6 4

10

Rf 5

16

14

Risk %

CML A

M

B

Expected Return

%

Page 179: libvolume5.xyzlibvolume5.xyz/law/ballb/3rdyear/semester5/corporatelaw/corporate... · ABE Advanced Diploma in Business Administration Study Manual CORPORATE FINANCE Contents Study

171

© Licensed to ABE

Study Unit 8

The Capital Asset Pricing Model

Contents Page

Introduction 172

A. Risk, Return and CAPM 172 Systematic and Unsystematic Risk 172 Measuring Systematic Risk 173 Market Risk and Return 174 The Beta Factor and Market Risk 175 The Capital Asset Pricing Model Formula 177 Alpha Values 177 The CAPM and Share Prices 177 The CAPM and Gearing 178

B. Calculation of Betas 178

C. Validity of the CAPM 179 CAPM Assumptions 179 Limitations of CAPM 180

D. Practical Applications of CAPM 180 CAPM and Portfolio Management 180 CAPM and Capital Investment Decisions 182

E. The Arbitrage Pricing Model 182

Answers to Practice Question 185

Page 180: libvolume5.xyzlibvolume5.xyz/law/ballb/3rdyear/semester5/corporatelaw/corporate... · ABE Advanced Diploma in Business Administration Study Manual CORPORATE FINANCE Contents Study

172 The Capital Asset Pricing Model

© Licensed to ABE

INTRODUCTION

In the last study unit we discussed portfolio theory and said that it underpins the Capital Asset Pricing Model (CAPM). CAPM was developed in the 1960s by Sharpe and Litner building on the work of Markowitz and portfolio theory. The model at its simplest brings together aspects of share valuations, the cost of capital, and gearing, and thus has important implications in financial management.

For our purposes, we can make the assumption that there are two basic functions associated with the CAPM:

� Attempting to establish the “correct” equilibrium market value of a company’s shares.

� Calculating the cost of a firm’s equity (and thus the weighted average cost of capital), as an alternative approach to the dividend valuation model which we considered in a previous unit.

The model also implies equilibrium between risk and the expected return for each security and can be used by the financial manager in the assessment of risk in either individual company shares or a portfolio of securities.

A. RISK, RETURN AND CAPM

There is risk associated with investment in any security, and we said in the last study unit that the greater the risk, the greater the required return from the investment. However, one type of stock which has a low risk, and which is assumed in portfolio theory to be risk-free, is Treasury bills (because, as we have already seen, the Government is unlikely to renege on its commitments to pay the returns agreed on the bills). The difference between a higher return and that achieved at the risk-free rate is known as the excess return, and it will differ between securities depending on the market’s perception of the relative risk of each.

The only way for an investor to avoid risk altogether is to invest solely in government securities, but in doing so the investor will trade off risk for a lower return than might otherwise have been made.

Systematic and Unsystematic Risk We showed in the last study unit that risk comprises financial and business risk. We also saw that investors tend to diversify their portfolios to reduce their risk whilst maintaining their return. The risk which can be diversified away is known as unsystematic risk, and is unique to a particular company. It is independent of political and economic factors, and may arise, for example, as a result of bad labour relations causing strikes, the emergence of improved competitor products or adverse press reports. It is diversified away because the factors causing it are different for different companies and cancel each other out.

The risk related to the market, however, cannot be diversified away (if it could then the return on the market would not be higher than the risk-free rate), and is known as systematic or market risk. Systematic risk is unavoidable risk. Systematic risk may also vary between projects. Such risk may arise as a result of government legislation, from adverse trends in the economy or from other external factors over which the company has no control.

The two types of risk are significant, because in building a portfolio of shares the investor will want to minimise unsystematic risk.

Page 181: libvolume5.xyzlibvolume5.xyz/law/ballb/3rdyear/semester5/corporatelaw/corporate... · ABE Advanced Diploma in Business Administration Study Manual CORPORATE FINANCE Contents Study

The Capital Asset Pricing Model 173

© Licensed to ABE

We noted earlier that research has found that if a portfolio has between 15 and 20 shares selected at random then the unsystematic risk in the portfolio should be eliminated. Increasing the size of the portfolio up to this level will certainly reduce the level of unsystematic risk (see Figure 8.1).

Figure 8.1: Systematic and systematic risk

Although by definition unavoidable, the degree of systematic risk will be a variable factor between different industries – shares in different companies will have systematic risk characteristics which are different from the market average because the market considers some investments to be riskier than others (for example, food retailers are lower risk than those in the fashion industry). When an investor holds a portfolio which is balanced throughout with all available stocks and shares, or a unit trust which mirrors the market, he will incur systematic risk which is equal to the average systematic risk in the market as a whole.

We can also see that individual investments will have their own levels of systematic or market risk.

Measuring Systematic Risk The CAPM is principally concerned with:

� How systematic risk is measured.

� How systematic risk affects the required returns and share price.

In order to measure systematic risk, we use the beta factor (ββββ).

The CAPM also includes some fundamental assumptions which we can summarise as:

(a) Investors in shares (as opposed to risk-free investments, which are generally government securities) require a return which is in excess of the risk-free rate as a form of compensation for taking the systematic risk of the investment.

(b) Investors should not require a premium for unsystematic risk as this may be diversified and removed from the portfolio (as discussed earlier).

(c) As the systematic risk is higher for some companies (as measured by their β factor) the investor will expect a greater return and will continue to do so as the β gets larger.

The financial manager may, incidentally, adopt a similar approach to investment in one or more new projects. When a company is considering an investment in a new project, there will be a degree of

Page 182: libvolume5.xyzlibvolume5.xyz/law/ballb/3rdyear/semester5/corporatelaw/corporate... · ABE Advanced Diploma in Business Administration Study Manual CORPORATE FINANCE Contents Study

174 The Capital Asset Pricing Model

© Licensed to ABE

risk involved. The greater the perception of risk in the venture, the greater will be the expected return (assuming, of course, that the directors are willing to sanction the investment in the first place).

Market Risk and Return The CAPM was formulated principally to evaluate investments in stocks and shares (“the market”) as opposed to investment projects under consideration by companies. The model is based on the comparison of systematic risk within individual investments and shares, with that in the market as a whole (hence systematic risk also being described as market risk). Market risk, in its simplest form, is the average return of the market.

Market risk is, of course, something which is almost impossible to determine with any degree of accuracy, as it is based on the total expected market return. As the components of the market fluctuate consistently, so the systematic risk attached to shares will also change. Therefore CAPM must make one major and fundamental assumption – that there is a linear relationship between the return obtained from one single investment and the market average.

Let’s look at an example.

Our aim is to demonstrate at a basic level how the return from one investment compares with the market:

Company A Whole Market Price at start of period 110 490 Price at end of period 130 510 Dividend paid 6.5 40.1

The return on Company A’s shares (Rs) and the return on the general market portfolio of shares (Rm) may now be calculated as follows:

periodofstartatPrice

Dividend loss)(orgainCapital +

Therefore:

Rs = 110

6.5 110)(130 +− = 0.24

Rm = 490

40.1 490) (510 +− = 0.12

Statistical analysis of “historical” returns from Company A and from the “average” market may suggest that a linear relationship exists. Thus, the linear relationship can be demonstrated through collecting comparative figures from Company A and average market returns (say on a month by month basis). The results can then be plotted on to a scatter diagram and a line of best fit can then be drawn with linear regression (see Figure 8.2).

Page 183: libvolume5.xyzlibvolume5.xyz/law/ballb/3rdyear/semester5/corporatelaw/corporate... · ABE Advanced Diploma in Business Administration Study Manual CORPORATE FINANCE Contents Study

The Capital Asset Pricing Model 175

© Licensed to ABE

Figure 8.2: Relationship of returns between one company and the market

This approach to analysis could bring out three important issues, namely:

� The return from Company A (Rs) and the return from the market (Rm) will tend to rise or fall together.

� The return from Rs may be higher or lower than Rm because the systematic risk of an individual security differs from that of the whole market. Company A is an illustration of an investment which provides generally higher returns than the market and is therefore considered more risky than the average.

� The graph may not always produce a line of good fit. This typically happens when there is insufficient data to be plotted, and the data available is being affected by both unsystematic and systematic risk.

Negative returns may also be possible, which may happen when share prices drop suddenly. This will then amount to a capital gains loss, thereby equating to a negative return.

Our example demonstrates the relationship between an individual company’s systematic risk and that of the market fairly predictably. The measure of the relationship between the returns of the company and those of the market can then be developed in the beta factor (β) for that company. The line of best fit, also known as the characteristic line, will dictate the beta factor – the steeper the line, the greater will be the beta factor.

The Beta Factor and Market Risk The beta factor is a measure of a share’s volatility in terms of market risk.

We can identify three possibilities for that measurement:

� Where β > 1, the shares would be described as aggressive, i.e. they would outperform the Stock Market whichever way the general trend in prices was moving.

Return from companyA’s shares (Rs)

Return from whole market (Rm)

Line of best fit

Page 184: libvolume5.xyzlibvolume5.xyz/law/ballb/3rdyear/semester5/corporatelaw/corporate... · ABE Advanced Diploma in Business Administration Study Manual CORPORATE FINANCE Contents Study

176 The Capital Asset Pricing Model

© Licensed to ABE

� Where β = 1, the shares would be described as neutral, i.e. they would follow the general trend of the Stock Market.

� Where β < 1, the shares would be described as defensive, i.e. they would be less risky than the market generally.

As you will see, the market as a whole is assigned the value of “1”. If a company’s beta factor is 2, this would indicate that it would return twice as much as the market generally. Therefore, it would be expected that, if the market return (Rm) rose by 5%, then the return in a company (Rs) with a beta factor of 2 would rise by 10%. Variations in the company’s return (Rs) outside this would be specifically due to the impact of its own unsystematic risk, which is unique to that company.

We should remember that another essential characteristic of the CAPM is that unsystematic risk can be cancelled out by diversification of the portfolio. In a simple example, Company Y’s shares do worse than the average market returns and Company Z’s do better (as originally predicted by the beta factor). The net effect will be self-cancelling and therefore the unsystematic risk has been removed from this hypothetical portfolio.

In such circumstances, the average return on the portfolio will be dependent upon:

� Changes in the average market return, and

� The beta factors of the shares which make up the portfolio.

We will now look at a further example to highlight these points.

Example

Suppose that:

(a) The return on government stock is 10%.

(b) The average market return is 15%.

(The difference of 5% is therefore the excess return as we described earlier.)

The difference between the risk-free return and the expected return on an individual investment can be measured as the excess return for the market as a whole, multiplied by the beta factor of the investment.

Now suppose that we take the example of a company with a β of 1.4, the risk-free return is 9%, and the expected market return is 13%. The expected return on the company’s shares would exceed the expected market return by:

1.4(13 − 9)%, or 5.6%

(The total expected return would be 14.6% (9 + 5.6).)

If the market as a whole fell by an average of 3%, to 10%, then the total expected return on the company would also fall as follows:

9% + 1.4(10 − 9) = 10.4%.

(The fall is represented by 1.4 × 3% = 4.2%)

Page 185: libvolume5.xyzlibvolume5.xyz/law/ballb/3rdyear/semester5/corporatelaw/corporate... · ABE Advanced Diploma in Business Administration Study Manual CORPORATE FINANCE Contents Study

The Capital Asset Pricing Model 177

© Licensed to ABE

The Capital Asset Pricing Model Formula We can now move on to specify the formula for the CAPM.

The formula is based on the risk-free rate of return, the excess rate of return and the beta factor of the security in question. It is expressed as:

(Rs − Rf) = β(Rm − Rf)

or Rs = Rf + β(Rm − Rf)

where: Rs = expected return from an individual investment

Rf = the risk-free rate of return

Rm = the market rate of return (the return on the all share index)

β = the beta factor of the investment.

Alpha Values The alpha value of a share is used to measure the amount by which the return on that investment is either above or below what is expected, given its level of systematic risk.

Example

A company’s shares have a β of 1.2, and an alpha of +2%. The market return is 10% and the risk-free rate is 6%.

The expected return = 6% + 1.2(10 − 6)% = 10.8%.

The current return is 10.8% + 2% = 12.8% = expected alpha return.

You should note that alpha values are only temporary rates and can be (+) or (−). They will tend towards zero for shares over time and will for a diversified portfolio actually be zero if the portfolio is taken as a whole.

Where alpha values are positive, they may attract investors, because of an implied abnormal return, and the reaction will be a temporary increase in the share price.

The CAPM and Share Prices The CAPM can also be used to predict share values as well as estimating returns from investments carrying different levels of risk. This is shown in the example below.

Company A and Company B pay an annual return of 34.04p per share and this is expected to carry on indefinitely. The risk-free rate is 8% whilst the average market rate is 12%. Company A’s β = 1.8 and Company B = 0.8.

We will now calculate the expected return and predict the market value of each share, as follows:

The expected return for A = 8% + 1.8(12% − 8%) = 15.2%.

The expected return for B = 8% + 0.8(12% − 8%) = 11.2%.

Using the dividend valuation model, the expected price of the share can be calculated:

Predicted share value in A = 0.15234.04 = 224p.

Predicted share value in B = 0.11234.04 = 304p.

Page 186: libvolume5.xyzlibvolume5.xyz/law/ballb/3rdyear/semester5/corporatelaw/corporate... · ABE Advanced Diploma in Business Administration Study Manual CORPORATE FINANCE Contents Study

178 The Capital Asset Pricing Model

© Licensed to ABE

The CAPM and Gearing Whilst we will look at gearing in detail in the next study unit, you will already be aware that the level of gearing in a company will affect the risk of its equity. Correspondingly, the β factor will alter. The extra risk for which the investor is being compensated is systematic risk which should be reflected in the company’s β factor.

We will return to this concept later in your studies.

B. CALCULATION OF BETAS

In the last unit we explained that the slope of the capital market line (also known as the characteristic line) dictates the beta factor of a security, and as such it can be calculated by measuring the gradient of the securities market line.

The gradient can be calculated using regression analysis. To calculate the gradient (and thus the beta) you would use one of the following regression formulae:

(1) β = )variance(mm)s, covariance(

where: m = return from the market

s = returns from the security.

i.e. the covariance of returns on an individual security with the market as a whole divided by the variance of the market returns

(2) β = 22 x)(xnyxxyn

Σ−ΣΣΣ−Σ

where: n = number of pairs of data for x and y

(3) β = m

mss

σρσ

where: σm = the standard deviation of returns on the market

σs = the standard deviation of returns of the company’s equity

ρsm = the correlation coefficient between the total returns on the company’s equity and the total returns on the shares of the individual company.

Example

Returns on Jack plc’s shares have a standard deviation of 12% – twice as high as that of the market. It is estimated that the correlation coefficient of the market and Jack plc’s returns is 0.45. If the estimated market return is 17% and the return on government bonds is 9% – calculate:

(a) The beta of Jack plc’s shares;

(b) The cost of equity for Jack plc.

Using the above formula (3), we can calculate Jack’s beta – remember that Jack’s standard deviation is twice that of the market and therefore the market standard deviation is 12/2 = 6.

β = m

mss

σρσ = 6

45.0 12× = 0.9

The beta of Jack plc’s shares is therefore 0.9.

Page 187: libvolume5.xyzlibvolume5.xyz/law/ballb/3rdyear/semester5/corporatelaw/corporate... · ABE Advanced Diploma in Business Administration Study Manual CORPORATE FINANCE Contents Study

The Capital Asset Pricing Model 179

© Licensed to ABE

The cost of equity is the return shareholders expect to obtain from holding their shares (i.e. it is the expected return from the investment) and can be calculated using the capital asset pricing model:

Rs = Rf + β (Rm −−−− Rf)

Government debt can be assumed (unless told otherwise) to be risk-free, and thus the return on it is equivalent to the risk-free rate.

RJack = 9 + 0.9(17 − 9) = 16.2

Therefore the cost of equity for Jack plc is 16.2%.

C. VALIDITY OF THE CAPM

You may feel that this is a very theoretical area, and many of the underlying assumptions taken from economic theory are unrealistic. However, the lack of realism is unimportant if the model can correctly predict the return on a security or portfolio for a given level of risk.

CAPM Assumptions We can list the various assumptions underlying the CAPM and assess the validity of each as follows.

� Investors are risk averse and require greater return for taking greater risks.

Empirical evidence supports this.

� There are equal borrowing and lending rates

Generally borrowing rates are higher than lending rates. However, the CAPM can be modified to incorporate this and the results remain the same.

� There are no transaction costs

The existence of transaction costs means that investors may not undertake all required transactions to make their portfolios efficient, thus the CML may be a band rather than a line.

� There are no market imperfections

Market imperfections do exist and may mean that unsystematic risk may be of some importance.

� Homogeneous expectations

Clearly not all investors have the same view on the prospects of securities. However, when the assumption is relaxed the CAPM has been found to still maintain its predictive abilities.

� No taxation

The existence of taxation may mean that shareholders prefer capital gains or dividends. However, when this assumption is relaxed the CAPM has still been found to maintain its predictive abilities.

� There is no inflation

Inflation clearly exists and may be seen as an additional risk. However, when incorporated into the model, the model can still predict the required returns accurately.

It is difficult to test the CAPM because the model deals with expected returns and all securities, and it is only possible to record actual results and those securities included in market indexes. (Market indexes generally contain only a sample of the securities available to investors.) Empirical research

Page 188: libvolume5.xyzlibvolume5.xyz/law/ballb/3rdyear/semester5/corporatelaw/corporate... · ABE Advanced Diploma in Business Administration Study Manual CORPORATE FINANCE Contents Study

180 The Capital Asset Pricing Model

© Licensed to ABE

suggests that although CAPM is not a perfect model of the real world it does provide a reasonable model of risk/return trade off. For example, low beta shares do provide lower expected returns than higher beta shares; however, low beta shares often provide returns above, and high beta shares returns below, those which the model would predict. The CAPM is used in practice as a decision-making tool in the choice of portfolios in both the UK and the US.

Limitations of CAPM The practical use of CAPM is limited by two major factors. These are:

� the acceptability of the assumptions; and

� the problems of using the model, given that the assumptions are accepted.

The most critical of the assumptions is that individual investors are able efficiently to diversify away unsystematic risk. The assumption of efficient diversification is itself dependent on many other assumptions, including those of a perfect capital market, rationality of investors, etc. Also, you should remember that CAPM is based on a one-year time period, and its extension to multiple time periods requires the economic environment and returns on the project relative to the market to remain reasonably stable. It must therefore be used with care when evaluating projects over longer periods.

Further problems include those of estimating returns on projects and the market under different economic conditions; the probabilities of these different conditions occurring; and the determination of the risk-free rate. There are several government securities and their return depends on their term to maturity.

There are many reasons why entrepreneurs may not diversify enough as required by CAPM. One compelling reason is that managers simply do not want to diversify from a business that they know well, and perceive considerable difficulties in moving outside of their experiences. Similarly, managers may not wish to be actively restrained from “playing the markets”, whatever the arguments in favour of diversifying away risk. Moreover, there is considerable effort and overhead involved in an individual investor attempting to manage a portfolio of investments actively over any length of time. Managers also argue, quite correctly, that it is for the shareholders to diversify their own risk and construct a portfolio to their own preferences, rather than any individual company representing a fully balanced portfolio itself.

While these views appear “irrational” from a purist modeller’s viewpoint, you can argue that there is nothing rational in acting against your instincts and preferences.

Outside the very short term, the market imperfections of lack of divisibility of investments, fixed charges, imperfect opportunities and poor information mean that the model has poor predictive ability. Furthermore, each investment or project should have its own discount rate according to its systematic risk as measured by its beta co-efficient. The discount rate in any one year then depends on the risk-free rate of interest and the market risk premium in that year. There are ways of forecasting such vagaries but such are the complexities of doing so that it is beyond the scope of this discussion.

D. PRACTICAL APPLICATIONS OF CAPM

CAPM and Portfolio Management The capital asset pricing model has several practical uses, and beta factors of individual securities are published by a number of analysts (e.g. London Business School) for investment and other purposes. The capital asset pricing model can be used by investors in selecting their portfolios of shares, thus reflecting the individual’s risk preference. A risk-taker would select a portfolio with a beta greater

Page 189: libvolume5.xyzlibvolume5.xyz/law/ballb/3rdyear/semester5/corporatelaw/corporate... · ABE Advanced Diploma in Business Administration Study Manual CORPORATE FINANCE Contents Study

The Capital Asset Pricing Model 181

© Licensed to ABE

than 1 (large gains/large losses) whereas someone more cautious would select a beta equal to or less than 1. Portfolios of securities have beta factors which are the weighted average of the betas of the securities within the portfolio.

Example

Jessica wishes to know the expected return on her portfolio, when the risk-free rate is 7% and the return on the market is expected to be 20%. Her portfolio is made up as follows:

Security Percentage of Portfolio Beta Factor of Security R plc 15 0.2 S plc 10 1.2 T plc 5 1.8 U plc 30 0.9 V plc 25 0.2 W plc 15 0.8

Firstly, let us calculate the portfolio’s beta. This is simply the weighted average of the individual betas:

Portfolio’s beta = (15% × 0.2) + (10% × 1.2) + (5% × 1.8) + (30% × 0.9) + (25% × 0.2) + (15% × 0.8)

= 0.03 + 0.12 + 0.09 + 0.27 + 0.05 + 0.12 = 0.68

The expected return on the portfolio would be

Rp = Rf + β (Rm − Rf) so Rp = 7 + 0.68(20 − 7) = 15.84%

Whilst considering the above, a general rule for investors is to buy high beta shares in a bull market and sell them in a bear market (and replace them with shares having low betas in a bear market).

As discussed above there are several problems with the CAPM which affect its use in portfolio management:

� Whilst beta factors are fairly stable over time, errors in calculating them and genuine statistical variations mean that companies should use industry betas rather than individual company betas in order to obtain reliable results. In addition, changing market demands (e.g. telephones are now considered more of a necessity than 30 years ago) and changing cost structures in firms (firms tend to have a greater proportion of fixed costs than in the past) means that a company’s beta will not remain stable over time.

� The risk-free rate is assumed to be equal to returns on government bonds – but there are several government securities with different terms and interest rates.

� The model deals with expected returns but it is the historical excess return on the market (Rm − Rf) that is used.

� The CAPM and portfolio theory are based on perfect, efficient markets with rational investors who have diversified away all unsystematic risk. In reality, however, investors may not have well-diversified portfolios and will thus be concerned about their total risk. The model also ignores the costs of trading and insolvency – which do exist in the real world and the investor must consider them.

Page 190: libvolume5.xyzlibvolume5.xyz/law/ballb/3rdyear/semester5/corporatelaw/corporate... · ABE Advanced Diploma in Business Administration Study Manual CORPORATE FINANCE Contents Study

182 The Capital Asset Pricing Model

© Licensed to ABE

CAPM and Capital Investment Decisions If CAPM is accepted, it might be concluded that, when deciding whether to invest in a particular project, management should be concerned with its systematic risk and not with its overall risk. If the β factor can be estimated, then it is possible, using the formula given earlier, to calculate the minimum required return on the project, based on the systematic risk of the project. The model can thus be used to compare projects of different risk classes, unlike the NPV method which does not consider risk in its choice of discount rate.

In using the model, management are determining a required rate of return based on market and risk-free rates of returns, the returns on the project and its variation to the market; in so doing they are assuming that shareholders wish them to evaluate such projects as though they were stocks and shares in the market, and that shareholders are fully diversified themselves and have no desire for the company to diversify on their behalf.

E. THE ARBITRAGE PRICING MODEL

The problems associated with the CAPM have led to the development of other models, including the Arbitrage Pricing Model (APM).

The CAPM is often criticised for its simplified relationship between risk and return. The APM, however, assumes that the return on security is based on a number of factors, each of which is independent of the others; and that the expected rate of return on a security is a function of the risk premiums discussed below plus the risk-free rate. The model states that:

r = E(rj) + β1F1 + β2F2 + β3F3 + β4F4 + ……….e

where: E(rj) = the return expected from the security

β1 = the sensitivity to changes in factor 1

F1 = the difference between the expected and actual values of factor 1

β2 = the sensitivity to changes in factor 2

F2 = the difference between the expected and actual values of factor 2

β3 = the sensitivity to changes in factor 3

F3 = the difference between the expected and actual values of factor 3

β4 = the sensitivity to changes in factor 4

F4 = the difference between the expected and actual values of factor 4

e = a random term

In order to determine the factors to which returns on the securities are sensitive, and which form the basis of risk factors, factor analysis is undertaken. Key factors identified include:

� Changes in the expected level of industrial production

� Unanticipated changes in the term structure of interest rates

� Unanticipated changes in inflation

� Changes in the risk premium on bonds

Page 191: libvolume5.xyzlibvolume5.xyz/law/ballb/3rdyear/semester5/corporatelaw/corporate... · ABE Advanced Diploma in Business Administration Study Manual CORPORATE FINANCE Contents Study

The Capital Asset Pricing Model 183

© Licensed to ABE

If it is expected that a certain combination of securities will produce higher returns than indicated by the model then arbitrage trading will occur with the aim of improving expected returns. When no arbitrage opportunities remain, the expected return on a security will be:

E(rj) = rf + β1(r1 − rf) + β2(r2 − rf) + β3(r3 − rf) + β4(r4 − rf) + ……….

where: rf = the risk-free rate

r1 = the expected return on a portfolio which has unit sensitivity to factor 1 and zero sensitivity to any other factor

r2 = the expected return on a portfolio which has unit sensitivity to factor 2 and zero sensitivity to any other factor

r3 = the expected return on a portfolio which has unit sensitivity to factor 3 and zero sensitivity to any other factor

r4 = the expected return on a portfolio which has unit sensitivity to factor 4 and zero sensitivity to any other factor

The advantages of APM compared to CAPM are:

� The need to determine the market portfolio is removed.

� Systematic risk is broken into small components which need not be determined initially.

� APM explains the pricing of securities in relation to each other, as opposed to CAPM which explains pricing in relation to the market as a whole.

The APM, however, does have disadvantages:

� There is a need to identify those factors to which a security is sensitive.

� The model is based on simplifying assumptions including portfolio theory, and perfect competition.

The APM, in common with CAPM, is a method of dealing with risk and return in uncertain conditions. Empirical evidence has cast doubt upon, but not disproved, either model; however, Hill (1995) states that CAPM is the model used in practice to determine the performance of portfolios, mainly because the APM has not as yet been fully developed.

Page 192: libvolume5.xyzlibvolume5.xyz/law/ballb/3rdyear/semester5/corporatelaw/corporate... · ABE Advanced Diploma in Business Administration Study Manual CORPORATE FINANCE Contents Study

184 The Capital Asset Pricing Model

© Licensed to ABE

Practice Question

Calculate the return on a particular share with a beta factor of 0.7, given the following data:

Return on government securities: 6.5%

Market return: 9%

What would happen if market return:

(a) Increased to 12%?

(b) Fell to 5%?

Now check your answer with the one given at the end of the unit.

Page 193: libvolume5.xyzlibvolume5.xyz/law/ballb/3rdyear/semester5/corporatelaw/corporate... · ABE Advanced Diploma in Business Administration Study Manual CORPORATE FINANCE Contents Study

The Capital Asset Pricing Model 185

© Licensed to ABE

ANSWERS TO PRACTICE QUESTION

The return on the share = 6.5 + 0.7(2.5) = 8.25%

(a) The result of the increase in market return is that the return on the share rises as follows:

6.5 + 0.7(5.5) = 10.35%

(b) The result of the fall in market return is that the return on the share falls as follows:

6.5 + 0.7(−1.5) = 5.45%

Page 194: libvolume5.xyzlibvolume5.xyz/law/ballb/3rdyear/semester5/corporatelaw/corporate... · ABE Advanced Diploma in Business Administration Study Manual CORPORATE FINANCE Contents Study

186 The Capital Asset Pricing Model

© Licensed to ABE

Page 195: libvolume5.xyzlibvolume5.xyz/law/ballb/3rdyear/semester5/corporatelaw/corporate... · ABE Advanced Diploma in Business Administration Study Manual CORPORATE FINANCE Contents Study

187

© Licensed to ABE

Study Unit 9

Capital Structure

Contents Page

Introduction 188

A. Capital Gearing 188 Gearing Ratios 189 Valuation Basis 191 Scientific Approaches 191

B. Factors Determining Capital Structure 192 Ability of Earnings to Support the Structure 192 Attitudes of Capital Suppliers 193 Patterns of Assets and Trading 194 Demand Patterns 195 Attitudes of Management and Proprietors 195

C. Theory of Capital Structure 196 Traditional View of Capital Structure 196 Modigliani and Miller 197 Impact of Taxation on the Cost of Capital and Capital Structure Decisions 200

D. Capital Gearing and the Effects on Equity Betas 202

Page 196: libvolume5.xyzlibvolume5.xyz/law/ballb/3rdyear/semester5/corporatelaw/corporate... · ABE Advanced Diploma in Business Administration Study Manual CORPORATE FINANCE Contents Study

188 Capital Structure

© Licensed to ABE

INTRODUCTION

Gearing is the proportion of debt within a company’s capital structure, measured as debt/equity generally at market values. We have seen in previous study units that a high level of gearing increases the financial risk of a firm and the required return of shareholders. A high level of gearing may also affect the return required on debt. Thus, the level of gearing of a firm could impact on the company’s WACC, and obviously the optimal level of gearing is where the company’s cost of capital is minimised.

However, whether gearing does affect the cost of a company’s capital is an area of debate in finance – the two main schools of thought are the traditional view and the theories of Modigliani and Miller (known commonly as MM).

Before going on to discuss them we shall consider gearing in some detail – looking at the principal factors which influence the financial manager in choosing capital instruments to maintain balance in the overall capital mix. We have also talked about some of the practical ideas for day-to-day working.

Perhaps the most important point to emerge is that capital gearing is not a simple ratio calculation with firmly defined ingredients, but more of a multi-dimensional problem. A series of factors interact to establish a capital mix, and an appreciation of those factors is important before beginning to attempt financial management in this area.

A. CAPITAL GEARING

The mix of the various types of capital employed within a business is referred to as the capital gearing or leverage of the organisation. Total fixed and current assets have to be financed. Some will be financed by equity capital, i.e. the ordinary shares and the reserves belonging to the shareholders, and some will (usually) be financed by debt capital, i.e. all fixed-interest-bearing financial instruments.

There are two basic states to be distinguished.

� High Gearing

When the proportion of debt compared with equity is high, the structure is said to be high-geared. Typical examples may arise in heavy manufacturing firms where investment in long life, high cost plant means that large sums have to be invested using borrowed funds.

� Low Gearing

When the proportion of debt capital to equity capital is low, the structure is said to be low-geared. Service industries and supermarket chains generally have low gearing ratios, because they do not have to invest heavily in plant and machinery. Supermarkets are typically cash-based entities and will often receive payment from customers who shop there before they have to settle with their suppliers. As a result, their need to resort to external financing is minimal unless they embark on a major store opening, or refurbishment, programme.

The gearing ratio of a business will, therefore, be largely determined by the nature of its operations. It follows that particular industries will show common characteristics. Where a prospective investor, or lender, is considering an investment, he or she will look at the typical gearing ratio for that market sector to compare the efficiency of the business at managing its financing needs, and will query significant variances which cannot obviously be obtained from the published accounts.

Page 197: libvolume5.xyzlibvolume5.xyz/law/ballb/3rdyear/semester5/corporatelaw/corporate... · ABE Advanced Diploma in Business Administration Study Manual CORPORATE FINANCE Contents Study

Capital Structure 189

© Licensed to ABE

Generally speaking, the accepted “norm” in the UK is to maintain a balance of debt capital to total capital of 1:2, i.e. to finance half of the total assets with debt capital.

Gearing Ratios Capital mix and capital problems can be analysed by a number of different gearing ratios, the principal ones being:

(a) Prior chargecapitalEquity capital

(b) capital chargePrior + capitalEquity

capital charge Prior

(c) employed capital TotalcapitalchargePrior

You should always make it clear which ratio(s) are being used and how any figures are arrived at.

� Prior charge capital is anything appearing as a charge on the profit of the business prior to taxation and dividend. The term includes debentures and long-term debt, and possibly short-term debt.

� Total capital employed, in its simplest form, will be the total assets less current liabilities. However, you should note that certain items may or may not be included – examples are deferred tax and minority interests. You should always remember to state how you have arrived at your assumptions.

The following is a short example to help to clarify these points.

Consider the following balance sheet.

Page 198: libvolume5.xyzlibvolume5.xyz/law/ballb/3rdyear/semester5/corporatelaw/corporate... · ABE Advanced Diploma in Business Administration Study Manual CORPORATE FINANCE Contents Study

190 Capital Structure

© Licensed to ABE

PQR plc Balance Sheet at 31 December ....

£000 £000 £000 Fixed assets 4,250 Current assets Debtors 200 Stock 400

600 Creditors: amounts falling due within one year Creditors 100 Bank loans 175 Overdrafts 50 325

Net current assets 275

Net assets 4,525 Creditors: amounts falling due after more than 1 year Debentures 500 Bank loans 500 (1,000)

3,525

Capital and reserves £000 Ordinary shares 2,000 Profit and loss account 1,000 Preference shares 500 Share premium account 25 3,525

Applying the different gearing ratios we get the following interpretations of capital mix.

(a) Prior chargecapital

Equity =

25000,1000,2500500500++

++ = 025,3500,1 × 100% = 49.59%.

Note that prior charge capital is made up of:

£000Debentures 500Bank loans (of more than 1 year) 500Preference shares 500

1,500

If short-term loans and overdrafts were included in prior charge capital, this figure would become (1,500 + 175 + 50) = 1,725 and the gearing ratio would rise to 57.02%. Equity is taken as total capital and reserves excluding preference shares.

Page 199: libvolume5.xyzlibvolume5.xyz/law/ballb/3rdyear/semester5/corporatelaw/corporate... · ABE Advanced Diploma in Business Administration Study Manual CORPORATE FINANCE Contents Study

Capital Structure 191

© Licensed to ABE

(b) capital chargePrior +Equity capital chargePrior =

500,1025,3500,1+

× 100% = 33.15%.

Again, by including short-term borrowings, the gearing ratio would rise to:

725,1025,3725,1+

× 100% = 36.32%.

(c) employed capital Totalcapital charge Prior = 1 500

4 525,,

× 100% = 33.15%.

It is not really appropriate to include short-term borrowing in this particular ratio because it has already effectively been allowed for in the calculation of net assets.

Valuation Basis Opinions are divided as to whether the relevant debt and equity contents should be valued in terms of book values or market values. Many businesses may well revalue their investment in bricks and mortar (real property) in view of a decline in market values.

By using market values of ordinary shares, a value will automatically be placed upon the share capital and the shareholders’ reserves, because the market is assumed to have taken the value of reserves into account in determining the market price of the shares. Similarly, the market value of debt capital will be taken to reflect more realistically the market opinion and therefore the risk of that debt. A market-based approach is, of course, dynamic in the sense that the gearing ratio will alter as soon as market values alter.

This is fine to an extent with quoted companies whose share prices can be readily determined at any given point in time. Problems occur, however, with the private company, partnerships and sole traders in view of the difficulty of attempting to arrive at a market capitalisation. Supporters of the book value approach will, in attempting to take this into account, argue that market values may not always reflect the real long-term position. For instance, strike action in a particular industry may have an adverse impact on the securities of a company in the short term which will, by virtue of the market-based approach, be reflected in a temporary and unrepresentative gearing ratio.

It is of fundamental importance to see that all assets are correctly valued, and you should note that book values may not always be realistic, as a result of changes in the property market impacting on the valuation of land and buildings or customer fashions reducing (or increasing) stock values, etc. A decision will also have to be made as to whether to include intangibles such as goodwill, patents and brand names. Goodwill may have arisen through paying more than the book value to acquire an asset, or group of assets, and it will generally be deducted from the total asset values since it represents a historic figure which may no longer apply.

Scientific Approaches Beyond the simplistic target of financing only 50% of assets, more scientific approaches can be applied. For example, long-term debt may be limited to a chosen percentage of equity funds. This approach has the following problems:

� The maximum may become the norm.

� Unless maturity dates for debt finance are widely separated, the cash implications of finding large sums for redemption may cause problems.

Page 200: libvolume5.xyzlibvolume5.xyz/law/ballb/3rdyear/semester5/corporatelaw/corporate... · ABE Advanced Diploma in Business Administration Study Manual CORPORATE FINANCE Contents Study

192 Capital Structure

© Licensed to ABE

Short-term debt should be restricted to satisfying short-term needs. If a rollover of debt is required, the financing of changing interest commitments, as well as the possibility of not being able to arrange refinancing, can lead to difficulties.

Another more scientific approach is to consider the number of times a fixed interest payment will be covered by annual earnings, which gives an indicator of financial risk. The main problems here are:

(a) The determination of an optimum financial risk measure for a particular company.

(b) Earnings may not always be representative of the cash available to meet interest payments. Debtors cannot be used to finance debt until they have paid.

Because of (b) above, a rather more detailed approach is to base the level of corporate indebtedness on the ability of the cash flow to support it. A potential debt provider is more likely to be encouraged to supply funds where he or she can see that the cash flow to fund the interest payments is planned to be available.

Whilst failing to reach an answer to the optimal gearing level question, the theorists have highlighted a number of factors a firm should consider, including:

� Its future taxable capacity

� Risk and volatility of future earnings

� Interest cover

� Likely costs of financial distress

� Availability of other sources of finance

� Debt capacity (i.e. assets that can be secured, presence of covenants).

B. FACTORS DETERMINING CAPITAL STRUCTURE

Ability of Earnings to Support the Structure When the assets to be financed cost £100 and the earnings generated by them are £10, then such a level of earnings could only service the £100 if the return expected by the ordinary shareholders for a class of risk of this type was 10%. Thus, all the earnings would have to be paid out as dividends.

If the dividend required was, say, 12%, then an alternative structure would be necessary to overcome the problem that the earnings were only £10. Examples of two alternatives are given below (in both cases we will continue to use our £100 basis).

Capital Earnings Required £ £ Ordinary shares 50 Ordinary shares at 12% 6 Debentures 50 Debentures at 8% 4

Capital 100 Earnings 10 Or we could have:

Page 201: libvolume5.xyzlibvolume5.xyz/law/ballb/3rdyear/semester5/corporatelaw/corporate... · ABE Advanced Diploma in Business Administration Study Manual CORPORATE FINANCE Contents Study

Capital Structure 193

© Licensed to ABE

Capital Earnings Required £ £ Ordinary shares 40 Ordinary shares at 12% 4.8 Preference shares 30 Preference shares at 7% 2.1 Debentures 30 Debentures at 8% 2.4

Capital 100 Earnings 9.3 Available for reserves 0.7

10.0 Simple though the example is, it should clarify in your mind how the financial manager can combine securities to arrive at the optimum capital structure for the company. As we can see, by using less risky, fixed-interest capital, it should be possible to reduce the demands on equity amounts. In other words, the earnings expectation can be geared down.

The earnings of the capital, the company’s policy in paying dividends or distributing retained earnings, and the return required by the providers of capital will all influence the pattern of finance that the business is able to raise. In turn the financial manager will take account of present, and predicted, future interest rates in an assessment of the most suitable security to be issued.

Attitudes of Capital Suppliers Potential suppliers of capital or equity will take account of other factors in addition to the rate of return offered by the company.

� Providers of debt capital will consider the security offered and the ability of the business to meet its interest payments (i.e. the interest cover). In the first of our two examples above, debenture interest is covered 2½ times by the earnings of 10%. Typically an unsecured lender would look for cover of between three and five times and we can therefore assume that security would be required in this case.

� Providers of equity capital must allow all other forms of capital to be serviced before their dividend can be paid. They will look closely at the debt holder’s stake as the volume of debt will significantly affect ordinary dividends in times when earnings fall.

Let us consider the following, which assumes total payout and no retention. Taxation has been ignored.

Page 202: libvolume5.xyzlibvolume5.xyz/law/ballb/3rdyear/semester5/corporatelaw/corporate... · ABE Advanced Diploma in Business Administration Study Manual CORPORATE FINANCE Contents Study

194 Capital Structure

© Licensed to ABE

Highly Geared Company

Lowly Geared Company

Ordinary shares £1,000 £9,000 8% Debentures £9,000 £1,000

Capital £10,000 £10,000

Year 1: Earnings £1,500 £1,500 Debenture interest £720 £80

Available for dividend £780 £1,420 Dividend % 78% 15.8%

Year 2: Earnings £720 £720 Debenture interest £720 £80

Available for dividend – £640 Dividend % NIL 7.1%

Debenture interest is, of course, a fixed charge, and the effect of having to service payment when earnings fall is clearly demonstrated. Ordinary shareholders will only be entitled to their dividend after this fixed charge has been met. In Year 1 the earnings are high and the shareholders in the highly geared company obtain a higher return than those in the low geared business. The reverse position is shown when earnings are low, and in our example the shareholders in the highly geared company receive nothing.

The effect of the mixture of debt and equity effectively gears up the effect of fluctuating profits and will generally influence the decision of an ordinary shareholder whether or not to invest. Where gearing is high, dividends can be expected to fluctuate in response to profit fluctuations and it will impact on the share prices in due course.

Hence profit maximisation does not always operate in the best interests of the shareholders’ future wealth. An influx of debt capital may help to generate additional profit, but there will be a risk that it will disturb the financial gearing ratio, with the result that the market will then demand a higher return in order to compensate for what it sees as increased risk. This may result in the share prices falling and the reduction of the shareholders’ wealth in capital gains terms, without a significant increase in future dividend to compensate for the fall.

Concepts of profit maximisation and shareholder wealth must be set against a relative time background. They should not be viewed as simple, absolute requirements. In planning the mix of debt and equity capital, the financial manager must take account of the risk attitude of existing and potential investors.

Patterns of Assets and Trading To some extent at least, the pattern of assets in most companies will dictate the gearing. The use of secured debt capital will, for instance, require some tangible assets on which the security can be perfected. If the business has few tangible assets, it will have to raise its finance through alternative capital instruments.

Page 203: libvolume5.xyzlibvolume5.xyz/law/ballb/3rdyear/semester5/corporatelaw/corporate... · ABE Advanced Diploma in Business Administration Study Manual CORPORATE FINANCE Contents Study

Capital Structure 195

© Licensed to ABE

A second consideration will be the nature of the principal trade of the firm. A stable, well established business, such as a bakery, will generally have less difficulty raising debt capital than, for example, a company engaged in extensive research in aero-engine development and manufacture. This is because the market will consider the former, being well tried and tested, to be less risky.

Companies in, or about to enter, risk activities, such as developing new markets overseas, will be very likely to raise their financing requirements through risk capital (i.e. equity). Companies planning less risky activities, such as a large new building for their own use, will often resort to the use of debt capital because of the ease and relative cheapness with which it can be made available.

Demand Patterns Progressing from the previous point, the demand for the products of a company, or the nature of the industry as a whole, will impact on the amount of debt capital which can be raised. We will consider this under three headings:

(a) Industry and individual demand

When industrial demand does not continue to grow for a protracted period, no matter how well an individual company is performing within that industry, it will eventually suffer the same problems as the industry as a whole. Careful thought should be given to taking on additional debt capital by a buoyant firm in a declining industry, as the eventual drop in orders may make the financing commitments through interest payments difficult to maintain. This may not, of course, be the case where a firm in these circumstances were to raise capital for a diversification project outside the industry concerned.

(b) Sales stability

A steady sales record is generally considered to be a better pointer to future stability of sales performance than a volatile record. A steady record will give confidence to investors and should facilitate raising debt capital.

(c) Competition

Where a company trades in an industry that demands special skills (e.g. computers), or where a large initial investment has to be made on entry to the market (e.g. steel processing), there will be less chance of new competition entering the market. The established business in such markets should find it relatively easy to raise debt capital, because the market will be confident in the firm’s ability to service its debt payments, as it is unlikely to be faced with new and aggressive competitors in the future.

Where market entry is easy and relatively cheap, the certainty that income will continue for a business already established in that industry will be reduced. Investors will typically be more cautious about providing debt capital because of the uncertainty that it can be financed into the future.

Attitudes of Management and Proprietors Many people are instinctively conditioned to avoid borrowing external funds if it can be avoided. Even where borrowing is inevitable, they will try to minimise the extent of their commitments. They have an attitude of “I’ve never owed anybody anything”, a view which perhaps influences their approach to capital gearing.

It is true that secured borrowing, such as a mortgage debenture secured on the company’s premises, may restrict the business in its free use of the building it occupies. Some managers would prefer to retain absolute control of their assets and only use equity. To them the difference in the cost of debt

Page 204: libvolume5.xyzlibvolume5.xyz/law/ballb/3rdyear/semester5/corporatelaw/corporate... · ABE Advanced Diploma in Business Administration Study Manual CORPORATE FINANCE Contents Study

196 Capital Structure

© Licensed to ABE

and equity would be an opportunity cost of having unencumbered use of their buildings. Whether the concept of control by suppliers of debt capital is really valid is open to conjecture, as generally little control is ever exercised until interest payments are missed.

An alternative way in which some managers approach debt capital is to borrow the maximum amount available at present interest rates. This is only limited by financing ability, available security and risk. Such a policy may leave the business open to trouble if interest rates rise (if the rates are not fixed or capped), if costs rise, or if sales fall. The advantage is that increased debt financing may enable the business to make full use of its resources in a profitable way. In addition to which, in conditions of rising inflation, the “real” cost of financing fixed-rate debt will decrease as payments will be made out of “future” pounds, the value of which will have been eroded by inflation.

C. THEORY OF CAPITAL STRUCTURE

We noted earlier that the two main schools of thought are the traditional view and the theories of Modigliani and Miller (MM). Both schools of thought are based on a number of assumptions. To simplify the theories and to highlight their conclusions, we note these assumptions at the outset (although some are later relaxed).

(a) There is no taxation.

(b) There are constant earnings, which are fully paid out as dividends.

(c) There is a widespread expectation of the prospects of the company.

(d) Business or operating risk is constant.

(e) There are no market imperfections such as transaction costs.

(f) Companies are immediately able to alter gearing, e.g. by redeeming or issuing debt.

Traditional View of Capital Structure This view states that as the level of gearing increases, the cost of equity increases and the cost of debt initially remains constant, but once a certain level (not defined) of debt is reached, it starts to increase. The WACC initially falls due to the increasing levels of the cheaper debt, but then starts to increase to reflect the increasing cost of equity (and at higher levels of gearing the increased cost of debt). We can show this graphically (Figure 9.1):

Page 205: libvolume5.xyzlibvolume5.xyz/law/ballb/3rdyear/semester5/corporatelaw/corporate... · ABE Advanced Diploma in Business Administration Study Manual CORPORATE FINANCE Contents Study

Capital Structure 197

© Licensed to ABE

Figure 9.1: Traditional View of Capital Structure

The minimum cost of capital is shown at point A.

Modigliani and Miller To understand the work of MM we must first discuss arbitrage. Arbitrage is any transaction which makes an immediate, risk-free profit, and occurs in a situation when two identical goods or products (including shares) are sold in the same market but at different prices. Obviously such a situation would not last very long – traders would buy at the lower price and sell at the higher price (thus making a profit) until the forces of supply and demand force the lower and higher price, and thus the market, into equilibrium.

In their 1958 paper MM assumed that there are perfect capital markets (i.e. no taxes or transaction costs, rational investors and so forth). They concluded that the value of the firm depends on its assets and the operating income derived from them, and that there is no optimal capital structure. Firms should thus concentrate on maximising the net present value of investments.

The theory is based on the principle of arbitrage and can be illustrated by the following example.

A plc and B plc are identical except that B plc has £60,000 debt outstanding. The cost of the debt is 5%. If the traditional theory is correct B plc will have the higher cost of equity to offset the risk of holding debt. The cost of equity (Ke) in A plc is 15% and in B plc is 16.5%

A plc B plc £ £

Net operating income 20,000 20,000 Interest on debt – 3,000 Earnings available to shareholders 20,000 17,000 Ke 15% 16.5%

Market value of equity (Earnings/ Ke) 133,333 103,030 Market value of debt – 60,000

Total value of firm 133,333 163,030

Page 206: libvolume5.xyzlibvolume5.xyz/law/ballb/3rdyear/semester5/corporatelaw/corporate... · ABE Advanced Diploma in Business Administration Study Manual CORPORATE FINANCE Contents Study

198 Capital Structure

© Licensed to ABE

MM argue that the difference in market values of two identical (except for financing mix) firms will not remain because the arbitrage mechanism will bring the value of the firms into equilibrium. The arbitrage process will occur via investors engaging in home-made leverage, i.e. they will borrow and invest in A plc thus imitating B plc. Some examples will show how this works.

Example

Charlie owns 10% of B plc. His investment of £10,303 (10% of the equity value) is made up of:

£ 10% of the value of the firm 16,303less 10% proportionate share of debt 6,000

10,303 Charlie’s net return, therefore, is £1,700 (10% of £17,000).

Charlie’s wealth could be improved if he:

(a) Sold his equity in B plc for £10,303.

(b) Borrowed £6,000 at 5% (which matches his proportionate share of debt in B plc, and because MM assume perfect capital markets individuals can borrow at the same rate as companies).

(c) Invested the £16,303 in A plc.

This would give Charlie a net return of:

£ Return on equity in A plc (Ke = 15% × £16,303) 2,445.45less Interest on debt (5% × £6,000) 300.00

Net return 2,145.45 Clearly other investors would follow Charlie’s lead and the forces of supply and demand would increase the price of A plc shares and thus lower its cost of equity and vice versa for B plc until their prices were in equilibrium and no arbitrage opportunities remained.

Shareholders therefore can obtain the benefits from gearing on their own account by duplicating the capital structure of the firm and there is thus no benefit to be obtained to a firm from simply changing its capital structure.

If a firm cannot change its value by changing its capital structure, then its weighted average cost of capital (WACC) must remain constant because the value of any firm equals the NPV of future earnings divided by its costs of capital.

This leads to one of MM’s famous “propositions”:

WACCg = WACCug = earnings before interest/WACC

where: g = geared

ug = ungeared

The value of the geared firm is thus equal to the value of the ungeared firm which is equal to the earnings before interest/WACC.

MM then argued that as a company increases its level of gearing the cost of equity increases due to increased financial risk (financial risk arises because there are more fixed charges to pay before

Page 207: libvolume5.xyzlibvolume5.xyz/law/ballb/3rdyear/semester5/corporatelaw/corporate... · ABE Advanced Diploma in Business Administration Study Manual CORPORATE FINANCE Contents Study

Capital Structure 199

© Licensed to ABE

shareholders can obtain any returns), and the increase equals the savings made from the lower cost of debt.

MM’s second proposition states that:

The savings from debt beingcheaper than equity ==== The increase in cost of equity

due to increased risk The increase in financial risk can be calculated as follows:

(Keug −−−− Kd)eg

dVV

where: Vd = value of debt

Veg = value of equity in the geared firm

Keug = cost of equity in the ungeared firm

Kd = the cost of debt

We showed above that the WACC of an ungeared and an identical geared firm are the same and, therefore, the cost of equity in the geared firm must equal the cost of equity in the ungeared firm plus a premium for the financial risk discussed above. Therefore:

Keg = Keug + (Keug − Kd)eg

dVV

where: Vd = value of debt

Veg = value of equity in the geared firm

Keug = cost of equity in the ungeared firm

Kd = the cost of debt

Keg = cost of equity in the geared firm

The model is shown in Figure 9.2:

Figure 9.2: MM Without Tax

Kd

Keg

Level of Gearing

Keug Cost of Capital

Page 208: libvolume5.xyzlibvolume5.xyz/law/ballb/3rdyear/semester5/corporatelaw/corporate... · ABE Advanced Diploma in Business Administration Study Manual CORPORATE FINANCE Contents Study

200 Capital Structure

© Licensed to ABE

Example

Scat plc and Millie plc are identical in every way except that Millie has 25% debt at the risk-free rate of 9%, whereas Scat is all equity financed. Scat’s cost of equity is 12%. Calculate both companies’ WACC.

Scat plc is all equity financed, and therefore its WACC = Ke = 12%.

For Millie:

Keg = Keug + (Keug − Kd)eg

dVV

= 12% + 1/3(12 − 9)

= 13%

WACCg = 25% × 9% (debt) + 75% × 13% (equity)

= 12%

Therefore, WACCg = WACCug

There are several criticisms of MM 1958 theory:

(a) Market imperfections do exist, e.g. there are transaction costs when buying/selling shares.

(b) Personal borrowing cannot be substituted for corporate borrowing because most individuals do not have the capacity to borrow at the levels companies can, nor can they obtain the same rates.

(c) Debt cannot be assumed to be risk-free because companies and individuals can become bankrupt.

(d) MM ignored bankruptcy, and as such their theory may not be valid at very high levels of gearing where the risk of bankruptcy is greatest.

(e) The 1958 paper ignores taxation.

Impact of Taxation on the Cost of Capital and Capital Structure Decisions In 1963 MM modified their work to include the effects of corporation taxes. In many countries (including the UK and the USA) debt interest is allowable against corporation taxes. Tax relief on debt interest is therefore a gain for the shareholders. MM thus argued that the cost of capital declines with gearing, and the firm’s value increases by the present value of the tax relief on debt interest.

This can be shown in a graph (Figure 9.3) and illustrated by an example.

Page 209: libvolume5.xyzlibvolume5.xyz/law/ballb/3rdyear/semester5/corporatelaw/corporate... · ABE Advanced Diploma in Business Administration Study Manual CORPORATE FINANCE Contents Study

Capital Structure 201

© Licensed to ABE

Figure 9.3: MM With Tax

Example

Max plc has earnings before interest of £100,000. Its capital structure is made up of 10% debt at a cost of 5%, and 90% equity. The cost of equity is 10%. Max plc operates in Connahland which does not allow debt interest against taxation, but is considering doing so under a new law. Calculate the change in Max plc’s WACC and market value if the law is passed. Corporation tax in Connahland is 25%.

The weighted average cost of capital before the law would be:

WACC = (Ke × Proportion of equity) + (Kd × Proportion of debt)

= 10% × 90% + 5% × 10%

= 9.5%

Therefore, the value of firm = £100,000/9.5% = £1,052,632

If the law was passed the WACC would be:

WACC = (Ke × Proportion of equity) + (Kd (1 − t) × Proportion of debt)

= 10% × 90% + 5 (1 − 0.25)% × 10%

= 9.375%

Therefore, the value of firm = £100,000/9.375% = £1,066,667

Thus the cost of capital would reduce by 0.125% and the value of the firm would increase by £14,035.

Therefore, the cost of equity for a geared company given above becomes:

Keg = Keug + eg

dVV (Keug − Kd)(1 − t)

and WACCg = WACCug(1 − E + DDt ).

where: D = market value of debt capital in geared company

E = market value of equity in a geared company

Cost of Capital

Kd after tax

WACC

Level of Gearing

Ke

Page 210: libvolume5.xyzlibvolume5.xyz/law/ballb/3rdyear/semester5/corporatelaw/corporate... · ABE Advanced Diploma in Business Administration Study Manual CORPORATE FINANCE Contents Study

202 Capital Structure

© Licensed to ABE

In the example of Scat and Millie above, consider what the situation would be if corporation tax was 35%. Scat plc is all equity financed, and therefore its WACC = Ke = 12% (unchanged from above).

However, for Millie:

Keg = Keug + eg

dVV (Keug − Kd)(1 − t)

= 12% + 1/3(12 − 9)(1 − 0.35)

= 12.65%

and WACCg = WACCug(1 − E + DDt )

= 12(1 − (1 × 0.35)/4

= 10.95%

or more traditionally to prove the formula:

WACCg = 25% × 9 (1 − 0.35)% (debt) + 75% × 12.65% (equity)

= 10.95%

∴ WACCg = WACCug and the conclusion could be drawn that a company should borrow as much as possible – the value of the geared firm will also be greater than the value of the ungeared firm by the tax saving on debt.

However, a firm may reach a point at which it has no taxes to offset debt interest against – this is known as tax shield exhaustion. When such a point is reached there are no longer any benefits to be gained by increasing the level of the firm’s gearing.

The theory, like the previous one, has its limitations in that it again ignores bankruptcy and agency costs. It also ignores personal taxation. (In 1977 Miller considered personal taxes and concluded that there was no optimal debt level.)

Empirical testing of these theories is very difficult in practice and as such the optimal level of gearing (if one exists) remains an unresolved issue. Strictly speaking, they are all hypotheses rather than theories.

D. CAPITAL GEARING AND THE EFFECTS ON EQUITY BETAS

The level of gearing has a significant impact on a firm’s beta and has to be considered when estimating the required rate of return for a new project.

A company’s systematic risk, reflected in its beta, is made up of two main types – operating risk and financial risk. We saw above that a geared company has a higher financial risk, and thus a higher beta, than the equivalent ungeared company.

You do not need to be able calculate betas for geared companies, but you should understand how the calculation is undertaken.

MM show, using their 1963 formula that:

βg = βug [1 + Vd(1 − t)/Veg]

Page 211: libvolume5.xyzlibvolume5.xyz/law/ballb/3rdyear/semester5/corporatelaw/corporate... · ABE Advanced Diploma in Business Administration Study Manual CORPORATE FINANCE Contents Study

Capital Structure 203

© Licensed to ABE

Using this formula it is possible to calculate the operating beta (or equity beta) of a firm. The operating beta shows the risk of the firm’s activities as opposed to its financing structure.

Example 1

Sam plc is an all-equity firm with a beta of 1.5. Sugar plc is identical in all respects except that it has 50% debt in its structure. If the rate of corporation tax is 30% calculate the beta of Sugar.

βg = βug [1 + Vd(1 − t)/Veg]

βSugar = 1.5[1 + 1(1 − 0.3)/1]

βSugar = 2.55

It is important to consider differences in gearing when using one company’s beta to estimate another company’s because, as noted above, increasing the level of gearing a firm has increases its beta. The equity beta of the first company must be found by “ungearing” its company beta, and then “re-gearing” it to match the second company’s capital structure. The procedure is shown in the next example.

Example 2

Arnold Ltd is about to be floated on the Stock Exchange and wishes to estimate its beta. It is very similar to Oliver plc which has a beta of 1.4, except that Oliver plc has 30% debt and Arnold Ltd has 40% debt. Estimate the beta of Arnold using the above information. Assume the tax rate is 33%.

First we have to “ungear” Oliver:

βg = βug [1 + Vd(1 − t)/Veg]

1.4 = βug [1 + 30 (1 − 0.33)/70]

1.4 = βug 1.287

βug = 1.4/1.287

βug = 1.09

Now we re-gear this equity beta for Arnold Ltd’s capital structure:

βg = βug [1 + Vd(1 − t)/Veg]

βg = 109 [1 + 40 (1 − 0.33)/60]

βg = 1.58

Note the higher beta for the higher geared firm, illustrating MM’s formula.

Whilst this is a useful tool to know you must be aware of the limitations in using the formula to estimate betas for firms. The limitations include the general limitations of the CAPM that we discussed above and in the previous unit. In addition, different firms have different cost structures, opportunities for growth and are of different sizes; as such no one firm can be seen to be identical to another.

The model assumes that debt is risk-free. However, in reality corporate debt has a beta of approximately 0.25; it has the effect of overstating geared betas and understating ungeared betas.

Page 212: libvolume5.xyzlibvolume5.xyz/law/ballb/3rdyear/semester5/corporatelaw/corporate... · ABE Advanced Diploma in Business Administration Study Manual CORPORATE FINANCE Contents Study

204 Capital Structure

© Licensed to ABE

Page 213: libvolume5.xyzlibvolume5.xyz/law/ballb/3rdyear/semester5/corporatelaw/corporate... · ABE Advanced Diploma in Business Administration Study Manual CORPORATE FINANCE Contents Study

205

© Licensed to ABE

Study Unit 10

Corporate Dividend Policy

Contents Page

Introduction 206

A. Key Influences on Dividend Policy 206 Retention Policy 206 Availability of Internal Funds 207 Profit Available for Distribution 207 Profits and Dividend Level 208 Effect on Share Prices 209 Concept of Signalling – Investor and Market Expectations 209 Shareholder Expectations 210 Company Law on Distributable Profits 210 Other Influences 210

B. Theories of Dividend Policy 211 Fundamental Theory of Share Values 211 Clientele Effect 212 Modigliani and Miller’s Dividend Irrelevance Theory 212 Dividend Relevancy Theory 212

C. Practical Aspects of Dividend Policy 213 Approaches to the Level of Dividend 214 Non-Dividend Transactions 214

Page 214: libvolume5.xyzlibvolume5.xyz/law/ballb/3rdyear/semester5/corporatelaw/corporate... · ABE Advanced Diploma in Business Administration Study Manual CORPORATE FINANCE Contents Study

206 Corporate Dividend Policy

© Licensed to ABE

INTRODUCTION

We saw in earlier study units that a firm’s dividend policy is one of the three key decisions it must make. Corporate dividend policy is the decision between dividends and capital gains (dividends may include scrip dividends, share splits and other perks).

There is a debate in this area as to whether the market value of a company’s shares (and thus the value of the firm and shareholder wealth) is affected by its dividend policy. Before looking at the theoretical arguments, though, we shall consider some of the practical influences on a company’s choice of dividend policy, including the key decisions of investment and financing policy..

A. KEY INFLUENCES ON DIVIDEND POLICY

The major source of internally generated capital is retained profits. Once profits have been earned, the factor which most affects the amount of retentions is the corresponding amount declared and paid out as dividends. For this reason we shall consider retention policy and dividend policy together.

There are, basically, two opposing positions which could be adopted:

� That a company should pay out all its earnings as dividends and go to the market for additional capital as required. It is said that, in this way, the successful companies with the higher rates of dividend will be best able to raise capital and to flourish, at the expense of the less successful.

� That a company should retain all its earnings and pay no dividends. Investors would maximise their wealth in terms of capital gains, for the company would capitalise its retentions and issue bonus shares for the shareholder to sell if he wished.

Retention Policy The company uses its funds in the pursuit of profit and, where that profit is sufficiently large, it will pay a dividend to shareholders. The surplus then remaining is referred to as retentions and will be available to finance growth and the replacement, as necessary, of the company’s assets.

These retentions of profits which are ploughed back into the business are internally generated capital.

Retentions of profit arise in two forms:

(a) As amounts set aside out of profits prior to determining the amount available for dividends, i.e. provisions of profit.

(b) As the surplus remaining when the shareholders’ dividends have been paid, i.e. retentions. Equally, we could argue that these funds have been invested by shareholders through them foregoing dividends.

Naturally the board must consider the desirable levels of retentions in order to fund future projects and growth. Retained earnings are the most important source of finance for UK companies, providing over 60% of all funding requirements over recent years.

The main reasons for this may be as follows:

� Company managers often mistakenly believe that there is no cost involved when retained earnings are used. As we shall see later there is, in fact, an opportunity cost derived from the idea that shareholders have consented to re-invest these earnings in the company, but it is true to say that there is no cost involving the outlay of cash.

Page 215: libvolume5.xyzlibvolume5.xyz/law/ballb/3rdyear/semester5/corporatelaw/corporate... · ABE Advanced Diploma in Business Administration Study Manual CORPORATE FINANCE Contents Study

Corporate Dividend Policy 207

© Licensed to ABE

� The dividend policy is determined by the directors, who see retained earnings as a ready source of cash to invest in their favoured projects without the need, trouble or expense involved in consulting or raising funds from shareholders and other outsiders.

� By using retained earnings the directors avoid the expense of issue costs and, perhaps more importantly, minimise the risks involved in losing control of the company following an issue of shares or secured debentures.

These considerations are balanced by the shareholders’ need for at least a minimum return of the profits and satisfaction of their investment expectations. Equally, though, a company in search of funds will not be viewed favourably if it is over-generous with its dividends or pays over-generous salaries to its owner-directors (if a limited company).

Availability of Internal Funds The obvious advantage of internally generated funds is that they become available without the formalities of issuing houses, brokers, offering of security and so on. They are completely free of formality but, obviously, the required volume of capital at the required time cannot be made available as easily as it can with external funds, i.e. profits arise as the result of trading and not simply to ordered dates.

Profit Available for Distribution The first consideration is in respect of provisions set aside out of profit.

� Depreciation

The annual charge for depreciation in the profit and loss account does not arise due to an outflow of cash at the time the charge is made. The cash outflow associated with the procurement of an asset (usually) occurs when the asset is first acquired. The outflow is treated as capital expenditure and recorded in the balance sheet. Thereafter, depreciation charges “filter” the capital expenditure from the balance sheet to the profit and loss account at periodic intervals.

Note, however, that the charge in the profit and loss account which actually represents a charging of calculating proportions of the original cost of the asset, reduced by any anticipated scrap value, is not a cash flow-backed item, but a recording of portions of an historic cost incurred on previous occasions.

� Other Items

There are many items where this characteristic of non-cash flow backing occurs, e.g. provision for doubtful debts; provision for plant maintenance; provision for major repairs.

The effect of creating provisions for depreciation and other items is to reduce the profit available to pay out dividends and/or make retentions. Because of this, amounts of cash are held back in the business and are not paid as dividends. They become available for other uses, e.g. fixed asset purchase.

It is important to note that usually, with provisions, a sum of money is not physically separated from the rest in the bank account to substantiate the provision made. The cash not now required for dividends, because provisions have been made, can be put to general use. Some companies may in fact create a separate fund into which the cash backing for their provisions is put. This requires cash to be paid out and an investment purchased. In this case the creation of provisions does not comprise internally generated funds.

Page 216: libvolume5.xyzlibvolume5.xyz/law/ballb/3rdyear/semester5/corporatelaw/corporate... · ABE Advanced Diploma in Business Administration Study Manual CORPORATE FINANCE Contents Study

208 Corporate Dividend Policy

© Licensed to ABE

Where, however, the creation of provisions, their eventual spending and consequent replacement is a more or less continuous process, then such provisions represent a significant proportion of internally generated funds.

In practical terms it is recognised that at least some prudence is necessary, at least in times of changing prices, to prevent the “real” value of the business being depleted. You can see this in current cost accounting, where revaluation surpluses and deficits arising from changes in the prices of fixed assets and stock etc. are taken to a non-distributable reserve. The idea behind this is that if such amounts were distributed it would imply that the operating capability of the business had been eroded.

You should not, however, think that the “current cost profit attributable to shareholders”, apparent from current cost accounts, can prudently be distributed. Other matters must be considered, such as cash availability, capital expenditure plans, changes in volume of working capital, the effect on funding requirements of changes in production methods and efficiency, liquidity, and new financing arrangement, as well as the effect of price changes on the finance required.

Profits and Dividend Level Assuming that there are sufficient profits available, it is extremely difficult to determine what influences a board of directors in declaring a particular level of dividends and hence making a particular level of retentions.

You might think that dividends would be increased in line with increases in profits. However, there is a noticeable tendency in practice for there to be a time lag before dividends are increased following an increase in profits. This is because companies like their dividend rate to be maintained and to increase steadily rather than fluctuate from year to year. Directors will tend to wait until an increase (or decrease) in profits appears to be sustained before building it into the rate of dividend. This controlled, stable approach is felt to produce confidence by investors in the financial management of the company.

From this approach a behaviour pattern can be determined for the process of fixing dividend levels. The decision becomes strongly related to past behaviour:

� What was last year’s dividend?

� What are this year’s available profits?

� Should the dividend be increased or decreased?

� If so, by how much?

� Can that position be sustained?

The effect of this cautious behaviour is to produce a time delay between increases in profits and consequent increases (or decreases) in dividends.

This time delay is important for two reasons:

(a) As a company’s profits increase – and since the directors will tend to delay dividend increases – retentions will increase. If a company’s profits decrease, retentions will be decreased.

(b) Generally, when an increase in dividend rate is announced, it will be fair to assume that the directors feel that the increased level will be capable of being sustained in the future. Alternatively, dividend level will only be reduced if the directors feel that the existing level cannot be maintained in the future.

Page 217: libvolume5.xyzlibvolume5.xyz/law/ballb/3rdyear/semester5/corporatelaw/corporate... · ABE Advanced Diploma in Business Administration Study Manual CORPORATE FINANCE Contents Study

Corporate Dividend Policy 209

© Licensed to ABE

Effect on Share Prices The impact on investors of the level of both dividends paid and earnings retained need to be considered.

(a) Dividends

The market price of a share is a single point indicator of all the expectations and interpretations of the future investment suitability of the company by its shareholders. “Shareholders” will represent a complete cross-section of the financial institutions and the investing public, all with their differing motives and requirements from their shareholding. Thus, so many factors are built into the share price that it is impossible to isolate any one factor with certainty. Generally, however, in line with reason (b) stated above, if a dividend rate is increased, the investors will assume that the new level is likely to be at least maintained and, since the yield from the shares has increased, and increase in market price will usually follow. In the long term, share prices tend to follow the yield of the share.

Although not all shareholders will be holding shares for the dividends offered – they will be primarily interested in capital gains – dividends are undoubtedly a tangible return from the investment and are likely to have a marked effect upon share prices.

(b) Retentions

To make retentions is to defer the time when shareholders receive dividends. By making retentions (“ploughing back” profits) the company is growing and will be able to earn larger profits, pay bigger dividends, grow still bigger and so on. So the promise is that by retaining funds the company can invest them (in itself) and gain a better return than if they had paid the funds over to the shareholders for them to invest elsewhere (i.e. externally to the company).

There is thus an implied promise that dividends in the future will be increased. This promise may well be seen as more risky than tangible amounts of dividend at the present moment, and usually a less marked increase in share prices follows retention than follows increased dividends.

There have been many surveys and studies on this topic, involving different sized companies and types of industry with different shareholding patterns, set against varying periods of economic activity, rates of inflation, world situations and Stock Market confidence. Not surprisingly, there are conflicting findings by the various authorities, not only because of the factors indicated but primarily because such studies attempt to quantify, in mathematical terms, the behaviour of a large and diverse group of human beings.

Concept of Signalling – Investor and Market Expectations This brings us to the concept of “signalling”. In reality investors do not have perfect information, particularly about the prospects of the company. The pattern of dividend payments is therefore taken as a key to estimating future performance.

An increase in dividends is taken as a signal of increased management confidence and leads investors to increase their estimates of future earnings thereby causing a rise in the share price. A dividend cut, on the other hand, is taken as a bad sign and the share price may decline.

In practice, many factors are already discounted by the market in the prevailing share price, so movement only really occurs on the announcement of a dividend if the amount is different to that which the market was expecting anyway. The dividend announced merely confirms the market’s expectations. This gives directors the opportunity to enhance expectations by an unexpectedly high

Page 218: libvolume5.xyzlibvolume5.xyz/law/ballb/3rdyear/semester5/corporatelaw/corporate... · ABE Advanced Diploma in Business Administration Study Manual CORPORATE FINANCE Contents Study

210 Corporate Dividend Policy

© Licensed to ABE

dividend (which must be sustainable in future years) or of reducing expectations by an unexpectedly low dividend.

Shareholder Expectations No matter what their preference for dividends as opposed to capital growth in the value of their shares, all shareholders will hold some expectations about what the dividend should be. Often this is based on prior dividends and a vague idea of an acceptable pay-out ratio, so that as profits grow there is an expectation that dividends should keep in step.

Furthermore, it is generally accepted amongst shareholders that dividends should match those declared last year or show an improvement. A stable dividend is taken (quite wrongly, perhaps) as a sign of a stable company. This often forces directors to declare too large a dividend when losses have been incurred in order to “save face” and prevent the share price falling. Having to “pass” either a final or interim dividend became a subject of great concern during the recession of the early 1990s.

Company Law on Distributable Profits The Companies Acts lay down stringent rules which govern the power of a company to declare a dividend. Under S.263 of the 1985 Act, for instance, it is restricted to the maximum of the aggregate of accumulated realised profits less accumulated realised losses. Thus, in this instance, it is the current net balance of distributable reserves which is the important figure.

In addition, public companies are also constrained by S.264 in that a dividend may only be paid if net assets, after payment of the dividend, are equal to or greater than the total of called-up share capital plus any non-distributable reserves. The latter includes:

� Share premium account

� Capital redemption reserve

� Accumulated unrealised profits less accumulated unrealised losses not yet written off

Thus, the difference here is that unrealised profits and losses must also be taken into consideration, e.g. asset revaluations which have not yet yielded a profit or loss (depending on whether the revaluation was up or down).

Public companies usually pay dividends twice a year: an interim dividend after the interim results, and a final dividend, but only after the final results (and therefore this final dividend) have been approved by the shareholders.

Remember, shareholders have the power to reduce the dividend payable, but not to increase it. Company directors are therefore in a strong position and for all practical purposes shareholders are often obliged to accept the proposed dividend.

Other Influences

(a) Personal taxation of shareholders

Different shareholders will suffer different rates of personal taxation on any dividend income which they receive. Thus, if a dividend of 10p per share is declared, those paying income tax at 25% will receive a net amount of 7.5p, whereas those paying 40% will only receive 6p. Therefore, if the rate of capital gains tax is lower than the shareholder’s marginal rate of taxation, he will prefer a situation in which less is distributed as dividend but is instead retained within the business to provide capital growth.

Page 219: libvolume5.xyzlibvolume5.xyz/law/ballb/3rdyear/semester5/corporatelaw/corporate... · ABE Advanced Diploma in Business Administration Study Manual CORPORATE FINANCE Contents Study

Corporate Dividend Policy 211

© Licensed to ABE

(b) Government policy: dividend restraint

From time to time the Government has operated a policy of dividend restraint as part of a (prices and) incomes policy. No such constraints exist at present. Equally governments could restrict the flow of funds to investors outside its borders or to certain groups or individuals if they were so disposed.

(c) Profitability

As we have seen, the profitability of a company is a key factor in the amount which can be paid out in dividends. If profits are volatile it is unwise to commit the firm to a high pay-out.

(d) Inflation

In times of high inflation dividends based on historic cost profits can lead to distribution of the company’s capital almost inadvertently, thereby reducing the operating capacity of the business.

(e) Growth

Rapidly growing companies may prefer to re-invest the bulk of their earnings rather than distribute them as dividends. This is often the case with newly-formed companies. Alternatively some companies (perhaps those backed by venture capital) will be obliged to offer higher dividends because of their relatively riskier investment.

(f) Other sources of finance

Unquoted companies in particular may find it difficult to access other sources of finance. Retained earnings are important and dividends will therefore tend to be small.

(g) Control

By using internally generated funds ownership or control is not threatened and directors are free to use such funds as they see fit rather than convince new investors of the benefits of their schemes.

(h) Cash flow considerations

A company declares a dividend out of its Profits After Taxation. This is a dividend net of any tax, and the full amount will have to be paid.

(i) Corporate taxation

Companies are obliged to pay Advanced Corporation Tax (ACT) on all dividends paid.

B. THEORIES OF DIVIDEND POLICY

There is a debate as to whether the market value of a company’s shares (and thus the value of the firm) is affected by its dividend policy. This is reflected in the several theories regarding dividend policy, all of which have one common premise – that the aim of dividend policy should be to maximise shareholder wealth (which depends on both current dividends and capital gains).

Fundamental Theory of Share Values This model, also known as the traditional model, states that the level of dividends paid is important.

The fundamental theory of share values (which assumes that the market value of the company depends on the size and growth rate of dividends paid, and the rate of return required by shareholders) values the company using the dividend growth model. The implications of this theory are that

Page 220: libvolume5.xyzlibvolume5.xyz/law/ballb/3rdyear/semester5/corporatelaw/corporate... · ABE Advanced Diploma in Business Administration Study Manual CORPORATE FINANCE Contents Study

212 Corporate Dividend Policy

© Licensed to ABE

shareholders will want management to pursue a distribution/retention policy which will maximise the level of, and growth in, dividends.

Clientele Effect There may not, however, be an optimal distribution/retention policy that the firm can adopt to meet the needs of all shareholders, because of the different taxes (capital gains and income tax) and tax rates borne by different investors. A company should choose and maintain one policy which maximises one group of shareholders’ wealth. Shareholders will then migrate to companies which operate a policy in line with their needs – this is known as the clientele effect.

The changes in the treatment of tax credits in the 1997 Budget will have a major impact on the preferred dividend policy of pension funds. Until the 1997 (Labour) Budget, pension funds were able to claim back tax credits on dividends received. As a result of the Budget changes dividend yield will reduce in significance, as will the preference for dividends over capital gains by this particular clientele.

Modigliani and Miller’s Dividend Irrelevance Theory Modigliani and Miller’s dividend irrelevance theory argues that the value of a company is determined by the NPV of the investments undertaken by the company, and not by any distribution policy.

MM showed that changes in the value of a firm’s shares are not dependent on the actual pattern of dividends paid (you do not need to know the workings of proof for this theory). They argue that if a company issues a dividend from retained earnings, and then needs to raise cash for an investment, the loss on shares of the additional finance is exactly equal to the dividend paid, and a company should therefore be indifferent as to its dividend policy. Moreover, whilst accepting the existence of the clientele effect, MM state that the type of clientele a firm has will have no effect on a firm’s value.

This argument assumes perfect capital markets and rational investors, and these assumptions are the basis of the criticisms of MM’s model:

(a) It assumes that there are no transaction costs or difficulties in borrowing.

(b) It assumes that share issue costs are zero, and ignores the potential problems of capital rationing.

(c) There is no taxation. In reality:

� The timings of ACT payments discourage high dividend payouts by companies.

� Taxpayers may have a slight preference for capital gains.

� Some taxpayers will be indifferent between capital gains and dividends.

(d) Shareholders receive all relevant information about the company’s reinvestment plans. This ignores competition and different perceptions of risk.

Dividend Relevancy Theory There are several arguments that support the idea that the level of dividend will have an effect on the value of the firm – the dividend relevancy theory.

� The effects that different tax rates have on investors’ preferences.

� Market imperfections mean that a positive NPV project will not be automatically reflected in the firm’s share price, but its effects will be shown over time, whereas a reported dividend has a much quicker impact on the share price.

Page 221: libvolume5.xyzlibvolume5.xyz/law/ballb/3rdyear/semester5/corporatelaw/corporate... · ABE Advanced Diploma in Business Administration Study Manual CORPORATE FINANCE Contents Study

Corporate Dividend Policy 213

© Licensed to ABE

� Empirical evidence shows that investors prefer a constant dividend policy with either constant dividends or dividends growing at a constant rate helping them plan their finances.

� If capital rationing exists then it may be cheaper (because of issue costs and so forth) for the firm to retain its earnings and pay a lower level of dividend.

� Uncertainty as to the future means that shareholders may prefer a certain dividend to an uncertain capital gain.

C. PRACTICAL ASPECTS OF DIVIDEND POLICY

In practice a company must consider several factors when determining its dividend policy. Note that it is the directors who determine dividend policy.

� It has to match its dividend policy to its clientele, to prevent a mass buying and selling of its shares.

� If a company faces a takeover, management might declare an increased dividend as a defence. The market might perceive the increased dividend as a sign of improved future profitability of the company and its share price will rise. This makes it more expensive for any potential takeover bid. In 1996 National Power paid a special dividend of £1 per share. At the time the company was subject to a hostile takeover bid from a US electricity company.

� High dividend payouts reduce the risk that shareholders may lose all their investment.

� Dividends act as a signal to indicate the future prospects of the company – a sudden cut in dividends indicates that the company is experiencing difficulties, and vice versa for an increase in dividends.

� The market expects companies in general to follow industry practice.

� The company should finance as much investment as possible using retained earnings, to avoid finance issue costs, and an issue of equity capital may have an impact on the status quo of shareholding control and may leave the company open to a takeover bid.

� The necessity for companies to repatriate overseas profits.

� The level of profits made and the legal position regarding the payment of dividends in its country (or countries) of operation.

� Repayment of debt and any restrictions on dividend payments under loan agreements.

� The liquidity of the company should be considered, together with the timing of cash flows (the company must remain solvent and have enough cash to pay any dividends it declares).

� The level of inflation affects the level of dividend which can be paid – companies must ensure they have sufficient capital to maintain their operating capability.

� The company’s gearing level can have an effect on its dividend policy – any reduction in retained earnings will increase its gearing ratio.

� The company may have a large amount of cash which it may wish to return to its shareholders, especially if its future cash flow predictions are strong. A large amount of surplus cash was another reason for the 1996 National Power special dividend mentioned above.

Page 222: libvolume5.xyzlibvolume5.xyz/law/ballb/3rdyear/semester5/corporatelaw/corporate... · ABE Advanced Diploma in Business Administration Study Manual CORPORATE FINANCE Contents Study

214 Corporate Dividend Policy

© Licensed to ABE

Approaches to the Level of Dividend Three approaches may be identified.

(a) Dividend cover

This is the earnings per share divided by the net dividend per share. This indicates the number of times the same dividend could have been paid on the shares from the current year’s earnings alone.

One dividend policy is to pay out a fixed dividend per share each and every year, which could be fixed in either real or money terms. This is the most common policy, with most companies going for stable, slightly rising dividends per share – the “ratchet” dividend policy.

(b) The pay-out ratio

Pay-out ratio is the relation of dividends paid to ordinary shareholders to the earnings available to be paid out to ordinary shareholders. As such, it includes previous cumulative earnings.

Another dividend policy is to maintain a constant pay-out ratio. Naturally the dividend will fluctuate each year depending on the level of retained earnings and any movements in reserves. This policy is quite rate in listed companies, though many firms may work towards some notional target pay-out ratio.

(c) Residual value

A third, less common, dividend policy is to use retained earnings to fund all projects which show a positive net present value (NPV) at the firm’s weighted average cost of capital (WACC) and pay out any remaining funds as dividends.

This is possibly the most objective dividend policy for rational investors but the concerns of “signalling” (see above) often overrule such a common sense approach.

Non-Dividend Transactions You should not forget that there are alternatives to dividend payouts, as follows.

(a) Scrip issues

A scrip issue, also referred to as a capitalisation, or a bonus issue, involves the conversion of reserves into capital, causing a fall in the reserves. Shareholders receive additional shares in proportion to their holding. Unlike a rights issue no additional funds are brought into the company – the shareholders do not “pay” for these shares. There is then more equity in circulation with the result that the market value will generally fall in the short term, thus making it more attractive to potential investors. The reserves used are either from a credit balance in the profit and loss account, or from reserves specifically marked for the payment of shares, and authority is required from the articles of association and the AGM.

(b) Scrip dividends

Scrip dividends are a conversion of profit reserves into issued share capital offered to shareholders in lieu of a cash dividend. Enhanced scrip dividends are those where the value of shares is greater than the cash dividend offered as an alternative. Such dividends are of benefit to the company as they maintain cash within the business and are not liable to advance corporation tax. There may, however, be tax complications arising for individual investors.

Page 223: libvolume5.xyzlibvolume5.xyz/law/ballb/3rdyear/semester5/corporatelaw/corporate... · ABE Advanced Diploma in Business Administration Study Manual CORPORATE FINANCE Contents Study

Corporate Dividend Policy 215

© Licensed to ABE

(c) Stock split

A stock split is the splitting of existing shares into smaller shares, e.g. each ordinary share of 50p is split into two of 25p, in order to improve marketability of the company’s shares. It can also be used to send signals that the company is expecting significant growth in EPS and dividends per share, and for this reason the resulting market price of the split shares is higher than the simple split price would be. For example, if a share with a market value of £10 was split into two shares their price would be higher than £5. Reserves are not affected.

(d) Shareholder concessions

A number of companies offer discounts of one form or another to their shareholders. These can be thought of as a dividend in kind and are a useful marketing tool or publicity exercise, e.g. Sketchley offers a reduction in dry cleaning bills to its shareholders.

While these are often attractive to small shareholders (institutions rarely qualify, in any case) they are often not a good reason for investment. The concession can be removed at any time and often a significant holding must be maintained.

An advantage, however, is that they are tax exempt!

Page 224: libvolume5.xyzlibvolume5.xyz/law/ballb/3rdyear/semester5/corporatelaw/corporate... · ABE Advanced Diploma in Business Administration Study Manual CORPORATE FINANCE Contents Study

216 Corporate Dividend Policy

© Licensed to ABE

Page 225: libvolume5.xyzlibvolume5.xyz/law/ballb/3rdyear/semester5/corporatelaw/corporate... · ABE Advanced Diploma in Business Administration Study Manual CORPORATE FINANCE Contents Study

217

© Licensed to ABE

Study Unit 11

Working Capital and Short-Term Asset Management

Contents Page

Introduction 219

A. Working Capital 219 Rate of Turnover of Working Capital 219 Ratios Associated with the Assessment of Working Capital 221

B. Overtrading 222

C. Cash Management 224 Cash Flow Planning 224 Margin of Safety 225 Cash Management Problems 226 Cash Ratios 226 Factors Affecting Cash Resources 227 Cash Management Models 227

D. Management of Stocks 230 Cost of Stockholding 230 Stock Turnover Ratios 231 The 80:20 Rule 231 Economic Order Quantity 232 Just-In-Time (JIT) Method of Procurement 234

E. Management of Debtors 235 Debtors’ Turnover Ratio 235 Actions Available to the Company 235 Credit Control 236 Establishing Credit Limits and Terms 238 Debt Recovery and Management 239 Management Control Information 240

(Continued over)

Page 226: libvolume5.xyzlibvolume5.xyz/law/ballb/3rdyear/semester5/corporatelaw/corporate... · ABE Advanced Diploma in Business Administration Study Manual CORPORATE FINANCE Contents Study

218 Working Capital and Short-Term Asset Management

© Licensed to ABE

Credit Insurance 240 Trading Abroad 241

F. Creditor Management 242

G. Short-Term Finance and Investment 243 Management of Short-Term Finance 243 Short-Term Investments 243 Specialist Sources Of Finance 244

Answers to Practice Questions 250

Page 227: libvolume5.xyzlibvolume5.xyz/law/ballb/3rdyear/semester5/corporatelaw/corporate... · ABE Advanced Diploma in Business Administration Study Manual CORPORATE FINANCE Contents Study

Working Capital and Short-Term Asset Management 219

© Licensed to ABE

INTRODUCTION

Funds employed in an organisation can be split for planning purposes into long-term and short-term funds. As we have seen in an earlier module, the financial manager will attempt to match the funding available to the business to the life of the investment it is required for. In this study unit we will consider the management of short-term funds applied to funding current assets.

We shall start by looking at the definition of working capital, its constituent parts and relationship to fixed capital, and some common ratios that can be applied as a management tool. We then go on to consider the implications of cash management. Balance in cash flow management is very important: too little cash resources highlights impending danger, but too large a reserve of cash will mean that potential future earnings will be impaired.

The financial manager must predict the needs of the business and make suitable funding or investment arrangements – planning the effective use of the financial resources, applying to the working capital of a business. We shall discuss the specialist financial products that are available to help to reduce the cash tied up in debtors, and look at ways in which the company can decide on an optimum period of credit to be granted within the general constraints of the markets in which it operates.

A. WORKING CAPITAL

Working capital is the total amount of cash tied up in current assets and current liabilities, and is calculated by deducting the total amount of current liabilities from the total amount of current assets. Thus if A plc has current assets of £10m and current liabilities of £6m then its working capital resources are £4m. Working capital is sometimes expressed as the current ratio (see below).

The finance needed to fund a firm’s required level of working capital can be either short or long term.

It is essential to ensure that a firm has sufficient working capital to allow it to operate smoothly and have sufficient funds to pay its bills when they arise, including taking account of the effects of inflation. However, an organisation should be careful not to over-provide working capital and cause unnecessary cost, a phenomenon known as overcapitalisation.

Over-investment in working capital leading to excessive stocks, debtors and cash, coupled with few creditors, is known as overcapitalisation. Such a situation will lead to lower return on investment and the use of long-term funds for short-term assets. Indicators of overcapitalisation include long turnover periods, high liquidity ratios and a low sales/working capital ratio.

Rate of Turnover of Working Capital An organisation needs to control the rate of turnover of working capital constituents. Whilst reducing the rate of turnover reduces the level of working capital required, it may lead to overtrading (see later).

The rate of turnover of working capital can be determined by calculating the working capital cycle (also called an operating cycle, trading cycle or cash cycle), which shows the relationship between investment in working capital and cash flow.

Page 228: libvolume5.xyzlibvolume5.xyz/law/ballb/3rdyear/semester5/corporatelaw/corporate... · ABE Advanced Diploma in Business Administration Study Manual CORPORATE FINANCE Contents Study

220 Working Capital and Short-Term Asset Management

© Licensed to ABE

The following is an example of a typical working capital cycle:

Raw materials in stock 20 daysWork-in-progress 21 daysFinished goods stock 38 daysPeriod between despatch and invoice to customer 10 daysPeriod from invoicing to customer payment 60 days

Total 149 days If the supplier of the raw materials required payment after 60 days, the company would need to fund the cost of the goods sold for a period of 89 days, and of course wages and other expenses would have to be paid during the period. Steps taken to speed up the rate of working capital turnover, e.g. reducing stock levels, therefore means reducing the company’s investment in working capital.

To illustrate this point further, let’s look at an example.

Example

A company sells £20m of goods throughout the 50 weeks of the working year. As the sales are partly through retail outlets and partly through mail order, daily sales from Monday to Friday can be considered to be equal. The firm banks its takings on Thursday of each week and the incremental cost of banking is £50. The company’s account is always overdrawn and it pays interest on this overdraft of 15% pa (in this example to be applied daily on a simple interest basis).

Management wish to know whether there will be a benefit to banking twice weekly on Monday and Thursday. Investigate the possibility.

£20m over 50 weeks of the year gives a turnover of £400,000 per week and £80,000 per day for a five-day week.

We will now assess the banking alternatives being considered:

Day Receipts Banking Thursday only Banking Monday & Thursday

£000s Days Interest

Charged “£ days”

£000s Days Interest

Charged “£ days”

£000s

Monday 80 3 240 0 0 Tuesday 80 2 160 2 160 Wednesday 80 1 80 1 80 Thursday 80 0 0 0 0 Friday 80 6 480 3 240

960 480 Banking only on Thursday has the same effect as having an overdraft of £960,000 for one day each week. In terms of interest, the cost of this is:

£960,000 × (15% ÷ 365) × 50 = £19,726.*

The annual interest cost of banking twice weekly is:

£480,000 × (15% ÷ 365) × 50 = £9,863.*

(*For interest purposes, we are using a calendar year.)

Page 229: libvolume5.xyzlibvolume5.xyz/law/ballb/3rdyear/semester5/corporatelaw/corporate... · ABE Advanced Diploma in Business Administration Study Manual CORPORATE FINANCE Contents Study

Working Capital and Short-Term Asset Management 221

© Licensed to ABE

Annual incremental banking costs at £50 per time are:

Once weekly, 1 × £50 × 50 = £2,500

Twice weekly, (2 × £2,500) = £5,000.

The total cost of banking on Thursdays only is:

£19,726 + £2,500 = £22,226.

Banking on Mondays and Thursdays costs:

£9,863 + £5,000 = £14,863, a saving of £7,363 pa, assuming constancy of all the factors.

There is one even better solution, still banking twice weekly. Can you decide what it is?

Day Receipts Banking Tuesday & Friday

£000s Days Interest

Charged “£ days”

£000s

Monday 80 1 80 Tuesday 80 0 0 Wednesday 80 2 160 Thursday 80 1 80 Friday 80 0 0

320 The calculation is as follows:

£320,000 × (15% ÷ 365) × 50 = £6,575, and the total cost is:

£6,575 + £5,000 = £11,575.

An alternative way to compare the different banking methods would be to count the number of days interest is charged. Interest is charged when the money is with the business and not in the bank. Paying takings in more quickly means a reduction in interest charged on the overdraft (and less risk of loss or theft).

The summary of the days’ interest is as follows:

Banking Thursday: 12 days

Banking Monday and Thursday: 6 days

Banking Tuesday and Friday: 4 days

If such an exercise was to be conducted over a period of several years then discounted cash flows (see later study unit) would be used.

Ratios Associated with the Assessment of Working Capital In attempting to control working capital a financial manager will use some of the ratios we discussed earlier in the course. However, unlike someone external to the company he will not be restricted by balance sheet figures but will be able to monitor the ratios continually.

Page 230: libvolume5.xyzlibvolume5.xyz/law/ballb/3rdyear/semester5/corporatelaw/corporate... · ABE Advanced Diploma in Business Administration Study Manual CORPORATE FINANCE Contents Study

222 Working Capital and Short-Term Asset Management

© Licensed to ABE

The main ratios that the financial manager will use in this area are the current ratio, quick ratio and acid test ratio.

(a) The current ratio (or working capital ratio) is measured as:

sliabilitieCurrent

assetsCurrent , expressed as a ratio, e.g. 2:1

Whilst there is a target ratio of 2:1, the acceptability of the ratio calculated will depend on the nature of the business, but current liabilities exceeding current assets generally indicate that the business may have problems. In common with all ratios it is important to monitor its trend in order to ascertain whether there are potential problems developing.

(b) The quick asset ratio removes those items which cannot easily and quickly be converted into cash at their full value (i.e. stock) and is calculated as:

sliabilitieCurrent Stock assetsCurrent −

Again there is no ideal ratio; the acceptability of the one calculated depends on the industry (although the target is 1:1). In addition, it is the trend over time that is important.

(c) The acid test ratio is the amount of cash which the firm has to service its current liabilities and is measured as:

sliabilitieCurrent sinvestment Quoted + Deposits +Cash

Again it is the trend that is of most importance.

Companies with poor acid test ratios need to have standby overdraft facilities in order to ensure that the short-term need to service payments of current liabilities can be met. However, remember that too much cash will mean that the firm is underutilising its resources and that a better return could be available elsewhere.

B. OVERTRADING

An increase in a company’s turnover is basically good, but it should be part of a planned strategy with a permanent increase being supported by a matching permanent increase in the working capital of the company. This will most commonly be achieved by retention of profits and/or an injection of share capital. Inflation could increase the requirement more than the funding injected.

Overtrading is a common phenomenon for growing companies, and occurs when a business overextends itself by having insufficient capital to match increases in turnover. Increasing turnover will result in higher stock and debtor levels which will need to be funded. Another cause of overtrading is the repayment of a loan when the business has insufficient cash to fund it. Whilst there will be some corresponding growth in creditors, sustaining growth on trade credit alone is unlikely to be successful in the longer term.

In such a situation increasing the firm’s overdraft and reducing the level of credit allowed to debtors are other possible sources of finance. However, both will prove difficult in practice; the latter especially may create problems preventing the required growth the firm desires.

Overtrading is illustrated by the following example:

Page 231: libvolume5.xyzlibvolume5.xyz/law/ballb/3rdyear/semester5/corporatelaw/corporate... · ABE Advanced Diploma in Business Administration Study Manual CORPORATE FINANCE Contents Study

Working Capital and Short-Term Asset Management 223

© Licensed to ABE

Example

Year 1 Year 2 £ £ £ £

Fixed Assets 80,000 120,000 Current Assets Stock 20,000 40,000 WIP 20,000 50,000 Debtors 50,000 80,000 Cash 5,000

95,000 170,000 Current Liabilities Creditors 45,000 118,000 Bank 20,000 60,000

65,000 178,000 30,000 (8,000)

110,000 112,000 Financed By: Share Capital 100,000 100,000 Profit & Loss Account 10,000 12,000

110,000 112,000

Sales £500,000 £1,000,000 Gross Profit £100,000 £100,000 Gross Profit % 20% 10% Net Profit £30,000 £2,000 Net Profit % 6% 0.2%

The important points to note are:

(a) Turnover has doubled, but the gross profit percentage has halved. Discounts for quicker payment may have caused this, as could lower sale prices to win more orders.

(b) Net profit percentage shows a big decline. Increased wages and bonuses, or writing off obsolete stock may have caused this.

(c) Stock and WIP have more than doubled. (Has obsolete stock been written off?)

(d) Although sales have risen by 100%, the increase in debtors is only 60%.

(e) Surplus cash from Year 1 has been used and bank borrowing has increased significantly.

(f) Creditors have increased by 162% for a rise in turnover of 100%. Credit periods have extended, and problems could arise if they have not been negotiated.

(g) Whilst fixed assets have increased, it may not be symptomatic of the increased trade. The expenditure may be part of a planned cycle, and indeed, if the machines are more productive

Page 232: libvolume5.xyzlibvolume5.xyz/law/ballb/3rdyear/semester5/corporatelaw/corporate... · ABE Advanced Diploma in Business Administration Study Manual CORPORATE FINANCE Contents Study

224 Working Capital and Short-Term Asset Management

© Licensed to ABE

they will benefit the increased business volumes. It is unwise to increase capital expenditure from short-term finance such as trade credit and bank overdraft, however.

(h) The positive current and quick asset ratios have disappeared, indicating a worsening in the short-term financing position.

(i) The proprietors’ stake for the two years is:

Year 1 Year 2 £ £

Total Assets 175,000 290,000 Financed By:

Capital 110,000 62.9% 112,000 38.6% Creditors 45,000 25.7% 118,000 40.7% Bank overdraft 20,000 11.4% 60,000 20.7%

175,000 290,000 There has been a dramatic decline in the proportion funded by the equity holders, and should the bank limit be reached, no more trade credit be available and debtors be unwilling to pay more quickly, then the firm could go out of business despite a full order book and the potential to be successful.

Methods to relieve the situation could include:

� Faster debt collection, although too much pressure may lose customers.

� More efficient stock-holding.

� Slower payment to creditors, but there are limits that will be acceptable.

� Increased bank financing, although the bank will probably expect a capital injection from outside the business as well.

� Slowing down the rate of growth in turnover, allowing work in progress to be finished and stock sold, thereby reducing the amount of working capital needed.

C. CASH MANAGEMENT

Every organisation must have adequate cash resources (including undrawn bank overdraft facilities) available to it to meet its financial commitments of day-to-day trading (e.g. wages and taxation). Cash is also required to meet contingencies, to take advantage of discounts and other opportunities available, and to finance expansion. Firms should, though, avoid holding too much cash with the resulting underutilisation of resources.

Cash Flow Planning In order to understand cash management you need to be aware of the difference between profits and cash flow. From your accountancy studies you will be aware that profit is the amount by which income exceeds expenditure when both are matched on a time basis. Cash flow, however, is the actual flow of cash in and out of the organisation with no adjustments made for prepayments or accruals.

Page 233: libvolume5.xyzlibvolume5.xyz/law/ballb/3rdyear/semester5/corporatelaw/corporate... · ABE Advanced Diploma in Business Administration Study Manual CORPORATE FINANCE Contents Study

Working Capital and Short-Term Asset Management 225

© Licensed to ABE

A business which has insufficient cash may be forced into liquidation by its unpaid creditors even if it is profitable. A lack of cash can be seen by an increasingly late payment of bills. Management therefore needs to plan and control cash flow to prevent liquidation. In the short term this is done by cash flow budgeting, which can be daily, weekly, monthly or yearly, ensuring that the organisation has sufficient cash inflows to meet its outflows as they become due. Such budgets should fit in with the overall budgetary scheme that the company operates. If a shortage is expected then the firm can arrange finance, perhaps by increasing its overdraft, to overcome the problem.

Other remedies that can be used to deal with short-term expected cash shortages are:

� Accelerating cash inflows from debtors.

� Postponing cash outflows by delaying payment to creditors; whilst this is considered to be a cheap alternative (creditors rarely charge interest), such an alternative increases the risk of insolvency of the firm.

� Postponing capital expenditure (or negotiating extended payment terms with the supplier).

� Reversing past investment decisions, such as selling off non-essential assets.

� Rescheduling loan repayments (with the lender’s agreement).

� Reducing the level of dividend to be paid.

� Deferring (after discussion with the Inland Revenue) tax payments (but there will be an interest cost to doing this).

Despite it being bad policy to finance long-term assets with short-term funding, where the financial manager can determine from the cash budget that sufficient funds will become available, it may be possible to operate such a funding policy without detriment to the firm.

In order to help cash management of groups, a facility called cash pooling may be requested from the group’s bank. This process of cash pooling allows the offsetting of surplus and deficits held at the bank by the group’s companies using a dummy account. The net balance is the one on which interest is payable or chargeable, and the group can then decide how to allocate this cost or income.

For those groups which have overseas subsidiaries involved in intra-group trading, then the group may net off the transactions between its members on a multilateral basis. Whilst there are some countries which limit or prohibit netting (e.g. Italy and France), the groups should benefit from reduced transaction costs.

A further method of cash management that may be adopted by a multinational firm is to centralise cash management, holding funds in one of the major financial centres such as London or New York, with only the minimum level required for day-to-day purposes being held by subsidiaries. The remittance of funds back to the parent can be done via the group’s bank, or telegraphic transfer, but there may be limitations imposed by overseas governments on the level of remittances.

Margin of Safety No forecast will ever be 100% accurate and the further into the future the projections are made, the greater will be the margin of error. In cash budgeting the balance at the end of each period represents a “margin of safety”, whereby the company buys peace of mind at the expense of profitable utilisation of cash. The size of the balance must be related to the certainty or otherwise of the predicted inflows and outflows, and the availability of back-up resources, such as overdraft facilities available. A cash-based business, such as a food supermarket, will have more certainty of its cash inflows than a business selling principally on extended trade terms. Where, therefore, cash inflow can be predicted with relative accuracy, provision for a margin of safety can be smaller.

Page 234: libvolume5.xyzlibvolume5.xyz/law/ballb/3rdyear/semester5/corporatelaw/corporate... · ABE Advanced Diploma in Business Administration Study Manual CORPORATE FINANCE Contents Study

226 Working Capital and Short-Term Asset Management

© Licensed to ABE

Cash Management Problems There are several reasons why a business may encounter problems with its cash flow, including:

� Overtrading – which we discussed above.

� Growth – a firm may need to finance new assets to replace old and obsolete ones.

� Loss-making – if a business continually trades at a loss for a protracted period cash problems will materialise.

� Inflation – the replacement costs of stock will be at a higher price when there is inflation. However, competitive pressure may prevent a corresponding increase in selling price.

� Payment delays – either due to the business’s inefficiency or external delays.

� Bad debts – a large customer going into liquidation can create severe problems with a company’s cash flow.

� Large items of expenditure – fixed asset purchases or the redemption of loans can drain cash resources rapidly if insufficient plans have been made.

� Seasonal trading – this can cause short-term difficulty, particularly if a retailer’s stock, bought in especially for seasonal trading (e.g. Christmas), proves unpopular and does not sell.

A company experiencing problems with its cash flow should ensure that the invoice department is informed immediately when goods are despatched, and that instructions for payment are made clearly to customers.

Cash Ratios Ratio analysis can help in cash management and serve as an indicator of the cash-holding position. The main ratios are:

(a) Cash Holding – this ratio indicates the proportion of current assets which are held as cash. Generally, the financial manager will want to keep this figure at the safe minimum to be able to service immediate current outflows.

The ratio is measured as:

CashCurrent assets

The ratio may increase when a business is deliberately accumulating cash to meet forthcoming needs, e.g. capital expenditure or repayment of debt capital.

(b) Cash Turnover – this ratio is used to determine how frequently cash is turned over and is expressed as:

balancecash Average

period theduring Sales

The ratio assumes that an average cash holding is used, typically calculated as:

2balancecash Closing + balancecash Opening

However, note that cash flows will not be constant, especially if there is seasonal trade.

Page 235: libvolume5.xyzlibvolume5.xyz/law/ballb/3rdyear/semester5/corporatelaw/corporate... · ABE Advanced Diploma in Business Administration Study Manual CORPORATE FINANCE Contents Study

Working Capital and Short-Term Asset Management 227

© Licensed to ABE

For example, if sales for the year were £72,000 and the average cash holding was £9,000 then using the above formula:

balancecash Average

period theduring Sales = 000,9000,72 = 8 times, i.e. cash was turned over every 45.6 days.

A higher rate of cash turnover will generally be taken to imply the effective use of cash – the more frequent the turnover, the lower the level of cash needed. High rates of turnover, however, will appear as the result of maintaining too low a level of cash, and as such these ratios should be viewed together, i.e. the maximum turnover rate consistent with adequate holdings levels. Any proposed measures of improving cash flow management must be carefully evaluated to ensure that the costs do not outweigh the benefits.

Cash-based ratios will vary widely in different industries, e.g. turnover of cash in a food supermarket will be rapid, but in a major engineering concern it may take months to turn over once. Viewing one set of ratios for just one period will in itself disclose very little about the management of the firm and its trading prospects, and calculated ratios should be compared over time, and with industry norms.

Factors Affecting Cash Resources The amount of cash an organisation is holding can be affected by a number of unforeseen events:

� New competitors and/or new products may adversely affect demand for a company’s products.

� Consumers may change their purchasing habits, e.g. people are becoming increasingly aware of benefits derived from the use of environmentally friendly products.

� Upward movements in interest rates will reduce the amount of cash available to firms that are in a net borrowing situation.

� Businesses dependent on trading (both buying and selling) will be affected by movements in foreign exchange rates.

� Strikes or other disasters may halt production, or at least significantly reduce it, with a resultant fall in sales volume.

There is often a considerable amount of money tied up in the “float”, i.e. in the process of converting the cheque sent by the debtor into cash in the receiver’s bank. Delays during the process are those in receiving the cheque (transmission delay) and in the lodging and clearing of the cheque. The use of systems such as bank giro, BACS (Bankers’ Automated Clearing Services Ltd), standing orders, direct debits and CHAPS (Clearing House Automated Payments System) help to reduce these delays. In addition a company could collect local cheques itself, and should certainly ensure that cheques are banked on the day of receipt whenever possible.

Cash Management Models A number of models have been developed to help companies manage their cash. These range from simple spreadsheet models to more complicated models such as the Miller-Orr model discussed later. The aim of these models is to trade-off the lost interest of holding idle cash balances against the problems of having insufficient cash, and thus help companies determine the optimal minimum and maximum levels of cash holding. The models can generally be manipulated in order to allow the organisation to determine which factors need to be carefully managed in order to maintain optimal cash balances. Like all models they have their drawbacks and rely on good quality information being input to produce worthwhile results.

Page 236: libvolume5.xyzlibvolume5.xyz/law/ballb/3rdyear/semester5/corporatelaw/corporate... · ABE Advanced Diploma in Business Administration Study Manual CORPORATE FINANCE Contents Study

228 Working Capital and Short-Term Asset Management

© Licensed to ABE

(a) Baumol’s or the Inventory Model of Cash Management

This model states that the total cost for holding an average level of cash is minimised when:

Q = iFS2

where: Q = total amount of cash needed to raise for the time period

F = fixed cost of obtaining new funds (e.g. issue costs of shares)

S = amount of cash to be used in the time period

i = interest foregone (opportunity cost) of holding cash or near cash equivalents (this is a variable cost of obtaining funds)

and the average total cost of holding an average level of cash incurred in a period is:

2Qi + Q

FS

Example

Sooty plc requires £6,000 cash per annum. Any cash raised will have an associated fixed cost of £300 and an interest rate of 15%. The interest rate on short-term securities is 10%. Advise Sooty as to the level of finance it should raise at any one time.

The cost of holding cash for Sooty is the difference between the cost of the funds and that earned on short-term securities, i.e. 15% − 10% = 5%.

Therefore, substituting into the above formula:

Q = 05.0

000,63002 ××

= £8,485

This level should be raised every £8,485/£6,000 = 1.4 years.

Whilst this model provides a good basis for cash management, especially for firms which use cash at a steady rate, it ignores costs associated with having a cash deficit (e.g. interest on an overdraft), and any costs which may increase with increases in the amount of cash held. The model has been found in practice to be poor at predicting the amounts of cash required in future periods, and of little help in those firms where there are large and irregular inflows and outflows of cash.

(b) Miller-Orr Model

The Miller-Orr model, which was developed to produce a more useful model than the Baumol model, sets upper and lower limits to the level of cash a firm should hold. When these points are reached the firm either buys or sells short-term marketable securities in order to reverse the trend of cash flows. In order to set these levels, the variability of cash flows needs to be determined along with the costs of buying and selling securities, and the interest rate.

The steps in using the model are:

(i) Determine the lower level of cash the firm is happy to have. This is generally set at a minimum safety level, though in theory it could be zero.

(ii) Determine the variation in cash flows of the firm (perhaps over a three or six month period).

Page 237: libvolume5.xyzlibvolume5.xyz/law/ballb/3rdyear/semester5/corporatelaw/corporate... · ABE Advanced Diploma in Business Administration Study Manual CORPORATE FINANCE Contents Study

Working Capital and Short-Term Asset Management 229

© Licensed to ABE

(iii) Calculate the spread of transactions, using the following formula:

Spread = 3 × 3rateInterest

cost)n Transactio flowcash of Variance (0.75 ××

(iv) Calculate the upper limit – this is the sum of the lower limit and the spread.

(v) However, in order to minimise the costs of holding cash, securities should be sold when a pre-calculated level (the return point) is reached. The return point is the sum of the lower limit and 3

1 of the spread.

We will look at an example to show application of the model. You will only be required to understand how calculations are undertaken in your examination.

Example

Pat Ltd faces an interest rate of 0.5% per day and its brokers charge £75 for each transaction in short-term securities. Their managing director has stated that the minimum cash balance that is acceptable is £2,000, and that the variance of cash flows on a daily basis is £16,000. What is the maximum level of cash the firm should hold, and at what point should it start to purchase or sell securities?

Following the above procedure:

(i) Determine the lower level of cash the firm is happy in having; this has been set at £2,000.

(ii) Determine the variation in cash flows of the firm – this has been found to be £16,000.

(iii) Calculate the spread of transactions:

Spread = 3 × 3rateInterest

cost)n Transactiocashflow of Variance(0.75 ××

= 3 × 30.005

75)16,000(0.75 ××

= £1,694

(iv) Calculate the upper limit – this is the sum of the lower limit and the spread; upper limit = £2,000 + £1,694 = £3,694.

(v) However, in order to minimise the costs of holding cash, securities should be sold when a pre-calculated level (the return point) is reached. The return point is the sum of the lower limit and 1

3 of the spread = £2,000 + 13 (1,694) = £2,565.

Thus the firm is aiming for a cash holding of £2,565 (the return point). Therefore, if the balance of cash reaches £3,694 the firm should buy £3,694 − £2,565 = £1,129 of marketable securities, and if it falls to £2,000 then £565 of securities should be sold.

The model is useful in that it considers the level of interest rates (the higher the rates the lower the spread, and thus the less cash that is needed to be held before the return point and the upper limit is reached), and transaction costs (the higher the transaction costs the greater the spread and thus the less transactions are needed). In addition, the variability of cash flows are considered – those which are more variable are allowed a greater degree of freedom. The major problem of the model is that it does not take into consideration the fact that several cash flows of the firm can be predicted accurately (e.g. dividend payments), it having been developed to deal with uncertainty in cash flow management.

Page 238: libvolume5.xyzlibvolume5.xyz/law/ballb/3rdyear/semester5/corporatelaw/corporate... · ABE Advanced Diploma in Business Administration Study Manual CORPORATE FINANCE Contents Study

230 Working Capital and Short-Term Asset Management

© Licensed to ABE

D. MANAGEMENT OF STOCKS

Stock, or inventory as it is also known, comprises four main types:

� Raw material

� Work-in-progress

� Finished goods

� Miscellaneous items, e.g. tools, stationery, fuel, etc.

As with other working capital items, the problem for the financial manager is that of maintaining balance. He or she must ensure that:

(a) There will be sufficient raw material stock available to satisfy production needs and enough finished goods stock to meet customers’ requirements.

(b) At the same time, the amount of capital employed in stocks is minimised. (Stockholding has been called the graveyard of industry by financial commentators in the past.)

Achieving balance can be particularly difficult in the distributive, wholesaling industries, where customers expect a range of goods to be carried, but only place orders infrequently when they need a non-routine item. The distributor is faced with the problem of finding a balance between his ability (and reputation) for good service, and the need to restrict capital tied up in slow moving lines.

Cost of Stockholding Holding stock is generally an expensive cash utilisation of a firm’s working capital resources. Stock is basically money in another form, and some of the principal costs associated with stockholding will include:

(a) Cost of stock, less any available discount (e.g. for bulk purchasing)

(b) Providing finance – since stock is money, there is the cost of financing it, which may be taken as the weighted average cost of capital. There is also an opportunity cost of capital to consider, as funds tied up in stocks cannot be used for other, more profitable investments and so potential income will be forgone.

(c) Stock handling – included under this heading will be the costs of the stores installation, which may include racks, bins, paperwork systems, insurance and maintenance cost, security and so forth.

(d) Holding losses – these costs include evaporation, deterioration, obsolescence, theft, damage in stores and in transit. There may well be, of course, holding gains, particularly during times of inflation, but any gain will usually be offset by the usually higher costs of funds in such periods.

(e) Procurement costs – the costs of obtaining stock. These include:

(i) Clerical and administrative costs of procurement, e.g. salaries, purchasing office, telephones, letters, etc.

(ii) Transportation costs.

(iii) Related costs of tooling, production, scheduling, etc. associated with internal order, where stocks are produced internally.

Page 239: libvolume5.xyzlibvolume5.xyz/law/ballb/3rdyear/semester5/corporatelaw/corporate... · ABE Advanced Diploma in Business Administration Study Manual CORPORATE FINANCE Contents Study

Working Capital and Short-Term Asset Management 231

© Licensed to ABE

(f) Shortage or stock-out costs – the costs of being without stock for a period of time. These include

(i) Loss of contribution through the lost sale caused by stock-out.

(ii) Loss of future custom to competitors.

(iii) Idle time caused by breaks in production.

(iv) Overtime, rescheduling and related costs, arising from the need to expedite a “rush” order.

(v) Lost production.

(vi) Higher prices.

There are practical problems in quantifying many of these items, since they will typically be unknown until the effect of stock-out is known.

Stock Turnover Ratios When turnover, or the throughput, of an item increases, the level of stockholding will generally be reduced. Management will therefore seek to verify the rate of stock turnover in order to establish whether the position is satisfactory, or whether they will need to take action. Like all ratios, the real benefit comes from comparison over successive periods of time against the overall plan for the business.

The ratio is expressed as:

period in the consumed materials ofCost periodin days of No. period of endat ngStockholdi × = No. of days stock in hand

For example, a company has a closing stock of £32,600 and an annual consumption of stock of £220,700. Applying the ratio:

700,220600,32 × 365* = 54 days.

(*This is for a full year.)

If we make some simplifying assumptions to the effect that stock is consumed evenly throughout the year and the year end position is representative of the remainder of the year (i.e. there are no seasonal trends) we can say that the stock turned over every 54 days, i.e. 6.8 times. The ratio may also use the average of opening and closing stock as a more representative figure rather than just closing stock.

In some circumstances, similar calculations can be made expressing the relationship between stock and sales, again arriving at turnover intervals in terms of days and times. You should bear in mind that a separate calculation may be required for each type of stock, also breaking down its constituent parts into raw materials, finished goods and work in progress.

The 80:20 Rule This rule is known as Pareto’s Law after the Italian economist whose career spanned the late 19th and 20th centuries.

It is often found that a large percentage (the 80%) of stock is made up of only a small number (the 20%) of physical items. In these circumstances it is usual to adopt the procedure of dual control. The smaller number of high value items is subjected to a detailed stock control system. The larger number of low value items is made the subject of a more routine inventory system based on minimum

Page 240: libvolume5.xyzlibvolume5.xyz/law/ballb/3rdyear/semester5/corporatelaw/corporate... · ABE Advanced Diploma in Business Administration Study Manual CORPORATE FINANCE Contents Study

232 Working Capital and Short-Term Asset Management

© Licensed to ABE

and maximum reorder levels. The reasoning behind this is that strict control of 80% of the value should be quite sufficient to maintain an appropriate rate of stock turnover and control stock availability.

Pareto analysis is sometimes called ABC Analysis (not to be confused with Activity Based Costing). In this instance stock is broken down into three types depending on its value and usage.

� A items are those which are probably low volume usage but relatively high cost. In terms of control they are usually treated on an individual basis. They may represent 20% in number and perhaps 80% of the value.

� B items represent 30% of the items with, say, 15% of the value. In terms of control, they will typically be monitored by the use of a reorder policy.

� C items are the high volume, low priced items where close control is unimportant. They can be controlled by bulk issue methods such as “two-bin” systems.

The Pareto rule can also be graphed by comparing the cumulative value of the items (which could be in terms of cost, turnover, usage, etc.) against the cumulative number of items, as shown in Figure 11.1.

Note that the proportions can change so that, for instance, the top 80% in value may be represented by 30% of the items and so on.

Figure 11.1

We may conclude the subject of stock management by noting that this is another area where the financial manager, in his or her broad overall management capacity, oversees what is, in fact, a specialist activity. Stock control has grown into an autonomous subject with a considerable background of mathematical modelling, both deterministic and probabilistic, seeking to optimise suitable stockholding and holding cost minimisation, against the background of production difficulties and product prices.

Economic Order Quantity Economic Order Quantity (EOQ), a deterministic model, is defined by CIMA as:

“A quantity of materials to be ordered which takes into account the optimum combination of: � Bulk discounts from high volume purchases � Usage rate � Stockholding costs

Page 241: libvolume5.xyzlibvolume5.xyz/law/ballb/3rdyear/semester5/corporatelaw/corporate... · ABE Advanced Diploma in Business Administration Study Manual CORPORATE FINANCE Contents Study

Working Capital and Short-Term Asset Management 233

© Licensed to ABE

� Storage capacity � Order delivery time � Cost of processing the order”.

There are several assumptions underlying this model:

(a) No bulk discounts.

(b) There is a known, constant stockholding cost.

(c) There is a known, constant ordering cost.

(d) The rates of demand are known.

(e) There is a constant, known price per unit.

(f) No lead time, i.e. replenishment is made instantaneously (the whole batch is delivered at once).

Hence, the reorder quantity which minimises costs is the amount which minimises the combination of:

� Stockholding

� Stock reordering.

The combined cost can be expressed algebraically as:

( Q2

× h) + (C × dQ

)

where: h = cost of holding one unit of stock per annum (or other relevant period)

C = cost of ordering a consignment from a supplier

d = annual demand (or other period)

Q = reorder quantity

Therefore Q ÷ 2 is the average stockholding per annum (or other time period).

This can be illustrated graphically as shown in Figure 11.2.

Figure 11.2

Page 242: libvolume5.xyzlibvolume5.xyz/law/ballb/3rdyear/semester5/corporatelaw/corporate... · ABE Advanced Diploma in Business Administration Study Manual CORPORATE FINANCE Contents Study

234 Working Capital and Short-Term Asset Management

© Licensed to ABE

From this you can see that the larger the reorder quantity, the larger will be the stockholding cost. However, as the number of orders during the year decreases, ordering costs will be reduced. Alternatively, the smaller the reorder quantity, the smaller the stockholding costs, but the number of orders will increase, hence ordering costs will increase.

It can be proved that combined costs are minimised, i.e. the reorder quantity is most economic, when:

Q = hCd2

Example

Annual demand for material is 300 units, the ordering cost is £2 per order, the units cost £20 each and it is estimated that the carrying costs will be 15% per annum. Determine the EOQ and the numbers of orders to be placed per year.

Substituting into the EOQ formula:

C = £2 per order

d = 300 units

h = £20 × 15% = £3 per unit

EOQ = h

2Cd = 330022 ×× = 20 units.

Therefore, dQ

= 20

300 = 15.

Therefore, place 15 orders per year for 20 units.

The EOQ model given above is for replenishment stock in one batch. Where replenishment takes place gradually, e.g. where items are manufactured internally and placed into stock when they are completed, the formula must be adjusted slightly as:

EOQ =)R

d h(12Cd

where: R is replenishment rate per annum

Firms using the EOQ method may decide, because of uncertainties as to demand or lead time, to hold a “safety level” of stock to reduce the likelihood of a stock-out caused by excessive demand or an extra long lead time. The cost of this safety stock would be its quantity multiplied by the unit stockholding cost. The EOQ is still the quantity ordered.

Just-In-Time (JIT) Method of Procurement The aim of this method is to minimise the costs of holding stock, and goods are obtained from suppliers when required rather than holding stocks of materials and components. It is only suitable where a stock-out would not be disastrous (e.g. a hospital must have sufficient stocks of drugs and other medical equipment to ensure that it can treat its patients).

In order to introduce JIT production flows need to be even and predictable, a criteria which is aided if the firm adopts a policy of Total Quality Management (TQM) removing waste and other bottlenecks. The aim of TQM is to have 100% quality with zero defects.

Page 243: libvolume5.xyzlibvolume5.xyz/law/ballb/3rdyear/semester5/corporatelaw/corporate... · ABE Advanced Diploma in Business Administration Study Manual CORPORATE FINANCE Contents Study

Working Capital and Short-Term Asset Management 235

© Licensed to ABE

The advantages of JIT include:

� A reduction in stockholding costs.

� Simplification of the accounting for raw materials.

� Materials can be purchased at discounted prices.

� A reduction in production lead times.

� A reduction in scrap and work returned for correction.

� A reduction in labour costs per unit due to increased productivity.

� A healthier current ratio, and lower working capital requirements.

E. MANAGEMENT OF DEBTORS

Perhaps the management of debtors carries more importance than the management of stocks. Such an hypothesis will be based on the fact that at least stocks, even when poorly managed, remain on the premises and under the control of the company. Debtors represent the capital of the company placed in the hands of others in the form of goods over which the company has effectively lost all control.

The issue is, in common with all aspects of working capital, one of balance. With debtors, the factors to be balanced are:

(a) Giving credit or discounts – which acts as an aid to sales and potential profitability – and the period over which credit is, or has to be, extended.

(b) The cost of giving credit – effectively the company is lending out its precious working capital resources at 0% interest, thus needing extra capital.

(c) The cost of being unable to use that capital for more profitable projects.

(d) The cost of eventual bad debts should they arise (administrative costs, collection costs, etc.).

Debtors’ Turnover Ratio Competent financial managers will need some “rule of thumb” calculations to monitor the daily position relating to debtors and the debtors’ turnover represents such a management tool. It is expressed as:

year theduring made Sales

year theof end at the Debtors × 365 = No. of days’ sales outstanding.*

(*Where the period is for less than one year, the figure of 365 will be adjusted accordingly.)

Therefore a company with debtors of £68,400 and sales during the year of £272,500 has:

500,272400,68 × 365 = 92 days of sales outstanding and yet unpaid by debtors.

As with stock turnover, if we assume that the balance sheet level of debtors is representative of the entire year, and that sales arise evenly through the year with no seasonal trends to take into account, then we can say that debtors settle on average every 92 days.

Actions Available to the Company The physical possession of the company’s capital passes from its hands when the goods are transferred. Broadly there are five areas of attention to which the company can pay heed in

Page 244: libvolume5.xyzlibvolume5.xyz/law/ballb/3rdyear/semester5/corporatelaw/corporate... · ABE Advanced Diploma in Business Administration Study Manual CORPORATE FINANCE Contents Study

236 Working Capital and Short-Term Asset Management

© Licensed to ABE

attempting to influence the level of indebtedness of its customers and the inherent risks that this brings. These are:

(a) Checking out the credit standing of the customer, and re-checking for any adverse changes periodically.

(b) Setting a limit to the level of credit granted to individual customers and laying out terms of trading clearly in writing.

(c) Implementing formal collection procedures for delinquent accounts.

(d) Negotiating cash discounts for the most valued accounts.

(e) Taking out credit insurance.

Credit Control Credit control can be defined as:

“minimising the risks involved in handing over goods upon the strength of a promise to pay in future”.

The importance of the credit control department varies with the nature of the business in which it functions. Some companies sell entirely for cash – this is generally the case with a supermarket chain, for example. Companies who deal with small retail traders often have a major problem in that granting and controlling credit will involve them in a large number of accounts. This will be a time-consuming process in view of the numbers involved.

Other firms, making large industrial machines, may have few accounts and may even receive progress (stage) payments from customers as certain stages of the project are completed.

Bearing in mind these variations, the financial manager should consider the volume of business that will be sold on credit terms, the number of customers requiring credit and the records to be maintained when organising the credit control department. Important relationships will develop between credit control, and other departments, notably sales and marketing who may resent “their” hard won customers being subjected to status checks. We will now consider some of the most important aspects which affect credit control.

(a) The credit controller

This function may be combined with the sales ledger account department. This is a convenient arrangement because credit limits and the actual amount permitted from one period to another can be seen quite easily from the sales ledger records.

(b) Processing customers’ orders

When a new customer is involved, it is essential to check his or her creditworthiness and the following are useful methods:

� Direct methods

(i) Information obtained by the sales person from reports and interviews.

(ii) The credit controller’s own judgment, either from local knowledge, or from studying published accounts (or both).

The accounts will help to identify the length of time taken to pay other suppliers. A “rule of thumb” check which can be performed is:

Page 245: libvolume5.xyzlibvolume5.xyz/law/ballb/3rdyear/semester5/corporatelaw/corporate... · ABE Advanced Diploma in Business Administration Study Manual CORPORATE FINANCE Contents Study

Working Capital and Short-Term Asset Management 237

© Licensed to ABE

Average purchases per day = 365

materials of purchases Annual

then,

dayper purchases Average

creditors Trade = Number of days purchases in creditors.

(iii) Valuation of the company’s fixed assets.

(iv) Establishing a progressive and carefully managed system of credit, based on the track record of the customer’s ability and willingness to pay.

� Indirect methods

Methods available include:

(i) A report from the customer’s bank.

(ii) References from people who have had dealings with the potential customer.

(iii) The use of trade protection associations.

(iv) Consulting official records.

(v) Journals, newspapers and other publications.

(vi) Trade associations.

Generally, it is useful to have a number of opinions before granting credit, and whilst direct methods can be efficient they are potentially time consuming. Experience will, however, be needed to interpret the guarded wording in many third party reports. It is often what they don’t say, rather than what they do, that will lead the experienced credit controller to a safe conclusion.

The merits and otherwise of indirect methods include:

� Bank reports

These are often slow and must be made between two banks. The wording will always be guarded and remember that the bank may not be aware of all of their customer’s other commitments.

Bank reports possibly have greatest value in foreign trade, where they should be the best placed of all sources to assess the potential risks of the proposed transaction.

� Trade referees

Some buyers “nurse” specific accounts so that they can use them as referees and may even give friends’ names without disclosing the relationship. The best use of trade references is as a gauge of potential volume.

� Trade protection associations

Information is generally supplied only to members for their own use. Infolink is one of the major players in this market, and it can provide a status report on any form of business organisation, including sole traders and partnerships.

� Commercial credit houses

These organisations are commercial firms who specialise in the collection of credit information. Examples are Stubbs and Kemps, Seyde & Company and Dun and

Page 246: libvolume5.xyzlibvolume5.xyz/law/ballb/3rdyear/semester5/corporatelaw/corporate... · ABE Advanced Diploma in Business Administration Study Manual CORPORATE FINANCE Contents Study

238 Working Capital and Short-Term Asset Management

© Licensed to ABE

Bradstreet. As with the trade protection associations, a status report can be obtained for a reasonable fee. Some only report on incorporated entities, however.

In addition to the reporting service, credit ratings are set for most businesses above a certain size and published periodically in books available to members. The rating systems use symbols, often letters, to indicate the likely credit limit that could be set for specific companies.

Establishing Credit Limits and Terms Credit limits may be set in advance, in anticipation of a new account, but often no action will be taken until the first order on credit terms has been received. At this stage a credit limit should always be set and this will generally follow enquiries into the affairs of the new customer. As well as external reports, some of the factors that may be taken into account in assessing a new credit limit may include:

� The managerial efficiency and integrity of the business concerned.

� Affording facilities as good as those available from competitors (providing the business is not being offered because competitors have not been paid!).

� The capital employed in the customer’s business.

� The nature of the business and the sector in which it operates.

Consideration may also be given to relaxing the credit period allowed to customers if this is thought to be a profitable course of action. This may be of particular benefit when interest rates are low (and therefore the cost of financing is similarly low) in order to rebuild trade which has fallen off as a result of recession. However, care should be taken to ascertain the degree of risk in such a course of action. To assess the viability of extending credit periods given to customers, the company would undertake a calculation along the following lines:

Example

A company plans to extend its credit period from one to two months, with the intention of increasing its sales by £½m on top of its current level of £1¼m. The current profit is around 5% and the increased sales would require increased working capital of £50,000 (excluding the debtors). The required rate of return is 15%.

The company assumes that all customers take advantage of the new terms, and it calculates that the increased level of debtors would be:

£187,5001211,250,000

1221,750,000 =

×−

×

so the total increase in working capital is £(50,000 + 187,500) = £237,500.

The financing costs are £(237,500 × 15%) = £35,625.

The increased profits from the new policy are:

£(500,000 × 5%) = £25,000.

Therefore the new credit policy would not be worthwhile as £10,625 less would be made.

Page 247: libvolume5.xyzlibvolume5.xyz/law/ballb/3rdyear/semester5/corporatelaw/corporate... · ABE Advanced Diploma in Business Administration Study Manual CORPORATE FINANCE Contents Study

Working Capital and Short-Term Asset Management 239

© Licensed to ABE

Debt Recovery and Management An important function of credit control is to ensure that debts are collected as quickly as possible. In order to induce the customer to pay promptly, it is a common feature for the terms of payment to include a discount of around 2½% for prompt settlement, say within seven days.

Whether or not the discount can be justified depends upon the circumstances. If the discount really does prompt settlement within a much shorter period it may be justified, but, even then, if the money is simply added to the existing credit balance on current account, it would be much better to wait for a more usual payment period of 28 days and then receive the full, undiscounted payment.

An example may serve to underline this point.

Example

A company with annual sales of £1.2m on credit allows two months for payment. It is considering the introduction of a scheme whereby a 2% discount is offered for the payment of debts within 15 days from the date of invoicing, thereby reducing the period allowed to one month. The company would expect annual sales to fall to £1m with 30% of debtors taking advantage of the discount. If the company requires a return of 15% on its investments, would the offer of the discount be worthwhile?

It is possible to answer this question very simply by calculating the implied annual cost of the discount and then comparing it to the rate that the firm could receive if it possessed the money. Therefore, in the example, the cost would be:

rd) (100

(rd)discount of Rate−

× discount)for offered Period periodcredit (Original365

= 982 × )15 60(

365− = 16.5%.

From this you can see that it would not be worthwhile, as the firm could only earn 15% on the funds collected.

It is also possible to show the impact of a new policy in a different way:

Current level of debtors: 1.2m × 122 = £200,000.

Level of debtors following new policy on discount:

×+

× £1m of %70

121£1m of %30

36515 = 12,329 + 58,333 = £70,662.

Therefore the value of debtors is projected to fall by £(200,000 − 70,662) = £129,338.

The value of the reduction in perpetuity is £129,338 × 15% = £19,400.

The cost of the discount to the company is 2% × £300,000 = £6,000, which would appear to make the offer of a discount worthwhile, but this does not take account of the effect of the fall in profits as a result of the tighter credit control policy.

Thus, if the company would expect to obtain a profit margin of 7.5% on sales, then, in addition to the £6,000 cost, it would lose (7.5% × £200,000) = £15,000 in profits, making a total cost of £21,000. As the value is only £19,400 to the company, the discount is not worthwhile.

Irrespective of discounts, reminders should be sent out periodically unless payment on invoice is expected. One method of spreading the workload caused by sending out invoices and statements is known as cycle billing, and this involves sending out bills weekly or daily. This overcomes problems

Page 248: libvolume5.xyzlibvolume5.xyz/law/ballb/3rdyear/semester5/corporatelaw/corporate... · ABE Advanced Diploma in Business Administration Study Manual CORPORATE FINANCE Contents Study

240 Working Capital and Short-Term Asset Management

© Licensed to ABE

facing the credit control department when everything is processed, for example, in the last few days of the month.

When payments are really overdue, it is essential to take action without delay. This may take one or more of the following forms:

� Sending further reminders.

� Asking a sales person to call to collect.

� Late reminders threatening legal action.

� A call from the credit controller or a debt collector.

� A solicitor’s letter and, finally, legal action.

Management Control Information Reports supplied internally to management will include details of any overdue accounts, potential bad debts emerging, volume of new business transacted on credit, previous problems since reconciled, and any other specific difficulties affecting credit control. Significant ratios will often be included, such as the ratio of debtors to creditors, the ratio of credit sales to debtors, and the ratio of total sales to credit sales. A statement of outstanding debtors illustrating the age of those debts will also be included, but may vary considerably between different industries.

A typical statement of outstanding debtors is illustrated below.

Statement of Outstanding Debtors as at ............

Overdue Accounts Remarks Account Name Over 1

Month Over 2 Months

Over 3 Months

Over 4 Months

Over 5 Months

£ £ £ £ £ Atkins M 40 Cheque

promised Brown B 60 Court action

pending etc.

Total

Sales for year to date £................................ Sales for current period £................................ Current balances outstanding £................................

Credit Insurance There is often merit in covering debtors with credit insurance, particularly when the company is exporting. The Export Credits Guarantee Department (ECGD) can assist exporters in a number of fields of operation, including specific projects, capital goods construction and service sectors, by providing insurance cover against the main commercial and political risks which may arise during manufacturing and credit periods. ECGD also provides guarantees of payment to banks providing export finance at favourable interest rates.

Page 249: libvolume5.xyzlibvolume5.xyz/law/ballb/3rdyear/semester5/corporatelaw/corporate... · ABE Advanced Diploma in Business Administration Study Manual CORPORATE FINANCE Contents Study

Working Capital and Short-Term Asset Management 241

© Licensed to ABE

A new company, NCM UK Limited, created from the privatisation of ECGD Insurance Services Group in 1991, provides credit insurance to UK exporters on short terms of payment, although this type of cover is also available from other companies.

Policies which may be taken out will either be:

(a) Whole turnover policies, which cover total sales for the year; or

(b) Specific account policies, which cover a specific account.

Selection of a policy will largely be determined by the nature of the business conducted by the company and, if the company has a number of substantial accounts, a whole turnover policy will almost certainly be preferred.

Each proposal will be taken on its merits and insurance cover is given on a sound proposition where there is a satisfactory prospect that payment will be made.

The risks covered will include:

� The insolvency of the buyer.

� The prevention of, or delay in, the transfer of payment to the merchant in circumstances outside the control of both the merchant and the buyer.

� War preventing the export of the goods or performance of contracts, or the delivery of goods to the buyer’s country (contracts and shipments).

� War, revolution or civil disturbance in the buyer’s country specifically.

� The failure or refusal of a government buyer to fulfil the terms of the contract.

Trading Abroad A knowledge of the procedures followed when intending to export goods is essential, and the main factors are summarised below:

(a) Sell direct or through a merchant

It will be necessary to decide whether to sell direct or through a specialised export merchant. Exporting is often a complicated process to which the manufacturer can devote insufficient time. As a result, the company may be well advised to seek help from a recommended export merchant who understands, and has established contacts in, the chosen market.

(b) Winning customers

Help can be obtained from the DTI and through Business Link when an exporter is deciding on where his best market is located. Banks, trade associations and overseas agents can also fulfil a useful role, but care should be taken in choosing the latter since he or she may already deal in competing products.

(c) Assessing the credit standing of buyers

An initial appraisal of the credit standing should be made and then a review carried out at regular intervals.

(d) Complying with regulations

There are government restrictions in most countries, and in Britain an export licence may be required for certain transactions. Other countries may have regulations affecting factors such as:

� Exchange controls.

Page 250: libvolume5.xyzlibvolume5.xyz/law/ballb/3rdyear/semester5/corporatelaw/corporate... · ABE Advanced Diploma in Business Administration Study Manual CORPORATE FINANCE Contents Study

242 Working Capital and Short-Term Asset Management

© Licensed to ABE

� Trade restrictions on imports.

� Customs duties.

In Europe there are many Directives which lay down minimum requirements. Examples include the Machinery Directive (1/1/93), and the Efficiency Requirements for New Hot-Water Boilers Fired with Liquid or Gaseous Fuels Directive (1/1/94).

(e) Securing payment

Once the goods have been sold, the exporter may have to wait a considerable period before he is paid by the overseas buyer. To be able to operate overseas it is quite usual to obtain the backing of a bank or an accepting house.

The DTI’s Project and Export Policy Division (PEP) co-ordinates all government interest in, and support services for, large overseas projects. PEP collaborates closely with all appropriate government departments at home and with the diplomatic service posts abroad. Where appropriate, the Division can bring the full weight of government support to bear, including ministerial and diplomatic initiatives.

F. CREDITOR MANAGEMENT

A firm needs to determine what policy to adopt in the management of its creditors. In doing this it must consider the following factors:

� The need to ensure continuing supplies as and when required, by maintaining good relations with regular suppliers.

� The level of credit required, and the ability to extend it when the firm has a cash flow shortfall.

� The advantages of having a high level of trade credit as a method of reducing the level of working capital required.

� The possibility of extending credit, but this provides the firm with a poor credit rating and problems in obtaining additional credit.

� Whether to accept or reject early payment discounts (this decision is made in the same way as offering discounts to a firm’s customers – the benefits of accepting the discount (additional cost) must outweigh the costs (interest foregone) of paying the debt early).

This latter point illustrates the often forgotten cost of trade credit which is often assumed to be free but there is a cost of any early payment discounts foregone. An additional intangible cost may be the loss of supplier goodwill. In addition, recent legislation now allows suppliers to charge interest on overdue accounts. This will further add to costs if payments to suppliers are delayed.

The cost of early payment discounts foregone can be calculated as:

t365 s100

s ×−

where: s = the size of the discount offered in percentage terms, and

t = the reduction in payment period required in the payment period.

This gives the cost as a percentage which should then be compared to the interest rate that the company would obtain for investing the cash for the same number of days as required to obtain the reduction in payment period.

Page 251: libvolume5.xyzlibvolume5.xyz/law/ballb/3rdyear/semester5/corporatelaw/corporate... · ABE Advanced Diploma in Business Administration Study Manual CORPORATE FINANCE Contents Study

Working Capital and Short-Term Asset Management 243

© Licensed to ABE

G. SHORT-TERM FINANCE AND INVESTMENT

Management of Short-Term Finance We have seen that short-term finance is a major source of working capital and that it can be obtained from a variety of sources, most of which we have already discussed earlier in this course, i.e.

� Bank overdrafts

� The issuing of short-term debt instruments (for larger companies)

� Taking trade credit from suppliers

There are two further methods available to firms that you need to know about and are explained here. In addition, there are a number of specialist sources of finance, which we discuss later in this section..

(a) Bills of Exchange

This method is a common source of finance for international trade. The settlement of the cost of a trading transaction may be by means of a bill of exchange (also called trade bills). This occurs when the seller draws a bill on the buyer asking them to pay, on a certain future date, the price of the goods supplied, which is then accepted by the purchaser (by signing and returning it to the seller). The purchaser is thus formally acknowledging his debt to the seller. The seller can then use the bill of exchange as security in order to obtain money from the seller’s bank.

A bank may also agree to accept a bill from its customer in exchange for an agreement that the customer will repay the bank. The cost for arranging this finance is the discount (i.e. the full amount of the bill is not advanced). The more secure the bill (e.g. from a bank as compared to a trader) the “finer” or lower the discount.

(b) Acceptance Credits

This is a facility offered by banks for large companies with a good reputation. The company draws bills of exchange on the banks, generally for 60, 90 or 180 days, denominated in whichever currency most matches the needs of the company. The bills can be drawn on, as and when required, throughout the length of the agreement which can be for up to five years, provided the credit limit is not exceeded. The bill is then sold in the discount market and the proceeds passed to the company (less the bank’s commission). At maturity the company reimburses the bank the full value of the bill, and the bank pays the holder of the bill.

A major advantage of accepting credits is that they can be sold at a lower discount than trade bills. The cost of them is also fixed, allowing for easier budgeting and may be lower in times of rising interest rates than that of an overdraft. The credit is also guaranteed for the length of the agreement, unlike an overdraft.

Short-Term Investments We have seen that firms may have a surplus of cash, either deliberately in order to meet purchase costs in the near future or to take advantage of high interest rates, or because of higher than expected profit levels or a lack of investment opportunities. These fluctuations in cash can arise for a variety of reasons, a major one being seasonal fluctuations in trade.

Short-term surplus of funds should be invested in short-term marketable securities. Treasurers will employ surplus funds to obtain the best possible returns with maximum security, and in evaluating the alternatives a variety of factors must be taken into account including:

� Risk

Page 252: libvolume5.xyzlibvolume5.xyz/law/ballb/3rdyear/semester5/corporatelaw/corporate... · ABE Advanced Diploma in Business Administration Study Manual CORPORATE FINANCE Contents Study

244 Working Capital and Short-Term Asset Management

© Licensed to ABE

� Liquidity

� Term

� Cost

� Return

� Accessibility – deposits with banks, finance houses and local authorities are only accessible when the term finishes – debt instruments and equities can, however, be sold when cash is needed

� Type and level of interest rate

� Taxation

� Complexity

� Minimum/maximum criteria

� Image/policy

Investment opportunities which are available include:

� Money market deposits

� Treasury bills

� Equities

� Commercial Paper

� Bonds

� Gilts

� Certificates of Deposit

� Bank and other financial institution deposits, the interest rate varying with the institution, the type of account and the amount deposited

� Longer-term debt instruments

� Eurocurrency deposits

� Finance house deposits

� Local authority deposits

� Sterling Bankers’ Acceptances (bank bills)

� Local authority bills

� Bills of exchange (trade bills)

Specialist Sources Of Finance The exporter may obtain debtor financing through forfaiting. In the home market, factoring, invoice and block discounting are facilities to cater for the financing of credit afforded to customers. We will look at each of these in turn.

(a) Forfaiting

This is a specialist form of trade funding, geared to exports, which first emerged during the 1950s; it is currently growing in popularity as the range of markets in which forfaiters are prepared to operate is expanding. Forfaiters usually operate from within a separate department

Page 253: libvolume5.xyzlibvolume5.xyz/law/ballb/3rdyear/semester5/corporatelaw/corporate... · ABE Advanced Diploma in Business Administration Study Manual CORPORATE FINANCE Contents Study

Working Capital and Short-Term Asset Management 245

© Licensed to ABE

within a bank, and they provide non-recourse finance (essentially funds which are only repayable if the exporter fails to perform under his contract) to exporters who seek to boost and secure foreign cash flow.

Forfaiters provide exporters with non-recourse finance by buying trade debts from the exporter at a discount. The debts are guaranteed (the forfaiter calls this avalised) by an acceptable bank but, despite this, the forfaiter will closely monitor country risk in all areas in which he or she is prepared to operate.

For example, a German exporter of machine tools may be offered a series of promissory notes maturing at, say, six-monthly intervals over a period of five years by his Algerian customer. Unless the importer (customer) is of undoubted standing and there is seen to be minimal country risk, the notes must be avalised (guaranteed) in a form acceptable to the forfaiter. On receiving the notes from the importer, the exporter may then sell them at a discount to a bank in his own, or in a third, country. At this point the exporter has no further obligations, other than those relating to quality and fitness of the products sold and the fulfilment of any contracted service obligations.

� Essential features of forfaiting

(i) The promissory note must carry a first-class name (of the highest credit standing), or it must carry the guarantee (aval) of a first class bank.

(ii) The obligation of the importer (or guarantor) to pay must not depend on the exporter performing under the contract.

(iii) A standard commitment fee of around 1% per month will be payable to the forfaiter (this may vary with the perceived risk) and discount rates will reflect current money costs in the market and the length of the period before the forfaiter receives payment.

� Financial instruments used

The principal financial instruments used in a forfait financing are bills of exchange drawn by the exporter on the importer, and promissory notes issued by the importer in favour of the exporter. Promissory notes tend to be favoured by exporters because, as the endorser of a promissory note, the exporter can free himself of any liability by means of a without recourse clause in the endorsement. With a bill of exchange, the exporter must obtain the agreement of the forfaiter not to take proceedings against him in the event of non-payment by the importer.

� Security for the forfaiter

Security will be aimed at reducing the forfaiter’s risk, and also facilitates rediscounting the instrument if necessary.

(i) Aval is an irrevocable and unconditional guarantee to pay on the due date. It is written directly on to the bill or note, rather than being contained within a separate document. The words “per aval” are written on the instrument, together with the banker’s signature.

(ii) Guarantee is evidenced by a separate document which must not be conditional upon the performance of the exporter, i.e. it must be an unconditional obligation. It should also be transferable in order to permit rediscounting of the forfaited instrument.

Page 254: libvolume5.xyzlibvolume5.xyz/law/ballb/3rdyear/semester5/corporatelaw/corporate... · ABE Advanced Diploma in Business Administration Study Manual CORPORATE FINANCE Contents Study

246 Working Capital and Short-Term Asset Management

© Licensed to ABE

(b) Factoring and Invoice Discounting

These facilities are specifically designed to meet the needs of the expanding business operating in the home market. As a business carries on trade and builds up its debtor/creditor relationships, value is passed in the form of goods not yet paid for. A factor (factoring company) will advance money against the security of debts owed to a business. In this way, cash can be released more swiftly than waiting for the debtors to pay within normal trade terms.

There are many variations of factoring, the factor’s services frequently being tailored to the customer’s needs. The factor may take the entire sales ledger balances, or only specific, larger debts arising from individual transactions. The factor may actually buy the debt(s) or he or she may merely collect on the customer’s behalf, or the customer may collect on the factor’s behalf.

Factoring grew as a way to facilitate trade credit and is now an important part of commercial life. Subject to the factor’s approval, debts can be purchased free of recourse and therefore the company passing its debts to the factor will:

� Not have to deal with bad debts.

� Lose the administrative burden of the supervision of trade credit.

� Have an assured cash flow. Generally, the factor will advance around 80% of the value of the debt within two or three days of an approved debt being passed to him or her.

Factoring is particularly useful to firms trading in markets that require a considerable period of trade credit and to companies that are expanding rapidly, as it will leave other lines of credit open for use elsewhere in the business.

Costs will vary with the perceived level of risk, the volume of business and the period over which credit is granted. There will usually be a monthly facility fee payable by the customer, whether or not funds are drawn from the facility.

(c) Block Discounting

This is also a facility available for companies trading in the home market. It emerged in the 1960s and is used by companies who offer retail (white and brown) goods on instalment credit (hire purchase or rental) and initially supported the massive growth in television rental.

The facility is administered in a similar way to factoring, but the company will lodge its instalment credit agreements with the block discounter, rather than its invoices. The facility is usually undisclosed to the end-user customer and the company will collect rentals and instalments due on behalf of the block discounter.

Because the block discounter has a right to the future cash flows arising under the terms of approved agreements, he will generally agree to a higher gearing than a clearing bank, and a financial gearing of 3:1 for a company selling goods extensively on instalment credit may often be quite acceptable.

(d) Sales Aid Financing

As an alternative to offering their own trade credit, suppliers of capital equipment and vehicles both to the home market and abroad may offer finance at the point of sale through a third-party finance company. This is known as sales aid finance and is especially prevalent in the UK retail motor trade where rates may be subsidised by the manufacturer, the dealer or both, to attract business and private customers by easing the burden of expenditure on a major capital item.

Page 255: libvolume5.xyzlibvolume5.xyz/law/ballb/3rdyear/semester5/corporatelaw/corporate... · ABE Advanced Diploma in Business Administration Study Manual CORPORATE FINANCE Contents Study

Working Capital and Short-Term Asset Management 247

© Licensed to ABE

Whilst a simple loan document is the most common financial instrument used in the personal sector, sophisticated financial packages are often negotiated for the business user, these being based around either hire purchase or leasing, including operating leasing in the guise of rental or contract hire.

The benefit to the supplier (and the manufacturer when they are not one and the same) is that the finance company will generally settle at the time the goods are supplied and there is no need to extend trade credit. Additionally, the payment is received in full, generally without any recourse in respect of bad debts (other than those arising through poor quality merchandise). Suppliers will often work in conjunction with the finance company to present a “financial package” in such a way that less discount is passed to the customer than would be the case with a cash sale. Commissions may be generated from the sale of finance and related insurance products, which serve to boost the profit generated by a credit-based sale.

Larger organisations are often able to persuade the finance company to operate on an undisclosed basis. In these circumstances, all documentation will be designed in the supplier’s house style and will use the supplier’s rather than the finance company’s logo.

Page 256: libvolume5.xyzlibvolume5.xyz/law/ballb/3rdyear/semester5/corporatelaw/corporate... · ABE Advanced Diploma in Business Administration Study Manual CORPORATE FINANCE Contents Study

248 Working Capital and Short-Term Asset Management

© Licensed to ABE

Practice Questions

1. The final accounts for Daish, a small trading firm, are given as follows:

Income Statement

Nov 30, 2002 Nov 30, 2001 Nov 30, 2000 £000 £000 £000 £000 £000 £000Sales 3,860 2,800 2,000

Opening Stock 200 100 100 Purchases 2,700 1,750 1,000 Closing Stock 400 200 100

Cost of Sales 2,500 1,650 1,000

Gross Profit 1,360 1,150 1,000Operating and Financial Expenses 800 500 400

Net Profit 560 650 600

Balance Sheet

Nov 30, 2002 Nov 30, 2001 Nov 30, 2000 £000 £000 £000 £000 £000 £000Fixed Assets 1,312 800 600Current Assets: Stock 400 200 100 Debtors 220 200 150 Cash 10 630 2 402 30 280

Current Liabilities: Creditors 350 200 110 Bank Overdraft 380 730 150 350 50 160

Total Assets less Current Liabilities 1,212 852 720Long Term Liabilities: Bank Loan 250 250 225

Net Assets = Capital 962 602 495

Required:

Daish is concerned about its management of working capital.

(a) Explain the changes in the cash operating cycle for this firm over the years indicated.

(b) Do you think the firm is overtrading? Suggest possible future action based on your analysis.

Page 257: libvolume5.xyzlibvolume5.xyz/law/ballb/3rdyear/semester5/corporatelaw/corporate... · ABE Advanced Diploma in Business Administration Study Manual CORPORATE FINANCE Contents Study

Working Capital and Short-Term Asset Management 249

© Licensed to ABE

2. Babb plc specialises in the manufacture of sportswear. Sales in the current year have been £5.2 million. The terms of sale are 2% discount 14 days, net 28 days, although those customers not taking the discount actually take longer than 28 days to pay on average. The current level of debtors is £500,000, included in which are the one half of Babb’s customers who take advantage of the cash discount. 1% of credit sales become bad debts. The net operating margin (excluding bad debts and discounts) for Babb is 25% of sales.

The company is considering a change in its credit policy to 4% discount 14 days, net 28 days. It anticipates the following effects of this change:

� Sales to increase by 10% pa

� 75% of customers to take advantage of the discount

� The period of time before payment for customers not taking the discount to increase by one week

� Bad debts to fall to 0.5% of sales

Babb’s cost of finance is 12%.

Required:

(a) Calculate the financial implications of this change in credit policy and give your advice on the proposed change.

(b) Debtors and credit control are, of course, only one aspect of working capital management. What policies may produce savings in other items of working capital? Discuss some of the potential difficulties attached to the management of working capital.

Now check your answer with the one given at the end of the unit.

Page 258: libvolume5.xyzlibvolume5.xyz/law/ballb/3rdyear/semester5/corporatelaw/corporate... · ABE Advanced Diploma in Business Administration Study Manual CORPORATE FINANCE Contents Study

250 Working Capital and Short-Term Asset Management

© Licensed to ABE

ANSWERS TO PRACTICE QUESTIONS

1. (a) The cash operating cycle is the period between the payment of cash to creditors and the receipt of cash from debtors. We need to calculate appropriate ratios:

(i) Debtors’ payment period = sday 365salesCredit

debtors Trade × :

� Year ended Nov 30, 2002: 365860,3220 × = 20.8 days

� Year ended Nov 30, 2001: 365800,2

200 × = 26.1 days

� Year ended Nov 30, 2000: 365000,2

150 × = 27.3 days

(ii) Creditors’ payment period = sday 365PurchasesCreditors ×

� Year ended Nov 30, 2002: 365700,2

350 × = 47.3 days

� Year ended Nov 30, 2001: 365750,1

200 × = 41.7 days

� Year ended Nov 30, 2000: 365000,1

110 × = 40.1 days

(iii) Stock turnover ratio = sday 365sales ofCost stock Average ×

� Year ended Nov 30, 2002: 365500,2

2)400200( ×÷+ = 43.8 days

� Year ended Nov 30, 2001: 365650,1

2)200100( ×÷+ = 33.2 days

� Year ended Nov 30, 2000: 365000,1

2)100100( ×÷+ = 36.5 days

Cash operating cycle:

� Y/e 30 Nov 2000 Average days before receipt of cash on sales = 36.5 + 27.3 = 63.8 days Average days before payment of creditors = 40.1 days Operating cycle = 23.7 days

� Y/e 30 Nov 2001 Average days before receipt of cash on sales = 33.2 + 26.1 = 59.3 days Average days before payment of creditors = 41.7 days Operating cycle = 17.6 days

Page 259: libvolume5.xyzlibvolume5.xyz/law/ballb/3rdyear/semester5/corporatelaw/corporate... · ABE Advanced Diploma in Business Administration Study Manual CORPORATE FINANCE Contents Study

Working Capital and Short-Term Asset Management 251

© Licensed to ABE

� Y/e 30 Nov 2002 Average days before receipt of cash on sales = 43.8 + 20.8 = 64.6 days Average days before payment of creditors = 47.3 days Operating cycle = 17.3 days

Over the period, it appears that the company has continued to improve the cash operating period from 23.7 days to 17.3 days. However, a closer look at the ratios reveals some potential problems:

� Although the company has improved its debt collection procedures and has reduced the payment period from 27.3 days to 20.8 days, the stock turnover ratio in the year ended 30 November 2002 is longer, indicating that it is taking longer to convert stock purchases into actual sales.

� The creditors’ payment period in the year ended 30 November 2002 is longer than previous years, indicating that the company is making more use of creditors as a source of short-term finance. This can be dangerous in the long term as liquidity problems can arise.

(b) To consider whether the firm is overtrading we need to look at some ratios:

Year ended Nov 30 2002

Year ended Nov 30 2001

Year ended Nov 30 2000

Turnover £3,860,000 £2,800,000 £2,000,000 Gross Profit Ratio: 100860,3

360,1 × 100800,2150,1 × 100

000,2000,1 ×

= 35.2% = 41.1% = 50.0%

Net Profit Ratio: 100860,3560 × 100800,2

650 × 100000,2

600 ×

= 14.5% = 23.2% = 30%

Quick Asset Ratio: 730230

350202 160

180

= 0.3:1 = 0.6:1 = 1.1:1

Acid Test Ratio: 73010

3502 160

30

= 0.01:1 = 0.006:1 = 0.2:1

Current Ratio: 730630

350402

160280

= 0.9:1 = 1.1:1 = 1.8:1

The company shows significant signs of overtrading:

� An increase in turnover over the period.

� A decrease in gross profit and net profit ratios over the same period.

� A deterioration in stock turnover ratios.

� Increasing liquidity problems shown by quick asset and acid test ratios.

Page 260: libvolume5.xyzlibvolume5.xyz/law/ballb/3rdyear/semester5/corporatelaw/corporate... · ABE Advanced Diploma in Business Administration Study Manual CORPORATE FINANCE Contents Study

252 Working Capital and Short-Term Asset Management

© Licensed to ABE

� Increasing reliance on short-term finance, e.g. increase in bank overdraft and creditors’ payment period.

Future action should include:

� Efforts to increase stock turnover, e.g. advertising campaign

� Reduction in expenses to improve net profit percentage

� Seeking of alternative methods of finance to finance any expansion, e.g. bank loan

2. (a) The financial implications are:

(i) Increase in net operating profit

Current level of sales = 25% × £5.2m = £1.3m

After change in policy = 25% × £5.72m = £1.43m

(ii) Increase in discount allowed

Current level = £5.2m × 2% × 50% = £0.052m

After change in policy = £5.72m × 4% × 75% = £0.1716m

(iii) Savings on debtors

Assume average time taken to pay (customers not taking cash discount) = y weeks

Current situation:

Make up of debtors = (50% × 52

m2.5£ × 2) + (50% × 52

m2.5£ × y) = £0.5m

∴ y = 8 weeks

New situation (taking one week longer to pay):

Total debtors = (75% × 52

m72.5£ × 2) + (25% × 52

m72.5£ × 9) = £412,500

∴ Reduction in debtors = £500,000 − £412,500 = £87,500

Savings = 12% × £87,500 = £0.0105m

(iv) Reduction in bad debts

Current level = 1% × £5.2m = £0.052m

After change in policy = 0.5% × £5.72m = £0.0286m

Summary of changes: Saving Increased cost £m £m (i) Increase in net operating profit 0.13 (ii) Increase in discount allowed 0.1196 (iii) Savings due to reduced debtors 0.0105 (iv) Reduction in bad debts 0.0234

0.1639 0.1196 Overall benefit = £0.0443m

Page 261: libvolume5.xyzlibvolume5.xyz/law/ballb/3rdyear/semester5/corporatelaw/corporate... · ABE Advanced Diploma in Business Administration Study Manual CORPORATE FINANCE Contents Study

Working Capital and Short-Term Asset Management 253

© Licensed to ABE

This is a relatively small benefit given quite favourable assumptions have been made, e.g. a 10% rise in sales and a 50% reduction of bad debts. The cost of discount allowed is high and it is debatable whether the proposed change is actually viable.

(b) You could base your answer on the following points:

(i) Stock control – improvements using computerised systems and techniques such as economic order quantity and just-in-time. Achieving faster stock turnover can reduce costs of stockholding.

(ii) Cash control – use of cash flow forecasts can help identify likely surpluses and deficits of cash. Surpluses can be invested and short-term overdrafts arranged to cover deficits.

(iii) Creditors – it may be possible to delay payments to creditors but this can have adverse effects on relationships with suppliers and the company could incur interest payments on overdue accounts.

Page 262: libvolume5.xyzlibvolume5.xyz/law/ballb/3rdyear/semester5/corporatelaw/corporate... · ABE Advanced Diploma in Business Administration Study Manual CORPORATE FINANCE Contents Study

254 Working Capital and Short-Term Asset Management

© Licensed to ABE

Page 263: libvolume5.xyzlibvolume5.xyz/law/ballb/3rdyear/semester5/corporatelaw/corporate... · ABE Advanced Diploma in Business Administration Study Manual CORPORATE FINANCE Contents Study

255

© Licensed to ABE

Study Unit 12

Capital Investment Decision Making 1: Basic Appraisal Techniques

Contents Page

Introduction 257

A. Future Cash Flows and the Time Value of Money 257

B. Return on Investment (Accounting Rate Of Return) 258

C. Payback 259

D. Discounted Cash Flow 260

E. Net Present Value (NPV) 261 Future Value Over Time 263 Conventions Used in NPV Calculations 264 Net Terminal Values 264 Annuities or Uniform Series 265 Multiple Time Periods 266 NPV Profile 266 Perpetuities 267

F. Internal Rate of Return 267 Pitfalls with IRR 269 Dual Rate of Return Method 269

G. Cost/Benefit Ratio 270

(Continued over)

Page 264: libvolume5.xyzlibvolume5.xyz/law/ballb/3rdyear/semester5/corporatelaw/corporate... · ABE Advanced Diploma in Business Administration Study Manual CORPORATE FINANCE Contents Study

256 Capital Investment Decision Making 1: Basic Appraisal Techniques

© Licensed to ABE

H. Comparison of Methods 270 Non-Discounted Methods 270 Discounted Methods 270

I. Impact of Taxation on Capital Investment Appraisal 271

Answer to Practice Question 273

Appendix: Discounting Tables 274 Single Payment Compounded Forward Factor 275 Uniform Series Compounded Forward Factor 276 Single Payment Present Worth (Discount Tables) 278 Uniform Series Present Worth (Cumulative Discount Tables) 280 Annuity Tables 282

Page 265: libvolume5.xyzlibvolume5.xyz/law/ballb/3rdyear/semester5/corporatelaw/corporate... · ABE Advanced Diploma in Business Administration Study Manual CORPORATE FINANCE Contents Study

Capital Investment Decision Making 1: Basic Appraisal Techniques 257

© Licensed to ABE

INTRODUCTION

At the beginning of this course we saw that one of three major decision areas undertaken in an organisation is that of investment appraisal, and the financial manager has to employ appraisal techniques in order to decide which projects to accept and which to reject. In this and the following study unit, we will consider the financial models and techniques that are commonly used in capital investment appraisal.

Capital investment decisions will largely shape the future of the business and its ability to manage its future operations. They are, though, generally difficult and expensive to reverse and must, therefore, be right first time. Appraisal of the implications of a decision is, then, essential.

The criteria for the appraisal of projects may be based on legal requirements (e.g. to meet health and safety legislation) or social and staff welfare needs (e.g. the provision of canteens). However, in the majority of cases, it will be on economic grounds – the key being that projects accepted meet preset financial criteria, generally a return greater than the cost of the capital needed to finance it. In addition, they must also seek to maximise shareholder wealth, by maximising long-term returns.

The methods of capital investment appraisal which we shall examine are as follows:

� Return on Investment (ROI) or Accounting Rate of Return (ARR)

� Payback

� Net Present Value (NPV)

� Internal Rate of Return (IRR)

� Cost/Benefit Ratio

� Adjusted Present Value (APV) (which we shall consider in the next unit)

These methods do not all fulfil the criteria set above, but they are the most widely used in practice. A useful exercise for you whilst working through the following sections is to assess the validity of the different techniques, in view of the above criteria and of the primary objective of corporate finance, that of shareholder wealth maximisation.

A. FUTURE CASH FLOWS AND THE TIME VALUE OF MONEY

There are several different techniques used in organisations when making the investment decision. When evaluating an investment opportunity it is important that all cash flows arising from that opportunity are considered, and that the timing of these different cash flows is also taken into account.

You may wonder why the timing of cash flows is so important. This is because £1 received today is worth more than £1 received in 12 months’ time, because of the subjective time preference of investors, i.e. individuals prefer to consume income now rather than in the future.

There are other reasons sometimes mooted for the existence of the time value of money, but whilst these have some credence they do not fully explain the situation. The common reasons are that:

� There is a risk involved in investing. Whilst there is uncertainty involved in capital investment, techniques have been developed to deal with this within discounted cash flow (DCF) analysis (see later).

Page 266: libvolume5.xyzlibvolume5.xyz/law/ballb/3rdyear/semester5/corporatelaw/corporate... · ABE Advanced Diploma in Business Administration Study Manual CORPORATE FINANCE Contents Study

258 Capital Investment Decision Making 1: Basic Appraisal Techniques

© Licensed to ABE

� There is a risk that inflation will erode buying power during the period under consideration. However, even if there is zero inflation, techniques which consider the time value of money are still used in investment appraisal. (We shall consider methods of dealing with inflation in the next study unit.)

Let us assume that you could deposit your £1 for 12 months at a 10% annual interest rate. After one year interest of £1 × 10% = 10p would be added to your £1 and your investment would have grown to £1.10. We can therefore say that the future value of your £1 today, at an interest rate of 10%, is £1.10.

Cash therefore has a time value and we express the values today (i.e. at the start of an investment project) of future cash flows as the present value of the investment.

B. RETURN ON INVESTMENT (ACCOUNTING RATE OF RETURN)

This approach expresses the profit after tax arising from an investment as a percentage of the total outlay on the investment. The profit is expressed after depreciation as it is argued that the original capital is being recovered. The result is compared to a predetermined company (or group, or division) target, an investment being accepted if the result meets or exceeds the target.

When using ROI to compare projects which are mutually exclusive (i.e. the acceptance of one prevents the adoption of the other) the project which gives the highest ARR is the one that should be accepted (provided it meets or exceeds the target ARR).

Difficulties arise with this method when the duration of the investment (and hence the profitability from it) extends for more than one year, as it then becomes necessary to determine some representative profit and investment value for the duration of the project. This is usually achieved by a form of averaging, in view of the difficulty of estimating profit generation several years in the future.

We will now look at an example to clarify this.

An investment in a new machine costing £1,000 will generate the earnings shown below over the five years of its projected useful life. Depreciation will be on a straight line basis to reflect the estimated manner in which the value of the machine will decline over these years.

Year 1 2 3 4 5 Budgeted profits 600 600 500 500 300 less Depreciation (200) (200) (200) (200) (200)

400 400 300 300 100 Tax (say) 120 120 105 105 35

Net profit 280 280 195 195 65 We first need to establish the average profit arising from the investment and then compare this with the investment:

Average profit = 565 + 195 + 195 + 280 + 280 = £203.

Page 267: libvolume5.xyzlibvolume5.xyz/law/ballb/3rdyear/semester5/corporatelaw/corporate... · ABE Advanced Diploma in Business Administration Study Manual CORPORATE FINANCE Contents Study

Capital Investment Decision Making 1: Basic Appraisal Techniques 259

© Licensed to ABE

Firstly we can compare this with the original investment to give the rate of return:

investmentOriginal

profitAverage = 1100 000,1

203 × = 20.3%

Alternatively we can compare it with the average investment. This has traditionally been calculated as the original investment divided by 2 – thus, here, it would be £1,000 ÷ 2 = £500.

investmentAverage

profitAverage = 1100 500

203 × = 40.6%.

This is not a very satisfactory way to make the calculation, because the figure of £500 has no actual basis in reality. Furthermore, it produces a rate of return which does not represent the facts of the case. The annual returns on the original investment in our example are as shown below:

Year 1 000,1280 = 28%

Year 2 000,1280 = 28%

Year 3 000,1195 = 19.5%

Year 4 000,1195 = 19.5%

Year 5 000,165 = 6.5%, giving a total of 101.5%.

Dividing this result by 5 gives us (101.5% ÷ 5) = 20.3%, i.e. half the figure of 40.6% which we calculated by using the traditional formula for counting the investment involved.

Further problems arise because this method fails to recognise that a net profit of £65 in five years’ time is barely significant in today’s terms, even when there is a low rate of inflation. In other words, the method fails to recognise time value of money. This is a cornerstone of discounted cash flow methods as we shall shortly see.

Another issue with this type of analysis is that profits are the results of receipts and outgoings and they do not represent cash transactions and the cash flow arising is not taken into account during the term of the investment.

Probably the greatest merit in this method of analysis is its simplicity, it being based in conventional accounting terms and requiring only limited analytical skill to carry it out and to interpret the conclusions that can be drawn from it.

C. PAYBACK

This method simply measures the time period taken until the profits generated from the investment equal the initial cost of the investment. The aim is to calculate how much time will elapse before the capital project “pays back” the original amount invested from the profits generated by it. (Either cash receipts or accounting profits can be used – cash receipts would be preferable for the reasons noted above.) The result is compared to a predetermined company (or group, or division) target, an investment being accepted if the result meets or is less than the target length of time. When comparing different projects the one with the quicker payback period would be the one chosen.

Consider the following example.

Page 268: libvolume5.xyzlibvolume5.xyz/law/ballb/3rdyear/semester5/corporatelaw/corporate... · ABE Advanced Diploma in Business Administration Study Manual CORPORATE FINANCE Contents Study

260 Capital Investment Decision Making 1: Basic Appraisal Techniques

© Licensed to ABE

Project A Project B Investment outflow Year 0 (600) (600)

Cash inflows Year 1 400 700 Year 2 200 400 Year 3 800 400

Total inflows 1,400 1,500 Payback for Project A is two years, assuming that the cash inflows occur at equal periods and in equal amounts during each year. Payback in the case of Project B is just over ten months.

Payback focuses on risk in considering the period during which the investment remains outstanding. The sooner the investment is returned, the safer the project should be. You should note, however, that the method takes no account of cash inflows after payback, neither is there any attempt to consider reinvestment possibilities for incoming funds during the period prior to payback.

Perhaps, therefore, we should view payback as more of a risk appraisal tool than a performance measure.

D. DISCOUNTED CASH FLOW

Discounted cash flow (DCF) analysis (also known as present value analysis) is a technique used to determine the net value of a project in terms of today’s money. It considers the time value of money, and the cost of capital to the organisation. By using a discounted cash flow method it is possible to convert all future cash flows to their present value and then to assess them on a like-for-like basis. The net effect of all the cash inflows and outflows resulting from a project being discounted back to present values is known as a project’s net present value (NPV).

In order to convert cash flows arising from a project into their present values, it is necessary to establish the cash inflows and outflows arising from it, and what cost of capital should be used to evaluate such projects.

The cash flows, or sufficient information to determine them, will always be provided as given information and they should be recorded, and the year in which they occur, in a logical manner (you will see in this and the next study unit how this is done).

The cost of capital used in evaluating such projects is generally the required rate of return of those investing in the firm – which we have seen to be its weighted average cost of capital (WACC). To calculate the cost of equity you should use either the dividend growth model or CAPM, depending on the information provided. The resulting WACC will be slightly different, although both methods have advantages and disadvantages because they are based on different underlying assumptions. (Note that CAPM is generally used in the APV technique discussed in the next study unit.) However, we will discuss situations where an alternative rate should be used. Note that you may be presented with the cost of capital to be used, and you should always consider the information provided when determining the figure to be chosen or calculated.

This required rate of return forms the basis of the discount factors which are used to convert cash flows to their present values.

Page 269: libvolume5.xyzlibvolume5.xyz/law/ballb/3rdyear/semester5/corporatelaw/corporate... · ABE Advanced Diploma in Business Administration Study Manual CORPORATE FINANCE Contents Study

Capital Investment Decision Making 1: Basic Appraisal Techniques 261

© Licensed to ABE

The formula used for deriving the discount factor is:

ni) + 1(1

where: n represents the number of periods, and

i represents the cost of capital per period.

In practice discount factor tables are available to look up the relevant factor (see the appendix to this study unit). In order to use the discount factor tables you need to read across the year, and down from the cost of capital you are considering to obtain the discount factor. For example, with a cash flow arising 10 years in the future, for a company with a 5% cost of capital, the discount factor to use would be 0.614, giving a present value of the cash flow × 0.614.

There are also several computer packages available for the purposes of investment evaluation. Tables also exist for future cash flows.

In the assessment of the future cash flows generated by investment projects, the two most commonly used methods which use this discounted cash flow analysis are:

� Net present value (NPV)

� Internal rate of return (IRR)

The DCF methods concentrate on cash inflows and outflows associated with the project over its full life span, so you can see that they are superior to the other methods discussed so far. Another major advantage is that they consider the timing of cash flows associated with the project.

E. NET PRESENT VALUE (NPV)

The concept of net present value (NPV) is of vital importance in the field of corporate finance, and we have already made several references to it in this course.

In order to determine the NPV of a project, we need to list all the cash flows related to the project. The net cash flows are then discounted at the cost of capital using the formula shown above.

The decision rule in using the NPV technique is that if the NPV is positive the project should be accepted, and if the NPV is negative then the project should be rejected. The reasoning behind this is that when there is a positive NPV, the project offers you a return in excess of your cost of capital and acceptance of such a project will increase the wealth of the company. For a negative NPV project, the cost of capital is not covered and acceptance of such a project will reduce the value of the firm. The primary objective of the firm is, of course, to maximise shareholder wealth by maximising the value of the firm. The value of a company will increase by the NPV of a project provided that its WACC remains unchanged. The increase in wealth will be reflected in the share price because of the efficient market hypothesis (EMH).

The use of the NPV technique is best seen by considering an example.

Page 270: libvolume5.xyzlibvolume5.xyz/law/ballb/3rdyear/semester5/corporatelaw/corporate... · ABE Advanced Diploma in Business Administration Study Manual CORPORATE FINANCE Contents Study

262 Capital Investment Decision Making 1: Basic Appraisal Techniques

© Licensed to ABE

Example

A project has an initial cost of investment of £10,000 in a machine, and the following expected cash inflows:

Year 1 = £6,000 Year 2 = £6,000 Year 3 = £6,000 Year 4 = £5,000 Year 5 = £5,000

No scrap value is expected from the machine. The cost of capital is expected to be 10% throughout the five years of the project. Should the project be accepted?

The way to make this decision is to turn these future cash flows into present values by either the use of discount factor tables (see appendix), or the formula noted above. (Note that we shall use both of these methods in this study unit.)

Present value of machine revenues:

Year Net Cash Flows Formula Disc. Factor NPV £ £ 0 −10,000 - 1 −10,000

1 +6,000 1)1.01(1

+ 0.909 5,454

2 +6,000 2)1.01(1

+ 0.826 4,956

3 +6,000 3)1.01(1

+ 0.751 4,506

4 +5,000 4)1.01(1

+ 0.683 3,415

5 +5,000 5)1.01(1

+ 0.621 3,105

18,000 +11,436 In straight cash flow terms, the opportunity presented by the machine purchase is as follows:

£ Cash revenues resulting 28,000less Cost of machine 10,000

Cash gain 18,000 This cash gain is received over a period of five years, which for the flows given is the same as £11,436 now.

(Note that this method deals with liquidity, not profitability, and cash flows are used, not accounting flows (e.g. capital outlay, not depreciation).)

Page 271: libvolume5.xyzlibvolume5.xyz/law/ballb/3rdyear/semester5/corporatelaw/corporate... · ABE Advanced Diploma in Business Administration Study Manual CORPORATE FINANCE Contents Study

Capital Investment Decision Making 1: Basic Appraisal Techniques 263

© Licensed to ABE

Future Value Over Time Another way of regarding net present value is to say that, if the cost of capital is 10% over a five year period:

£11,000 in one year’s time is the same as £10,000 now

£12,100 in two years’ time is the same as £10,000 now

£13,310 in three years’ time is the same as £10,000 now

£14,641 in four years’ time is the same as £10,000 now

So the future value of £10,000 now (assuming a constant cost of capital of 10% throughout the period) is:

Year 1 £11,000

Year 2 £12,100

Year 3 £13,310

Year 4 £14,461

Example

Consider the following calculation. Suppose a company has a machine in mind which will produce a stream of revenues as follows:

£Year 1 400Year 2 600Year 3 200

1,200 Note that the value of the machine is not £1,200, but is the present value of these cash flows which, using discount factor tables, are calculated as:

Cash Flow DF PV of Cash Flow £ £

Year 1 400 0.9091 363.64 Year 2 600 0.8265 495.90 Year 3 200 0.7513 150.26

1,009.80 The value of the machine is therefore £1,009.80.

In the absence of cost of capital figures the project could be compared with the interest rate that would be achieved by investing the initial outlay in investments of a similar level of risk.

Page 272: libvolume5.xyzlibvolume5.xyz/law/ballb/3rdyear/semester5/corporatelaw/corporate... · ABE Advanced Diploma in Business Administration Study Manual CORPORATE FINANCE Contents Study

264 Capital Investment Decision Making 1: Basic Appraisal Techniques

© Licensed to ABE

Conventions Used in NPV Calculations � Presentation

The best method of answering such questions is to set out all your cash flows and discount factors in a table, putting each year’s cash flows in the same column or row. Examples of correct presentation are provided throughout this course.

� Signs

The normal method is to regard cash inflows as positive and cash outflows as negative.

� Years

A simplifying assumption is made that all cash flows occur in discrete steps at the end of the year, but all initial outlays are regarded as occurring at the end of Year Zero, and thus are not discounted (and are given a discount factor of 1).

� Time periods

Time periods always commence at the present, which is Year 0.

� Relevant figures

The only relevant figures to put into an evaluation are those cash flows arising as a result of accepting the project. Thus the concept of relevant cost, which you will have met elsewhere in your studies, is the concept which should be applied in investment appraisal. Sunk costs and apportioned overheads are, as such, ignored in DCF calculations.

Items such as depreciation, which are accounting and not cash flows, are ignored (the value of fixed assets is already taken into account in the initial outlay required for the project and in any scrap value from the assets at the end of or during the project).

Interest and repayment of loan principals are not included in the cash flows because this would be double counting, since the sums have already been included in the cost of capital used.

Net Terminal Values If the (opportunity) cost of capital in a business is 10% per annum, this will represent the required rate of return from the project. This is sometimes referred to as the cut-off rate or the criterion rate. Where the capital investment (as measured by the internal rate of return, or yield) is expected to generate less than this return, the viability of proceeding with the investment, based on the financial analysis, will be open to question since the return will be less than the cost of financing it.

Example

An investment in Project R of £20,000 is expected to generate cash receipts of £6,000 at the end of Year 1, £8,000 at the end of Year 2, and £10,000 at the end of Year 3. The business is able to invest money at an annual rate of 10% (the opportunity cost).

From the cash flows below it is clear that Project R produces insufficient return on the investment. Its net terminal value is −£560, and that indicates that greater benefit will accrue if the money is invested in a security (or with a deposit-taking institution) at the rate of 10% pa which it is assumed is freely available.

Page 273: libvolume5.xyzlibvolume5.xyz/law/ballb/3rdyear/semester5/corporatelaw/corporate... · ABE Advanced Diploma in Business Administration Study Manual CORPORATE FINANCE Contents Study

Capital Investment Decision Making 1: Basic Appraisal Techniques 265

© Licensed to ABE

End of: Cash Flow £

Compound Factor *

Terminal Value £

Year 0 (20,000) 1.331 (26,620) Year 1 6,000 1.210 7,260 Year 2 8,000 1.100 8,800 Year 3 10,000 1.000 10,000

Net Terminal Value = (560) * The compound factors reflect the fact that the receipts at the end of Year 1 are available for

investment for two years, and the receipts at the end of Year 2 for one year, i.e. for Year 2 the formula is calculated as:

(1 + i)n

where: i = the interest rate per period; and

n = the number of periods.

Therefore, (1 + 0.1)2 = 1.210.

Annuities or Uniform Series In certain cases the level of cash flow will be uniform from year to year. In such cases there is an easier method of calculating the net present value than using the discount factor tables.

The easier method is to use cumulative discount tables (provided in the appendix to this study unit) which are simply the addition of discount factors over a number of years. To use these tables you need to read down the correct cost of capital column, and across the row for the number of years the cash flows remain constant, in order to obtain the cumulative discount factor. You should, however, note that if the uniformity of cash flows does not start in Year 1, the discount figure you should use would be the difference between the discount factor at the last year for the constant flows, less that for the year before the flows commenced. The resulting present value is then discounted at the rate for the year before the constant cash flows started. An example should help to clarify this.

Example

A company is considering investing in a project which will cost £5,000, and will yield cash inflows of £2,000 in Year 1 and £3,000 for the following three years. There is no scrap value at the end of the project. Should the investment be accepted if the company’s cost of capital is 10%?

First we need to set out the cash flows as before:

Year Cash Outflow Cash Inflow Discount Factor Present Value £ £ £

0 −5,000 1 −5,000 1 2,000 0.909 1,818 2 3,000 3 3,000 2.487 × 0.909 6,782 4 3,000

3,600

Page 274: libvolume5.xyzlibvolume5.xyz/law/ballb/3rdyear/semester5/corporatelaw/corporate... · ABE Advanced Diploma in Business Administration Study Manual CORPORATE FINANCE Contents Study

266 Capital Investment Decision Making 1: Basic Appraisal Techniques

© Licensed to ABE

We discount the series of constant cash flows back to Year 1 using the (4 − 1) cumulative discount factor, and then discount that amount back one year from Year 1 to the present time.

As the project gives a positive net present value it should be accepted.

Multiple Time Periods These can be catered for by using the basic annual tables, as follows. Simply divide i by the number of times in the year that discounting occurs and multiply the years by the number of periods per year.

For example, find the PV of a uniform series of receipts of £500 discounted half-yearly for the two years, where i = 8%.

i = 8/2 = 4%

t = 2 × 2 = 4 years £500 × 3.630 = £1,815

You will not normally be requested to deal with other than annual cash flows.

NPV Profile For any project, a range of costs of capitals can be discounted and the results plotted against the resulting NPVs. This is often called an NPV Profile Curve, and it is simply a diagrammatic representation of the NPV possibilities. A profile curve is shown in Figure 12.1.

Figure 12.1: NPV Profile of Two Projects X and Y

Page 275: libvolume5.xyzlibvolume5.xyz/law/ballb/3rdyear/semester5/corporatelaw/corporate... · ABE Advanced Diploma in Business Administration Study Manual CORPORATE FINANCE Contents Study

Capital Investment Decision Making 1: Basic Appraisal Techniques 267

© Licensed to ABE

You will note that a study of the NPV profile reveals certain useful information, as follows:

(a) The net present value of a project at any selected cost of capital can be read directly from the graph, e.g.

(i) Project X has an NPV of £90,000, if the cost of capital is 10% (point “a” on the curve).

(ii) Project Y has an NPV of just below £20,000, if the cost of capital is 17% (point “b” on the curve).

(b) Where two project curves are shown, any point of intersection marks the spot below which one project is more profitable than the other, and above which the other project is more profitable.

The project curves for X and Y intersect at 11.5%. Above this rate of interest Y is more profitable, below it X is the more profitable project with a higher NPV.

(c) Where the curves cut the horizontal axis, the NPV of each project is nil (£0), the cost of capital being equal to the project’s yield; this is known as a project’s internal rate of return (see later). X has an internal rate of return of 15%, and Y has one of 19%.

Perpetuities When a project yields a sum for ever then its present value can be calculated as:

PV = iflowCash

because the cumulative present value of £1 in perpetuity (i.e. each year for ever) is £1/i.

F. INTERNAL RATE OF RETURN

We stated above that the internal rate of return of a project is that cost of capital which makes the net present value of a project to be equal to zero (the higher the cost of capital the lower the net present value of cash flows). It is this percentage rate of discount which we compare with the company’s cost of capital. If the cost of capital required to reduce the future cash flows to zero is greater than the company’s cost of capital, then the project will be accepted because it produces a positive return for the business.

You can see this in Figure 12.2, which represents the approach of taking a number of cash inflows and outflows and discounting them over a constant period of time at an increasing discount rate.

Figure 12.2

Page 276: libvolume5.xyzlibvolume5.xyz/law/ballb/3rdyear/semester5/corporatelaw/corporate... · ABE Advanced Diploma in Business Administration Study Manual CORPORATE FINANCE Contents Study

268 Capital Investment Decision Making 1: Basic Appraisal Techniques

© Licensed to ABE

A discount rate of 0% clearly has no effect. As the percentage rate increases, the NPV falls until at a particular cost of capital, the NPV is equal to (or approximately equal to) zero, i.e. NPV = 0. This is the project’s internal rate of return. The internal rate of return would then be compared to the company’s cost of capital; if it were higher it would be accepted, and if it were lower it would be rejected, because it is not generating sufficient funds to satisfy the needs of the suppliers of the company’s finance. This method can also be used to compare the cost of financing using alternative financial instruments.

Essentially each cash flow is appreciated to have an element of capital and interest within it. Once the interest element is removed by discounting the instalments by the discount rate, the payments, now comprising only the capital element, will equal the initial investment at the IRR point.

The internal rate of return can be found by trial and error – in order to establish the precise discount rate several attempts will usually be required using different discount rates until the sum of the cash flows equals the initial investments. Further adjustments may well be needed where the cash flow payments required by the transactions are of an uneven nature. A good starting place for such appraisals is two-thirds of the project’s ROI.

An alternative approach is to plot an NPV profile using two or more discount rates and to read the approximate internal rate of return from the graph. This is known as interpolation.

Example

An investment of £20,000 is expected to generate cash receipts of £6,000 at the end of Year 1, £8,000 at the end of Year 2, and £10,000 at the end of Year 3. The business can invest money at an annual rate of 10% (its opportunity cost).

Our aim will be to find the discount rate which reduces all the cash flows to zero. In our example the NPV is almost zero when we use an IRR (discount rate) of 9%, and from the table below you will see that the return on Project R is just under 9%. As the required rate of return is 10% (or more), the project would be rejected.

The table below shows the calculations at a discount rate of 9%. (The IRR will be determined by trial and error.)

End of Year Cash Flows Discount Factor Present Value £ 9% £ 0 (20,000) 1.000 (20,000) 1 6,000 0.917 5,502 2 8,000 0.842 6,736 3 10,000 0.772 7,720

NPV = (42) We should remember that a project with a higher IRR may not necessarily be better than a project with a lower IRR. Let’s just stop to think why.

The main reasons for this relate to the actual amount of money which is to be returned. A 30% return may sound excellent, but if the investment is only a few hundred pounds, it may not be worth devoting the time necessary if a larger project with a lower IRR will generate better cash returns.

It is also important to recognise that IRR assumes that surplus cash flows are to be invested at the project’s own “internal” rate. However, this may not be possible in many months’ time when market forces have changed, with the result that the returns actually available have fallen.

Page 277: libvolume5.xyzlibvolume5.xyz/law/ballb/3rdyear/semester5/corporatelaw/corporate... · ABE Advanced Diploma in Business Administration Study Manual CORPORATE FINANCE Contents Study

Capital Investment Decision Making 1: Basic Appraisal Techniques 269

© Licensed to ABE

Pitfalls with IRR Before attempting any detailed analysis, it is important to be aware of the potential pitfalls which can arise when, for instance, the net cash flow changes during the investment period from positive to negative, or vice versa. When this happens multiple internal rate solutions are possible, and for the sake of clarity we shall give an example.

Example

An initial investment of £3,950 generates the following cash flows in the next three years:

£ Year 0 (3,950 ) Year 1 13,102 Year 2 (14,500 ) Year 3 5,350

2 The overall return on the initial investment is 2, hardly a significant contribution on the face of it. However, the cash flows actually satisfy an internal rate of return of 5%, 10% and 15% as shown below!

Year Cash 5% 10% 15% £ £ £ £ 1 13,102 12,473 11,909 11,398 2 (14,500) (13,151) (11,977) (10,962) 3 5,350 4,622 4,018 3,520

3,952 3,944 3,950 3,956 This problem can arise when the cash flow is positive during one period and negative during another. Every change of sign gives rise to an additional solution.

Dual Rate of Return Method It is possible to resolve this potential problem by using the Dual Rate of Return Method (sometimes known as the Extended Yield Method).

To do this, it is initially necessary to identify all periods in which the cumulative cash flows produce a cash surplus, when discounted at the internal rate of return. These cash flows are then discounted at a predetermined rate (the rate required from the project), in order subsequently to calculate the internal rate of the remaining cash flows.

This is a particularly important technique when conducting the evaluation of a project which has surplus cash generated during only a part of its term.

Page 278: libvolume5.xyzlibvolume5.xyz/law/ballb/3rdyear/semester5/corporatelaw/corporate... · ABE Advanced Diploma in Business Administration Study Manual CORPORATE FINANCE Contents Study

270 Capital Investment Decision Making 1: Basic Appraisal Techniques

© Licensed to ABE

G. COST/BENEFIT RATIO

This ratio, sometimes referred to as the Profitability Index (PI), is calculated by the formula:

outflow investmentPresent

inflow future of PV (discounted at the cost of capital)

A project offering a PI of greater than 1.0 should be accepted. In the case of competing projects, the highest over 1.0 will generally be preferred.

Thus, where capital rationing is important, the PI can be used to help to “rank” projects in order of relative profitability.

H. COMPARISON OF METHODS

As with other areas of financial modelling, each method of investment appraisal has its drawbacks, and most firms use three or four of the different methods.

Non-Discounted Methods The ARR ignores both the timing of cash flows and the opportunity cost of capital, but it is used in practice in approximately half of all companies.

The payback method ignores the time value of money, and total cash flows over a project’s life once the payback period has been reached. It is often used in practice, however, as a screening device, being considered to provide a fair approximation to NPV if cash flows follow a pattern. It is useful when firms have liquidity problems or are perhaps producing novelty products which require a quick repayment of investment.

Discounted Methods We noted above that it is generally preferable to use a method of investment appraisal that discounts cash flows, the two main methods being IRR and NPV. Both these techniques are acceptable in capital investment appraisal if an organisation can accept all projects which are beneficial to the organisation.

However, when there is capital rationing, NPV is the better method, as IRR can mis-rank the projects. This superiority can be proven using the incremental approach.

Example

Zak plc has only the space to implement either Project X or Project Y; both projects last one year and have the following details:

Cash Flow Yr 0

Cash Flow Yr 1

IRR NPV @ Cost of Capital 10%

Project X (10,000) 20,000 100% 8,182 Project Y (20,000) 35,000 75% 11,818

NPV and IRR give different rankings. To find which is the better we need to use the incremental cash flow approach.

Page 279: libvolume5.xyzlibvolume5.xyz/law/ballb/3rdyear/semester5/corporatelaw/corporate... · ABE Advanced Diploma in Business Administration Study Manual CORPORATE FINANCE Contents Study

Capital Investment Decision Making 1: Basic Appraisal Techniques 271

© Licensed to ABE

Firstly, let us accept Project X which is the preferred project using the IRR technique. If this is the correct choice, then the incremental cash flows of Y − X will not produce an IRR which is acceptable (i.e. it will be lower than the cost of capital).

Consider the differences to cash flows if we move from Project Y to Project X:

Cash Flow Yr 0

Cash Flow Yr 1

IRR NPV @ Cost of Capital 10%

Change = (10,000) 15,000 50% 3,636 IRR of 50% is acceptable – therefore using IRR as an appraisal tool we should accept X and (Y − X), but X + (Y − X) = Y. Thus, Y should be chosen (as per NPV) and we have used IRR to prove that NPV is the superior method.

NPV is also preferable because it is very useful for evaluating interrelated projects, and it emphasises the size of return given. IRR also has the problem that it can give multiple IRRs.

The advantage of IRR over NPV is that IRR is better at highlighting the rate of return against the cost of capital.

Despite the advantages of NPV over IRR, IRR is more popular in practice.

I. IMPACT OF TAXATION ON CAPITAL INVESTMENT APPRAISAL

It is important to consider the effects of taxation on investment appraisal – not just as part of a numerical question, but also because it can affect the outcome of the decision as to whether to invest.

Taxation must be taken into account in DCF analysis, since tax payments can seriously affect the cash flows arising over the project’s life, and you should always include additional columns or rows in your DCF analysis to allow for taxation.

Capital investment appraisal is affected by taxation in four main ways:

(a) Annual profits are taxed – in practice nine months after the end of the accounting year – but you must read the question carefully to see what assumptions the examiner has made. The tax outflow may be included one year after the year in which the taxable income arose, i.e. there will be a time lag of one year, making necessary a five-year table for a four-year investment scheme.

(b) Interest on debt is allowable against corporation tax, and this will affect the discount rate used.

(c) Tax losses must be included and they will normally be shown as a tax receipt one year later.

(d) The existence of grants and capital allowances which help reduce the tax bill – thus making the project likely to be accepted. You must always read the question carefully for details of any such allowances. The most common allowance is the 25% reducing balance writing down allowance.

Capital allowances on plant, machinery and so on are provided by means of writing down allowances (WDAs). These relate to an annual percentage of the tax written down value (WDV) of a qualifying asset. WDV is the original cost less capital allowances previously given. WDA is currently 25% on plant and machinery, and 4% on industrial buildings, based on a reducing balance method of computation.

Page 280: libvolume5.xyzlibvolume5.xyz/law/ballb/3rdyear/semester5/corporatelaw/corporate... · ABE Advanced Diploma in Business Administration Study Manual CORPORATE FINANCE Contents Study

272 Capital Investment Decision Making 1: Basic Appraisal Techniques

© Licensed to ABE

First year allowances (FYAs) were introduced for small and medium-sized businesses for the period 2 July 1997 to 1 July 1998 (50% for non long-life assets) then again from 2 July 1998 to 1 July 2000 (40% for non long-life assets) and only apply for machinery and plant.

To apply capital allowances in practice we should assume, for example, that once an item of plant and equipment has been purchased, then the corresponding tax allowance at the current tax rate will effectively reduce the outlay for the item, because there will be less tax to pay. The payment of tax and, hence, the cash benefit will usually be in the year following the year in which the asset was acquired.

When appraising a capital investment scheme you not only need to take taxation into account, but you must also consider the effect that a change in the tax rate or law (e.g. the removal of a government grant) would have on a project. You must also consider the possibility of any additional subsidies the firm may be able to claim, and their effect on the value of the project and, therefore, of the company.

Practice Question

A project has an initial cost of investment of £25,000. It is expected to produce the following cash inflows:

Year 1 £3,000 Year 2 £4,500 Year 3 £9,000 Year 4 £11,500

No scrap value is expected. The cost of capital is expected to be 9% over the four years. Should the project be accepted?

Now check your answer with the one given at the end of the unit.

Page 281: libvolume5.xyzlibvolume5.xyz/law/ballb/3rdyear/semester5/corporatelaw/corporate... · ABE Advanced Diploma in Business Administration Study Manual CORPORATE FINANCE Contents Study

Capital Investment Decision Making 1: Basic Appraisal Techniques 273

© Licensed to ABE

ANSWER TO PRACTICE QUESTION

Present value of revenues:

Net Cash Flows £

Discount Factor NPV£

0 (25,000) 1 (25,000) 1 3,000 0.917 2,751 2 4,500 0.842 3,789 3 9,000 0.772 6,948 4 11,500 0.708 8,142

3,000 (3,370) The project would not be accepted.

Page 282: libvolume5.xyzlibvolume5.xyz/law/ballb/3rdyear/semester5/corporatelaw/corporate... · ABE Advanced Diploma in Business Administration Study Manual CORPORATE FINANCE Contents Study

274 Capital Investment Decision Making 1: Basic Appraisal Techniques

© Licensed to ABE

APPENDIX: DISCOUNTING TABLES

On the following pages you will find sets of discounting tables for:

� £1 compounded annually

� Uniform series (annuity) compounded annually

� Present value of £1

� Present value uniform series (annuity)

� Present value of an annuity

Page 283: libvolume5.xyzlibvolume5.xyz/law/ballb/3rdyear/semester5/corporatelaw/corporate... · ABE Advanced Diploma in Business Administration Study Manual CORPORATE FINANCE Contents Study

Capital Investment Decision Making 1: Basic Appraisal Techniques 275

© Licensed to ABE

Single Payment Compounded Forward Factor

Page 284: libvolume5.xyzlibvolume5.xyz/law/ballb/3rdyear/semester5/corporatelaw/corporate... · ABE Advanced Diploma in Business Administration Study Manual CORPORATE FINANCE Contents Study

276 Capital Investment Decision Making 1: Basic Appraisal Techniques

© Licensed to ABE

Uniform Series Compounded Forward Factor

(Continued over)

Page 285: libvolume5.xyzlibvolume5.xyz/law/ballb/3rdyear/semester5/corporatelaw/corporate... · ABE Advanced Diploma in Business Administration Study Manual CORPORATE FINANCE Contents Study

Capital Investment Decision Making 1: Basic Appraisal Techniques 277

© Licensed to ABE

Uniform Series Compounded Forward Factor (Continued)

Page 286: libvolume5.xyzlibvolume5.xyz/law/ballb/3rdyear/semester5/corporatelaw/corporate... · ABE Advanced Diploma in Business Administration Study Manual CORPORATE FINANCE Contents Study

278 Capital Investment Decision Making 1: Basic Appraisal Techniques

© Licensed to ABE

Sing

le P

aym

ent P

rese

nt W

orth

Fac

tor

(Dis

coun

t Tab

les)

(Con

tinue

d ov

er)

Page 287: libvolume5.xyzlibvolume5.xyz/law/ballb/3rdyear/semester5/corporatelaw/corporate... · ABE Advanced Diploma in Business Administration Study Manual CORPORATE FINANCE Contents Study

Capital Investment Decision Making 1: Basic Appraisal Techniques 279

© Licensed to ABE

Sing

le P

aym

ent P

rese

nt W

orth

Fac

tor

(Dis

coun

t Tab

les)

(C

ontin

ued)

Page 288: libvolume5.xyzlibvolume5.xyz/law/ballb/3rdyear/semester5/corporatelaw/corporate... · ABE Advanced Diploma in Business Administration Study Manual CORPORATE FINANCE Contents Study

280 Capital Investment Decision Making 1: Basic Appraisal Techniques

© Licensed to ABE

Uni

form

Ser

ies P

rese

nt W

orth

Fac

tor

(Cum

ulat

ive

Disc

ount

Tab

les)

(Con

tinue

d ov

er)

Page 289: libvolume5.xyzlibvolume5.xyz/law/ballb/3rdyear/semester5/corporatelaw/corporate... · ABE Advanced Diploma in Business Administration Study Manual CORPORATE FINANCE Contents Study

Capital Investment Decision Making 1: Basic Appraisal Techniques 281

© Licensed to ABE

Uniform Series Present Worth Factor (Cumulative Discount Tables) (Continued)

Page 290: libvolume5.xyzlibvolume5.xyz/law/ballb/3rdyear/semester5/corporatelaw/corporate... · ABE Advanced Diploma in Business Administration Study Manual CORPORATE FINANCE Contents Study

282 Capital Investment Decision Making 1: Basic Appraisal Techniques

© Licensed to ABE

Annuity Tables

Present value of an annuity of 1, i.e. 1 (1 + r)

r

n− −

where: r = interest rate and where n = years

Interest Rates

(continued over)

Page 291: libvolume5.xyzlibvolume5.xyz/law/ballb/3rdyear/semester5/corporatelaw/corporate... · ABE Advanced Diploma in Business Administration Study Manual CORPORATE FINANCE Contents Study

Capital Investment Decision Making 1: Basic Appraisal Techniques 283

© Licensed to ABE

Annuity Tables (Continued)

Interest Rates

Page 292: libvolume5.xyzlibvolume5.xyz/law/ballb/3rdyear/semester5/corporatelaw/corporate... · ABE Advanced Diploma in Business Administration Study Manual CORPORATE FINANCE Contents Study

284 Capital Investment Decision Making 1: Basic Appraisal Techniques

© Licensed to ABE

Page 293: libvolume5.xyzlibvolume5.xyz/law/ballb/3rdyear/semester5/corporatelaw/corporate... · ABE Advanced Diploma in Business Administration Study Manual CORPORATE FINANCE Contents Study

285

© Licensed to ABE

Study Unit 13

Capital Investment Decision Making 2: Further Considerations

Contents Page

Introduction 286

A. Allowance for Risk and Uncertainty 286 Limiting the Payback Period 286 Inclusion of a Risk Premium in the Discount Rate 286 Certainty Equivalents 286 Attaching Probabilities to Cash Flows 287 Simulation Models 287 Sensitivity Analysis 287 Other Approaches 288

B. Impact of Inflation on Investment Appraisal 288

C. Capital Rationing 289

D. Lease Versus Buy Decisions 291

E. Post-Appraisal Audit 293

F. Adjusted Present Value (APV) 293 Side Effects of Different Financing Methods 293

G. Use of the Capital Asset Pricing Model 296

H. Worked Examples 297

Answers to Practice Questions 310

Page 294: libvolume5.xyzlibvolume5.xyz/law/ballb/3rdyear/semester5/corporatelaw/corporate... · ABE Advanced Diploma in Business Administration Study Manual CORPORATE FINANCE Contents Study

286 Capital Investment Decision Making 2: Further Considerations

© Licensed to ABE

INTRODUCTION

In this study unit we shall continue our look at investment appraisal by considering other factors that impact on the decision, including inflation and uncertainty. We shall also consider a further technique of investment appraisal – that of adjusted present value (APV) which builds upon the work we have already covered on NPV and Modigliani and Miller.

You should note, though, that capital investment appraisal is not, however, a mechanistic process where data is processed and an irrevocable “correct” answer emerges. The methods are only as good as the data input to them. Despite the refinement of using risk, inflation and other allowances, capital investment appraisal still requires the decision-maker – the human being – to assimilate the available data and take a risk, using human senses backed with management information.

A. ALLOWANCE FOR RISK AND UNCERTAINTY

All investments are subject to risk from unforeseen factors such as new legislation or changes in fashion, which makes the original estimates of costs and sales, etc. no longer valid. There are several accepted methods for incorporating risk into a capital investment appraisal and the more common ones are outlined below.

Note, though, that underlying all these approaches to allowing for risk is the problem that we are attempting to quantify in real numbers some aspect of the future which is largely subjective to the person and firm carrying out the appraisal.

Limiting the Payback Period One common method of dealing with risk and uncertainty is to limit the length of time a project can pay back its investment. The justification used is that the further into the future are the estimates of cash flows, the more inaccurate they are likely to be. Alternatives based on this approach are to state that a project must also have a positive NPV in addition to meeting its payback criterion, or that the discounted cash flows must pay back within a certain time period (i.e. the present values of cash flows occurring during the payback period must be positive).

Where there are constant cash inflows the payback of discounted cash flows can be found by dividing the initial outlay by the annual cash flow and seeing at what point in time the resulting figure is less than the cumulative discount factor.

Inclusion of a Risk Premium in the Discount Rate The inclusion of a risk premium in the discount rate means that, if the normal discount rate to be used were, say, 12%, then an additional amount – say 4% – might be allowed to cover for risk, giving a 16% discount rate in total. Obviously the premium added is subjective and is thus correspondingly weak. Higher discount rates impact more significantly on the more distant cash flows so that two projects – one short and one long in life-span – would be treated differently for risk by this method.

Certainty Equivalents Under this method the expected cash flows arising from a project are subjectively adjusted by management to their equivalent riskless amounts. Different risk factors can be used for each cash flow item and for each year – the greater the risk the smaller will be the certainty equivalent value for cash inflows and the larger the certainty equivalent value for each outflow. The risk factors are generally expressed as percentages, e.g. 80% of income, 20% of costs.

Page 295: libvolume5.xyzlibvolume5.xyz/law/ballb/3rdyear/semester5/corporatelaw/corporate... · ABE Advanced Diploma in Business Administration Study Manual CORPORATE FINANCE Contents Study

Capital Investment Decision Making 2: Further Considerations 287

© Licensed to ABE

Attaching Probabilities to Cash Flows Another method of looking at each cash flow separately is to attach individual probabilities to each cash inflow and outflow.

This method can also be used to calculate the worst possible outcome and its attached probability, or the probability that the NPV is negative or zero (i.e. it is a measure of risk).

For example:

Value Probability £

130 0.2 130 0.6 145 0.2

We are stating probability values for the possible value of the outlay, so the “expected” value is:

£ £130 × 0.2 = 26130 × 0.6 = 78145 × 0.2 = 29

133 (“Expected”, here, is used in its statistical sense.)

Similarly, probabilities can be attached to the other inflows and outflows, to arrive at an expected NPV. You should note that this procedure is rather more complicated mathematically; this simple outline is given to indicate the formal methods of allowing for risk in DCF appraisals.

Simulation Models An alternative to calculating an expected value when given probability estimates is to use a simulation model to establish a probability distribution of the project’s expected NPV.

Simulation models, such as the Monte Carlo Simulation, are operated on computers using random numbers. The model of the cash flows is constructed and a range of random numbers is assigned to each possible value for those variables which are uncertain. The computer generates a series of random numbers and uses them to assign values to the variables. The results can then be used (probably by the computer) to calculate the NPV for each set of random numbers generated. The average, and range, of possible NPVs can be determined and used in the decision as to whether to accept the project or not. From this we can move to the distribution of the NPV or IRR and, in the decision-making situation we can match the means and standard deviations of competing projects, matching expected revenues (mean) against the risk of achieving the same (standard deviation).

Sensitivity Analysis Basically, an investment project usually represents, as we have seen, an outflow of cash, followed at later time intervals by an associated series of inflows. Allowing for risk aims to quantify the likely effects of the results of the project not conforming to the pattern anticipated when the project was assessed. Sensitivity analysis is an attempt to add to the number of dimensions in an appraisal picture by indicating what the picture will look like if certain dominant items in it are flexed away from their original quantification.

Page 296: libvolume5.xyzlibvolume5.xyz/law/ballb/3rdyear/semester5/corporatelaw/corporate... · ABE Advanced Diploma in Business Administration Study Manual CORPORATE FINANCE Contents Study

288 Capital Investment Decision Making 2: Further Considerations

© Licensed to ABE

You will also appreciate that each of the pieces of data that collectively form an “investment” will have a relative order of importance as to the resulting effect (of its being the one to go wrong) on the overall project. Clearly, some aspects will be vitally important to the project; others will be only marginally important. Some can go wildly astray; others deviate only a fraction before the viability of the project is placed in jeopardy.

For example, a particular investment in, say, a drilling machine will not be greatly influenced if it is found that more water-based coolant is required than was allowed for; but if the expensive driving belts prove to require unexpectedly frequent renewal that would, obviously, be more significant.

Here is risk in a new light, then – recognising that the component aspects of an investment project can all have a probability of occurring as anticipated – and, further, that the various aspects can have a separate and differing significance to the project.

Sensitivity analysis seeks to provide the decision-maker with more than just a “go/no go” statement, and aims to bring in all the “maybes” and “ifs”, i.e. each aspect of the investment can be flexed slightly, and then a re-evaluation made, to see if the project is still acceptable or not.

The decision-maker can be presented with an appraisal which says: “This project is OK as it stands on paper; costs can be allowed to increase by x% or sales decline by y% before it becomes an unacceptable project”.

The sensitivity of a project to alterations in the original appraisal input data is analysed so that the horizon of the decision-maker is widened, in order that they can see the proposed project in its overall situation if things start to vary from those anticipated.

Other Approaches � A simple approach would be to have a “worst”, “likely” and “best” estimate of each projected

cash flow. In this way a project could be appraised in each of the available lights.

� Linear programming (LP) could be used in a situation where a certain number of constraints and variables exist. Some variables are known to be “slack” and LPs can be produced with differing values input to these slacks.

� Option theory can be used in investment appraisal. When a project is undertaken it often provides additional options – to abandon a project, to make follow-on investments and to wait before undertaking an investment – and such options may need to be considered when undertaking investment appraisal. Such calculations are best undertaken using a computer model. (Options and option theory are considered in detail in the a later unit.)

B. IMPACT OF INFLATION ON INVESTMENT APPRAISAL

The rate of inflation can have a major impact on a project, and management must be made aware of any inflation assumptions made in NPV calculations. Remember that any projected inflation figures are only estimates, and that the rate of inflation can vary between different years, and widely between different elements of cost, but to produce estimates for every element of cost for every year is not cost-effective.

The greater the rate of inflation the greater the minimum rate of return required by investors, in order to compensate for loss of income due to a declining value of money.

When considering the choice of the cost of capital to use in investment appraisal we need to discuss the difference between the real and money (or nominal cost) of capital. The real cost of capital is that in present value terms, whereas the nominal or money cost of capital is that cash flow actually paid

Page 297: libvolume5.xyzlibvolume5.xyz/law/ballb/3rdyear/semester5/corporatelaw/corporate... · ABE Advanced Diploma in Business Administration Study Manual CORPORATE FINANCE Contents Study

Capital Investment Decision Making 2: Further Considerations 289

© Licensed to ABE

out. If you are asked in the examination about inflationary influences on DCF calculations, the following formula will be useful to mention:

Real cost of capital = i + 11 m −

where: m = the money cost of capital, and

i = rate of inflation

When determining which discount factor to use you should note that the money rate of interest should be used when money cash flows are used, and real rates of interest if the cash flows are expressed in real terms. Thus, if you are required to adjust cash flows for inflation, you should use the money rate of interest, and if you are required to ignore inflation, or to treat all cash flows as real cash flows, then you should use the real cost of capital. It is essential to read the information provided carefully and state all assumptions – if inflation is not mentioned then you should state “I am assuming that all cash flows are real cash flows, and the correct cost of capital to use is the real cost of capital”.

Where the decision-maker is aware that inflation is going to occur they may wish to allow for it in their calculations. The correction may be either specific or general, as with adjustments for risk.

� Specifically, the cash flows for each year can be adjusted for the rate of inflation expected to occur during that year, in an attempt to reduce all the data to a common base level.

� Generally, an allowance can be made in the discount rate to cover the expected rate of inflation. Expectations about inflation are unlikely to be accurate and if they are significant to the outcome of the project we should use the risk and uncertainty techniques discussed above.

Inflation can also have an impact on gearing and the resulting cost of capital. Inflation may result in a company increasing its selling price, which may have a major impact on demand for a company’s goods or services.

C. CAPITAL RATIONING

An organisation is said to be in a capital rationing situation when it has insufficient funds to accept all projects with a positive NPV. A decision therefore must be made as to which projects to choose. The technique used depends on whether capital rationing only exists for the current period (single period capital rationing) or whether it will be limited for several periods, and whether the projects being considered are divisible (can be undertaken in whole or in parts) or non-divisible (can only be undertaken as a whole or not at all).

In a situation where there is single period capital rationing and divisible projects, management should choose the projects which give the highest NPV per £1 of capital invested (i.e. maximising the return from the limiting factor).

Example

The management of Rosie Ltd have found that for the following year the company has only £100,000 available for investment. The company’s cost of capital is 20%. They are currently considering four independent and divisible projects, as set out in the following table.

Page 298: libvolume5.xyzlibvolume5.xyz/law/ballb/3rdyear/semester5/corporatelaw/corporate... · ABE Advanced Diploma in Business Administration Study Manual CORPORATE FINANCE Contents Study

290 Capital Investment Decision Making 2: Further Considerations

© Licensed to ABE

Project Investment Required NPV at 20% £ £

W 100,000 48,000 X 20,000 16,000 Y 30,000 18,000 Z 45,000 21,000

How should Rosie Ltd. proceed?

First we need to calculate the NPV per £1 of capital invested, i.e. the return achieved per unit of limiting factor, and rank the projects according to the results.

Project Investment Required NPV at 20% NPV/£1 Invested Ranking £ £

W 100,000 48,000 0.48 3 X 20,000 16,000 0.80 1 Y 30,000 18,000 0.60 2 Z 45,000 21,000 0.47 4

The available £100,000 can now be allocated:

Project Investment NPV £ £

X 20,000 16,000 Y 30,000 18,000 W (balance use 1/2) 50,000 24,000

100,000 58,000 This combination of projects gives the maximum return to the company and should be accepted by the management.

If the projects are not divisible then this method may not give the optimal decision because there is likely to be unused capital. This unused capital could be invested and will earn interest. The best method of solving such a problem is trial and error by comparing the NPV available from the different possible combinations of projects, remembering to calculate any interest that would be received on the unused capital.

For multi-period capital rationing the timing of cash flows from each project is important, but again management are attempting to maximise NPV per unit of scarce resource – capital. When there are divisible projects, linear programming can be used, and when there are non-divisible projects integer programming would be used.

Page 299: libvolume5.xyzlibvolume5.xyz/law/ballb/3rdyear/semester5/corporatelaw/corporate... · ABE Advanced Diploma in Business Administration Study Manual CORPORATE FINANCE Contents Study

Capital Investment Decision Making 2: Further Considerations 291

© Licensed to ABE

D. LEASE VERSUS BUY DECISIONS

Because purchasing involves a large (in relative terms) capital outlay, which may involve borrowing, an organisation may consider leasing an asset rather than purchasing it outright. Purchase also leads to a commitment to particular assets which may succumb to new technology very quickly; leasing might provide an easier avenue to new technology, as and when it arrives. When considering whether or not to lease an asset it is assumed that the funds would come from borrowing rather than from the general pool of retained earnings, thus the decision could be seen as one of lease or borrow.

The methods used to consider hire purchase of an asset are the same as for considering leasing an asset, but when looking at the non-financial aspects of the decision you should remember that when the HP term is over the asset belongs to the firm.

The methods discussed below are based on finance leases – operating leases are simply a form of renting and for them the only relevant cash flows are the lease payments and the tax saved from offsetting these payments against tax.

The traditional method of considering the financial implications of whether or not to lease an asset is done in two stages; first a decision is made as to whether or not to purchase the asset based on the operating cash flows arriving from it using the NPV techniques (with the discount factor being the WACC or other rate generally used by the organisation) that we have already discussed. If it is decided to purchase the asset (i.e. the NPV is positive) then a decision is made as to how to finance the asset (i.e. whether to lease it or fund it some other way). This latter decision is made by discounting the differential cash flows which would arise from leasing at the company’s (after tax – if tax-payers) cost of borrowing.

When considering a lease or buy decision you must be careful to consider all the taxation implications. The Finance Act 1991 states that depreciation is allowable as an expense against taxation in the form of capital allowances, as is the interest element of the finance charge. If the purchase decision involves borrowing to buy, remember that debt interest is allowable against tax; lease payments may also be allowable. Again, it is important to read the information provided very carefully, and to state any assumptions you make; a generally accepted simplifying assumption is that lease payments are all fully allowable against tax – something you may assume unless told otherwise.

You must also be careful to include all cash flows – including the sale of the asset (if bought), any extension fees of the lease, any maintenance costs and so on. Whilst it is possible to calculate the comparison in one table you are less likely to make mistakes if you calculate the NPV of the two options separately, and then compare them.

Other factors that may need to be taken into account are:

� The company’s liquidity and cash flow position – it may not be in the position to purchase an expensive asset outright.

� The choice of lease or buy will have an effect on reported profits, and may affect the market’s view of the firm.

� If the asset was to be purchased outright there would be an opportunity cost of what other uses the funds could be put to.

� Expenses of maintenance, insurance and so on may differ between leasing and purchasing.

The costs of leasing may be far lower than a company’s cost of capital, and as such there is a danger in using the traditional approach that a project may be rejected before its financing decision is considered when it would be worthwhile at the lower cost of capital.

Page 300: libvolume5.xyzlibvolume5.xyz/law/ballb/3rdyear/semester5/corporatelaw/corporate... · ABE Advanced Diploma in Business Administration Study Manual CORPORATE FINANCE Contents Study

292 Capital Investment Decision Making 2: Further Considerations

© Licensed to ABE

In order to overcome this problem, some financial managers take the decision the other way round – deciding which is the cheaper method of financing and then evaluating the project at the cheaper cost.

A second and more preferable method of overcoming this problem is to evaluate the project as though it is purchased, and then as though it is leased. The correct decision is the one which provides the highest NPV. If neither NPV is positive the project should be rejected.

Example

A company is to make the decision whether to lease or buy a new Toyota Corolla car for use in the business. The cost of the car is £14,000 and it has an estimated useful life of five years. Due to the very high mileage the car is likely to do in this period, there is no estimated residual value. Assume tax is payable at 31% on operating cash flows one year in arrears. Capital allowances of 25% on a reducing balance basis are given on the car.

The company has the option to lease the car under a finance agreement for five years at an annual cost of £3,200, payable at the year end. If the company were to purchase the car it would need to borrow the full amount at 12% interest. Which is the most cost-effective option?

We first need to calculate capital allowances if the car is purchased:

£ Year 1: 25% × £14,000 3,500Year 2: 25% × £10,500 2,625Year 3: 25% × £7,875 1,969Year 4: 25% × £5,906 1,477

9,571

Year 5: £14,000 − £9,571 4,429 We now need to look at the cost of both options.

Leasing

Assume that lease payments are eligible for a full tax allowance. (Note that the discount rate of 12% must be adjusted for tax relief on borrowing costs, giving 12% × (100% −−−− 31%) = 8%.

Year Cash Flow Discount Factor NPV £ 8% £

1 Lease cost (3,200) 0.926 (2,963) 2 Lease cost (3,200) 0.857 (2,742) Tax saving on lease cost (× 31%) 992 0.857 850

3 Lease cost (3,200) 0.794 (2,541) Tax saving on lease cost (× 31%) 992 0.794 788

4 Lease cost (3,200) 0.735 (2,352) Tax saving on lease cost (× 31%) 992 0.735 729

5 Lease cost (3,200) 0.681 (2,179) Tax saving on lease cost (× 31%) 992 0.681 676

6 Tax saving on lease cost (× 31%) 992 0.630 625

(9,109)

Page 301: libvolume5.xyzlibvolume5.xyz/law/ballb/3rdyear/semester5/corporatelaw/corporate... · ABE Advanced Diploma in Business Administration Study Manual CORPORATE FINANCE Contents Study

Capital Investment Decision Making 2: Further Considerations 293

© Licensed to ABE

Purchase

Year Cash Flow Discount Factor NPV £ 8% £

0 Cost of car (14,000) 1.000 (2,963) 2 Tax saved (capital allowances)

31% × £3,500

1,085

0.857

930

3 Tax saved 31% × £2,625 814 0.794 646 4 Tax saved 31% × £1,969 610 0.735 448 5 Tax saved 31% × £1,477 458 0.681 312 6 Tax saved 31% × £4,429 1,373 0.630 865

(10,799) The leasing option is therefore the cheaper one and should be adopted.

E. POST-APPRAISAL AUDIT

An important aspect of capital investment appraisal should be a post-appraisal audit. It is by checking back on forecasts, when the actual facts become known, that a certain amount of stringency is brought into the situation. Of course, we can only calculate actual findings for the project which was implemented. The project which was displaced or rejected cannot be measured, so appraisal audit is confined to those events which did occur, not those prevented from occurring by the selection of another alternative.

F. ADJUSTED PRESENT VALUE (APV)

Side Effects of Different Financing Methods Conventional methods of capital investment appraisal (e.g. NPV and IRR) discount the relevant future cash flows at the organisation’s WACC. This is fine for simple, small projects, but it ignores changes in financing arrangements and differing levels of systematic risk that may arise from accepting a new project. These changes in financing arrangements often occur because a new share or debt issue may be raised to pay for the capital project.

The interaction that may occur between the financing and investment decisions can be dealt with in capital investment appraisal by adjusting the WACC using the CAPM (see earlier in the course) or by using the adjusted present value technique which considers the above points.

The steps in using an APV approach are:

(a) Find the ungeared cost of equity for the firm.

(b) Estimate the base case NPV – this is done by discounting the project’s cash flows at the ungeared cost of equity.

(c) Evaluate and discount the side effects of the project and its financing methods, examples of these being:

� Tax

Page 302: libvolume5.xyzlibvolume5.xyz/law/ballb/3rdyear/semester5/corporatelaw/corporate... · ABE Advanced Diploma in Business Administration Study Manual CORPORATE FINANCE Contents Study

294 Capital Investment Decision Making 2: Further Considerations

© Licensed to ABE

� Financing

� Issue costs

� Subsidised loans

(d) Add the discounted side effects to the base case NPV to give the APV.

(e) If the APV is positive accept the project.

This process can be best illustrated by an example. Although it is useful for you to understand these workings you will not be expected to undertake similar calculations concerning APV in your examination.

Example

Amigo is considering investing in a project which will cost £75,000. The cost of the project will attract writing down allowances at 25% on a reducing balance basis. The project will have a net cash flow of £37,500 per annum for three years, after which time it will be scrapped for a minimal sum. To finance this project £15,000 will be taken from retained earnings, and the remainder will be provided by Amigo’s bank for a cost of £1,000. The rate of interest on the loan is to be 12%.

The industry average beta for projects of this type is 2.5, and its average gearing ratio is 3:4 (debt: equity). The risk-free rate is 10%, and the market return is 15%. Assume corporation tax is 33%, paid a year in arrears. Assess the project’s viability using an APV technique.

Note that it would be very difficult to assess this project with any reliability otherwise than by APV.

(a) Find the ungeared cost of equity for the firm using the following formula:

βg = βug [1 + eg

d

V)t1(V − ]

Using the industry average beta, rather than an individual company’s beta, is a good technique as this removes many of the problems with betas discussed earlier in the course.

Unless told otherwise assume the debt is risk-free.

βg = βug [1 + Vd(1 − t)/Veg]

2.5 = βug [1 + 3 (1 − 0.33)/4]

2.5 = βug [1.5025]

βug = 2.5/1.5025

βug = 1.66

Using CAPM 10% + (15% − 10%) 1.66 = 18% = base case discount rate.

(b) Estimate the base case NPV – this is done by discounting the project’s cash flows at the ungeared cost of equity.

Calculating the writing down allowances:

Timing Book Value WDA Tax Rate Tax Relief Yr 2 75,000 × 0.25 = 18,750 × 0.33 6,188 Yr 3 56,250 × 0.25 = 14,063 × 0.33 4,641 Yr 4 42,187 (balancing allowance) × 0.33 13,922

Page 303: libvolume5.xyzlibvolume5.xyz/law/ballb/3rdyear/semester5/corporatelaw/corporate... · ABE Advanced Diploma in Business Administration Study Manual CORPORATE FINANCE Contents Study

Capital Investment Decision Making 2: Further Considerations 295

© Licensed to ABE

Base case present value:

Year 0 1 2 3 4 Outlay (75,000) Cash flows 37,500 37,500 37,500 Tax (12,375) (12,375) (12,375) WDA 6,188 4,641 13,992

After tax cash flows (75,000) 37,500 31,313 29,766 1,617 Discount factor (18%) 1 0.8475 0.7182 0.6086 0.5158

PV cash flow (75,000) 31,781 22,489 18,116 834 Base case present value = (1,780) (note that this is negative).

(c) Evaluate and discount the side effects of the project and its financing methods.

(i) Present value of debt arrangement fee = (£1,000) – assume paid in Year 0

Tax relief = £1,000 × 0.33 = £330

Claimed Yr 2 = £330 × 0.7972 (applying 12% discount rate) = £263

(ii) Present value of tax relief, discounted at the pre-tax cost of debt which reflects the low risk of the tax relief (note that the after-tax rate is not used because this would double-count the tax relief.):

Interest (allowable against tax unless stated otherwise) × Annual tax relief on interest:

£60,000 × 0.12 = £7,200 × 0.33 (tax relief) = £2,376

Year 2: £2,376 × 0.7972 = £1,894Year 3: £2,376 × 0.7118 = £1,691Year 4: £2,376 × 0.6355 = £1,510

PV of tax relief = £5,095 (d) Add the discounted side effects to the base case NPV to give the APV.

Adjusted present value:

£ Base-case present value (1,780) Present value of tax shield 5,095 Arrangement fee (1,000) PV of tax relief on arrangement fee 263

Adjusted present value 2,578 (e) As the APV is positive accept the project.

Page 304: libvolume5.xyzlibvolume5.xyz/law/ballb/3rdyear/semester5/corporatelaw/corporate... · ABE Advanced Diploma in Business Administration Study Manual CORPORATE FINANCE Contents Study

296 Capital Investment Decision Making 2: Further Considerations

© Licensed to ABE

The advantages of the APV method are that:

� It is easier to calculate than attempting to adjust the discount rate for all the side effects, and as such it is versatile and flexible.

� It allows management to evaluate all the effects of the method of financing a project, and thus to see where the benefits of the investment are coming from. This may be especially useful if the benefits are arising from, say, a government incentive which may be withdrawn quickly thus making the project unviable.

The disadvantages of the APV method are that:

� It is dependent on the theories of MM (1963) (see module a) for its underlying assumption and thus the problems with MM are also inherent with APV.

� It can be difficult to identify all the costs associated with the method of financing and choosing a correct interest rate to discount them with.

� It can be long and complicated.

G. USE OF THE CAPITAL ASSET PRICING MODEL

The principal input that the capital asset pricing approach can make into capital budgeting is in determining the discount rate. However, there are two important limitations:

� The discount rate is determined as at a point in time, yet most projects run for several years; and

� There are so many market imperfections that the rate may be influenced more by “real world” distortions than determined by the model itself.

Furthermore, it is possible to have conflicting outcomes in appraising the same project depending on the source of the discount rate. If the rate is based on the weighted average cost of capital (WACC) method, a project may be rejected because it gives insufficient return, say, whereas under CAPM it would be acceptable because such low return is offset by low systematic risk.

Rigid use of a cost of capital method will tend towards acceptance of higher risk and avoiding sufficiently profitable but lower risk projects. Such conflicts are rare, but can occur in practice.

If CAPM is accepted, it might be concluded that, when deciding whether to invest in a particular project, management should be concerned with its systematic risk and not with its overall risk. If the beta factor can be estimated, then it is possible, using the formula given earlier, to calculate the minimum required return on the project, based on the systematic risk of the project. The model can thus be used to compare projects of different risk classes, unlike the NPV method which does not consider risk in its choice of discount rate.

In using the model, management are determining a required rate of return based on market and risk free rates of returns, the returns on the project and its variation to the market; in so doing they are assuming that shareholders wish them to evaluate such projects as though they were stocks and shares in the market, and that shareholders are fully diversified themselves and have no desire for the company to diversify on their behalf.

Page 305: libvolume5.xyzlibvolume5.xyz/law/ballb/3rdyear/semester5/corporatelaw/corporate... · ABE Advanced Diploma in Business Administration Study Manual CORPORATE FINANCE Contents Study

Capital Investment Decision Making 2: Further Considerations 297

© Licensed to ABE

H. WORKED EXAMPLES

The following examples are typical of the types of problem you may expect to face.

1. A company has estimated the expected cash flows for four possible projects as follows:

Year Project 0 1 2 3 4 5

1 (500) 200 200 300 200 200 2 (400) 100 100 100 100 500 3 (150) 40 50 60 70 100 4 (1,000) 500 400 300 200 100

Note that all figures in £000s.

Required:

(a) Rank these projects in order of acceptability using:

(i) Payback;

(ii) Net present value at 20% cost of capital;

(iii) Internal rate of return.

(b) Explain which project should be accepted if the projects were mutually exclusive and there was no capital rationing.

(c) Explain which project(s) should be accepted if the projects were independent and indivisible, and the company had £1 million to invest.

(d) Explain which project(s) should be accepted if the projects were independent and divisible, and the company had up to £1.1 million to invest.

(e) Explain briefly the relative advantages and disadvantages of the appraisal methods used in (a) above.

The following discount factors are provided for use as necessary:

Year Net Present Value Value of an Annuity 20% 30% 20% 30%

1 0.8333 0.7692 0.833 0.769 2 0.6944 0.5917 1.528 1.361 3 0.5787 0.4552 2.106 1.816 4 0.4823 0.3501 2.589 2.166 5 0.4019 0.2693 2.991 2.436

Page 306: libvolume5.xyzlibvolume5.xyz/law/ballb/3rdyear/semester5/corporatelaw/corporate... · ABE Advanced Diploma in Business Administration Study Manual CORPORATE FINANCE Contents Study

298 Capital Investment Decision Making 2: Further Considerations

© Licensed to ABE

Answer

1. (a) (i) Payback method

Project 1 The £500,000 initial outlay is returned as follows: (200,000 + 200,000 + 1

3 [300,000]) = 2.3 years

Project 2 The £400,000 initial outlay is returned as follows: (100,000 + 100,000 + 100,000 + 100,000) = 4 years

Project 3 The £150,000 initial outlay is returned (40,000 + 50,000 + 60,000) = 3 years

Project 4 The £1,000,000 initial outlay is returned as follows: (500,000 + 400,000 + 1

3 [300,000]) = 2.3 years

So the ranking is: 1st equal – Project 1 and Project 4 3rd – Project 3 4th – Project 2

(ii) With NPV at 20% Project 1 Year Cash Flow Factor NPV

£000 £000 0 (500) 1.0000 (500.00) 1 200 0.8333 166.66 2 200 0.6944 138.88 3 300 0.5787 173.61 4 200 0.4823 96.46 5 200 0.4019 80.38

155.99

Project 2 Year Cash Flow Factor NPV £000 £000 0 (400) 1.0000 (400.00) 1 100 0.8333 83.33 2 100 0.6944 69.44 3 100 0.5787 57.87 4 100 0.4823 48.23 5 500 0.4019 200.95

59.82

Page 307: libvolume5.xyzlibvolume5.xyz/law/ballb/3rdyear/semester5/corporatelaw/corporate... · ABE Advanced Diploma in Business Administration Study Manual CORPORATE FINANCE Contents Study

Capital Investment Decision Making 2: Further Considerations 299

© Licensed to ABE

Project 3 Year Cash Flow Factor NPV £000 £000 0 (150) 1.0000 (150.00) 1 40 0.8333 33.33 2 50 0.6944 34.72 3 60 0.5787 34.72 4 70 0.4823 33.76 5 100 0.4019 40.19

26.72

Project 4 Year Cash Flow Factor NPV £000 £000 0 (1,000) 1.0000 (1,000.00) 1 500 0.8333 416.7 2 400 0.6944 277.8 3 300 0.5787 173.6 4 200 0.4823 96.5 5 100 0.4019 40.2

4.8 The ranking from Project 1 to 4 runs also from 1 to 4 in order.

(iii) IRR method

NPV at 30%, when combined with the NPV at 20%, can be used to determine the IRR by use of a formula: Project 1 Year Cash Flow Factor NPV

£000 £000 0 (500) 1.0000 (500.00) 1 200 0.7692 153.80 2 200 0.5917 118.30 3 300 0.4552 136.60 4 200 0.3501 70.00 5 200 0.2693 53.90

32.60

Page 308: libvolume5.xyzlibvolume5.xyz/law/ballb/3rdyear/semester5/corporatelaw/corporate... · ABE Advanced Diploma in Business Administration Study Manual CORPORATE FINANCE Contents Study

300 Capital Investment Decision Making 2: Further Considerations

© Licensed to ABE

Project 2 Year Cash Flow Factor NPV £000 £000 0 (400) 1.0000 (400.00) 1 100 0.7692 76.90 2 100 0.5917 59.20 3 100 0.4552 45.50 4 100 0.3501 35.00 5 500 0.2693 134.70

(48.70)

Project 3 Year Cash Flow Factor NPV £000 £000 0 (150) 1.0000 (150.00) 1 40 0.7692 30.80 2 50 0.5917 29.60 3 60 0.4552 27.30 4 70 0.3501 24.50 5 100 0.2693 26.90

(10.90)

Project 4 Year Cash Flow Factor NPV £000 £000 0 (1,000) 1.0000 (1,000.00) 1 500 0.7692 384.60 2 400 0.5917 236.70 3 300 0.4552 136.60 4 200 0.3501 70.00 5 100 0.2693 26.90

(145.20) To determine the internal rate of return for each project we can apply the formula:

X +

−× )XY(

b + aa

where: X = the lower rate of interest used

Y = the higher rate of interest used

a = the difference between the present values of the outflow and the inflows at X%

b = the difference between the present values of the outflow and the inflows at Y%.

Page 309: libvolume5.xyzlibvolume5.xyz/law/ballb/3rdyear/semester5/corporatelaw/corporate... · ABE Advanced Diploma in Business Administration Study Manual CORPORATE FINANCE Contents Study

Capital Investment Decision Making 2: Further Considerations 301

© Licensed to ABE

If one NPV is positive (i.e. a) and the other negative (i.e. b) the formula is:

X +

−×

−X)(Y

baa

So to take each project in turn:

Project 1:

IRR = 20 +

−× 20) (30 188.59155.99 = 28.3%

Project 2:

IRR = 20 +

−× 20) (30 108.52

59.82 = 25.5%

Project 3:

IRR = 20 +

−× 20) (30 37.62

26.72 = 27.1%

Project 4:

IRR = 20 +

−× 20) (30 150.0

4.8 = 20.3%

This then ranks the projects in the order:

1st – Project 1

2nd – Project 3

3rd – Project 2

4th – Project 4

(b) With mutually exclusive projects and no capital rationing, the company should select the one with the highest NPV, this being Project 1.

(c) In the situation of independent, indivisible projects where the company had £1 million to invest, the best approach would be to maximise the NPV obtainable. In this case, Projects 1 and 2 should be accepted.

(d) Looking at the situation of divisible, independent projects we would need to consider the highest profitability index, being:

Project 1 2 3 4

investment InitialInflows P/V −

50099.655

40082.459

15072.176

10008.1004

= 1.31 1.15 1.18 1.00 We would allocate our money to the most profitable first which would provide the following portfolio:

Project 1 – investment of £500,000Project 3 – investment of £150,000Project 2 – investment of £400,000

£1,050,000

Page 310: libvolume5.xyzlibvolume5.xyz/law/ballb/3rdyear/semester5/corporatelaw/corporate... · ABE Advanced Diploma in Business Administration Study Manual CORPORATE FINANCE Contents Study

302 Capital Investment Decision Making 2: Further Considerations

© Licensed to ABE

(e) Payback is useful in that it has the advantage of showing when the initial investment has been repaid. However, it also has the disadvantages of:

(i) Ignoring income after the payback period; and

(ii) Ignoring interest.

Net present value provides the advantage of considering both cash flows and interest over the project life but the disadvantage of ignoring risk.

Internal rate of return provides the advantage of an appraisal as a single percentage figure and thereby indicates a project yielding the highest return on investment. However, by the use of such a rate the cash flow effects can be hidden and the risk aspects ignored.

Page 311: libvolume5.xyzlibvolume5.xyz/law/ballb/3rdyear/semester5/corporatelaw/corporate... · ABE Advanced Diploma in Business Administration Study Manual CORPORATE FINANCE Contents Study

Capital Investment Decision Making 2: Further Considerations 303

© Licensed to ABE

2. Hurdlevack Ltd relies on the payback method of project evaluation, requiring that investments repay capital within three years. The board are currently considering the four following projects.

Project A B C D £ £ £ £

Sales 40,000 75,000 60,000 60,000 Direct costs 16,000 27,000 15,000 18,000 Depreciation 8,000 40,000 30,000 35,000 Interest 12,000 16,000 9,000 7,000

Initial investment 120,000 160,000 90,000 70,000

Project life 15 years 4 years 3 years 2 years

The engineering department has asked the board to evaluate these opportunities by means of a discounted cash flow technique. The finance department has been unwilling to use a discounted cash flow technique, because of difficulty in establishing an appropriate discount rate. It therefore proposes to calculate each project’s internal rate of return, and let the board determine appropriate hurdle rates.

Required:

(a) Calculate each project’s payback and state which of the opportunities is acceptable by this criterion.

(b) Calculate each project’s internal rate of return and using a hurdle rate (the minimum rate of return acceptable to the company) of 15%, state which of the opportunities is acceptable by this criterion.

(c) Suggest why the above two project appraisal methods do not give answers which are consistent with each other for the accept/reject decision.

(d) Briefly outline some of the elements which should be considered when determining the appropriate hurdle rate for an individual project.

Page 312: libvolume5.xyzlibvolume5.xyz/law/ballb/3rdyear/semester5/corporatelaw/corporate... · ABE Advanced Diploma in Business Administration Study Manual CORPORATE FINANCE Contents Study

304 Capital Investment Decision Making 2: Further Considerations

© Licensed to ABE

Answer

2. (a) Direct costs, interest charges and flows from sales are used in the calculation of payback periods, as follows.

Project A B C D £ £ £ £

Sales 40,000 75,000 60,000 60,000 Direct costs (16,000) (27,000) (15,000) (18,000) Interest (12,000) (16,000) (9,000) (7,000)

Net annual cash flow 12,000 32,000 36,000 35,000 As the payback is the time in which the cash flows repay the initial capital outlay, it can be derived as follows.

Project A B C D £ £ £ £

Capital outlay 120,000 160,000 90,000 70,000 Net annual cash flow 12,000 32,000 36,000 35,000

Payback period 10 years 5 years 2½ years 2 years Thus, C and D are acceptable.

(b) When looking at the internal rate of return (IRR) of the projects, we look at cash flows from sales and direct costs.

Project A B C D £ £ £ £

Sales 40,000 75,000 60,000 60,000 Direct costs (16,000) (27,000) (15,000) (18,000)

Net annual cash flow 24,000 48,000 45,000 42,000

Project A B C D £ £ £ £

Capital outlay 120,000 160,000 90,000 70,000 Net annual cash flow 24,000 48,000 45,000 42,000

Payback 5 years 3.33 years 2 years 1.67 years

From the annuity tables, the IRR is (approximately): 18% 8% 23% 13%

Thus A and C are acceptable.

Page 313: libvolume5.xyzlibvolume5.xyz/law/ballb/3rdyear/semester5/corporatelaw/corporate... · ABE Advanced Diploma in Business Administration Study Manual CORPORATE FINANCE Contents Study

Capital Investment Decision Making 2: Further Considerations 305

© Licensed to ABE

(c) The cash flows of a project which arise after the payback period are not accounted for under the payback appraisal method. Thus, where a project takes a relatively long time to “come on stream”, it will be rejected on payback terms, even if it has an acceptable IRR. As can be seen from the above, Project A is acceptable only under IRR. Payback emphasises liquidity but, if cash flows cease quickly, as in Project D, the payback criteria may be met but an adequate return on capital is not seen.

Project B is rejected by both appraisal methods because of the small cash return. Project C is the opposite, having high cash returns, and it is acceptable under both appraisal methods.

(d) The inherent risks associated with cash flows deriving from each project should be assessed in determining hurdle rates. As it is likely that investments will already be under way, these should also be considered, as they might provide benefits associated with a new project – i.e. diversification of risks.

In a perfect capital market, flows from a projected project may not be positively correlated with the earnings stream of the firm, and diversification may still not be wise. Where there are possibilities for private shareholder diversification, diversification in private and corporate terms may equate, and there would not be any benefit in corporate diversification. If this is so, then only the risk associated with the earnings stream from the project would not be diversified by portfolio investment.

Where the capital market is imperfect, it may be quite acceptable to diversify. A number of flows from various projects may alter the risk incurred by a firm very little, if at all. In such cases, the firm’s cost of capital may be used as a required rate of return on investments. However, if the cash flows of a project will significantly affect the risk of a company on average, where these are taken with the existing earnings stream, it is necessary to calculate the return on each available project combination linked with the existing earnings stream. By doing this, the “right” mix of investments can be assessed, to compensate for any change in earnings with any change in associated risk.

Page 314: libvolume5.xyzlibvolume5.xyz/law/ballb/3rdyear/semester5/corporatelaw/corporate... · ABE Advanced Diploma in Business Administration Study Manual CORPORATE FINANCE Contents Study

306 Capital Investment Decision Making 2: Further Considerations

© Licensed to ABE

Practice Questions

1. A new polishing machine is required. The polishing machine forms a part of the production process, and most products manufactured require polishing at various stages in their production. A polishing facility is expected to be required for as long as the factory remains in operation, and no closure can be anticipated.

Details of the two machines under consideration are set out below.

Machine A B Initial cost £50,000 £90,000 Life – years 4 7 Salvage value at end of: Year 4 – machine A £5,000 Year 7 – machine B £7,000 Annual running costs £10,000 £8,000

Both machines fulfil the same function and have equal capacities. The approximate discount rate is 10%.

Required:

Determine which machine should be purchased. Specify any assumptions made.

To what amount would the initial cost of machine A be required to alter in order for the two machines to be of equal financial attractiveness?

2. ABC Ltd, which is investigating the possible acquisition of XYZ Ltd, for diversification purposes, has asked you to advise the firm on the basis of the following information:

XYZ Ltd Summary Balance Sheet as at 30th September 2002

£000 £000 Ordinary shares 1,500 Land & buildings 900 Reserves 900 Plant (net of depreciation) 600 10% debentures 750 Investments 450 Creditors 300 Stock 600 Debtors 600 Cash 300

£3,450 £3,450 Profits: 2000 – £350,000

2001 – £300,000

2002 – £450,000

Page 315: libvolume5.xyzlibvolume5.xyz/law/ballb/3rdyear/semester5/corporatelaw/corporate... · ABE Advanced Diploma in Business Administration Study Manual CORPORATE FINANCE Contents Study

Capital Investment Decision Making 2: Further Considerations 307

© Licensed to ABE

You are also told the following:

� It is estimated that the investments have a market value of £675,000, and that the stock could be sold for £750,000. The other assets have a value as stated in the balance sheet.

� All of the investments and plant valued at £225,000 would not be needed by ABC Ltd.

� The investments have produced annual income of £45,000 per annum for the last five years, and are expected to continue to do so.

� ABC Ltd would repay the debentures at par, immediately after acquisition.

� ABC Ltd requires a return on capital of 10%.

Required

You are required to calculate the maximum price which ABC Ltd should be prepared to pay for XYZ Ltd on each of the following bases:

(a) Break-up value.

(b) Profitability.

(c) Discounted cash flow, assuming that the cash flows to be discounted are as follows:

£000 2003 450 2004 600 2005 450 2006 onwards 570

Present value factors at 10%

Year 1 0.90909 Year 2 0.82645 Year 3 0.75131 Year 4 0.68301

3. Stamford Ltd specialises in the production of plastic sports equipment. The company has recently developed a new machine for automatically producing plastic cricket bats. The machine cost £150,000 to develop and install, and production is to commence at the beginning of next week. It is planned to depreciate the £150,000 cost evenly over four years, after which time production of plastic cricket bats will cease. Production and sales will amount to 30,000 bats each year. Annual revenues and operating costs, at current prices, are estimated as follows.

Sales (£9.60 each) £288,000

Variable manufacturing costs £200,000

This morning, a salesman has called and described to the directors of Stamford Ltd a new machine, ideally suited to the production of plastic cricket bats. This item of equipment is distinctly superior to Stamford’s own machine, reducing variable costs by 30% and producing an identical product. The cost of the machine, which is also capable of producing 30,000 cricket bats per annum, is £190,000.

Page 316: libvolume5.xyzlibvolume5.xyz/law/ballb/3rdyear/semester5/corporatelaw/corporate... · ABE Advanced Diploma in Business Administration Study Manual CORPORATE FINANCE Contents Study

308 Capital Investment Decision Making 2: Further Considerations

© Licensed to ABE

Assume the following:

� Annual revenues and operating costs arise at the year-end.

� The general rate of inflation is 10% per annum.

� The company’s money cost of capital is 21%.

� The existing machine could be sold immediately for £12,000.

� If purchased, the new machine could be installed immediately.

� Either machine would possess a zero residual value at the end of four years.

Required:

(a) Calculation of the net present value of the two options open to the management, using the real cost of capital.

(b) Advice to the management as to which course should be followed, and an explanation of the significance of your calculations under (a).

Note: Ignore taxation.

Table of Factors for n ==== 4 Years

Interest Rate (per cent)

Present Value of £1

Present Value of £1 Received per Year

r (1 + r)−n r

r) + 1( 1 n−−

10 0.68 3.17 11 0.66 3.10 12 0.64 3.04 13 0.61 2.97 14 0.59 2.91 15 0.57 2.85 16 0.55 2.80 17 0.53 2.74 18 0.52 2.69 19 0.50 2.64 20 0.48 2.59 21 0.47 2.54 22 0.45 2.49

Page 317: libvolume5.xyzlibvolume5.xyz/law/ballb/3rdyear/semester5/corporatelaw/corporate... · ABE Advanced Diploma in Business Administration Study Manual CORPORATE FINANCE Contents Study

Capital Investment Decision Making 2: Further Considerations 309

© Licensed to ABE

4. An opportunity has arisen for your company to acquire the specialised product stocks of a bankrupt business for £50,000. The net proceeds from the sale of these stocks will be influenced by a number of factors originating from outside the company but the range of possibilities appears to be as follows.

Possible Amounts of Net Sales Proceeds

Probability

£ % Year 1 24,000 60

20,000 30 36,000 10

Year 2 60,000 50 48,000 30 20,000 20

The estimates for Year 2 are independent of those for Year 1. The company’s required rate of return is 20%.

Required:

(a) To calculate the expected net sales proceeds each year; and to state whether on this basis the project would yield the required rate of return.

(b) To tabulate the possible combinations of sales value over the two years, with their related probabilities, and from this data to calculate the overall percentage probability of the rate of return being less than the required 20%.

(c) Without making any further calculations, to explain how you would arrive at the standard deviation of net present value and the coefficient of variation for this project, and the use that might be made of those statistics.

Now check your answers with the ones given at the end of the unit.

Page 318: libvolume5.xyzlibvolume5.xyz/law/ballb/3rdyear/semester5/corporatelaw/corporate... · ABE Advanced Diploma in Business Administration Study Manual CORPORATE FINANCE Contents Study

310 Capital Investment Decision Making 2: Further Considerations

© Licensed to ABE

ANSWERS TO PRACTICE QUESTIONS

1. The technique known as the equivalent annual cost technique may be used here, as the service provided by the machines in question is to be required for the foreseeable future. The present value of operating each machine for its economic life is calculated, and then expressed as an annual equivalent. This latter figure is achieved by the following calculation:

period same for thefactor Annuity

life economicfor machine operating of luePresent va

Such a figure enables an easier comparison to be made of assets with differing economic lives, provided that at the end of its life an asset is replaced by a similar one. An assumption of unchanging technology and constant relative prices is necessary.

On this basis, the present value of the costs is as follows.

A B £ £ Initial cost 50,000 90,000 Running costs: £10,000 × 3.170 (4 years at 10%) 31,700 £8,000 × 4.868 (7 years at 10%) 38,944 less Salvage value: £5,000 × 0.683 (3,415) £7,000 × 0.513 (3,591)

Present value of cost (a) 78,285 125,353

Annuity factor (b) 3.170 4.868

Equivalent annual cost (a ÷ b) £24,696 £25,750

This shows that machine A should be chosen, as it is the less expensive. At the end of four years, then eight years, etc., new machines A would be purchased as replacements.

To be equally financially attractive, it would be necessary for the initial cost of machine A to rise until the equivalent annual cost of A was also £25,750. This would mean a present value total of:

£ £25,750 × 3.170 = 81,628 less Present value now 78,285

£3,343 increase So, it would be necessary for the initial cost of machine A to rise to £53,343 for the machines to be equally attractive.

Page 319: libvolume5.xyzlibvolume5.xyz/law/ballb/3rdyear/semester5/corporatelaw/corporate... · ABE Advanced Diploma in Business Administration Study Manual CORPORATE FINANCE Contents Study

Capital Investment Decision Making 2: Further Considerations 311

© Licensed to ABE

2. Maximum prices ABC Ltd would be prepared to pay for XYZ Ltd on the following bases.

(a) Break-up Value

This can be computed in two ways, bearing in mind that, even if the investment and plant worth £225,000 are not needed by ABC Ltd, they do represent part of the value of XYZ Ltd and any as yet unrealised profits in them constitute part of the value. As the company is being acquired as a whole, any sale of assets surplus to requirements and made subsequently would merely alter the values of industrial assets, and it would not affect the overall break-up value.

The value is calculated as follows:

Either Or Value at 30/11/2002 £000 £000 Land and buildings 900 900 Plant 600 375 Investments 675 – Stock 750 750 Debtors 600 600 Cash 300 300 Sum due on sale of investments/assets or cash proceeds realised

– 900

3,825 3,825 less: Debentures 750 750 Creditors 300 300

Value of business to ordinary shareholder £2,775 £2,775 So, therefore, the break-up value is £2,775,000, which purely puts a value to the assets as they stand and, to some extent, includes growth in the value of the assets over the historical cost. Potential future profitability is not reflected in this figure. To allow for this, a value of goodwill would need to be calculated in terms of “n” years’ purchase of average weekly profits less an allowance for the projected interest on capital. The number represented by “n” would be that typical to the particular industry or trade.

(b) Profitability

To obtain a maximum price on this basis, different assessments can be made.

Initially, it should be seen whether “profits” in the terms of the question relates to profits after payment of debenture interest and including the interest received from investments. An assumption is made that this is so.

Past profits are usually adjusted, in exercises such as this, in the light of changing circumstances which will prevail when the purchasing company takes over – with no investments and repaid debentures.

Two alternative methods can be used, with both unweighted and weighted profits. The number of years’ average profits to be taken into the market value needs to be assessed.

Page 320: libvolume5.xyzlibvolume5.xyz/law/ballb/3rdyear/semester5/corporatelaw/corporate... · ABE Advanced Diploma in Business Administration Study Manual CORPORATE FINANCE Contents Study

312 Capital Investment Decision Making 2: Further Considerations

© Licensed to ABE

As the company specifies a cost of capital, or capitalisation rate, of 10% a P/E ratio of 10 would be appropriate:

(i.e. Market value = rateation Crystallis

Earnings or Market value = P/E ratio × Earnings).

The valuations at 30th November 2002 are as follows:

Method 1 (unweighted)

Annual Profits (given)

Investment Interest

Debenture Interest

Adjusted Profit

£000 £000 £000 £000 2000 350 (45) 75 380 2001 300 (45) 75 330 2002 450 (45) 75 480

£1,100 £(135) £225 £1,190

Average annual unweighted profit is 3000,190,1£ = £396,666

Method 2 (weighted)

Adjusted Profit ×××× Weight Weighted Adjusted Profit

£000 £000 2000 380 1 380 2001 330 2 660 2002 480 3 1,440

£1,190 £2,480

Average annual weighted profit is 6000,480,2£ = £413,333

So, on a P/E basis of 10, the price would be:

Method 1 – £3,966,666

Method 2 – £4,133,333 (maximum)

The higher values seen here reflect the vendor’s ability to make a profit.

(c) Discounted Cash Flow

The cash flows represent the projected flows but excluding investment and debenture interest. They compare, therefore, with the adjusted profits assessments of value.

DCF looks at the value of the company as a discounted stream of future cash flows available to the purchaser.

This gives the net present value, as follows:

Page 321: libvolume5.xyzlibvolume5.xyz/law/ballb/3rdyear/semester5/corporatelaw/corporate... · ABE Advanced Diploma in Business Administration Study Manual CORPORATE FINANCE Contents Study

Capital Investment Decision Making 2: Further Considerations 313

© Licensed to ABE

Year Cash Flow DCF Present Value £000 (10%) £000 2003 450 0.90909 409.1 2004 600 0.82645 495.9 2005 450 0.75131 338.1

1,243.1 2006 onwards 570

Valued at: ratetion Capitalisa

Dividend × 0.75131

= 1.0000,570 × 0.75131 = 4,282,500

1,243,100 + 4,282,500 = £5,525,600

This treatment for £570,000 for 2006 onwards presents a value in perpetuity.

Therefore, to sum up:

Maximum value: £000

(a) Break-up value £2,775

(b) Profitability £4,133

(c) DCF £5,525

3. (a) Real cost of capital = i + 1i m −

= %10 = 1.111.0 = 10.0 + 1

10.0 21.0 −

Option 1, using the present machine, gives in real terms: £ Sales 288,000Variable costs of manufacture 200,000

Contribution 88,000 £88,000 for 4 years gives a PV £88,000 × 3.17 = £278,960

Option 2, using the new machine, gives: £ Sales 288,000Variable costs of manufacture 140,000

Contribution 148,000 £148,000 for 4 years gives a PV £148,000 × 3.17 = £469,160

Page 322: libvolume5.xyzlibvolume5.xyz/law/ballb/3rdyear/semester5/corporatelaw/corporate... · ABE Advanced Diploma in Business Administration Study Manual CORPORATE FINANCE Contents Study

314 Capital Investment Decision Making 2: Further Considerations

© Licensed to ABE

But costs arise on the changeover, as follows:

Cost of machine of greater efficiency £190,000 Less Proceeds of sale of old machine 12,000 £178,000

Present value following allowance for net purchase cost £291,160 As regards the development and installation costs of £150,000 relating to the new machine, this represents money spent. The sum does not come into the calculations otherwise.

Another way of looking at this is to make the calculations in money terms with an allowance for the 10% inflation.

Option 1

Year Sales Variable Costs Contribution PV £ £ £

21% Factor £

1 316,800 220,000 96,800 0.826 79,957 2 348,480 242,000 106,480 0.683 72,726 3 383,328 266,200 117,128 0.564 66,060 4 421,661 292,820 128,841 0.467 60,169

278,912 Option 2

Year Sales Variable Costs Contribution PV £ £ £

21% Factor £

1 316,800 154,000 162,800 0.826 134,473 2 348,480 169,400 179,080 0.683 122,312 3 383,328 186,340 196,988 0.564 111,101 4 421,661 204,974 216,687 0.467 101,193

469,079 less Costs of changeover 178,000

291,079 The factors are calculated as (1.21)n.

(b) The present machine makes a contribution of £88,000 pa for four years – £278,960 in all. The new and more efficient machine will increase the contribution level to £148,000 pa. However, there is the high initial cost of £190,000 less £12,000 sales receipts to be considered. This brings the PV of the more efficient machine down to £291,160. The marginal superiority in results of £12,200 needs to be carefully weighed against the risks of the project.

� Will the market for plastic cricket bats continue to be buoyant?

Page 323: libvolume5.xyzlibvolume5.xyz/law/ballb/3rdyear/semester5/corporatelaw/corporate... · ABE Advanced Diploma in Business Administration Study Manual CORPORATE FINANCE Contents Study

Capital Investment Decision Making 2: Further Considerations 315

© Licensed to ABE

� Might the variable production costs increase?

Either possibility could easily eliminate any advantage, so it is probably better not to invest in the new machinery.

The payback method is useful here, in the sense that, if we look at the return on the outlay of £178,000, the contribution pa increases by £60,000. This means that almost a full three years out of the project life of four years must elapse before the company is satisfied in payback terms. We must remember also that we have not considered interest, etc.

4. (a) Expected net sales proceeds:

Possible Cash Flow Probability Expected Value £ £

Year 1 24,000 0.6 14,400 20,000 0.3 6,000 36,000 0.1 3,600

24,000

Year 2 60,000 0.5 30,000 48,000 0.3 14,400 20,000 0.2 4,000

48,400

Cash Flow Discount Factor NPV £ (r ==== 20%) £

Y0 (50,000) 1.0 (50,000) Y1 24,000 0.83333 20,000 Y2 48,400 0.69444 33,611

Net present value = 3,611

The project yields a return in excess of 20% and should, therefore, be accepted.

(b) The table of possible combinations of sales values and their related probabilities are set out on the next page.

From this, the probability of the return being less than 20% is:

P(iii) + P(vi) + P(ix)

= 0.12 + 0.06 + 0.02

= 20%

Page 324: libvolume5.xyzlibvolume5.xyz/law/ballb/3rdyear/semester5/corporatelaw/corporate... · ABE Advanced Diploma in Business Administration Study Manual CORPORATE FINANCE Contents Study

316 Capital Investment Decision Making 2: Further Considerations

© Licensed to ABE

Page 325: libvolume5.xyzlibvolume5.xyz/law/ballb/3rdyear/semester5/corporatelaw/corporate... · ABE Advanced Diploma in Business Administration Study Manual CORPORATE FINANCE Contents Study

Capital Investment Decision Making 2: Further Considerations 317

© Licensed to ABE

(c) (i) Standard Deviation

Standard deviation comes from the following formula:

σ = 2)x p(x −Σ

where: σ = standard deviation

p = probability of a particular outcome

x = particular outcomes possible (i.e. NPVs)

x = expected net present value (i.e. 3,611)

Standard deviation measures the dispersion around the mean value (expected net present value). Its significance is seen in its size relative to the mean value. The greater the dispersion, the greater the risk of not achieving the expected outcome.

(ii) Coefficient of Variation

This can be calculated as follows:

Coefficient of variation = xσ

It is useful in the comparison of the dispersion of two or more distributions of projects of different sizes; the higher the coefficient, the more widely dispersed is the distribution.

Page 326: libvolume5.xyzlibvolume5.xyz/law/ballb/3rdyear/semester5/corporatelaw/corporate... · ABE Advanced Diploma in Business Administration Study Manual CORPORATE FINANCE Contents Study

318 Capital Investment Decision Making 2: Further Considerations

© Licensed to ABE

Page 327: libvolume5.xyzlibvolume5.xyz/law/ballb/3rdyear/semester5/corporatelaw/corporate... · ABE Advanced Diploma in Business Administration Study Manual CORPORATE FINANCE Contents Study

319

© Licensed to ABE

Study Unit 14

Managing Exposure to Risk – Financial Derivatives

Contents Page

Introduction 321

A. Principles of Hedging 321 Short and Long Positions 322 Closing out 322 Developing a Hedging Policy 324 Costs of Hedging 324

B. Interest Rates, Risk and Exposure 325 Interest Rate Structures 325 Impact of Interest Rate Changes 327 Managing Exposure to Interest Rate Risk 328

C. Internal Techniques of Managing Interest Rate Exposure 328 Matching 328 Smoothing 329

D. Futures Contracts 329 Characteristics 329 Hedging Interest Rate Exposure with Futures Contracts 331

E. Forward Rate Agreements (FRAs) 332 Characteristics 332 Hedging Interest Rate Exposure with FRAs 333

F. Interest Rate Swaps 333 Characteristics 333 Operation of Interest Rate Swaps 334

(Continued over)

Page 328: libvolume5.xyzlibvolume5.xyz/law/ballb/3rdyear/semester5/corporatelaw/corporate... · ABE Advanced Diploma in Business Administration Study Manual CORPORATE FINANCE Contents Study

320 Managing Exposure to Risk – Financial Derivatives

© Licensed to ABE

G. Options 335 Calls and Puts 336 Option Prices 340 Hedging and Trading Strategies using Options 343 Interest Rate Options 347 Application of Option Theory to Capital Investment Decision making 348

Page 329: libvolume5.xyzlibvolume5.xyz/law/ballb/3rdyear/semester5/corporatelaw/corporate... · ABE Advanced Diploma in Business Administration Study Manual CORPORATE FINANCE Contents Study

Managing Exposure to Risk – Financial Derivatives 321

© Licensed to ABE

INTRODUCTION

We have seen before that all business enterprises face a degree of risk. One facet of this risk is “business risk” – that arising from the very nature of the business itself. We can divide this type of risk into two groups.

� That which is inherent in the conduct of business itself and cannot be reduced or eliminated without ceasing trading – effectively closing down the business or selling it. (The owner(s) accept this type of risk in making their investment and expect some form of financial return as compensation.)

� That which arises as a consequence of the financial transactions taking place in the normal course of business. It is this element of risk exposure which the owner can seek to reduce or eliminate.

Essentially, we are concerned here with the possibility of incurring loss on certain types of transaction – principally, through making investments or taking out loans, which gives rise to interest rate risk, and through international trade which gives rise to exchange rate risk.

It is not possible to eliminate such risk completely, but it is possible to reduce it by way of hedging. Hedging the risk involves taking action now to reduce the possibility of a future loss, usually at the cost of foregoing any possibility of a gain. A simple example should explain this.

A firm knows that it will need certain goods in six months’ time. It is exposed to the risk that the price of these goods may rise in the meantime. This risk may be reduced by entering into a “forward contract” to purchase the goods in six months’ time at a price fixed now. However, if the price falls below the current price in the meantime, the firm will have lost the opportunity to make a gain.

Thus, the basis of hedging involves offsetting two transactions against each other:

� a cash transaction – the receipt or payment of money arising from normal business transactions (such as international trade or the management of funds); and

� taking a position on (buying or selling) a derivative instrument linked to the type of cash transaction.

There are a considerable number of such financial derivatives and in this unit we shall examine the principles of their operation and strategies for their use in relation to interest rate risk. (We shall consider the similar range of derivatives and strategies in relation to exchange rate risk in the next unit.)

The instruments we shall examine here are forwards and futures contracts, swaps and options. Options are somewhat different in that they offer the possibility of making gains as well as hedging risk, and we shall consider them in some depth.

A. PRINCIPLES OF HEDGING

As we noted above, hedging a risk involves taking action now to reduce the possibility of a future loss, usually at the cost of foregoing any possibility of a gain. It is a process whereby the exposure to potential loss caused by adverse movements in prices, interest rates and exchange rates may be limited.

A hedge against exposure to risk is invariably constructed by using a financial derivative. Again, as we noted above, there are a range of these instruments. They “derive” their value from the price of underlying assets such as foreign currencies, commodities and fixed income securities, etc. We can

Page 330: libvolume5.xyzlibvolume5.xyz/law/ballb/3rdyear/semester5/corporatelaw/corporate... · ABE Advanced Diploma in Business Administration Study Manual CORPORATE FINANCE Contents Study

322 Managing Exposure to Risk – Financial Derivatives

© Licensed to ABE

demonstrate the basic concept with an example using one such instrument – a futures contract – in relation to exchange rate exposure.

A UK firm is going to receive a $100,000 in one month’s time as a result of some consultancy work carried out in the USA. It is exposed to the risk that an (unfavourable) movement in the £/$ exchange rate before the payment is made will reduce its value. However, it may hedge this risk by using a futures contract. The futures contract will comprise a transaction to sell $100,000 in one month’s time (the time of its receipt) at a £/$ exchange rate fixed today. The potential loss of value on the payment is, then, limited to the difference between the current exchange rate and the agreed rate for the futures contract. This reduces the company’s exposure to any other adverse movements in the exchange rate, but also means that it cannot take advantage of a favourable movement in the rate.

Futures are one of the derivatives that allow a financial risk to be reduced, but do not (usually) allow any gain to come from favourable movements in the prices or rates underlying the instrument. Other such derivatives include forward contracts and swaps. However, these can be distinguished from option contracts which also allow a risk to be reduced, but do allow gains to be made from favourable movements.

Short and Long Positions We have seen that the basis of hedging involves offsetting two transactions against each other:

� a cash transaction; and

� buying or selling a derivative instrument linked to the type of cash transaction.

We can examine this relationship further.

When a firm holds cash or securities, it will want to maintain their current value. It is, though, exposed to the risk that its value will decrease due to adverse movements in prices or interest or exchange rates. It will, therefore, sell a derivative of equivalent value at a price fixed today (known as taking a short position on the derivative). This fixes the value of the cash or securities – possibly at a value less than the current value – but limits the exposure to any further losses. This position is known as a short hedge. It offsets a long position in respect of the transaction the cash market with a short position in respect of the derivative.

Conversely, a firm expecting an inflow of cash or securities at some time in the future (for example, through payments due or the purchase of an asset) will take out a long hedge to protect the value of those future cash flows or the asset. This will involve buying a derivative of equivalent value at a price fixed today – again fixing the value of the future cash/asset and limiting any further loss due to adverse movements. The effect is to offset a short position in the cash transaction with a long position on the derivative.

In both cases, the hedge works by ensuring that if the value of the asset/cash/securities falls below the agreed price, any further loss is counterbalanced by a gain from the selling/buying of the financial derivative.

Closing out In the vast majority of cases, the hedge does not lead to delivery of the underlying asset in the derivative – that is, neither the sale or the purchase of the underlying asset takes place. Rather, the contract is “closed out” by taking an opposite position in the derivatives market. Thus, a contract to buy can, in effect, be cancelled by one to sell, and vice versa. This is known as “reversing the trade”.

What this means is that a contract to buy a derivative at a particular price is matched by another contract to sell the same derivative at a price which may or may not be the same as in the first

Page 331: libvolume5.xyzlibvolume5.xyz/law/ballb/3rdyear/semester5/corporatelaw/corporate... · ABE Advanced Diploma in Business Administration Study Manual CORPORATE FINANCE Contents Study

Managing Exposure to Risk – Financial Derivatives 323

© Licensed to ABE

contract. The two contracts can then be closed out. If the reverse trade is exactly equal and opposite to the original, the financial position is neutral. This is, though, unlikely – in which case, there will be a gain or loss incurred, with a financial settlement being made between the parties involved.

Note that the transaction in the derivative is undertaken to hedge risk in respect of the underlying cash transaction. There is, therefore, in most cases, no need to take delivery of the contract and actually buy or sell the underlying asset – particularly as this may involve considerable financial costs. Instead, any gain/loss incurred in the process of closing out the derivatives would be offset against any loss/gain in respect of the underlying position in the cash market.

Although forward and futures contracts are similar (see later), it is more common to close out a futures contract than a forward contract. This is because futures contracts are freely tradable and the holder of a contract to buy, say, a quantity of currency but who no longer wishes to hold that position can cancel it out by purchasing a contract to sell the same amount on the same date. Indeed, it is estimated that only about 1% of futures contracts are settled by actual delivery. You should contrast this with the forward market where the chances of reversing a position are costly and rare, and as a result approximately 90% of such contracts are settled by actual delivery.

An example will illustrate the process in detail.

DeanCo is a purchaser of natural gas and is due to take delivery of 5,000,000 litres in January. In order to hedge against the possibility of gas prices increasing over the period up to January, the company takes a long position in a March futures contract. At the time, the price of March futures contracts is $14.50 per 1,000 litres, with each contract being for delivery of 1,000 litres. In January, the contract is closed out with both the spot price of gas and the March futures being $14.75 per 1,000 litres.

How effective is this strategy in hedging the risk?

The transactions involved are:

� buying five March futures contracts (the long position in March futures) each at a price of $14.50 × 1,000 (1,000 litres per contract);

� selling five March futures contracts in January (to close out the original contracts) each at a price of $14.75 × 1,000; and

� buying the 5,000,000 litres of natural gas on the spot market at $14.75 per 1,000 litres.

The total gain on closing out the futures contracts is:

(14.75 − 14.50) × 1,000 × 5 = $1,250

The outlay for the 5,000,000 litres on the spot market is:

5,000 × 14.75 = $73,750

The net cost to DeanCo is:

73,750 - 1,250 = $72,500

This equates to a cost of $14.50 per 1,000 litres

The gain from closing out the futures contract offsets some or all of the likely increase in the price of natural gas on the spot market over the period. The extent of this offset depends upon the spot price of gas at the time that the futures contract was taken out, which in this case was not given. If the price of gas had fallen over the period, there would have been a loss on closing out, but this would again be offset by the lower spot price paid on delivery.

Page 332: libvolume5.xyzlibvolume5.xyz/law/ballb/3rdyear/semester5/corporatelaw/corporate... · ABE Advanced Diploma in Business Administration Study Manual CORPORATE FINANCE Contents Study

324 Managing Exposure to Risk – Financial Derivatives

© Licensed to ABE

Irrespective of the spot price at the time of delivery, DeanCo is locked into a known price from the beginning. The effect of the hedge is to guarantee that the company never has to pay more than the $14.50 per 1,000 litres, although to achieve this guarantee, it has to forgo the opportunity to pay a lower price.

Developing a Hedging Policy Hedging exposure to risk can be costly and carries certain risks in itself if the techniques and instruments are inappropriately used. It needs, therefore, to be applied within a clear strategic framework which reflects the organisation’s objectives.

Whilst each company has to establish its own methods in accordance with the particular circumstances in which it operates – for example, the nature of the markets traded in and its attitude towards risk – there are a common set of issues which need to be addressed by all companies.

� Establish and make explicit the exposure to risk – It is essential to know the extent of exposure to exchange and interest rate risk at all times. This can then form the basis of decisions as to how to manage that risk.

� Establish lines of responsibility and limits of authority – The way in which exposure is reported, to whom and who acts on it are key issues. The lines of responsibility for action, and for reporting, must be clearly stated, understood by all concerned and monitored. Similarly, the limits on the volume and amount of derivative transactions traded need to be made explicit, as do the types of instrument which it is permitted to use. These must be supported by effective systems and procedures to monitor and control activities – particularly to prevent unauthorised dealing, which was one of the factors which undermined Barings Bank.

� Determine how much, and the types of, exposure to hedge – It may not necessarily be appropriate to adopt a “hedge everything” approach – this may be prohibitively expensive in relation to the extent of the exposure. However, selective hedging requires ranking exposure and making decisions as to which risks to hedge, which is not easy. Some general decisions could be made – for example, focusing on foreign currency receipts where the company is involved in considerable international trade, or on interest rate movements if the company is highly geared and is, therefore, vulnerable to interest rate risk.

� Identify appropriate methods of hedging – This will involve matching different types of derivative instrument to different types of risk and establishing guidelines for how long to hedge exposure. Note that there are certain methods of hedging risk internally (as we shall consider later in the unit) and these are generally far less expensive than using external methods using derivatives. Only the net exposure, after offsetting liabilities and assets internally, should be the subject of external hedging.

� Establish and maintain good banking relationships – It is important to develop and maintain good relationships with one or two specialist banks or other financial institutions which have the expertise to arrange certain transactions. Even the biggest and best treasury departments cannot effect all their own transactions and need the assistance of intermediaries with the facility to set up such devices as currency or interest rate swaps.

Costs of Hedging On the face of it, the cost of hedging essentially comprises the premiums paid out on the derivatives transactions to the facilitating banks and/or exchanges.

However, there is always a risk associated with trading in derivatives, some of which are very complex and may be inappropriately applied, particularly if the controls noted above are not effective.

Page 333: libvolume5.xyzlibvolume5.xyz/law/ballb/3rdyear/semester5/corporatelaw/corporate... · ABE Advanced Diploma in Business Administration Study Manual CORPORATE FINANCE Contents Study

Managing Exposure to Risk – Financial Derivatives 325

© Licensed to ABE

Thus, any losses incurred on the derivatives transactions not offset by gains in the underlying cash transaction must be counted as costs.

In addition to these direct costs, there is a further significant cost in the overheads involved in the hedging operation. These are tied up with the overheads of treasury management as a whole, and include not only the operations in respect of external hedging activities, but also in respect of the application of internal hedging techniques.

B. INTEREST RATES, RISK AND EXPOSURE

Interest rates are an important element of the economic environment and influence the foreign exchange value of a country’s currency, as well as acting as a guide to the sort of return shareholders might want and expect. Changes in market interest yields affect share prices.

Interest Rate Structures Every financial instrument has its own interest rate or range of rates. The market segmentation theory acknowledges that the demand for the great variety of financial products can be segmented into a number of different types, that different products are designed for different purposes and the interest charges are a fundamental part of the overall “package”.

The most common quoted rates include:

� Base rate – the “bottom line” rate set by banks. Most lending to individuals, smaller and medium-sized enterprises is at a certain margin above base rate.

� LIBOR – the London Inter-Bank Offer Rate at which banks lend to each other. Large and multinational companies’ lending is often tied into these more favourable LIBOR rates.

� Treasury bill rates – the rate at which the Bank of England sells Treasury bills to the discount market and a good indicator of longer-term rates as these bills will not be redeemed for some years.

� Gilts yield – again, rates attached to government securities and a good indicator of longer-term rates.

Why are there so many rates?

The reasons include:

� Risk – a higher risk must be compensated for by higher returns (i.e. higher interest charges).

� Need for profit – the “spread” between savings and lending-based products provides the intermediaries’ profit.

� Duration of lending – interest rates may depend on the date to maturity of the product – for example, Treasury stock might be short, medium or long-dated. The yield on a type of security varies according to the term (length) of the borrowing period, and generally, long-term loans yield a higher return than short-term ones.

� Size of loan – economies of scale and negotiating strength all have a part to play here.

� International rates – domestic rates will be influenced by speculators’ behaviour and the movements of funds internationally.

Page 334: libvolume5.xyzlibvolume5.xyz/law/ballb/3rdyear/semester5/corporatelaw/corporate... · ABE Advanced Diploma in Business Administration Study Manual CORPORATE FINANCE Contents Study

326 Managing Exposure to Risk – Financial Derivatives

© Licensed to ABE

(a) Risk and rates

Borrowers have different risks of default and this is reflected in the differences in the interest rates they are expected to pay – for example, the government will pay a lower rate of interest on its borrowing than a small, new company. The additional return required will be equal to the fall in expected value of the investment taking into consideration the default risk. We can see this by considering an example.

Jim plc issues 10% loan stock which is redeemable, at par, after 12 months. There is, however, a 20% chance that Jim will default on payment and there will be no return for the loan stockholders. If the interest rate on 12 month government stocks is 5%, calculate the yield on Jim’s loan stock. Ignore taxation and market risk.

The expected value of the loan stock after 12 months:

(0.8 × £110) + (0.2 × 0) = £88

Discounting at the rate on government stocks to reflect the additional return for the additional risks to find the current market price:

05.188£ = £83.81

The yield is then:

81.83

110 − 1 = 31.25%

The higher expected yield is required to compensate for the risk of default.

(b) Nominal (or money) rates and real rates of interest

Nominal rates of interest are those expressed in money terms, whereas real rates are those adjusted for the rate of inflation. The real rate is therefore a measure of the increase in the real wealth of the investor or lender.

The real rate is calculated as:

inflation of Rate 1

interest of rate Nominal 1+

+ − 1

If the nominal rate was 10% and inflation 5%, the real rate of interest would be:

05.110.1 − 1 = 4.76%

Real rates of interest are normally positive (i.e. the nominal rate exceeds the rate of inflation). Investors can therefore be assured that their real wealth is increasing.

In times of high inflation, however, a negative real rate is possible, although nominal rates will rise, too, as investors want to see a real return on their investments.

(c) Interest rates and financial management

When interest rates are low it might be advisable to:

� Borrow more, increasing the company’s gearing at lower fixed rates

� Borrow for longer terms

� Pay back loans with higher rates or roll them over for loans of lower rates

Page 335: libvolume5.xyzlibvolume5.xyz/law/ballb/3rdyear/semester5/corporatelaw/corporate... · ABE Advanced Diploma in Business Administration Study Manual CORPORATE FINANCE Contents Study

Managing Exposure to Risk – Financial Derivatives 327

© Licensed to ABE

Alternatively, when interest rates are high it might be advisable to:

� Substitute equity for debt finance and reduce the company’s gearing by investing surplus cash in short-term interest-bearing securities

� Borrow short term rather than at even higher longer-term rates

Impact of Interest Rate Changes It is clear that changing interest rates will lead to variations in a firm’s cashflows as receipts and payments fluctuate with movements in those rates. This can cause problems in the management of working capital. Rising interest rates can cause the value of a firm with a high level of gearing to fall, and the increased debt repayments may mean it losing competitiveness and even facing bankruptcy. In addition, rising interest rates may prevent a company extending its level of gearing.

Two other effects may be observed.

(a) Share prices

When interest rates change, the return expected by shareholders will change, too. If a shareholder expects, say, a 10% return on their investment and the annual divided per share was 15p then (ignoring any capital growth) the market value of a share should be:

10%

p15 = £1.50

If interest rates were to rise (to 12%, say) the shareholder would equally expect a bigger return for his shares and the price would have to fall (assuming the dividend pay out was constant) as follows:

12%

p15 = £1.25

I’m sure you can calculate what the share value would be if interest rates were to fall (say to 8%).

(b) Capital gains and losses

A final point to note is that, because of movements in the capital value of an investment, an increase in interest rates will lead to market values dropping.

As an example, if you hold gilts with a “coupon rate” of 10% at a time when the market is 10%, your market investment will be equal to the face value of the gilts, say £100.

If interest rates rise to 15% their market value will drop:

£100 × %15%10 = £66.67

If interest rates drop to 5% their market value will rise:

£100 × %5%10 = £200

Thus, a firm with any sizeable level of debt is exposed to problems arising from adverse movements in interest rates and needs to limit the risk associated with such exposure.

Page 336: libvolume5.xyzlibvolume5.xyz/law/ballb/3rdyear/semester5/corporatelaw/corporate... · ABE Advanced Diploma in Business Administration Study Manual CORPORATE FINANCE Contents Study

328 Managing Exposure to Risk – Financial Derivatives

© Licensed to ABE

Managing Exposure to Interest Rate Risk Most companies have some form of debt finance and as a consequence are exposed to interest rate risk.

Interest rates on a loan may be either floating or fixed.

� In the case of floating rates, payments will vary with the rate applied at the time and the risk is that rates may rise, increasing the cost of the loan and, in turn, affecting profits.

� Fixed rates have the advantage that costs remain constant and they are, consequently, seen as less risky. However, there remains a risk in that spot rates may fall below the fixed rate – in which case, a floating rate would be more advantageous.

Interest rate risk also applies to investments. Where companies have funds invested in interest bearing securities, fluctuations in interest rates will give rise to variations in income.

Where companies have both assets and liabilities that are sensitive to interest rate changes, gap analysis is the means used to measure the extent of exposure to interest rate risk. Under this, assets and liabilities which are sensitive to interest rate changes and mature at the same time are grouped together. The difference between the two represents the firm’s interest rate exposure:

� where interest sensitive assets are greater than liabilities, both with the same time to maturity, the gap is positive and the firm will lose out if interest rates fall;

� where interest sensitive liabilities are greater than assets, both with the same time to maturity, the gap is negative and the firm will lose out if interest rates rise.

The management of interest rate risk will involve hedging against adverse movements in order to contain the extent of any exposure. As with managing exchange rate risk (which we deal with in the next unit), this falls into two categories:

� internal, or natural, techniques – those which are effected entirely by the financial organisation and structure of the company itself; and

� external, or transactional, techniques – those using the range of derivative instruments which are effected by the use of third party services, such as banks and specialist exchanges.

Both types of technique provide effective means of covering exposure, depending on the circumstances. We shall consider internal techniques first and then work through the various external methods.

C. INTERNAL TECHNIQUES OF MANAGING INTEREST RATE EXPOSURE

There are two main internal means of reducing interest rate exposure.

Matching The principle of internal matching to hedge interest rate exposure essentially involves offsetting assets with liabilities – for example, increases in borrowing costs can be offset by increases in interest bearing deposits. For many firms, though, interest rate matching is not viable because the amounts received as income from interest bearing assets are rarely sufficient to offset more than a small proportion of their loan payments.

Page 337: libvolume5.xyzlibvolume5.xyz/law/ballb/3rdyear/semester5/corporatelaw/corporate... · ABE Advanced Diploma in Business Administration Study Manual CORPORATE FINANCE Contents Study

Managing Exposure to Risk – Financial Derivatives 329

© Licensed to ABE

Smoothing Smoothing refers to the maintenance of a balance between fixed rate and floating rate borrowing.

Floating rates enable a company to take advantage of any reduction in rates to reduce their borrowing costs, whereas fixed rates aid cash planning. Both, though, have disadvantages and it does not follow that one or other method should necessarily dominate. Having a mixture of fixed and floating rate liabilities means that advantage may be taken of the benefits of both. Thus, if interest rates rise, the increase in costs from the floating rate liabilities is to some extent offset by the relatively cheaper fixed rate payments.

D. FUTURES CONTRACTS

These contracts are fixed in terms of rate, delivery period and amount, and provide an interest rate commitment for a future period that is agreed at the outset.

Before we move on to consider the use of futures in hedging interest rate exposure, we shall look briefly at certain basic elements of the way in which they operate.

Characteristics Futures contracts are “exchange traded” derivatives. That means that they are bought and sold on organised exchanges such as the London International Financial Futures Exchange (LIFFE). As such, there are certain rules affecting the way in which they are transacted, and some specific terminology associated with them.

(Note that option contracts are also exchange traded derivatives and exhibit many of the same characteristics. Other types of derivatives – notably forward contracts – are not dealt with on dedicated exchanges and therefore have somewhat different characteristics.)

(a) Margin accounts

All exchanges offering futures contracts require traders to lodge some form of deposit as security against losses. This is the initial margin paid by the hedger to start a margin account.

At the end of each trading day, the daily gains or losses on the exchange for each trader are added to or subtracted from the margin account to arrive at a new balance. This process is known as marking to market, and the adjustment to the margin account is termed the variation margin. In most cases, despite the daily fluctuations in the variation margin, the margin account is sufficient to offset the likely effect of default. (The system proved its worth in the Barings case, where the SIMEX Exchange was left largely intact and none of the counterparties to Barings’ trade lost out.)

Note that, whilst all futures transactions have a buyer and a seller, because the exchange is regulating the trade and gains/losses are marked to the market through the variation margin, the obligation of futures traders is not to the party to their trade, but to the exchange itself (or, more precisely, to the clearing house holding the margin accounts).

(b) Contract size

Futures contracts on a particular exchange are all of a standard size. For example, the standard size for sterling interest rate futures contracts on LIFFE is £500,000. Thus, in order to hedge an exposure of £2 million, it would be necessary to take out four contracts, each of which would be for £½ million.

Page 338: libvolume5.xyzlibvolume5.xyz/law/ballb/3rdyear/semester5/corporatelaw/corporate... · ABE Advanced Diploma in Business Administration Study Manual CORPORATE FINANCE Contents Study

330 Managing Exposure to Risk – Financial Derivatives

© Licensed to ABE

(Similar standard sizes are also applied to currency futures contracts – for example, on the IMM Chicago Exchange, sterling contracts are for £62,500 and those for yen are for YEN12.5m.)

(c) Basis points and tick values

The smallest recorded movements in futures prices are expressed in basis points – one basis point being 0.01% of the amount of the underlying asset (see below). However, it is more usual to express the price movement of futures contracts themselves in ticks. The value of a tick will reflect the minimum gain or loss which may be recorded on a contract. This value varies from contract to contract depending on the standard size of the contracts traded.

To illustrate this, let us consider currency futures for a moment. The price movements on the contract are movements in one currency against another, and one basis point is 0.01% of the unit of currency – equating to a movement of 0.01 cent in the case of dollars or 0.01 pence for pounds, etc. So, for example, the minimum price movement recorded on a sterling contract on the Chicago exchange will be:

£62,500 × 0.0001$/£ = $6.25

This minimum recorded movement in price represents the minimum gain or loss which can be made on the contract and is known as the tick value. Thus, the tick value of a sterling futures contract on the Chicago Exchange is $6.25.

For interest rate futures, the price movements are movements in the interest rates themselves, and one basis point is a 0.01% change in interest rates. The tick value of an interest rate futures contract needs to reflect the proportion of the year over which the contract runs (since interest rates are quoted per annum). Thus, the tick value of a three month sterling interest rates futures contract on the LIFFE will be given by the price movement consequent upon the movement of one basis point for three months of a year:

Unit of contract × basis point × contract period = tick value

£500,000 × 0.0001 × 123 = £12.50

(c) Delivery dates of contracts

Exchange traded contracts – whether futures or options – all have pre-determined maturity dates on which delivery of the underlying asset occurs. For example, the contracts quoted on the Chicago Exchange relate to delivery dates at the end of March, June, September and December. This pattern applies to both currency and interest rate futures contracts.

As we have seen, most futures contracts are not delivered, but are closed out beforehand by the taking of an opposite position in the same derivative. Closing out clearly needs to be effected prior to the maturity date of the contract in order to cancel the obligation to buy or sell the underlying asset. Thus, the purchase of a December sterling contract would be closed out with the sale of a December sterling contract, prior to December.

Where a futures contract is required for a period which falls between the specified dates of available contracts, the trader will need to opt for the nearest delivery dates. For example, a company which wishes to hedge an interest rate exposure from February to May – a period which does not correspond with the available contracts – will need, in February, to take out a March futures contract and then roll the hedge forward by simultaneously closing this out in March and taking out a June contract on the same day. This will then be closed out in May with an equal and opposite trade.

Page 339: libvolume5.xyzlibvolume5.xyz/law/ballb/3rdyear/semester5/corporatelaw/corporate... · ABE Advanced Diploma in Business Administration Study Manual CORPORATE FINANCE Contents Study

Managing Exposure to Risk – Financial Derivatives 331

© Licensed to ABE

In theory, there is nothing to stop a hedger rolling over the hedge indefinitely, although this will incur transaction costs every time one is closed out and another opened.

(d) Prices of contracts

The prices of contracts are, as you would expect, a function of interest rates. They are calculated by reference to an index of 100 less the rate of interest applicable to the underlying instrument. Thus, an open (current) price of, say, a December contract of 93.070, corresponds to an interest rate 6.93% (100 − 6.93).

Thus, if interest rates rise, the price of an interest rates future contract falls. This inverse relationship is central to hedging with interest rate futures.

Hedging Interest Rate Exposure with Futures Contracts Any futures contract involves fixing the price today on a standard quantity of a financial instrument (in this case, interest rate futures) to be purchased or sold at a fixed future date. As we know, futures contracts are rarely held to maturity and the underlying asset delivered, but are closed out by taking an opposite position. Hedging interest rate exposure using interest rate futures is based on the gain or loss on the closing out of the futures contracts equating with the loss or gain arising from a change of interest rates on the company’s liabilities.

To fully appreciate the way in which interest rate futures are used, we need to remind ourselves that taking a short position on interest rate futures (i.e. selling them) hedges a rise in interest rates. Borrowers, then, will take a short position in order to hedge against higher interest rates, whilst lenders will take a long position in order to hedge against lower interest rates.

The following example illustrates the process of hedging using interest rate futures.

A company is going to take out a £2.5m loan in two months time (December) for three months. The three month LIBOR rate is currently quoted on the London exchange at 7¼% and the company wishes to hedge against a rise in interest rates in two months time by locking into an interest rate futures contract. Let us assume that the futures contract is quoted at 93.07 (as above). In December, it transpires that LIBOR rates had risen to 7½% and futures contracts were then quoted at 92.80.

To hedge against a rise in interest rates – which would result in a fall in futures prices – necessitates taking a short position (to sell) the three month December futures contracts at the price currently quoted. In the solution which follows, the contract runs to expiration and a cash settlement is received via the exchange’s settlement price system. Cash settlement at expiration is always based on the LIBOR rate on the last day of trading in those futures, which in this case is 92.80. All contracts are settled net – here, the difference between the quoted buying price of futures in December and the selling price of those contracts as quoted when they were entered into (in October).

(The same outcome would be achieved by closing out the original position through an equal and opposite trade – i.e. to buy December three month futures contracts – in December. Closing out is necessary when the period of the hedge does not coincide with the exchange’s delivery settlement date. However, in this example, the two dates do coincide which allows a choice of methods used for settling the profit/loss on the futures contracts.)

The first step is to ascertain the number of futures contracts required to hedge the position. The contracts available on LIFFE are in denominations of £500,000. So, to hedge the sum of £2.5m, the company needs to enter into five contracts.

Entering into a contract in October to sell five three month December interest rate futures fixes the price at 93.07. In December, the price has fallen to 92.80, resulting in a gain on expiration as follows.

Page 340: libvolume5.xyzlibvolume5.xyz/law/ballb/3rdyear/semester5/corporatelaw/corporate... · ABE Advanced Diploma in Business Administration Study Manual CORPORATE FINANCE Contents Study

332 Managing Exposure to Risk – Financial Derivatives

© Licensed to ABE

Gain = Number of contracts × movement in ticks × tick value

As we explained above, the tick value on LIFFE sterling futures contracts is £12.50.

Gain = 5 contracts × (92.80 − 93.07) ticks × £12.50

= 5 × 27 × £12.5

= £1,687.50

In December, the company will raise the £2.5m necessary by means of a loan at the now current three month LIBOR rate of 7½%. This results in an actual borrowing cost of £46,875 (£2.5m × 7.5% for ¼ of a year). We can compare this with the cost of interest at the October rate (7¼%) and derive the increased cost of the loan as follows:

Cost of increase = £2.5m × (7.5 − 7.25%) × 123 months

= £1,562.50

The efficiency of the hedge (the degree to which the hedge covers the exposure) is:

Efficiency = 50.562,1£50.687,1£ × 100%

= 108%

The gain from the futures position has more than offset the cost of the ¼% increase in interest rates. In fact, the success of the hedge results in an effective reduction of the interest rate on the loan This is shown as the actual interest minus the gain on the futures position as an annual percentage of the principal:

3

12 £2.5m

£1,687.50 - £46,875 × = 7.23%

The gain on the futures market has more than matched the loss in the cash market, and the company is only paying, effectively, 7.23% interest, even though interest rates rose over the two month period.

E. FORWARD RATE AGREEMENTS (FRAs)

These are forward contracts which provide for interest rates to be fixed in advance for a specified period commencing at some agreed future date. They are the interest rate equivalent of forward currency or commodity contracts, although unlike these forms of forward contract, there is no intended delivery of a sum of principle.

Characteristics Forward rate agreements are essentially an agreement between a bank and its customer to fix a future rate of interest to cover a specified amount of money to be lent or borrowed for a specified period of time in the future. They relate only to the predetermined rate of interest on a sum of money and not to the actual sum of money borrowed or lent. (That will be a separate transaction, conducted in the cash market in the usual way, to borrow/lend the money at whatever interest rate is prevailing at the time.) All that happens in respect of the FRA is that either the bank is compensated by the client, or vice versa, if actual interest rates vary from the pre-set interest rates agreed in the contract.

This means that, although there is an obligation to meet the terms of the interest rate contract, there is no obligation to actually borrow or lend the money. Provided that the compensating amount is settled between the two parties on the due date, that is where the obligation ends.

Page 341: libvolume5.xyzlibvolume5.xyz/law/ballb/3rdyear/semester5/corporatelaw/corporate... · ABE Advanced Diploma in Business Administration Study Manual CORPORATE FINANCE Contents Study

Managing Exposure to Risk – Financial Derivatives 333

© Licensed to ABE

FRAs are not standard contracts as traded on the futures exchanges, but are tailored to meet individual needs, though they tend to be only available for up to 12 months and on loans of £500,000 or more.

Hedging Interest Rate Exposure with FRAs Where a hedge on future borrowings is required, the purchase of an FRA locks the repayments on the loan to a specified interest rate and protects against any further rise in interest rates. On the other hand, selling an FRA provides a hedge on future lending by locking future cash inflows from the loan to a specified interest rate and protects against any further fall in interest rates. The rate at which the transaction is locked is a forward interest rate as quoted today, but taking effect at a specified date in the future.

Consider the following example.

Clock plc is going to take out a 90 day £10m loan in three months’ time at an interest rate linked to the three month LIBOR rate. As a hedge against an increase in interest rates over the intervening period it also takes out an FRA, the interest rate for which is set at 8% pa. What would be the consequences if, on taking out the loan, the LIBOR rates had fallen to 5%?

Since interest rates have decreased, Clock will have to pay the bank the interest rate differential on the sum of £10m. This is the amount by which the cost of the loan has fallen as a consequence of the fall in interest rates from 8% to 5%:

$10m × (0.08 − 0.05) × 36590 days = £73,972

In effect, the contract requires Clock to pay over this amount to the bank. However, the contract has to be settled prior to the start of the loan period (when the interest rate on the loan is known) and not, as you might expect, when the loan matures. The total amount of the reduction in costs over the period of the loan will, then, have to be discounted to reflect its present value. The discount rate will be that applying to the interest rate on the loan:

×+

×

36590 0.05 1

1 972,73£ = £73,071

The payment to the bank in three months time is, therefore, £73,071.

Note that Clock plc has not gained financially from the fall in interest rates, but had they increased, the company would have been compensated by the bank for the interest rate differential. FRAs only give protection from adverse interest rate movements – it is not possible to benefit from favourable movements. They do, however, effectively hedge the exposure.

F. INTEREST RATE SWAPS

An interest rate swap is an agreement between two parties under which each agrees to pay the other’s interest for an agreed period.

Characteristics The most common form of interest rate swaps involves the exchange of a fixed rate for a floating rate.

The interest rates are based on a “notional” underlying principal sum of money. Note, though, that it is not the principal which is swapped – merely the interest rate payments. (This is in contrast to currency swaps where both the principal and interest is swapped.)

Page 342: libvolume5.xyzlibvolume5.xyz/law/ballb/3rdyear/semester5/corporatelaw/corporate... · ABE Advanced Diploma in Business Administration Study Manual CORPORATE FINANCE Contents Study

334 Managing Exposure to Risk – Financial Derivatives

© Licensed to ABE

Whatever the form of the exchange, each party remains responsible for servicing its own debt and has to continue to make payments to its own lender at the agreed rate on the loan. Thus, a party agreeing to swap a fixed rate of interest for a floating rate would still be responsible for making the fixed rate payments to the lender of its funds. The effect of the swap is a compensating payment between the parties to settle the net difference arising from any differential rates on the interest payments.

Interest rate swaps work on the principle of comparative advantage (see below) and are more common than their cousins, currency swaps.

Swaps are popular because they are easy and cheap to arrange (with only legal fees to pay), and are flexible as to the size and duration to which they are applied.

Operation of Interest Rate Swaps The best way to explain the operation of interest rate swaps is through an example.

Hammer plc has a five year loan of £25m with a floating interest rate of LIBOR + 0.5%. It is concerned about fluctuating payments over the period of the loan and would like to convert to a fixed rate. Tongs plc also has a five year loan of £25m with a fixed rate of 7% and is seeking to reduce its repayments, preferably with a floating rate.

The rates for each party’s present agreement and those quoted for the alternative type of interest payment (fixed or floating), are as follows.

Company Fixed rate of interest

Floating rate of interest

Hammer Plc 6.5% LIBOR + 0.5%

Tongs Plc 7% LIBOR + 1.7%

How could a suitable arrangement be made whereby each party can take advantage of lower interest rates on each other’s loan?

Each company has the choice of either opting for a floating or fixed rate agreement at the rates quoted by the bank. Hammer has an interest rate advantage over that of Tongs, with its quoted rates for both fixed and floating rates of interest being lower than those available to Tongs. The differential interest rate advantage for Hammer is as follows:

� Fixed rate: 7% − 6.5% = 0.5%

� Floating rate: (LIBOR + 1.7%) − (LIBOR + 0.5%) = 1.2%

Combining these two absolute advantages gives an “apparent” advantage of 0.7% (1.2% - 0.5%). Because this apparent advantage exists, gains are possible via an interest rate swap. If this advantage did not exist, there would be no benefit to either party from a swap arrangement.

The strategy is for Hammer and Tongs to swap, in part or whole, their respective interest payments, applying the most advantageous rates to the terms of the swap – those available to Hammer. Under the swap agreement, Hammer will pay Tongs the fixed rate of 6.5% and Tongs will pay Hammer the floating rate of LIBOR + 0.5%. However, the swap payments need to take account of the “apparent” advantage of 0.7% and this will be apportioned between the two parties on some agreed formula. Let us assume that this will be equally apportioned (0.35% and 0.35%).

The effect on the interest cashflows of the two companies is shown in the following table.

Page 343: libvolume5.xyzlibvolume5.xyz/law/ballb/3rdyear/semester5/corporatelaw/corporate... · ABE Advanced Diploma in Business Administration Study Manual CORPORATE FINANCE Contents Study

Managing Exposure to Risk – Financial Derivatives 335

© Licensed to ABE

Cashflow Hammer plc Tongs plc

Obligation to lender

negative LIBOR + 0.5% 7%

Swap receipts positive LIBOR + 0.5% 6.15% (6.5% − 0.35%)

Swap payments negative 6.15% (6.5% − 0.35%)

LIBOR + 0.5%

Net payment negative 6.15% LIBOR + 1.35% (−7% + 6.15% − LIBOR + 0.5%)

The effect, then, is for Hammer to have secured a fixed interest rate 0.35% less than the rate it had been quoted (6.5%) and for Tongs to have secured a floating interest rate also 0.35% lower than the rate it had been quoted (LIBOR + 1.7%). The terms of the swap will have satisfied both parties in that they have acquired the type of rate desired (fixed or floating) and at a lower rate than originally quoted.

(Note that a different stream of cashflows would apply if the apparent advantage of 0.7% had been apportioned differently to the 50:50 model illustrated.)

Remember that each company remains responsible for the payments to their own respective lenders under the terms of the original loans.

The whole process is illustrated in Figure 14.1.

Figure 14.1: Interest rate swap

G. OPTIONS

Option contracts provide the purchaser of the option with the right – but not the obligation – to buy or sell the underlying asset or instrument at a price established in the contract. This price is referred to as the exercise or strike price. The purchaser of the option pays a non-refundable premium (the price of the option itself) to the seller of the option to establish the contract.

Hammer plc Tongs plc

Bank Bank

Fixed rate @ 6.15%

Floating rate @ LIBOR + 0.5%

Floating rate @ LIBOR + 0.5%

Fixed rate @ 7%

Page 344: libvolume5.xyzlibvolume5.xyz/law/ballb/3rdyear/semester5/corporatelaw/corporate... · ABE Advanced Diploma in Business Administration Study Manual CORPORATE FINANCE Contents Study

336 Managing Exposure to Risk – Financial Derivatives

© Licensed to ABE

There are a number of different types of option contract available on the various options exchanges around the world (the main ones being the LIFFE, New York, Philadelphia, Montreal and Chicago exchanges). These organised exchanges offer standard format options in which the main elements of the contract are pre-determined:

� size – each option contract relating to a particular volume of the asset/instrument concerned (for example, traded options in equities are in contract sizes of 1,000 units);

� exercise price – as determined by the market; and

� option period – with an expiry date usually of three months (or in multiples of three months).

There are two main types of standard format option contracts, as follows.

� Traditional options – These are all three month contracts which must be held to the expiry date, at which point the right to buy or sell the underlying asset/instrument must be exercised or allowed to lapse. There is no trading in these types of options.

� Traded options – These types of options may be bought or sold through the exchange continuously throughout their life. Such options are usually offered with three-month, six-month or nine-month expiry dates, each with a different exercise price and premium.

It is also possible to establish option contracts outside of the organised exchanges. Such options are known as “over the counter” (OTC) options, and the individual elements – size, exercise price, expiry date – maybe negotiated and agreed between the buyer and seller.

Options tend to be more expensive than other derivatives since they offer the possibility of making gains should the movement in the underlying asset be favourable (in contrast to futures and forward contracts). Indeed, the attractiveness of options as part of a financial strategy derives from the range of pay-off profiles available, depending on whether the option is to buy or sell the underlying asset/instrument and on whether the investor is the buyer or seller of the option (respectively, taking a long or short position on the option). The starting point for understanding the use of options, then, lies in identifying the outcomes associated with these characteristics.

Calls and Puts There are two types of option contract:

� a call option – which is an option to buy an asset/instrument; and

� a put option – which is an option to sell an asset/instrument.

(The underlying asset/instrument may be bonds, shares, currency, agricultural products, interest rate futures, etc., but for now let us assume that it simply refers to equities. So, we shall consider options as relating to the right to buy or sell a share.)

(a) Call options

These will be purchased where it is anticipated that the price of an asset will rise. The call option buyer (said to be holding a long call) will only exercise the option (i.e. buy the asset) if the market price of the asset is above the exercise price.

We can examine the characteristics of these options for both long and short positions by reference to a call option to buy shares in LTD at the exercise price of £25. We shall assume that the premium paid for the option (i.e. its price) is £1, which represents the value of the option at that instant in time.

Page 345: libvolume5.xyzlibvolume5.xyz/law/ballb/3rdyear/semester5/corporatelaw/corporate... · ABE Advanced Diploma in Business Administration Study Manual CORPORATE FINANCE Contents Study

Managing Exposure to Risk – Financial Derivatives 337

© Licensed to ABE

Long call

For the buyer of a call option, there are two scenarios:

� the share price falls below the exercise price of £25 – in which case the option will not be exercised, since the shares could be acquired on the market at the (lower) spot price, and the holder will, therefore, lose the price paid for the option (£1);

� the share price rises above the exercise price of £25 – in which case the option will be exercised, the shares acquired and then resold at the (higher) spot price, with the result that the holder makes a profit equal to the spot price less the exercise price (and the premium).

We can show the pay-off profile for this position diagrammatically, as in Figure 14.2.

Figure 14.2: Pay-off profile for a long call

Note that the risk exposure is limited to the amount of the premium in the case of the asset price falling below the exercise price, whereas the potential return is theoretically infinite.

Short call

For the seller of a call, the position is the exact opposite of the buyer’s position. He/she is obliged to sell the asset at the exercise price, but will only have to do so where the option is exercised by the buyer – i.e. if the market price of the asset is above the exercise price.

The potential profit is limited to the price of the option (here £1), but there is exposure to the risk of having to sell the shares at what might be a considerable loss.

Again, we can show the pay-off profile for this position diagrammatically (Figure 14.3).

Figure 14.3: Pay-off profile for a short call

Asset price

Prof

it/lo

ss

Exercise price £25

Pay-off profile

0

−2

−3

−1

3

2

1

Prof

it/lo

ss

Asset price

Exercise price £25

Pay-off profile

0

−2

−3

−1

3

2

1

Page 346: libvolume5.xyzlibvolume5.xyz/law/ballb/3rdyear/semester5/corporatelaw/corporate... · ABE Advanced Diploma in Business Administration Study Manual CORPORATE FINANCE Contents Study

338 Managing Exposure to Risk – Financial Derivatives

© Licensed to ABE

(b) Put options

These will be purchased where it is anticipated that the price of an asset may fall. The buyer of a put option (a long put) will only exercise the option (i.e. sell the asset) if the market price falls below the exercise price. This establishes a ceiling on any loss incurred if the option is not exercised, but allows profits to be taken if the price falls and the option is exercised.

Again, we shall examine the characteristics of these options for both long and short positions by reference to a put option to buy shares in LTD at the exercise price of £25. The premium is £1.

Long put

For the put option buyer, there are two scenarios:

� the share price rises above the exercise price of £25 – in which case the option will not be exercised and the buyer loses the price of the option (£1);

� the share price falls below the exercise price of £25 – in which case the option will be exercised, since the shares can be sold at the exercise price and then bought back at the (lower) spot price, with the result that the holder makes a profit equal to the exercise price less the spot price (and the premium).

Note that, even if the buyer of the put option does not already own the shares at the expiry date, he/she can still profitably exercise the option by acquiring them on the open market and then selling them at the (higher) exercise price. The profit is the same.

We show the pay-off profile for this position in Figure 14.4.

Figure 14.4: Pay-off profile for a long put

Note that the risk exposure is limited to the amount of the premium in the case of the asset price rising above the exercise price, whereas the potential return is the difference between the exercise price and the (lower) asset price.

Short put

For the seller of a put the position is, again, the exact opposite of the buyer’s position. He/she is obliged to buy the asset at the exercise price, but will only have to do so where the option is exercised – i.e. if the market price of the asset is below the exercise price.

Again, profit is limited to the price of the option (the £1 premium), whereas the risk exposure if the asset price falls is considerable.

The pay-off profile for this position is shown in Figure 14.5.

Prof

it/lo

ss

Exercise price £25

Pay-off profile

0

−2

−3

−1

3

2

1Asset price

Page 347: libvolume5.xyzlibvolume5.xyz/law/ballb/3rdyear/semester5/corporatelaw/corporate... · ABE Advanced Diploma in Business Administration Study Manual CORPORATE FINANCE Contents Study

Managing Exposure to Risk – Financial Derivatives 339

© Licensed to ABE

Figure 14.5: Pay-off profile for a short put

As can be seen from the above, the seller of an option carries a considerable downside risk. The premium charged is, therefore, a reflection of this risk – and we shall look at the pricing of options in the next section. However, it must be said that the “odds” are in favour of the seller – otherwise the option would not be sold in the first place.

For the buyers of options, we can see that they offer a ceiling on potential losses from fluctuating asset prices equivalent to the price paid for the option, whilst at the same time enabling a profit to be taken from a favourable change in those prices.

(c) Put-call relationships

The essence of developing strategies in the use of options lies in the relationship between the pay-offs associated with short and long positions on puts and calls.

Consider the taking of a long position on a call (purchasing a call option) together with a short put (selling a put option) on the same underlying asset, both with the same strike price. We can show the pay-offs associated with this position as follows. (Note that in this and the diagrams which follow, we have ignored the effect on the pay-off profile of paying the premium for the option.)

Figure 14.6: Pay-off profiles for a long call/short put

This position results, no matter what the price of the underlying asset, in that asset being acquired for “X” – the strike price. Can you work out why?

Prof

it/lo

ss

Asset price

Exercise price £25

Pay-off profile

0

−2

−3

−1

3

2

1

Long call

Asset price

Short put Exercise price

X

0

Prof

itLo

ss

Page 348: libvolume5.xyzlibvolume5.xyz/law/ballb/3rdyear/semester5/corporatelaw/corporate... · ABE Advanced Diploma in Business Administration Study Manual CORPORATE FINANCE Contents Study

340 Managing Exposure to Risk – Financial Derivatives

© Licensed to ABE

There are two possible scenarios at the expiry date:

� the price of the asset has risen above the strike price – in which case the short put will not be exercised (i.e. the right to sell will not be exercised), but the long call will be exercised (i.e. the right to buy will be exercised) and the asset acquired at the strike price; or

� the price of the asset has fallen below the strike price – in which case the long call would not be exercised, but the short put will be and the seller of the put will have to acquire the asset at the strike price.

An alternative strategy which equates with the above position would be to purchase the underlying asset itself at the outset (taking a long position on the asset). If this was done using borrowed funds at a risk free rate equivalent to the present value of the strike price at the expiry date (which would be “X"), then at the expiry date, the loan “X” will be repaid and the asset acquired for that price.

Thus, these two strategies have the same result – acquisition of the asset at “X”, the exercise price.

We can express this in terms of an equation as follows:

( )

PCr1

XS t −=+

where S = spot price of the underlying asset

X = exercise price

r = risk free rate of interest

t = time for the option to run (before expiry date)

C = value of the call option (the premium)

P = value of the put option (the premium)

This equation is known as the put-call parity.

Using this equation allows us to define different combinations of short and long positions on puts, calls and/or the asset itself which result in parity, in order to hedge against changes in the price of the asset. For example, a call option can be created synthetically by taking a long position on both the asset itself and a put option. This is confirmed by transposing the terms of the put-call parity equation above, as follows:

( )tr1

XPSC+

−+=

We can similarly express the value of a put option as:

( )tr1

XSCP+

−−=

Option Prices The price of an option contract is the premium paid on its formation or the market price at which the option may be traded.

There are two elements to the price of an option:

Page 349: libvolume5.xyzlibvolume5.xyz/law/ballb/3rdyear/semester5/corporatelaw/corporate... · ABE Advanced Diploma in Business Administration Study Manual CORPORATE FINANCE Contents Study

Managing Exposure to Risk – Financial Derivatives 341

© Licensed to ABE

� the intrinsic value – which is the gain to be made from exercising the option immediately; and

� the time value – which reflects the length of time that the option has left to run before expiry.

(a) Intrinsic value

The intrinsic value is the difference between the price of the underlying asset (at any time) and the exercise price. The following diagram illustrates this in relation to a long call.

Figure 14.7: Intrinsic value of a long call

When the spot price of the asset is at “A”, the intrinsic value of the call option is A − X. In such a position, the option is said to be “in the money” – meaning that the exercise price is above the current price of the underlying asset. (For a long put, the contract would be “in the money” where the exercise price is below the spot price.)

If the spot price is at “B”, the contract is said to be “out of the money”, meaning that the exercise price is below that of the asset (for a long call) or above that of the asset (for a long put).

Where the exercise price and the spot price are equal, the contract is said to be “at the money”.

The minimum intrinsic value of an option is zero. Why should this be expected?

If the contract is out of the money, the option will not be exercised. In the case of a call option, if the price of the underlying asset is below the exercise price, it would be more advantageous for the holder to purchase the asset in the market at the lower spot price and, thus, the option would be allowed to lapse. The opposite holds true for a put option in that, if the price of the underlying asset was above the exercise price, again the option would be allowed to lapse.

The maximum intrinsic value of a put option is limited by its exercise price. Why should this be expected?

A put option will only be exercised when the price of the asset is below the exercise price. The intrinsic value of the option will, therefore, rise as the asset price falls towards zero. At its maximum, with the asset price at zero, the gain from exercising the option will be equivalent to the exercise price itself.

(b) Time value

The value of an option at any particular time is not just a function of its intrinsic value – this can, of course, change over the period prior to its maturity. There is, therefore, a value

Asset price

Intrinsic value

Pay-off profile (long call)

Exercise price

X

0

Prof

itLo

ss

B A

Page 350: libvolume5.xyzlibvolume5.xyz/law/ballb/3rdyear/semester5/corporatelaw/corporate... · ABE Advanced Diploma in Business Administration Study Manual CORPORATE FINANCE Contents Study

342 Managing Exposure to Risk – Financial Derivatives

© Licensed to ABE

associated with the length of time that the option has left to run before expiry – and the longer this period, the greater the likelihood that a gain may occur on the contract. It follows, therefore, that an option which is not due to expire immediately, would possess an element of value associated with time, irrespective of whether it was in the money, out of the money or at the money.

The time value represents, therefore, the difference between the intrinsic value of the option and its market value.

For example, consider the following prices for call options on XYZ shares as shown below for two different exercise prices and three maturity dates. XYZ’s shares are currently trading at 231p.

Expiry date Exercise price January April July

230p 16.5p 35p 39.5p

250p 8p 26.5p 32p The intrinsic value of the 230p option is:

231p (asset price) − 230p (exercise price) = 1p

The time value associated with the January expiry 230p option is, therefore:

16.5p (option price) – 1p (intrinsic value) = 15.5p

You can see that the time values of those options with later expiry dates are considerably higher.

What are the intrinsic and time values of the 250p call options?

The call options with the higher exercise price are currently out of the money. Their intrinsic value, therefore, is zero. The prices of the options, then, reflect just the time values associated with them – i.e. 8p for the January expiry date, etc.

Note that the time value tends towards zero as the option nears maturity, leaving the price of the option equating with its intrinsic value.

There are two factors affecting time values.

� Potential for change in the price of the underlying asset

Clearly, the likelihood of the asset price changing and making the option contract in the money will influence the time value. This is, to some extent, a function of the length of time before the option expires (as seen above), but is also influenced by the volatility of the price of the underlying asset itself.

The more volatile the price, the more likely it is that it will put the option in the money – either by rising above the exercise price (for a long call/short put) or falling below it (for a short call/long put). Volatility of the underlying asset is in part a function of the general buoyancy, or otherwise, of the market itself.

Note that the greater the likelihood of market volatility making an option in the money for the buyer, the greater the need for the seller of the option to raise the premium in order to cover the additional risk.

Page 351: libvolume5.xyzlibvolume5.xyz/law/ballb/3rdyear/semester5/corporatelaw/corporate... · ABE Advanced Diploma in Business Administration Study Manual CORPORATE FINANCE Contents Study

Managing Exposure to Risk – Financial Derivatives 343

© Licensed to ABE

� Variability of interest rates

This is a less obvious influence on option prices than the volatility of the asset price. It derives from the alternative strategy of acquiring the asset itself with borrowed funds.

Buying a call option contract is, in effect, putting a deposit on the deferred purchase of the underlying asset. The balance of the cost of the asset (the exercise price) will be paid in the future, at the expiry date, by which time its present value will be lower. The higher the level of interest rates, and/or the longer the period before expiry, the lower the present value would be. This (effective) lowering of the exercise price adds value to the option premium and, thus, its time value.

Conversely, a lowering of interest rates, and/or a shorter time period before expiry, will increase the present value of the exercise price. This would add value to holding a put option and increase its time value.

Hedging and Trading Strategies using Options So far, we have considered the general nature of options and identified a number of their characteristics. We now move on to look at strategies for hedging and/or trading, based on these underlying principles. Initially, the focus will be on equities, but we shall move on to explore options involving futures in respect of hedging interest rate exposure.

Note that, whilst options relate to an asset – allowing the holder of the option to take a position on, potentially, buying or selling that asset in the future at a fixed price – the point of entering into an option contract is not, in the vast majority of cases, actually to acquire or sell the asset. Rather, the intention is to establish a position (in relation to the seller of the option) whereby profits may be taken if the movement of the asset price is favourable – i.e. rising above the exercise price in the case of a call or falling below it in the case of a put. At the expiry date, therefore, a view could be taken as to whether the option is to be exercised and, if so, the seller would simply pay the buyer the profit due on the transaction, without any need for the asset itself to change hands. (Note, from the examination of the positions of buyers and sellers above, that any profit due to the buyer comes directly from the seller of the option.)

Alternatively, the contract may be closed out before maturity in the same way as previously discussed in respect of futures contracts. In this case, sellers and buyers adopt a position in respect one option which is directly contrary to their position on another option. This effectively cancels out their rights/obligations under both positions and the two options can be closed out, with any profit or loss on the transactions being taken. Remember that the objective in taking positions with options is not necessarily to acquire or dispose of the underlying asset in question, but to hedge against fluctuations in prices or to generate profits from them. Adopting positions which lead to closing out may be the best way of cutting exposure to risk and/or taking profits.

(a) Strategies involving an option and share combined

In the first strategies we consider, we shall look at two combinations involving an option and the holding of the underlying asset itself:

� a covered call – whereby a long position on the share is combined with a short position on a call option; and

� a covered put – whereby a long position on the share is combined with a long position on a put option.

Page 352: libvolume5.xyzlibvolume5.xyz/law/ballb/3rdyear/semester5/corporatelaw/corporate... · ABE Advanced Diploma in Business Administration Study Manual CORPORATE FINANCE Contents Study

344 Managing Exposure to Risk – Financial Derivatives

© Licensed to ABE

Covered call

This is one of the most fundamental investment strategies, involving a portfolio of a long asset (purchase of the share itself) with a short call (selling of a call option) on the same asset.

The principle behind the strategy is that, if the price of the asset were to fall and cause a loss on the holding of the asset itself, this would be compensated by the inflow of funds from the premium received on the option. Of course, the loss on the asset may not exactly be offset by the premium, but it can, nevertheless, be contained to a large extent.

Consider the position in respect of a share purchased at 200p and a short call on the same share with a premium of 20p and an exercise price of 200p. We show the pay-off profile for the strategy diagrammatically in Figure 14.8.

(Note that the pay-off profiles for the long asset and the short call are shown by the dotted lines and that for the strategy as a whole by the heavy, unbroken line.)

Figure 14.8: Pay-off profile for covered call

Note that, if the asset price rises, the option will be exercised and the profit is limited to the premium on the call. If the asset price falls, the option will not be exercised and the investor is left with the loss on the asset itself, offset by the premium on the call option – the potential net loss here is 180p (200p - 20p) if the share price continues on a downward spiral to zero.

In order to protect against this downside risk, it would be necessary to adopt a slightly different hedging strategy.

Covered put

This portfolio combines a long asset (purchase of the share) with a long put (purchasing a put option) on the same asset. The principle here is that the put provides a floor price beyond which no further loss can occur.

We can illustrate this with a similar example to that above. Again the share is purchased at 200p and this time is combined with a long put on the same share with an option premium of 20p and an exercise price of 200p. The pay off profile is shown in Figure 14.9.

120 140 160 180 200 220 240 260 280

Share price (pence)

80

60

40

20

0

−20

−40

−60

Prof

it/lo

ss (p

ence

)

Long asset

Pay-off profile for strategy

Short call

Page 353: libvolume5.xyzlibvolume5.xyz/law/ballb/3rdyear/semester5/corporatelaw/corporate... · ABE Advanced Diploma in Business Administration Study Manual CORPORATE FINANCE Contents Study

Managing Exposure to Risk – Financial Derivatives 345

© Licensed to ABE

Figure 14.9: Pay-off profile for covered put

If the price of the asset falls, the investor can exercise the put and sell the asset at the exercise price of 200p, limiting the loss to the amount of the premium. Thus, the downside risk is minimised whilst any potential gain from a rising asset price can be taken.

(b) Strategies using a combination of options

The previous strategies of holding an asset together with a short call or long put are hedging strategies – they are designed to offset losses on the underlying asset itself. However, investors can seek to generate profits from the options themselves.

The simplest trading strategies would involve holding a single option on an underlying asset, anticipating a rise/fall in the asset price – a long call/put.

More sophisticated trading strategies involve combinations of options, known as a spreads, straddles and strangles.

Spreads

These involve taking simultaneous long and short positions using different options – either calls or puts – on the same underlying asset.

The following example illustrates this in respect of a “call spread” comprising a long call and a short call on the same share with the following premium and exercise prices:

Long call – premium (paid): 40p; exercise price: 160p

Short call – premium (rec’d): 20p; exercise price: 220p

120 140 160 180 200 220 240 260 280

Share price (pence)

80

60

40

20

0

−20

−40

−60

Prof

it/lo

ss (p

ence

)

Long asset

Pay-off profile for strategy

Long put

Page 354: libvolume5.xyzlibvolume5.xyz/law/ballb/3rdyear/semester5/corporatelaw/corporate... · ABE Advanced Diploma in Business Administration Study Manual CORPORATE FINANCE Contents Study

346 Managing Exposure to Risk – Financial Derivatives

© Licensed to ABE

Figure 14.10: Pay-off profile for call spread

The three scenarios which can result are:

� if the asset price falls below 160p, neither of the calls will be exercised and the investor is left with a net loss of 20p on the premiums;

� if the asset price is between 160p and 220p, the long call will be exercised and the asset acquired at the exercise price of 160p – constituting a gain of up to 60p (if the asset was at 220p) less the net loss of 20p on the premiums;

� if the asset price rises above 220p, both calls will be exercised, with the result that the asset will be acquired at 160p and then sold at 220p – constituting, again, a gain of 60p less the net loss of 20p on the premiums.

Thus, it can be seen that the strategy reduces the downside risk if the share price falls, although it places a ceiling on the maximum profit which can be generated if the share price rises.

Straddles and strangles

These are further examples of the ways in which different options can be combined to produce a portfolio matched to the investor’s own risk profile. In both cases, they involve the simultaneous holding of long or short positions on both a call and a put option with the same expiry date on the same underlying asset. The strategies differ in that the exercise prices for the put and call options are the same for a straddle, but vary in the case of a strangle.

Consider the following illustration of a long straddle with an exercise price of £5, the premiums being 20p for the call and 30p for the put.

120 140 160 180 200 220 240 260 280

Share price (pence)

100

80

60

40

20

0

−20

−40

Prof

it/lo

ss (p

ence

) Long call

Pay-off profile for strategy

Short call

Page 355: libvolume5.xyzlibvolume5.xyz/law/ballb/3rdyear/semester5/corporatelaw/corporate... · ABE Advanced Diploma in Business Administration Study Manual CORPORATE FINANCE Contents Study

Managing Exposure to Risk – Financial Derivatives 347

© Licensed to ABE

Figure 14.11: Pay-off profile for long straddle

The three possible scenarios are:

� if the price falls below £5, the call will not be exercised but the put will be, allowing the share to be acquired at the lower market price and then sold at the exercise price – the net profit being

(exercise price − market price) − cost of the premiums (50p);

� if the price remains at £5, neither of the options will be exercised, resulting in a net loss of 50p (the combined cost of the premiums);

� if the price rises above £5, the put would not be exercised but the call will be, allowing the share to be acquired at the exercise price and then sold on at the market price – the net profit being

(market price − exercise price) − 50p

This strategy allows profits to be taken if the asset price (share) rises or falls either side of the exercise price by an amount equal to the combined cost of the premiums (50p). It is, therefore, a suitable strategy where the investor considers that market volatility is likely to rise (either as a whole or in respect of the particular asset).

Interest Rate Options Option contracts may be used to hedge against fluctuating interest rates in a similar way to changing equity prices. They enable investors to set a floor or ceiling on interest rates at a later date, and yet still benefit by taking profits from the sale of the option prior to maturity.

An interest rate option gives the holder the right, but not the obligation, to fix a rate of interest on a notional loan or deposit of an agreed amount for a fixed term on a specific forward date.

Interest rate options may be standard exchange traded options – as, for example, those on LIFFE –or may be tailor-made away on the “over the counter” (OTC) market. Amongst the options available on LIFFE are options on interest rate futures such as three month sterling futures, three month eurodollar futures, three month Euro LIBOR futures and long gilt futures. Interest rate options on futures are now the norm on LIFFE because of their convenience, liquidity and flexibility. They are priced in a similar way to futures contracts.

400 420 480 500 520 540 560 580 600

Share price (pence)

100

80

60

40

20

0

−20

−40

Prof

it/lo

ss (p

ence

)

Long put

Pay-off profile for strategy

Long call

440 460

Page 356: libvolume5.xyzlibvolume5.xyz/law/ballb/3rdyear/semester5/corporatelaw/corporate... · ABE Advanced Diploma in Business Administration Study Manual CORPORATE FINANCE Contents Study

348 Managing Exposure to Risk – Financial Derivatives

© Licensed to ABE

The most common type of interest rate options offered on the OTC market are interest rate caps, floors and collars. The principal underlying each of these is based on the setting of interest rate limits which, if reached, trigger the exercise of the option, thus protecting the holder from any further adverse interest rate movements.

� In the case of a cap, the buyer receives compensation from the seller of the option if interest rates go above the capped rate.

� In the case of a floor, a lower limit to interest rates is set and an investor who purchases a floor will be compensated by the seller if interest rates fall below the predetermined limit.

� An interest rate collar consists of taking a position in an interest rate cap at the same time as taking a position in an interest rate floor. Where a hedge is required to protect against an increase in interest rates on borrowings, an interest rate cap is purchased along with selling an interest rate floor. The premium received from selling the floor is used to offset the premium paid on the cap, helping to reduce the overall cost of the hedge. The collar ensures, in this case, a minimum borrowing floor and a maximum borrowing rate. This is achieved by buying puts and selling calls.

Note that, as we have said before, interest rate futures lock in a particular interest rate and, as a result, the company is protected against adverse movements, but it cannot take advantage of any favourable movement. Interest rate options, on the other hand, enable the participator not only to protect against adverse movements of interest rates, but also to benefit from any favourable change in those rates.

Options are also available on interest rate futures and there are advantages to trading in option contracts on futures as opposed to the futures themselves. With a futures contract, both parties are required to deliver on the contract or must reverse the trade by closing out. With an options contract, the purchaser can let the option lapse if required and forgo the premium. This can, however, be expensive since the premiums can be high, especially if there is a great deal of volatility in the market.

Application of Option Theory to Capital Investment Decision making The theory underlying the operation of options is not only relevant to hedging and speculating, but is also relevant to the wider context of business. It lends itself to decision making in general, and capital investment decisions in particular.

There are many well established methodologies available to support business decision making – for example, capital investment decisions are invariably based on some form of discounted cash flow (DCF) analysis. However, many of these traditional methods do not provide the flexibility needed by management to adapt and revise decisions in response to changes in market conditions or technological developments. A knowledge and understanding of option theory and its application to decision making can provide a framework for this.

Companies are continually faced with decisions about the allocation of resources to competing opportunities. Decisions as to whether to invest now, invest at a future date, divest or do nothing underpin a series of other management decisions. Because each of these choices creates a different set of pay-offs, they can be thought of in terms of options, and option theory may provide a useful analytical tool.

In the context of capital investment decisions, there are a range of possible examples where this may apply.

Page 357: libvolume5.xyzlibvolume5.xyz/law/ballb/3rdyear/semester5/corporatelaw/corporate... · ABE Advanced Diploma in Business Administration Study Manual CORPORATE FINANCE Contents Study

Managing Exposure to Risk – Financial Derivatives 349

© Licensed to ABE

� The option to make follow-on investments if the immediate investment project succeeds

The fact that an organisation has undertaken the initial project will place it in an advantageous position for future investments and opportunities. This decision may be viewed as a call option on the assets which constitute the follow-on investment.

� The option to abandon a project

If it is seen that, once a project has been started, the returns are significantly less than was anticipated, then there may be a real possibility that the project should be abandoned. This decision may be viewed as a put option on the assets employed to date and which may be subsequently relieved or sold.

� The option to wait and see before investing

The option of delaying an investment might well be disadvantageous in that it may allow a competitor to take advantage of the opportunity. However, it may also be beneficial – waiting could allow time for new technologies and new operating skills to be exploited. The decision in these types of cases could be viewed as a call option on investment in the asset.

What is apparent, then, is that the price of the option is, in effect, valuing “flexibility” in the decision making process.

Discounted cash flow techniques are widely used to evaluate investment decisions, and they are entirely appropriate for evaluating the associated financial outcomes (in respect of mortgages and loans, etc.). However, the flow of outcomes from many such decisions is less than straightforward. In these cases, a greater degree of flexibility in approach is required and they lend themselves to a more proactive role in influencing the outcome.

For example, investors in the equity of a company are generally passive, depending on the degree of control they have over the firm. In effect, they are merely confined to monitoring the progress and performance of the business and exercising choice after the event. On the other hand, managers play a far more active role in influencing the final outcome. They have, so to speak, a range of options which, if exercised, could influence – and, crucially, allow advantage to be taken – of new opportunities. In this case, a DCF analysis, although valid as a tool in its own right, does not have the flexibility to handle many of the uncertainties faced by the decision maker.

Page 358: libvolume5.xyzlibvolume5.xyz/law/ballb/3rdyear/semester5/corporatelaw/corporate... · ABE Advanced Diploma in Business Administration Study Manual CORPORATE FINANCE Contents Study

350 Managing Exposure to Risk – Financial Derivatives

© Licensed to ABE

Page 359: libvolume5.xyzlibvolume5.xyz/law/ballb/3rdyear/semester5/corporatelaw/corporate... · ABE Advanced Diploma in Business Administration Study Manual CORPORATE FINANCE Contents Study

351

© Licensed to ABE

Study Unit 15

International Trade and Finance

Contents Page

Introduction 353

A. Theory and Practice of International Trade 353 Absolute and Comparative Advantage 353 Development of International Trade and Operations 356 Foreign Direct Investment 357 MNCs and World Trade 357

B. International Investment 358 The Investment Decision 359 International Investment Appraisal 360 Repatriation of Profits 361 Finance 361

C. Finance and International Trade 362 Terms and Methods of Payment 362 Finance for Exporters 363 Government and Quasi-Government Aid 369 Finance for Importers 369 Countertrade 371

D. Exchange Rates 372 Exchange Rate Concepts 372 Exchange Rate Systems 375 Influences on Exchange Rates 375

E. Risk and International Trade/Finance 379 Political or Country Risk 379 Foreign Exchange or Currency Risk 379

(continued over)

Page 360: libvolume5.xyzlibvolume5.xyz/law/ballb/3rdyear/semester5/corporatelaw/corporate... · ABE Advanced Diploma in Business Administration Study Manual CORPORATE FINANCE Contents Study

352 International Trade and Finance

© Licensed to ABE

F. Internal Methods of Managing Exchange Rate Risk and Exposure 380 Currency invoicing 380 Netting 381 Matching 381 Leads and Lags 382

G. External Methods of Managing Exchange Rate Risk and Exposure 382 Forward contracts 382 Currency swaps 383 Currency Futures 385 Currency Options 387 Money Market Hedge 388

Page 361: libvolume5.xyzlibvolume5.xyz/law/ballb/3rdyear/semester5/corporatelaw/corporate... · ABE Advanced Diploma in Business Administration Study Manual CORPORATE FINANCE Contents Study

International Trade and Finance 353

© Licensed to ABE

INTRODUCTION

The first part of this last study unit in your course considers international trade, focusing on the development of trading operations and the implications for such businesses in activities such as foreign investment and financing overseas projects.

We then move on to the major area of managing exchange rate risk. Firstly, here, we shall look at the workings of the foreign exchange market, examining the more practical aspects of the subject such as the different exchange rate systems and the influences on exchange rates. We then concentrate on the techniques and policies adopted by an organisation to minimise its exposure to risk.

A. THEORY AND PRACTICE OF INTERNATIONAL TRADE

International trade arises because of differences in the demand for and the supply of goods between countries.

Some countries may be unable to produce certain goods they require, or to produce them in sufficient quantities or in a cost-effective manner – for example, the USA is a net importer of oil. Conversely, on the supply side production of certain commodities, or the presence of certain commodities and resources, is unevenly distributed throughout the world – for example, oil in the Middle East. Such commodities/resources are often restricted to an area, and are relatively immobile between one country and another (especially natural resources and labour), but may be sold to those with a demand for the resource concerned. Countries thus tend to specialise in those areas in which they enjoy the greatest comparative advantage.

Absolute and Comparative Advantage It is easy to see why foreign trade is advantageous when a country can not, perhaps for reasons of climate or natural resources, produce particular goods which it desires but can purchase them from another country which can provide them. For example, the UK has no diamond reserves but has oil, whilst South Africa has no oil but has diamonds – thus both nations can benefit by exchanging one for the other. However, even countries such as France and Germany, which have many similar natural resources, still benefit from trading. This can be explained by the theory of absolute advantage.

(a) Absolute advantage

A country which is able to produce a good using less labour and capital than another country is said to have absolute advantage in the production of that good. Thus, we might consider that Germany has the absolute advantage over France in the production of high-performance cars (such as Porsche), but France has the absolute advantage in the production of inexpensive cars (such as Citroen). This means that due to the concentration of skilled labour and production facilities it is more efficient in terms of the usage of resources (minimising the capital and labour required) to produce Porsches in Germany and Citroens in France, and to import/export cars to meet the needs of the populations of the two nations. Such trade, by utilising the more efficient production methods, means that more goods will be produced from the fixed level of natural resources.

(b) Comparative advantage

In certain trading relationships (e.g. the USA and India) it can be shown that one of the countries has the absolute advantage in the production of all goods (the USA has a higher output per unit of labour and per unit of capital than India). However, specialisation of

Page 362: libvolume5.xyzlibvolume5.xyz/law/ballb/3rdyear/semester5/corporatelaw/corporate... · ABE Advanced Diploma in Business Administration Study Manual CORPORATE FINANCE Contents Study

354 International Trade and Finance

© Licensed to ABE

production and trade between countries can still be advantageous to both countries if each has a comparative cost advantage in the production of a good, and concentrates on the production of that good. Remember that when economists refer to comparative costs they are considering opportunity costs and not absolute costs – i.e. the production of other goods foregone by producing X rather than Y will differ between countries.

The concept of comparative cost advantage and how it may be applied to international trade can be explained through a very simple model based on two products (X and Y) each produced by two countries (A and B). Table 15.1 shows the production in tonnes per day of each of the products, achieved by packages of 10 factor (of production) units producing each product in each country – i.e. there are 40 factor units employed altogether:

Table 15.1: Comparative advantage – initial production (tonnes)

Production per Day of 10 Factor Units

Country A Country B Total

Product X 20 10 30

Product Y 30 20 50 Country A is more efficient in the production of both X and Y. It has an absolute cost advantage for both products. However, suppose one factor unit (assuming each unit in each package is equally efficient) is moved in A from producing X to Y, then this would result in the loss of 2X and a gain of 3Y. In other words:

� in Country A, the cost of a gain of 1Y achieved by factor movement is 32 X; but

� in Country B the same gain of 1Y achieved in the same way costs ½X.

Measured in terms of X, therefore, Y costs more in Country A than in Country B.

If we consider the results of other possible movements in the two countries we can produce the following analysis:

� Country A

Movement of one factor unit from X to Y loses 2X and gains 3Y, so to gain 1Y costs 2

3 X.

Movement of one factor unit from Y to X loses 3Y and gains 2X, so to gain 1X costs 2

3 (or 1½) Y.

� Country B

Movement of one factor unit from X to Y loses 1X and gains 2Y, so to gain 1Y costs ½X.

Movement of one factor unit from Y to X loses 2Y and gains 1X, so to gain 1X costs 2Y.

So in Country A, 1Y costs 23 X and 1X costs 1½Y, whereas in Country B, 1Y costs ½X and 1X

costs 2Y. Thus:

� X is cheaper in A measured by its opportunity cost in Y; and

� Y is cheaper in B measured by its opportunity cost in X.

Page 363: libvolume5.xyzlibvolume5.xyz/law/ballb/3rdyear/semester5/corporatelaw/corporate... · ABE Advanced Diploma in Business Administration Study Manual CORPORATE FINANCE Contents Study

International Trade and Finance 355

© Licensed to ABE

We can thus say that A has a comparative cost advantage in X and B has a comparative cost advantage in Y. When we take our measurements in opportunity cost, we see that although B has an absolute cost disadvantage in both products, it has a comparative cost advantage in one.

Let us try moving some factors in A from producing Y to producing X, where the country has a comparative cost advantage. Suppose we move 5 factor units, i.e. half the available package. This increases production of X by half to 30 tonnes per day, whilst it reduces the production of Y by half to 15 tonnes per day. We have gained in X but lost in Y. Suppose we make up this loss of Y by transferring factors in B, say transfer 8 factor units from X to Y. Production of X will fall by 8/10ths and of Y will rise by 8/10ths. We can now produce Table 15.2 showing the position after the movement of factor units.

Table 15.2: Comparative advantage – adjusted production (tonnes)

Production per Day of 10 Factor Units

Country A Country B Total

Product X 30 2 32

Product Y 15 36 51 Comparing Table 15.1 with Table 15.2, we see that production of X has risen from 30 to 32 tonnes per day, whilst production of Y has risen from 50 to 51 tonnes per day as a result of switching 5 factor units from Y to X in Country A and 8 factor units in Country B. If both X and Y are freely traded between the two countries then both gain as a result of the specialisation and trade. If the opportunity cost ratio for two goods were the same in two different countries then there would be no benefit from trade.

The belief that comparative cost leads to a growth in total production and trade and a more efficient use of scarce resources in the world has long been an argument in favour of free trade; the argument is that countries should specialise in those goods for which they have a comparative advantage and that all goods should be freely traded on an international scale. However, we need to be aware of the assumptions on which the assumed benefits are based:

� The factor units are transferable from one activity to the other.

� All factor units are equally efficient and there is no loss of efficiency after transfer – the newly transferred units produce just as much as those already operating in the activity.

� There is no unemployment either before or after the transfer, i.e. all units of one product have an opportunity cost in terms of the other.

� There is no transfer of resources from one country to the other. Clearly, the greatest gains would be achieved by transferring all factors from Country B to Country A where they could all produce more. If the comparative advantage is caused by superior management, then gains could be achieved by transferring management ability from A to B.

We have to recognise that, in the modern world:

� There is specialised production and factor immobility within countries. Factors cannot always be transferred from one activity to another, e.g. shipyard welders do not always make good computer programmers.

� There is unemployment and the opportunity cost of employing factors with no alternative employment is nil.

Page 364: libvolume5.xyzlibvolume5.xyz/law/ballb/3rdyear/semester5/corporatelaw/corporate... · ABE Advanced Diploma in Business Administration Study Manual CORPORATE FINANCE Contents Study

356 International Trade and Finance

© Licensed to ABE

� Multinational companies can and do transfer factors from one country to another. They can change the absolute cost of production by transfer of capital and managerial skill.

� There is no free trade, even without political controls, because modern international markets are becoming oligopolistic on a world scale.

� High transportation costs can reduce or even cancel out the advantages of international trade and specialisation.

As a result of these changes we have to be very cautious in applying the concept of comparative cost advantage to modern international trade, although there are examples of it applying in practice and countries specialising in the production of certain goods.

Development of International Trade and Operations The most common form of international trade remains exporting, either directly into a foreign market or indirectly using agents and distributors based in that market. There are, though, limitations on export opportunities – there may be restrictions such as tariffs and quotas imposed on exporting by the recipient country, and also, on occasions, from the home government itself (as, for example, on arms sales to certain countries). In addition, the logistics of exporting certain products, such as bulky machinery, may mean that it is more economical to produce the goods closer to the markets.

Developments whereby companies get increasingly involved with the foreign market include:

� Management contracts, whereby a company sells its management expertise to an overseas company in exchange for a level of fees, often in conjunction with a licensing agreement, involve minimum investment.

� Licensing, which may be used when penetration of the overseas markets needs to be fairly rapid, where the political risk is high, and when the company does not wish to invest large amounts of money. It may also be used when it is difficult to invest directly in a country or the remittance of funds to the parent is difficult. There are, however, disadvantages to licensing – quality may suffer, profits may not be very high and there is the risk of producing a competitor both in the overseas market and (via export) in other markets.

We saw in earlier study units that there has been an increasing globalisation of capital markets, allowing organisations to raise and trade capital relatively easily in overseas countries. This globalisation has allowed a significant increase in foreign direct investment (FDI). These opportunities for FDI have allowed a significant growth in the development of multinational companies.

There is no standard definition of a multinational company (MNC), but a working definition is:

“an enterprise which owns or controls subsidiaries, service or production facilities, in countries outside of that in which it is based”.

The basic form of MNCs is a parent or holding company and several (wholly or partly) owned subsidiaries and sub-subsidiaries.

You should note that it is control and/or ownership that is important in determining whether a company is an MNC – a company which merely imports/exports goods and/or services is not a multinational company. Where only a few countries are involved in the firm’s operations, the company is generally referred to as an international company, but when its operations span the globe it is typically referred to as a multinational company. The countries in which the subsidiary or facilities are based are known as host countries.

Page 365: libvolume5.xyzlibvolume5.xyz/law/ballb/3rdyear/semester5/corporatelaw/corporate... · ABE Advanced Diploma in Business Administration Study Manual CORPORATE FINANCE Contents Study

International Trade and Finance 357

© Licensed to ABE

MNCs are growing in number and size, with increases in output nearing 20% per annum. Whilst there are some medium-sized MNCs, there are also some very large companies (e.g. Ford) which have GNPs larger than several countries. Multinational companies include the Western world’s largest companies; their operations span the globe and they generate the bulk of international trade.

Foreign Direct Investment There are several reasons why a company may grow via FDI and become a multinational, and often in making the decision financial considerations are outweighed by strategic reasons. The common reasons quoted for FDI are listed below, but you should note that there is often more than one reason why a firm may decide to invest overseas.

� The provision of raw materials (e.g. oil, minerals) – many MNCs base part of their operations in the location of their raw materials, for cost or logistic reasons, or in order to comply with the political and legislative wishes of the host government.

� The provision of cheap and productive sources of labour is a reason given for many MNCs locating in the Far East and Mexico.

� Location in the market for the company’s goods, e.g. several Japanese car firms have located in Europe in order to supply the EU markets.

� Location in centres of knowledge, e.g. several Japanese and European companies have purchased firms in the US in order to gain access to technological knowledge in the field of electronics. An example of a firm investing in Britain for this reason are the plans of Microsoft to establish facilities in Cambridge.

� To permit diversification opportunities not available in the country’s domestic markets.

� To allow growth if there are limited opportunities for the firm “at home”; such growth can take the form of diversification, or horizontal or vertical integration.

� For companies based in countries with unstable or unpredictable political regimes FDI may be a way of ensuring safety from interference in, or expropriation of, their business. Such fears led to FDI in Australia and North America by Hong Kong-based firms prior to its repatriation by the People’s Republic of China.

The criteria by which investment opportunities are selected will vary between companies. MNCs often prefer to invest in their own domestic market, and they will only go abroad if they can secure a higher rate on capital by doing so.

Sometimes the MNC will undertake a foreign investment that is uneconomic, the main reasons for this being:

� To ensure an outlet for some other aspect of its worldwide operations (e.g. an oil company may construct a refinery as an outlet for its own crude oil production).

� To safeguard its existing interest (e.g. by producing a new but uneconomic car, in order to keep a brand name alive for the sake of a larger, successful model).

Studies indicate that many MNCs do not have a master plan for international investment but review each individual project on its merits. In many cases it is generally the viability of the project, rather than the finance available for investment, that is the key to the investment decision.

MNCs and World Trade Traditional economic theory insists that international trade arises from bargaining between independent traders in different countries with a buyer in one doing business with the seller in another.

Page 366: libvolume5.xyzlibvolume5.xyz/law/ballb/3rdyear/semester5/corporatelaw/corporate... · ABE Advanced Diploma in Business Administration Study Manual CORPORATE FINANCE Contents Study

358 International Trade and Finance

© Licensed to ABE

The arrival of the MNC distorts this theory as an ever larger proportion of international trade results from decisions taken at the nerve centres of these large industrial empires.

This is of increasing concern to governments for a number of reasons, including:

� Governments fear for their national interest which may be sacrificed for that of another country.

� Governments will be anxious about their own share of an expanding company’s investment programme.

� Governments will be anxious when there is a cutback in the investment in, and the business in, their own country.

MNCs are continually anxious to secure a high and ever-increasing level of new industrial investment. Countries and governments want to be on the receiving end of the MNCs’ investment, and compete for the large investment funds which MNCs can move around the world.

As a general rule, the subsidiaries of an MNC will be expected to earn a profit for their parent organisation and their financial policies and planning will be based on the objectives and interests of their parent. In theory each subsidiary should be capable of financing itself, whilst continuing to make a profit for the parent company. Of course this may not always be possible. New operations in a foreign country may mean that the company may not generate a satisfactory return in the early years. In such circumstances the parent may have to support the company by the provision of cash or guarantees.

Countries which receive foreign investments have to accept that foreign subsidiaries of MNCs will become net exporters of cash in the longer term. One of the main benefits to the host country, beyond the creation of jobs, arises from the payment of taxes and customs duties. Generally, too, the level of industrial investment will be increased, and the balance of payments of the host country will be improved as the MNC starts to export to other parts of the world.

Benefits are not always one way. Decisions to transfer reserves out of the host country may not be in its best interests and there are many examples where limits are placed on the import of capital and the sending home of high proportions of their foreign earnings. In addition, host countries may establish trade policy instruments which aim to safeguard the national interest – such as anti-dumping measures and other environmental policies.

These actions can serve to distort competition and restrict trade at both the national and international level, when applied contrary to competitive principles, and there is a careful balance to be struck between abuse of the host country’s interests and the ability of companies to develop their international trading realistically.

Note, though, that the actions of MNCs themselves can place restraints on competition. Practices such as orderly marketing, managed trade, export cartels, mergers to attain commanding market positions and displacement strategies can lead to serious trading barriers, in themselves serving to undermine the value of international trading agreements and concessions. This can generate political pressure to react to such competitive distortions by national trade policy measures which also serve to undermine international agreements.

B. INTERNATIONAL INVESTMENT

There are different forms of overseas investment available to an MNC – the choice made will depend on the reasons for the FDI (to extract the maximum possible profits from the subsidiary and as quickly as possible, or to develop a lasting presence in the country to the mutual benefit of all parties) and the options available in the country (there may be restrictions on the type of investment allowed and the

Page 367: libvolume5.xyzlibvolume5.xyz/law/ballb/3rdyear/semester5/corporatelaw/corporate... · ABE Advanced Diploma in Business Administration Study Manual CORPORATE FINANCE Contents Study

International Trade and Finance 359

© Licensed to ABE

level and type of funds permitted to be repatriated by the parent company imposed by the political regime in a country). The level of risk (see later) will also have a major impact on the company’s decision.

The most common forms of investment abroad are:

(a) The takeover of, or merger with, a firm already established in the target country. This option has the same advantages and disadvantages as a “domestic” takeover – established markets, production and distribution facilities, but often poor performance, financial status and management.

(b) Joint ventures with an overseas partner local to the area where investment is to take place, either for a fixed period for co-operation on a defined number of projects, or a continuing long-term joint-equity venture. Joint ventures are common in the Middle East and Japan where legislation prevents or makes difficult 100% foreign ownership of firms. It is also becoming increasingly popular in areas with high research and development costs, e.g. the aerospace and car industries.

(c) Firms may start up overseas subsidiaries or branches from scratch; again this route reflects the same advantages and disadvantages as domestic start-ups with the additional problems/opportunities that may occur as a result of differences between the two countries concerned.

The parent company will aim to achieve a cash return from overseas subsidiaries. There are several ways of achieving this:

� By dividends from subsidiaries

� By the parent company selling goods and services to the subsidiary

� Making a loan to a subsidiary at a high interest rate

� Levying charges on the subsidiary for management services provided by the parent company

The Investment Decision Points to be considered by a company prior to establishing new operations in an overseas territory include the following:

� Will the investment be temporary or permanent? For example, a temporary investment, such as a mine, will be worked out within a finite period of time. In such cases a loan will generally be the best method of finance, whereas for permanent investment equity will usually be the best approach.

� Is it best to operate as a branch or should a separate local company be created? The answer to this question will depend on local considerations, including the tax laws of the country concerned, as well as strategic policy regarding decentralisation of control of subsidiaries.

� How difficult will the repatriation of funds be in the future?

� Where borrowing is envisaged, should funds be borrowed locally, provided through the parent or raised through the Euromarkets, and what exchange risk (see later) will be involved?

� What is the cheapest and safest way of providing capital?

� Would it be politically wise to have a local participation in the chosen country?

Page 368: libvolume5.xyzlibvolume5.xyz/law/ballb/3rdyear/semester5/corporatelaw/corporate... · ABE Advanced Diploma in Business Administration Study Manual CORPORATE FINANCE Contents Study

360 International Trade and Finance

© Licensed to ABE

The decision to locate, or relocate, part of an MNC’s operations in a foreign country will follow detailed research and often complex negotiations. Of the issues which will primarily concern the management team, the following will be of particular significance:

� The political, economic and currency stability of the country.

� The availability and cost of suitably qualified labour and management, and the industrial relations record of the country.

� Support, assistance (and interference) of, and by, the government of the country.

� Communication facilities.

� Controls on the movement of products and currencies.

� The ability to acquire an established firm or whether a “green field” operation can be developed.

� The fiscal and international policy of the country.

� The level of competition already established in the region.

� The stability of the market for raw materials.

� Inflation and local taxes.

Entities trading internationally face particular difficulties based on different currency units which can cause potential problems in translating one unit of currency into another. They also have the problem of different laws, taxes, business practices and cultures which may be incompatible with existing operating methods.

International Investment Appraisal Investment appraisal in an MNC uses NPV, IRR or (occasionally) APV techniques, which we discussed earlier. However, different factors may have to be considered to reflect the differing nature of the investment:

� An increase in (unsystematic) risk caused by currency and political risk (see later) may lead to the firm increasing the discount rate used to evaluate projects; or the firm may use the discount rate determined by its overall systematic risk level (if it is assumed to be unchanged by the potential investment) and adjust the cash flows for the expected political and currency risk. In practice the discount rate is adjusted for political risk – because all cash flows would be affected by adverse political circumstances, whereas currency risk may have some beneficial, and some detrimental, effects on cash flows and as such is accounted for by adjusting cash flows.

� The different rates of inflation between the host country and that of the parent need to be considered.

� The different methods of repatriating cash flows (e.g. royalty fees, dividends) must be brought into consideration.

� The project must be shown to be beneficial both in the host country and its currency and in terms of funds remitted to its parent in order to fully justify it, both from the point of view of the parent company and in comparison with other (potential) projects in the host country.

� There may be financial incentives offered to the firm to encourage investment in a particular country.

Page 369: libvolume5.xyzlibvolume5.xyz/law/ballb/3rdyear/semester5/corporatelaw/corporate... · ABE Advanced Diploma in Business Administration Study Manual CORPORATE FINANCE Contents Study

International Trade and Finance 361

© Licensed to ABE

� Differences in accounting practices and tax systems in the host country may have a significant impact on the investment appraisal.

Taking into account the above points, an exercise to consider investment in an overseas project is the same as for a domestic one, with the added complication that cash flows have to be converted from the host currency to the domestic one.

Repatriation of Profits The aim of investment in overseas subsidiaries is to increase group profits for the MNC. In common with all groups, those companies with inter-divisional trade need to determine the level at which to set transfer prices for goods and services provided by one group member for another. The basis on which the transfer price is set – cost (either marginal or full), market price or some negotiated level – will affect the profit share of the group, and should be determined in order to maximise group profits by maximising the motivation of subsidiaries and circumventing any repatriation controls imposed by the host government.

Transfer prices of goods and services provided by group members for each other are only one way of obtaining cash returns from an overseas subsidiary; others include:

� Royalties charged to the subsidiary for making goods, or providing services, for which the parent holds the patent.

� Management charges levied in respect of services provided by “head office”.

� The subsidiary may borrow from its parent, and thus pay interest charges to it.

� Dividends which can be paid to the parent on the equity provided by it.

The choice of method, and level received from each (often by manipulating the various factors, e.g. the rate of interest for parental loans), will be determined by the requirements of the parent and the subsidiary, any exchange controls present, and the perceived risk of the investment.

This manipulation can make the interpretation of the accounts of MNCs and their subsidiaries extremely difficult. This problem is exaggerated by the differing accounting policies used in different countries; the differing choices made as to which exchange rates (actual (and at what date) or predicted) to use in setting forecasts and translating accounts into the parent company’s currency; and the different economic and political circumstances a subsidiary may be operating in.

Internal evaluation (i.e. by the companies themselves) tends to be most effective if based on budgets with operational management held responsible for those factors which they can control, including exchange rate variances which they should have been able to predict reasonably accurately. A common method of control in MNCs is to show a comparison against budgets in the parent company’s domestic currency to allow comparison between group members, but showing exchange rate variances separately.

Finance MNCs will often obtain their funds from the international markets, although working capital needs of individual subsidiaries may be raised locally. Because of the scale of operations there is rarely any difficulty in raising funds, but there may be some local limitation to the amount of borrowings which the host country is prepared to allow.

Eurobonds and Eurodollars which we discussed earlier in the course both have relevance to the international operations of a typical MNC. The borrower must be of high credit standing and must need large sums of money, as both markets are for the international gathering of capital resources, and are not aimed at the smaller organisation.

Page 370: libvolume5.xyzlibvolume5.xyz/law/ballb/3rdyear/semester5/corporatelaw/corporate... · ABE Advanced Diploma in Business Administration Study Manual CORPORATE FINANCE Contents Study

362 International Trade and Finance

© Licensed to ABE

MNCs often use international sources of funds, e.g. Eurobonds or overseas capital markets, to finance both themselves and their subsidiaries. A common method of financing an overseas subsidiary is to finance its fixed assets with a long-term loan in the host country’s currency, which permits the repayment of the loan using the profits generated in that country. This is known as the matching of assets and liabilities, and is a method used to reduce exchange rate risk (see later).

In deciding the financial structure of an overseas subsidiary, the parent needs to consider the level of gearing the subsidiary should have and from what source; how much equity should be placed by the parent in the subsidiary and how much from outside (and from what sources); and the level of reserves and working capital the subsidiary should aim for. These choices will be determined by political and legal restrictions in the parent’s country and the host country and the expected level of permanence of the investment (for a longer-term investment long-term sources of funds such as equity would be used in preference to short-term funds such as trade credit – which would be used for a short-term investment).

Moreover the firm, in common with all organisations, needs to consider the cost of capital which we discussed in depth earlier in the course. Whilst the factors affecting a domestic firm when deciding on its capital structure also need to be considered by an MNC, the decision is made somewhat harder by the greater choice of capital available from international capital markets, taxation and other legal restrictions in the various countries in which the MNC operates.

C. FINANCE AND INTERNATIONAL TRADE

Working capital finance requirements for overseas trade are likely to be greater than for solely domestic trade because of transport time, administrative delays and perhaps longer credit terms (90 days from shipment or 60 days from receipt).

Before considering financing methods themselves, it will be useful to briefly review the terms under which international trade transactions conducted.

Terms and Methods of Payment The most common form of settlement for the cost of a trading transaction is by means of a bill of exchange (also called trade bills). This occurs when the seller draws a bill on the buyer asking them to pay, on a certain future date, the price of the goods supplied, which is then accepted by the purchaser (by signing and returning it to the seller). The purchaser is thus formally acknowledging his debt to the seller. The seller can then use the bill of exchange as security in order to obtain money from the seller’s bank.

A bank may also agree to accept a bill from its customer in exchange for an agreement that the customer will repay the bank. The cost for arranging this finance is the discount (i.e. the full amount of the bill is not advanced). The more secure the bill (e.g. from a bank as compared to a trader) the “finer” or lower the discount.

Note that a bill of exchange is a method of facilitating payment and could therefore be used in several of the terms of payment we consider below. A bill of exchange may also be termed a draft.

There are a number of different ways in which payment for transactions may be effected.

(a) Open account

The exporter ships the goods and any documents of title direct to the importer. Payment is made by the importer in accordance with invoice terms, the exporter bearing the risk of non-payment.

Page 371: libvolume5.xyzlibvolume5.xyz/law/ballb/3rdyear/semester5/corporatelaw/corporate... · ABE Advanced Diploma in Business Administration Study Manual CORPORATE FINANCE Contents Study

International Trade and Finance 363

© Licensed to ABE

(b) Documentary collection

The exporter ships the goods and sends the documents of title through the banking system. There is a collection order which instructs the overseas bank regarding release of documents to the buyer. The exporter can instruct that the documents are either released against payment or against acceptance.

Open account trading status reports should be taken on the buyer, and insurance can also be taken out, if required.

(c) Documentary letters of credit

A documentary credit is a guarantee by the buyer’s bank (the issuing bank) that bills of exchange drawn by the exporter will be honoured, provided the credit terms have been fulfilled.

If the credit is irrevocable, it can only be modified or cancelled with the agreement of all parties.

Confirmed credits are ones which contain the additional guarantee of a bank in the exporter’s country to honour them, should the issuing bank default.

Documentary credits have an additional security over documentary collections. Banks deal in documents not in goods and, as the seller must comply with the instructions issued by the buyer’s bank, the seller will check those instructions before shipping the goods and, if he can comply with the instructions, he will get his money. Failure to comply with the instructions as detailed will mean that the seller’s bank must refer to the buyer’s bank and get permission to effect payment.

(d) Advance payment terms

The most advantageous method of payment from an exporter’s point of view is to receive cash for his goods before shipment. This method affords the greatest protection and allows the exporter to avoid tying up his own funds. Although less common than in the past, cash payment upon presentation of documents is still widespread.

Cash terms are used where there is political instability in the importing country or where the buyer’s credit is doubtful. In addition, where goods are made to order, prepayment is usually demanded, both to finance production and to reduce marketing risks.

Finance for Exporters Delays in receipt of payment for goods sold overseas can seriously affect a company’s cash flow, eventually reducing profitability. Banks and other organisations have therefore developed a wide range of finance facilities to assist exporters in financing their international business. Export credit can be split into two categories.

� Supplier credit – where the exporter sells goods to an overseas buyer on credit terms (for example, 30 days) and then obtains finance from a bank to cover the period of time between shipping the goods and receiving payment.

� Buyer credit – where the bank provides finance directly to the overseas buyer and the exporter receives payment upon shipment of the goods.

Payments made by banks in respect of various export finance schemes are paid either:

� With recourse – where the bank has the right to claim reimbursement for sums advanced to the exporter, in the event that the buyer does not pay.

Page 372: libvolume5.xyzlibvolume5.xyz/law/ballb/3rdyear/semester5/corporatelaw/corporate... · ABE Advanced Diploma in Business Administration Study Manual CORPORATE FINANCE Contents Study

364 International Trade and Finance

© Licensed to ABE

� Without recourse – where the exporter is not liable to repay finance received from a bank, if the buyer defaults.

If the exporter is cash-rich, he may be able to finance export sales from his existing bank balances. However, outlay can be considerable and, if credit terms are allowed to the buyer, the cost of raw materials, manufacturing and shipping will not be recouped in the form of the buyer’s payment for some time, and additional funding may be required. The following are the normal method of obtaining such funding.

(a) Bank overdraft

Probably the easiest way of financing export sales is by use of an overdraft facility agreed with the exporter’s bankers, though exporting companies are unlikely to use this method to finance all their exports, since other forms of finance which are specifically designed for export credit are available at lower cost.

(b) Advance against bills

This is short-term, with-recourse, finance obtained by an exporter who draws a bill of exchange, under the terms of the export contract, on the overseas buyer. The exporter presents the bill of exchange to the bank, which advances an agreed percentage of the face amount of the bill to the exporter and undertakes to present it to the buyer for collection. The bank charges a fee for this service, together with interest at a variable rate for the period of the advance.

(c) Negotiation of bills

This means that the bank buys the bill from the customer. The customer receives the face amount of the bill immediately. The bank sends the bill of exchange and the related shipping documents to the buyer’s bankers for collection and reimburses itself upon receipt of the proceeds, at the same time recovering its collection charges and interest for the period involved.

A negotiation facility must be specifically agreed with the exporter’s bankers, and funds made under this facility are on a with-recourse basis. Recourse is available to the banker upon dishonour of the bill, the charges and interest for this being fixed at the time of negotiation.

(d) Discount of a bill

Banks are prepared to discount bills of exchange which can be either:

� Drawn by an exporter on a buyer and accepted by that buyer; or

� Drawn by an exporter on a bank, under a letter of credit, and bearing a bank acceptance.

Bills are discounted with recourse to the customer and the discounting bank pays the face value of the bill less the discount charge which depends partly on the length of time the bill has to run to maturity and partly on the rate of discount which is usual for that type of bill. Finer rates are available for bills bearing a bank acceptance than for those accepted by an unknown or doubtful buyer.

(e) Acceptance credit facility

This is a facility offered by banks for large companies with a good reputation. The company draws bills of exchange on the banks, generally for 60, 90 or 180 days, denominated in whichever currency most matches the needs of the company. The bills can be drawn on, as and when required, throughout the length of the agreement, which can be up to five years, provided the credit limit is not exceeded. The bill is then sold in the discount market and the proceeds passed to the company (less the bank’s commission). At maturity the company reimburses the bank the full value of the bill, and the bank pays the holder of the bill.

Page 373: libvolume5.xyzlibvolume5.xyz/law/ballb/3rdyear/semester5/corporatelaw/corporate... · ABE Advanced Diploma in Business Administration Study Manual CORPORATE FINANCE Contents Study

International Trade and Finance 365

© Licensed to ABE

A major advantage of acceptance credits is that they can be sold at a lower discount than trade bills. The cost of them is also fixed, allowing for easier budgeting and may be lower in times of rising interest rates than that of an overdraft. The credit is also guaranteed for the length of the agreement, which is not the case with an overdraft.

Where goods are involved the bank generally has control over the documents and the goods.

An acceptance credit facility is normally only used by larger companies. Be careful not to confuse it with a documentary acceptance credit.

(f) Documentary acceptance credit

When an exporter presents documents under a confirmed irrevocable letter of credit to the confirming bank, he can obtain immediate finance, provided the documents comply with the terms of the letter of credit. The confirming bank will accept a term bill of exchange which can be discounted by the confirming bank, or the exporter can regain possession of the accepted bill and discount it with any bank for cash. Discount fees are paid by the exporter unless, under the terms of the letter of credit, the applicant/overseas buyer is responsible for such costs.

(g) Merchant bank finance

A merchant bank could provide most of the facilities already mentioned but, in addition, it can offer an accepting house acceptance facility. The exporter again hands over the documents as collateral security and draws a second bill on the accepting house for up to, say, 75% of the collection value and with a tenor slightly longer than the export bill, to allow receipt of proceeds before the accommodation bill matures. The merchant bank accepts the accommodation bill and discounts it in the market. For protection against risks, the merchant bank would expect not only to have control of the bill and the documents it is handling for collection, but also additional safeguards such as insurance cover.

(h) Factoring

Factoring is a without recourse form of finance and the factor will only enter into an agreement with an exporter after satisfactory reports have been obtained as to the exporter’s standing, the reliability of the overseas buyer, and trading conditions in the foreign country.

There can be many advantages for the exporter in employing a factor to deal with his debt collection:

� Credit risk is eliminated under the non-recourse agreement.

� There is no need for credit and political risk insurance.

� There is no need to take out forward exchange cover.

� Immediate financing is available on approved invoices, if required.

� There are no losses through bad debt.

� There is a reduced staffing requirement, since accounting, debt collection and the sales ledger are handled by the factor.

� It gives the exporter the opportunity of trading on “open account” terms, but with the security of also using more traditional instruments, such as letters of credit and bills of exchange.

� There is no need for any other source of status reports and credit information.

� Experienced credit managers are on hand, whose knowledge of language, local laws and trading customs are invaluable. A factor with established contacts is able to assess

Page 374: libvolume5.xyzlibvolume5.xyz/law/ballb/3rdyear/semester5/corporatelaw/corporate... · ABE Advanced Diploma in Business Administration Study Manual CORPORATE FINANCE Contents Study

366 International Trade and Finance

© Licensed to ABE

foreign buyer’s creditworthiness more easily and thoroughly than the exporter can from his own sources.

However, the disadvantages of factoring must also be considered:

� Costs – these vary, depending upon the extent of the service required by the exporter: administration of the exporter’s exports sales ledger; credit protection; financing.

� A factor is selective in choosing clients and debts.

� A factor may set an overall turnover limit for the exporter.

� A factor may set a limit on the amount owing at any one time by any one buyer.

� Terms may be limited to 120 days.

Factors lend or provide finance against debts which are already approved on the strength of the creditworthiness of the overseas buyer. The exporter’s own bankers may be prepared to effect an introduction to a factoring subsidiary of the bank. An exporter considering employing the services of a factor should always consult his bankers, since factoring can affect the value of a lending banker’s security.

(i) Export house finance

Export houses can be grouped into:

� Export merchants

These buy goods in their own right from suppliers and export them to their own buyers abroad for cash – usually within seven days. A merchant can therefore eliminate credit risk for the exporter, and transform the deal into the equivalent of a domestic cash sale.

� Export agent

An export agent acts as agent for the exporter, and the contract relationship between the buyer and seller is maintained. The exporter receives payment from the export agent upon shipment of the goods, and the overseas buyer is allowed a period of credit by the agent, which is provided from the agent’s own resources.

� Confirming house

A confirming house acts as agent for the overseas buyer, places an order with the exporter, and accepts a usance bill from the buyer which can be discounted at a fine trade bill rate. The buyer therefore receives a period of short-term credit, and the exporter need not be concerned with credit risk , since this is the equivalent of a domestic sale.

� Export finance house

This will provide non-recourse finance to the exporter under the terms of the export contract and agree credit terms to the buyer. Like the factoring company, it will deal with obtaining a credit assessment of the buyer and will relieve the exporter of the need for credit/risk insurance but it can arrange such, if required, or accept an assignment of a policy, as the banks do.

(j) Instalment finance

Some finance houses can arrange hire-purchase finance, covering a wide range of consumer and capital goods, through a network or credit union of associates in both buyers’ and sellers’ countries.

Page 375: libvolume5.xyzlibvolume5.xyz/law/ballb/3rdyear/semester5/corporatelaw/corporate... · ABE Advanced Diploma in Business Administration Study Manual CORPORATE FINANCE Contents Study

International Trade and Finance 367

© Licensed to ABE

The exporter will gain satisfaction from the arrangement (which is without recourse to himself), as he receives immediate payment, while the buyer receives deferred terms under a hire-purchase agreement. It is a relatively costly plan, and it may not work where there are exchange control restrictions or other monetary regulations.

(k) Leasing

An exporter sells the equipment to a leasing company, which then leases it to an overseas hirer. The exporter receives payment without recourse from the leasing company – usually after the equipment has been shipped and installed at the hirer’s premises.

There are two main types of lease seen in the international context:

� Cross-border leases, which are made directly from the leasing institution (often subsidiaries of major banks) in the exporter’s country to the overseas buyer.

� Local leasing facilities, which may be available, perhaps, through overseas branches or international leasing associations.

Both types of arrangement may be eligible for insurance cover. Where contracts are arranged between the leasing company and the foreign buyer, the insurance will be undertaken by the lessor. Here the exporter will have no risk, having sold the goods direct to the leasing company. In those cases where the exporter arranges his own leasing deals direct with the foreign buyer, he will himself be able to obtain cover.

(l) Forfaiting

Forfaiting is a means of providing exporting companies with trade finance on a without recourse basis, while their overseas buyer acquires a period of credit of up to seven years.

When forfaiting an exporter is giving up the right to claim payment for goods delivered to an overseas buyer. These rights are surrendered to the forfaiter (normally a bank, finance house or discount house) in return for cash payment at an agreed rate of discount.

Any type of trade debt can be forfaited and these debts can be in any form but they are usually either:

� A bill of exchange accepted by the buyer, or

� A promissory note issued by the buyer.

The forfaiter will calculate the discount rate, taking into account:

� The currency used

� The buyer’s credit rating

� The credit-risk factor for the buyer’s country

The discount rate is calculated as a margin above prevailing eurocurrency market rates for the period of credit and it varies in line with those rates, since this is the main source of funds which forfaiters tap to provide finance to exporters. Trade paper is discounted in any fully convertible currency, although US dollars, Swiss francs and Deutschmarks are usual, since these are the main eurocurrency market currencies.

A forfaiter will not finance a trade debt without the guarantee of a known international bank, although finance will be considered in respect of bills of exchange accepted, or promissory notes issued, by a first-class buyer, such as a government agency or a major multinational company.

Page 376: libvolume5.xyzlibvolume5.xyz/law/ballb/3rdyear/semester5/corporatelaw/corporate... · ABE Advanced Diploma in Business Administration Study Manual CORPORATE FINANCE Contents Study

368 International Trade and Finance

© Licensed to ABE

There are two main forms in which the debt instrument may be secured:

� The “Aval”

This is an irrevocable and unconditional guarantee to pay on the due date. The words “pour aval”, together with the bank’s signature, are written directly onto the bill or the promissory note and thus the bank becomes the debtor as far as the forfaiter is concerned.

The aval is not legally recognised in some countries, including England. However, this does not appear to have caused practical difficulties and, in fact, most forfaiters, including those in the UK, prefer the aval to a separate letter of guarantee.

� A Guarantee

This is a separate document whereby the guarantor undertakes to pay bills of exchange or promissory notes on their due dates. The guarantee must not, of course, refer to the underlying commercial transaction, since the forfaiter, having bought the debt instrument without recourse, will not wish to find payment is withheld because of a dispute between exporter and importer as to the goods or services supplied.

The aval or the guarantee is the forfaiter’s security which effectively eliminates any commercial risk.

The forfaiter may hold the bill or note until the maturity date, which is known as primary forfaiting, or sell it to another institution in what is known as the secondary forfaiting market.

A separate guarantee is generally less favoured than an aval, since it involves more work for the forfaiter and its transferability can cause problems.

Advantages of forfaiting for the exporter include:

� Forfaiting offers 100% finance on a without recourse basis and at a fixed rate, thereby enabling the exporter to build finance costs into the contract price. Finance is off-balance sheet, thus preserving existing bank credit facilities.

� In addition to removing interest risk, forfaiting also eliminates exchange, credit, political and transfer risks.

� Forfaiting is flexible, there being no distinction between types of goods and services and no constraints on origin.

� Forfaiting finance can be arranged very quickly, and documentation is brief and relatively simple.

� Forfaiting transactions are rarely published, and this aspect of confidentiality is often attractive to exporters.

� The ability to offer forfaiting in a tender may be necessary in order to remain competitive.

Disadvantages of forfaiting for the exporter include:

� Despite the greater degree of competition among financial institutions for forfaiting business, which has brought interest margins down, forfaiting tends to be relatively expensive.

� Forfaiting is generally limited to the major currencies, and forfaiters will not accept countries where too great a risk is perceived. Similarly, the forfaiting institutions will only accept the aval of a limited number of banks considered suitable.

Page 377: libvolume5.xyzlibvolume5.xyz/law/ballb/3rdyear/semester5/corporatelaw/corporate... · ABE Advanced Diploma in Business Administration Study Manual CORPORATE FINANCE Contents Study

International Trade and Finance 369

© Licensed to ABE

� The exporter normally has a responsibility to ensure that the debt instruments are validly prepared and guaranteed.

Government and Quasi-Government Aid Governments will often encourage MNCs to establish operations in their country, thereby creating wealth and increasing employment. This is often achieved through grants, subsidies, favourable loans and guarantees. Whether assistance is given to a business applying for such help will depend upon whether its proposals qualify.

Many different institutions are involved in providing aid to companies wishing to break into export markets, in addition to the provision of finance. The following institutions provide advice and help for this purpose.

� Chambers of Commerce

The International Chamber of Commerce produces publications aimed at aiding international trade, and also provides certificates of origin and carnets which enable goods to be temporarily imported without payment or import duty.

� Department of Trade and Industry

This is the main government department, under the presidency of the Secretary of State for Trade and Industry, concerned with overseas trade. It oversees export licensing, regional groups, and British Trade International whose role is to foster business competitiveness by helping UK firms secure overseas sales and investments, and by attracting high quality foreign direct investment. Other services include the export intelligence, information and marketing research services and the Information Service on Tariffs, Regulations and Licences, all with the aim of providing information and advice to exporters.

Financial assistance is also available from the DTI in helping trade associations or other representative bodies to assist UK companies to present their products to a specified overseas audience, e.g. at fairs and seminars.

� Export Credit Guarantee Department

The ECGD works closely with exporters, project sponsors, banks and buyers to help UK exporters compete effectively in overseas markets by arranging finance facilities and credit insurance for contracts where the private sector may be unable to help

� Overseas Development Administration

This is aid from the British government to overseas countries, and it includes both capital on concessional terms and technical assistance. The aid loans or grants for contracts of a developmental nature given are important to Britain’s exports, industry and employment.

Finance for Importers The importer must also have adequate finance available to enable him to purchase goods either for immediate resale or for processing prior to resale. Banks and other financial institutions have developed various products to make sterling and foreign currency financing available to importers.

These are similar in nature to the methods available to exporters and include the following.

Page 378: libvolume5.xyzlibvolume5.xyz/law/ballb/3rdyear/semester5/corporatelaw/corporate... · ABE Advanced Diploma in Business Administration Study Manual CORPORATE FINANCE Contents Study

370 International Trade and Finance

© Licensed to ABE

� Bank overdraft

In most cases an importer would not be able to finance all his purchases from an overdraft facility, since this is an expensive source of finance. Overdraft facilities may be secured or unsecured, depending on the financial standing of the importer.

� Bank loan

Bank loans are available to companies in both sterling and foreign currency. Some form of security, however, would be called for by the bank.

Importers may take advantage of favourable interest rates by borrowing in foreign currency, especially if they are able to make repayment of the loan from receivables denominated in the same currency. Eurocurrency loans are available for large national or international projects.

� Term documentary letter of credit

The importer can ask his bank to open a documentary letter of credit in favour of the seller, under which drafts are drawn, payable not at sight but at usance, i.e. 30, 60, 90, 120 or 180 days after sight. The exporter can readily discount such bills and get spot cash because of the standing of the accepting bank. The importer, however, does not have to provide cash until the maturity of the bill. As an alternative, where a documentary letter of credit is not considered appropriate, the importer’s bank may simply add his “pour aval” endorsement to the bill. This guarantees to the drawer that the bill will be paid at maturity.

� Produce loan or merchandise advance

A produce loan is made by a bank to an importer to enable him to pay for goods which he has contracted to buy. The goods are the security for the loan, which is repaid from the proceeds of sale.

Produce loans are granted for short periods only – long enough for the importer to be able to resell the goods and repay the loan with the proceeds – usually between seven days and three months.

� Acceptance credits

� Export credit

Export credit agencies have been established in several countries, to encourage the export of goods and services. The exporter can obtain credit facilities from these agencies at fixed and preferential interest rates, which enables credit finance to be made available to importers in other countries.

Rates of interest and lengths of credit terms are agreed by members of the Organisation for Economic Cooperation and Development (OECD) and these terms, which are offered by most national credit agencies, are reviewed on a regular basis.

� Confirming houses

The importer can receive short-term credit from the confirming house, to which he must pay a commission for the service provided.

In addition to direct financial assistance, banks also provide a range of further services. The principal service is effecting payment:

� Where open account or payment in advance terms are used, banks will effect payment by means of money transfers, telegraphic transfers, the SWIFT system, or by draft.

Page 379: libvolume5.xyzlibvolume5.xyz/law/ballb/3rdyear/semester5/corporatelaw/corporate... · ABE Advanced Diploma in Business Administration Study Manual CORPORATE FINANCE Contents Study

International Trade and Finance 371

© Licensed to ABE

� Opening of documentary letters of credit, including back-to-back and transferable letters of credit.

� Selling foreign currency to the importer to settle his purchases, both on the spot and on forward currency markets.

In addition, they will also obtain status reports on prospective suppliers, provide advice and practical assistance in complying with exchange control requirements, import licences, documentation, etc., secure travel facilities for importers seeking to make contacts overseas and assist in finding suppliers via the bank’s correspondent networks, and arranging introductions.

Countertrade Countertrade can be defined as a trading transaction whereby export sales are dependent on the exporter receiving imports, in one form or another, from the buyer. Countertrade developed rapidly during the 1980s as the international debt crisis worsened and many countries did not have the funds or credit facilities to pay for imports in the normal way. Countertrade has also been used to protect or stimulate the output of domestic industries. It is thought that 30% of world trade is arranged through some form of countertrade agreement, being common in developing countries; centrally-planned economies (as part of their political and economic policies); in those lacking foreign currency; and in buyers’ markets. It is less common between industrial countries, though it is used in high technology areas such as aviation and defence.

Types of countertrade transactions are:

� Counter purchase

This is the most common form of countertrade. As a condition of obtaining a sales order, the exporter agrees to purchase goods and services from the buyer’s country. Two parallel, but separate, contracts are arranged, and the value of the counter purchase varies from 10% to 100% of the export order.

The counter-purchase goods will often be unrelated to those exported – and indeed the counter-purchase agreement may be between parties not involved in the original transaction (although normally there will be recourse to the exporter if the counter-purchase obligations are not fulfilled).

� Barter

This is a direct exchange of goods for goods, and only one contract is involved. Such deals are rare, although they are still sought by several African and Latin American countries, or for large contracts involving oil.

� Buyback

Here, exporters of capital equipment agree to be repaid from the future output of the equipment supplied; such deals tend to be for large amounts and for long periods. They are sometimes referred to as “compensation” deals.

� Switch trading

Because of imbalances in trading agreements, one country may accumulate large surpluses owed by another country, e.g. Brazil may have a large surplus with Poland. These surpluses could be used by, for example, UK exports to Brazil being financed by the sale of Polish goods to the UK. Such “switch” or “swap” deals can be very complex and specialist intermediary organisations are often involved in arranging such transactions.

Page 380: libvolume5.xyzlibvolume5.xyz/law/ballb/3rdyear/semester5/corporatelaw/corporate... · ABE Advanced Diploma in Business Administration Study Manual CORPORATE FINANCE Contents Study

372 International Trade and Finance

© Licensed to ABE

� Offset

This type of countertrade is most commonly used in connection with high technology products, and involves an agreement by an exporter of such products to use materials and components produced in the importing country in his final product. Alternatively, an offset agreement may incorporate a requirement that bidders establish local production capacity.

� Evidence accounts

In some countries there is a requirement for goods imported to be matched by an equivalent amount of goods exported from that country, and where such trade is of a continuing nature it may be impractical to balance the business on an item-by-item basis. In these cases, a record of the balance of this trade may be kept by means of evidence accounts, such accounts being kept in balance on a year-by-year basis.

Countertrade is expensive, administratively difficult and often collapses – thus smaller and medium-sized companies tend to avoid it. One party may receive goods it is unable to trade in, or receive them at too high a price; or there may be too many parties involved to ensure the contract goes ahead.

The costs of countertrade include:

� Insurance costs.

� Advice fees, e.g. from banks, specialist consultants or third-party trading houses or brokers who may take up obligations of countertrade.

� The discount needed to dispose of goods can be up to 50% of low quality goods; this is known as disagio.

Countertrade transactions are fraught with danger, and the number of successful transactions form a relatively small percentage of those negotiated. However, the ability of an exporter to quote countertrade terms, even if the particular exporting transaction does not eventually result in countertrade, may mean the difference between success and failure in tendering for business. It can therefore be an important marketing tool.

Among the many dangers associated with countertrade are:

� Inability to dispose of large quantities of unmarketable goods taken as part of a countertrade deal.

� Loss of profits on the main exports because the costs of countertrade have not been fully considered.

� Cancellation of export orders because of the failure of other parties to meet countertrade obligations.

There are a number of specialists who can assist in countertrade, such as traders, brokers and banks.

D. EXCHANGE RATES

An exchange rate is the rate at which a unit of one currency can be exchanged for another – for any currency there is an exchange rate for each currency it can be traded with.

Exchange Rate Concepts There are three concepts involved in the way in which exchange rates are expressed which must be understood at the outset.

Page 381: libvolume5.xyzlibvolume5.xyz/law/ballb/3rdyear/semester5/corporatelaw/corporate... · ABE Advanced Diploma in Business Administration Study Manual CORPORATE FINANCE Contents Study

International Trade and Finance 373

© Licensed to ABE

(a) Buying and selling prices

Two prices are always quoted for any currency – a buying and a selling price. Thus, a UK bank would quote its exchange rates as follows:

Value of £ against other currencies

US Dollar 1.4910 – 1.4920 German Mark 2.6034 – 2.6059 Japanese Yen 164.710 – 166.030 French Franc 8.8468 – 8.8586

(Note that these rates vary on a daily basis and the figures given here are for illustration only.)

The lower of the two rates is the price at which the bank will buy sterling (for example, sell dollars in return for sterling) and the higher figure is the rate the bank will buy the exchange currency (for example, sell pounds for dollars). The wider the spread between these two quotes the larger the profit margin.

Note that the terms “bid” and “offer” rates are often used as an alternative to buy and sell rates, but here we shall stick to the latter terminology throughout.

(b) Spot and forward rates

This is an important distinction.

� The spot rate is the current rate of exchange for a unit of currency for immediate transactions and quotations for which no previous arrangements have been made, delivery to the buyer being made two days later.

� The forward rate refers to a rate of exchange fixed today for delivery at a defined date in the future – for example, one month forward and three months forward being for one month and three months in the future respectively.

Thus, for example, a German company which is contracted to receive $1 million in three month’s time could either:

� wait for three months and then sell the dollars at the (then) spot rate; or

� enter into a contract now to sell the dollars in three months’ time at the three month forward rate.

Whilst the company may expect that the spot rate in three months’ time may be higher than the current forward rate, there is an element of risk. By waiting and seeing what the spot rate is on the day of receipt of the dollars, the company stands to possibly gain or lose on the exchange at the prevailing spot rate in relation to the forward rate. On the other hand, the company can buy certainty about the exchange rate by selling the dollars forward at the (possibly) lower forward rate.

It is important to note that forward rates are not statements of what spot rates will be at the future date implied by the forward rate, (e.g. three months in the future for a three month forward rate), but are merely indications of the direction in which spot prices will move. Depending on the risk assessment, the forward rate can be expected to be higher or lower than the expected future spot rate. This is because a forward price comprises the spot price on the date of the contract plus (or minus) the interest differential reflecting the expected relative

Page 382: libvolume5.xyzlibvolume5.xyz/law/ballb/3rdyear/semester5/corporatelaw/corporate... · ABE Advanced Diploma in Business Administration Study Manual CORPORATE FINANCE Contents Study

374 International Trade and Finance

© Licensed to ABE

changes in interest rates. Thus, for example, if the forward rate of Deutschmarks against sterling is expected to fall it is because interest rates in Germany are expected to be higher than those in the UK. (We consider the impact of interest rates on exchange rates in more detail below.)

(c) Discount and premium

These refer to the strength of the forward rate in respect of the current spot rate. They identify whether the market considers the currency to be, respectively, weaker or stronger in the future, in relation to the exchange currency.

A currency with a forward discount implies that it is becoming weaker in relation to the exchange currency and will buy less of that exchange currency in the future. A forward premium would have the opposite effect.

The rates for one and three, or more, months in advance are often quoted in terms of the discount or premium applying. For example:

Value of £ against other currencies (spot and forward rates)

Spot Rates 1 Month Forward 3 Months Forward

US Dollar 1.4910 – 1.4920 0.97 – 0.95 c pm 2.63 – 2.60 c pm German Mark 2.6034 – 2.6059 1.5 – 1.25 pf pm 3.75 – 3.625 pf pm Japanese Yen 164.710 – 166.030 1.25 – 1.375 y dis 3.375 – 3.5 y dis French Franc 8.8468 – 8.8586 3 – 3.375 c dis 8.375 – 9 c dis

Thus, if the quoted spot rate for sterling against the US dollar is £1 = $1.4910 and the three month forward rate is £1 = $1.4647, the pound will buy less dollars in three month’s time compared to today. It is, therefore, at a discount relative to the dollar – or the dollar is at a premium relative to the pound. The size of the premium is the difference between the two rates – $0.0263 or 2.63 cents.

The discount or premium may also be expressed as a percentage annualised figure according to the following formula (in respect of the three month forward rate):

100% 3

12 rateSpot

rateSpot - rate Forward ××

= %10044910.1

4910.14647.1 ××−

= -7.06%

Thus, the dollar is said to be at an annualised premium of 7.06% against the pound.

The rule for calculating rates is:

� add a discount, deduct a premium.

This rule is important and you should remember it. It is essential to get the additions and subtractions correct.

Remember, too, that premiums and discounts are quoted in fractions of a currency. The DM is therefore quoted in pfennigs, the dollar in cents, the franc in centimes, and so on.

Page 383: libvolume5.xyzlibvolume5.xyz/law/ballb/3rdyear/semester5/corporatelaw/corporate... · ABE Advanced Diploma in Business Administration Study Manual CORPORATE FINANCE Contents Study

International Trade and Finance 375

© Licensed to ABE

Exchange Rate Systems An exchange rate is simply the price of one currency denominated in terms of another. As such, it is subject to very much the same processes as those which determine the price of any other good or service – the laws of supply and demand – and these will result in there being an equilibrium position.

We shall look at the supply and demand influences below, but first we should make an essential distinction between two different types of exchange rate system.

(a) Fixed exchange rates

This is where governments which are members of the international monetary system use their official reserves (which comprise foreign currency and gold) to maintain a fixed exchange rate. By adding to, or using, their official reserves the government ensures that the demand for, and the supply of, their currency are balanced (thus maintaining its price).

The exchange rate of each member currency is generally set against a standard – which could be gold, a major currency (e.g. the US $) or a basket of currencies. It is also possible for each currency in the system to be set against each other.

Fixed exchange rate systems encourage international trade by removing uncertainty. However, they restrict member states’ independence in setting domestic economic policies by requiring them to take appropriate action to maintain their exchange rate.

(b) Floating exchange rate systems

In such systems, exchange rates are left to, and are determined by, market forces, there being no use of the official reserves in maintaining the exchange rate level.

Floating exchange rate systems may be either free floating or, more commonly, managed floating.

Wide fluctuations of exchange rate values can occur under floating exchange rate systems creating problems of uncertainty for international trade. However, it is likely that the underlying economic conditions creating these fluctuations would have created severe problems for the working of a fixed exchange rate system – even creating instability.

Influences on Exchange Rates The demand for and supply of a currency, and hence its equilibrium price (or the exchange rate) is determined by number of factors, including:

� Income and expenditure in a country on both domestic goods (which would otherwise have been exported) and imports.

� Differential interest rates between countries.

� Differential inflation rates between countries.

� The balance of payments position between the countries concerned.

� Government economic policy, particularly in regard to influencing exchange rate levels.

� Speculators and the general view as to the economic prospects of a country.

The relationship between currency exchange rates, inflation and interest rates has occupied the attention of economists over a number of years. The differential interest rates, inflation rates, and the spot and forward exchange rates between two countries are all interconnected, and all impact on each other. In the absence of restrictions on international capital movements, this relationship can best be summed up in the form of a series of equilibrium models.

Page 384: libvolume5.xyzlibvolume5.xyz/law/ballb/3rdyear/semester5/corporatelaw/corporate... · ABE Advanced Diploma in Business Administration Study Manual CORPORATE FINANCE Contents Study

376 International Trade and Finance

© Licensed to ABE

(a) Inflation rates and exchange rates

In a perfectly competitive market, and assuming the absence of transport and transaction costs, the price of goods in one country should be the same, after adjusting for exchange rates, in another country. This is referred to as the law of one price.

The support for this rests with the fact that arbitrage can take place – if goods are cheaper in one country, then they can be purchased there and re-sold in another country at a higher price. This process would continue until, under the laws of supply and demand, the price of the goods in the first country would rise (through increasing demand) and/or the price of the goods in the second country would fall (through increasing supply). In due course, therefore, an equilibrium price will be established.

If prices are in equilibrium, but then change as a result of differential rates of inflation between two countries, we can expect the exchange rate to move in order to compensate for the gain/loss in the purchasing power of the exchange currency. So, if inflation in the US is 5% and in the UK it is 3%, then the exchange rate of the pound against the dollar would adjust to maintain the equilibrium in prices.

As a general rule, we can conclude that the purchase price for a commodity in country A must be equal to the purchase price for the same commodity in country B, adjusted for the exchange rate difference. That is, if inflation in one country relative to the other causes the price to rise, then there will be a proportional change in the exchange rate.

This gives rise to the theory of Purchasing Power Parity. This is based on the law of one price and states that the expected changes in the spot rate of exchange are linked to the inflation differentials between the respective two countries over the same period. Thus, the change in exchange rates over a time period is approximately equal to the difference in inflation between the two countries over the same period. A higher rate of inflation in one country will lead to a depreciation of its currency in terms of the other currency – ensuring that the real cost of purchasing the same good in different countries should be the same.

This is expressed in the following formula:

o

ot

SS S −

= i i1 + if d

d

where: So = the current spot rate

St = the spot rate at time t

if = the expected rate of inflation in the foreign country to time t

id = the expected domestic rate of inflation to time t

Consider the following example.

There is currently purchasing power parity between France and Germany with an exchange rate of 1DM to 3Ff. French inflation is expected to be 5%, and German inflation is expected to be 10%. Advise the German firm you are working for what the expected spot rate will be at the end of the year.

You are assumed to be working for a German firm so the domestic currency is the DM. Substituting into the above formula:

33 St − (1 DM to 3FF) = 0.10 + 1

0.10 05.0 − (inflation rates expressed as a decimal)

Page 385: libvolume5.xyzlibvolume5.xyz/law/ballb/3rdyear/semester5/corporatelaw/corporate... · ABE Advanced Diploma in Business Administration Study Manual CORPORATE FINANCE Contents Study

International Trade and Finance 377

© Licensed to ABE

∴ St − 3 = 3 × 10.1

)05.0 (− = 2.8636

We can check these calculations by comparing the prices at the end of the year.

A good now costing 10 DM will cost 10 × 3 = 30 francs. At the end of the year the same good will cost:

10 DM × (German inflation rate of 10%) 1.10 = 11 DM

30 francs × (French inflation rate of 5%) 1.05 = 31.5 francs

The price of the goods in Germany is 31.5 francs/2.8636 = 11 DM.

This method is often used to calculate exchange rates for use in investment appraisal if there are reasonable estimates of relative inflation rates available to the company. However, in reality purchasing power parity only holds over the long term, market imperfections impeding its effects in the short term.

Just as relative inflation rates help to determine exchange rates, alterations (by a country’s government or other factors discussed below) in a country’s exchange rate can also have an impact on its inflation rate.

A devaluation of a currency would lead to an increase in the price of imports and a reduction in the price of exports. For several countries, including the UK for which imports are price inelastic and exports are price elastic, this would lead to an increase in exports but have little impact on the level of imports. Thus, inflation would rise caused by dearer imports – the greater the proportion of goods and services consumed that are imported the greater the rise in inflation. An increase in the cost of goods and services will lead to higher wage claims which firms benefiting from increased exports would agree to. These higher wages would increase the level of inflation. Whilst the balance of trade may improve because of the increased export levels, any increases are likely to be offset by a net disinvestment of capital from the depreciating currency the overall impact being a further depreciation of the currency.

(b) Interest rates and exchange rates

We can observe the same processes at work in relation to interest rate differentials between countries as in inflation rate differentials.

We can see that, where there is a differential in the interest between two countries, investors wish to place their funds where the rates of interest are highest, so a country with higher interest rates will experience an influx of funds. This influx of funds will lead to an increased demand for the currency and thus increase its price (exchange rate). At a later date, when the investment matures, the funds will be converted back into the original currency of exchange and repatriated. This selling of the base currency will then cause its rate of exchange to fall relative to the exchange currency.

Just as purchasing power parity denotes the relationship of inflation rates to exchange rates, Interest Rate Parity (IPP) does the same for interest rates. This states that the expected changes in the spot rate of exchange (i.e. the difference between the spot and forward rates) are linked to the differential between interest rates in the two countries over the same period. Any gain from an increase in interest rates in one country relative to another will, then, be cancelled out by an adjustment in the exchange rate. The forward rate is then said to be at IPP.

Page 386: libvolume5.xyzlibvolume5.xyz/law/ballb/3rdyear/semester5/corporatelaw/corporate... · ABE Advanced Diploma in Business Administration Study Manual CORPORATE FINANCE Contents Study

378 International Trade and Finance

© Licensed to ABE

The theory can be stated as:

b

a

i + 1i + 1

= ratespot

rate forward

where i = interest rate, and a and b are the two countries under consideration.

Thus, if the 90-day interest rate in the US is 5.25% and in the UK for the same period is 6.75%, and the current spot rate is £1:$1.762, the 90-day forward rate may be calculated by substituting in the above formula:

0675.0 + 10525.0 + 1 =

1.762rate forward

∴ The forward rate = £1.737

(c) Relationship between interest rates and inflation (the Fisher Effect)

The relationship between expected levels of inflation and interest rates is described as the Fisher Effect after the economist who first documented it.

This states that the difference in nominal rates of interest between two countries will reflect the expected difference in inflation rates. In equilibrium, the real rate of return on capital is the same in both countries – that is, the rate of return, adjusted for inflation, in one country will equate with the rate of return, adjusted for inflation, in the other.

Interest rates can be money rates (i.e. the actual amount of cash paid) or real rates (i.e. money rates adjusted to remove the effects of inflation). In general the higher the expected rate of inflation the higher the rate of interest – to allow investors to obtain a high enough return after the effects of inflation have been considered, and as a tool of government to help reduce the rate of inflation.

The Fisher equation states that:

1 + the money or nominal rate of interest = (1 + the real rate of inflation) × (1 + the expected rate of inflation)

The Fisher Effect is developed in the International Fisher Effect which states that the ratio of nominal interest rates between two countries is equal to the ratio of their inflation rates, and those currencies with higher nominal interest rates will depreciate in relation to those with lower nominal interest rates. The belief behind this theory is that higher rates of interest are required to offset the effects of currency depreciation, and given free world-wide capital markets adjustments to spot exchange rates will mean that real rates of interest will be equal in different countries.

The International Fisher Effect can be expressed as:

1 + r1 + r

f

d

= 1 + i1 + i

f

d

where rd is the domestic money rate of interest and rf is the foreign money rate of interest.

Page 387: libvolume5.xyzlibvolume5.xyz/law/ballb/3rdyear/semester5/corporatelaw/corporate... · ABE Advanced Diploma in Business Administration Study Manual CORPORATE FINANCE Contents Study

International Trade and Finance 379

© Licensed to ABE

E. RISK AND INTERNATIONAL TRADE/FINANCE

In addition to normal business and financial risk, companies face extra risks connected with trading and investing overseas. These risks can be separated into political risk and foreign exchange risk.

Political or Country Risk Political risk (also known as country risk) includes the problems of managing subsidiaries geographically separated and based in areas with different cultures and traditions, and political or economic measures taken by the host government affecting the activities of the subsidiary.

Whilst a host country will wish to encourage the growth of industry and commerce within its borders, and offer incentives to attract overseas investment (such as grants), it may also be suspicious of outside investment and the possibility of exploitation of itself and its population. The host government may restrict the foreign companies’ activities to prevent exploitation or for other political and financial reasons. Such restrictions may range from import quotas and tariffs limiting the amount of goods the firm can either physically or financially viably import, to appropriation of the company’s assets with or without paying compensation. Other measures include restrictions on the purchasing of companies, especially in sensitive areas such as defence and the utilities – such restrictions could be an outright ban, an insistence on joint ventures or a required minimum level of local shareholders. In order to prevent the “dumping” of goods banned elsewhere (e.g. for safety reasons) a host government may legislate as to minimum levels of quality and safety required for all goods produced or imported by foreign companies.

Host governments, particularly in developing and underdeveloped countries, may be concerned about maintaining foreign currency reserves and preventing a devaluation of their national currency. In order to do this they may impose exchange controls. This is generally done by restricting the supply of foreign currencies – thus limiting the levels of imports and preventing the repatriation of profits by MNCs by restricting payments abroad to certain transactions. This latter method often causes MNCs to have funds tied up unproductively in overseas countries.

Foreign Exchange or Currency Risk Exchange rate risk applies in any situation where companies are involved in international trade. It arises from the potential for exchange rates to move adversely and, thereby, to affect the value of transactions or assets denominated in a foreign currency.

There are three main types of exchange rate risk to which those dealing overseas (importers, exporters, those with overseas subsidiaries or parents, and those investing in overseas markets) may be exposed.

(a) Transaction exposure

This occurs when trade is denominated in foreign currency terms and there is a time delay between contracting to make the transaction and its monetary settlement. The risk is that movements in the exchange rate, during the intervening period, will increase the amount paid for the goods/services purchased or decrease the value received for goods/services supplied.

(b) Translation exposure

This arises where balance sheet assets and liabilities are denominated in different currencies. The risk is that adverse changes in exchange rates will affect their value on conversion into the base currency.

Page 388: libvolume5.xyzlibvolume5.xyz/law/ballb/3rdyear/semester5/corporatelaw/corporate... · ABE Advanced Diploma in Business Administration Study Manual CORPORATE FINANCE Contents Study

380 International Trade and Finance

© Licensed to ABE

Any gains or losses in the book values of monetary assets and liabilities during the process of consolidation are recorded in the profit and loss account. Since only book values are affected and these do not represent actual cashflows, there is a tendency to disregard the importance of translation exposure. This is, though, a false assumption since losses occurring through translation will be reflected in the value of the firm, affecting the share price and hence, shareholders’ wealth and perceptions among investors of the firm’s financial health.

(c) Economic exposure

This refers to changes in the present value of a company’s future operating cashflows, discounted at the appropriate discount rate, as a result of exchange rate movements.

To some extent, this is the same as transaction exposure, and the latter can be seen as a sub-set of economic exposure (which is its long term counterpart). However, economic exposure has more wide ranging effects. For example, it applies to the repatriation of funds from a wholly-owned foreign subsidiary where the local currency falls in value in relation to the domestic currency of the holding company. It can also affect the international competitiveness of a firm – for example, a UK company purchasing commodities from Germany and reselling them in China would be affected by either a depreciation (loss of purchasing power) of sterling against the Deutschmark and/or an appreciation of yuan.

It can also affect companies who are not involved in international trade at all. Changes in exchange rates can impact on the relative competitiveness of companies trading in the domestic market vis-à-vis overseas companies when imports become cheaper. Thus, reduced operating cashflows may be a consequence of a strengthening domestic currency – a situation which has affected UK companies in the late 1990s.

The management of exchange rate risk will involve hedging against adverse movements in order to contain the extent of any exposure. At the operating level, the focus of attention is primarily on managing the exposure caused by transaction and economic risk, both essentially being underpinned by cashflows. The techniques which we shall examine in the following sections, then, relate essentially to these aspects of exposure, with the greater emphasis on transaction exposure

As with managing interest rate risk, these techniques fall into two categories:

� internal, or natural, techniques – those which are effected entirely by the financial organisation and structure of the company itself; and

� external, or transactional, techniques – those using the range of derivative instruments which are effected by the use of third party services, such as banks and specialist exchanges.

Although both types of technique provide effective means of covering the exposure, certain external techniques offer the possibility of taking advantage of favourable movements in exchange rates to generate profits.

F. INTERNAL METHODS OF MANAGING EXCHANGE RATE RISK AND EXPOSURE

There are four main internal means of reducing exchange rate exposure. These are based on methods of processing transactions and payments, and of offsetting assets and liabilities in different currencies.

Currency invoicing The first approach is simply to invoice foreign customers in the currency of the seller.

Page 389: libvolume5.xyzlibvolume5.xyz/law/ballb/3rdyear/semester5/corporatelaw/corporate... · ABE Advanced Diploma in Business Administration Study Manual CORPORATE FINANCE Contents Study

International Trade and Finance 381

© Licensed to ABE

Invoicing for goods supplied, and paying for goods received, in a company’s domestic currency removes the exchange rate risk for that company – but only one party to an exchange between foreign companies can have this facility, and the other bears the risk of exchange rate fluctuations. However, the advantages of removing exchange rate risk need to be weighed against those of invoicing in the foreign currency. These include marketing advantages such as the ease for the customer of dealing in his own currency and the possibility of purchasing at a discount if the foreign currency is depreciating relative to the domestic currency. In fact, often the only way to win a contract overseas is to deal in the currency of that market.

One way to prevent one or both parties being subject to exchange rate risk is for the firms involved to set a level of exchange rate to use for a transaction regardless of what the actual exchange rate is on the day the money is transferred.

Netting This is an internal settlement system used by multinational companies with overseas subsidiaries. It involves offsetting (netting out) the outstanding foreign exchange positions of subsidiaries against each other through a central point – the group treasury.

Suppose there are two overseas subsidiaries in different countries. Subsidiary A expects to receive a payment in one month’s time for the sale of goods to the value of $2m, while subsidiary B has to make a payment of $3m in one month’s time to a supplier. The central treasury can offset the two exposures and set up an external hedge for the net risk of $1m. This negates the need for two separate hedges to be carried out – the first to cover the $3m against a rise in exchange rates against the dollar and second to cover the $2m against a fall in exchange rates against the dollar. The single hedge is more efficient and cost-effective.

Matching This is the process of matching receipts in a particular currency with payments in the same currency. This prevents the need to buy or sell the foreign currency and thus reduces exchange rate risk to the surplus or deficit the firm has of the foreign currency. It is a cheap method of reducing or eliminating exchange rate risk provided that the receipts precede the payments, and the time difference between the two is not too long.

For example, where a company is selling to the US and has outstanding receipts denominated in $, it could purchase raw materials in the same currency. The one transaction will offset the other and minimise the exchange exposure that requires external hedging. It therefore does not matter whether the $ strengthens or weakens against the domestic currency.

Alternatively, a firm could match, say, dollar currency receipts from the export of goods to the US with a dollar loan. The receipts will be used to pay off the loan. This again secures the matching of an asset with a liability.

This process can be made easier either by having a bank account in the foreign country or a foreign currency account in a firm’s own country, and putting in all receipts and taking from it all payments in the overseas currency. The exchange rate risk on the surplus or deficit can be avoided by utilising one of the other methods of risk management.

Matching may also be used to reduce translation exposure – offsetting an investment in assets in one currency with a corresponding liability in the same currency. For example, the acquisition of an asset denominated in Yen could be achieved by borrowing funds in Yen. As the exchange rate against the Yen varies, the effect it has on the translated value of the asset and liability will increase and decrease

Page 390: libvolume5.xyzlibvolume5.xyz/law/ballb/3rdyear/semester5/corporatelaw/corporate... · ABE Advanced Diploma in Business Administration Study Manual CORPORATE FINANCE Contents Study

382 International Trade and Finance

© Licensed to ABE

in concert. The amount of the reduction in exposure will depend on the extent to which the expected economic life of the asset corresponds with when the loan matures.

Leads and Lags This final method of hedging internally involves varying payment dates to take advantage of the exchange rate – for example, paying either before or after the due date, depending on exchange rate movements. The effectiveness of this is dependent on how well exchange rate movements can be anticipated. A company will only pay in advance if it expects the domestic currency to weaken, but if it misreads the movement and the exchange rate strengthens, advance payment may prove expensive.

� Leads are advance payments for imports to avoid the risk of having to pay more local currency if the supplier’s currency increases in value.

� Lags involve slowing down the exchange of foreign receipts by exporters who anticipate a rise in the value of the foreign currency received. When this occurs, they will then benefit by an exchange rate in their favour.

The table below shows the scope for leading and lagging by financial managers of importers or exporters:

UK Exporter UK Importer Expectation of foreign currency Receiving foreign currency Paying foreign currency Devaluation Leads Lags Revaluation Lags Leads

Foreign Importer Foreign Exporter Expectation of sterling Paying in sterling Receiving sterling Revaluation Leads Lags Devaluation Lags Leads

A UK exporter would accelerate (lead) his receipts in the event of an anticipated devaluation, but he would delay (lag) his foreign receipts if a revaluation was expected, and so forth. In leading, he will need to borrow or otherwise raise the cash which will involve a cost of capital, whilst lagging will attract interest as there will be surplus for investment.

G. EXTERNAL METHODS OF MANAGING EXCHANGE RATE RISK AND EXPOSURE

Here we shall consider the methods of using derivatives – forwards, futures and options contracts – as considered in the last unit in relation to interest rate exposure, as well as currency swaps and money market hedges.

Forward contracts Forward foreign exchange contracts are a binding agreement between two parties to exchange an agreed amount of currency on a future date at an agreed fixed exchange rate. The exchange rate is fixed at the date the contract is entered into.

Forward contracts are binding and must be executed by both parties. As we saw in the previous unit, they are not exchange regulated and one problem of this is that one of the parties might default.

Page 391: libvolume5.xyzlibvolume5.xyz/law/ballb/3rdyear/semester5/corporatelaw/corporate... · ABE Advanced Diploma in Business Administration Study Manual CORPORATE FINANCE Contents Study

International Trade and Finance 383

© Licensed to ABE

However, they do not – like futures contracts – come in standard sizes and have fixed delivery dates. Rather, they are over the counter (OTC) instruments – in which the contract can be tailor made to suit the needs of the parties and delivery dates can range from a few days to upwards of several years.

In most cases, forward contracts have a fixed settlement date. This is appropriate where the cash transaction being hedged will take place on the same day that the forward contract is settled. However, there is no guarantee that the two days will tally – for example, a customer may be late paying – in which case the fixed settlement date is less than optimal. An alternative, to provide flexibility, is an “option date forward contract”. This offers a choice of dates on which the user can exercise the contract, although there is a higher premium payable on the contract for such an additional benefit.

The purpose of a forward exchange rate contract is to purchase currency at a future date at a price fixed today. As such, it provides a complete hedge against adverse exchange rate movements in the intervening period. Consider the following example.

A UK company needs to pay A$1m to a Australian company in three months’ time. The current spot and forward exchange rates for sterling are as follows:

A$/£

Spot 2.730 – 2.735

3 months forward 4 – 3 cents pm

What would be the cost in sterling to the UK company if it enters into a forward contract to purchase the A$1m needed?

Note the way in which the rates are quoted. The spot rate spread shows the sell and buy prices – the banks will sell A$s for sterling at the rate of A$2.730/£, and buy A$s in return for sterling at the rate of A$2.735/£. The forward rate is quoted in terms of the premium (“pm”) in cents which the Australian dollar is to sterling in the future. If the currency is at a premium, it is strengthening and the A$ will buy more pounds forward than it will spot or, conversely, the pound will buy less A$ forward than it will spot. (If the quoted forward rate had been, say, “3 cents dis” this would indicate a weakening of the currency.)

To calculate the cost of the forward contract, we need to convert the forward rate premium into an exchange rate. Because it is a premium, we need to subtract the amount from the spot to give the following sell/buy forward rates:

(2.730 – 0.04) – (2.735 – 0.03) = 2.690 – 2.705 A$/£

The cost of buying A$1m forward, therefore, is:

690.2000,000,1$A = £371,747

Whilst we have said that forward contracts are binding, they can be closed out by entering into an opposite contract to sell the currency – either at the spot rate or through a different forward rate. Partial close-outs can also be arranged where, for example, the full amount of the forward contract is not required. However, these arrangements are costly and, hence, rare.

Currency swaps In general, a swap relates to an exchange of cashflows between two parties – as we saw in relation to interest rate swaps in the previous unit. Thus, currency swaps relate to an exchange of cashflows in different currencies between two parties. They are agreements to exchange both a principal sum and

Page 392: libvolume5.xyzlibvolume5.xyz/law/ballb/3rdyear/semester5/corporatelaw/corporate... · ABE Advanced Diploma in Business Administration Study Manual CORPORATE FINANCE Contents Study

384 International Trade and Finance

© Licensed to ABE

the interest payments on it in different currencies for a stated period. Each party transfers the principal and then pays interest to the other on the principal received.

Swaps are arranged, through banks, to suit the needs of the parties involved.

The two key issues in setting up a currency swap are:

� the exchange rate to be used; and

� whether the exchange of principal is to take place at both commencement and maturity, or only on maturity.

The following example illustrates the general principles.

A German company is seeking to invest £20m in the UK and has been quoted an interest rate of 8% on sterling in London, whereas the equivalent loan in Deutschmarks is quoted at 7% fixed interest in Frankfurt. At the same time, a UK company wants to invest an equivalent amount in its German subsidiary and has been quoted an interest rate of 7.5% to raise a loan denominated in Deutschmarks on the Frankfurt Exchange. It could, however, raise the £20m in sterling in London at 5% fixed interest.

In the absence of a swap, each company would have to accept the quoted terms for its loan denominated in the foreign currency. This would result in both companies paying a higher rate than would apply if the loan was raised in their domestic currency. A swap agreement would involve each company taking out the loan in its own domestic currency and then exchanging the principals. Each company would pay the interest on the principal received – i.e. the other company’s loan – and at the end of the loan period, the principals would be swapped back.

The exchange rate to be applied is clearly crucial. If we assume that this is agreed as DM3 = £1, the swap would be conducted as follows.

� The UK company borrows £20m in England at an interest rate of 5% pa. It then swaps the principal of £20m for DM60m (at the agreed exchange rate) with the German company. The German company pays the interest payments on the £20m loan (at 5% interest) to the UK company, which then pays the bank. At the end of the loan period, the principal of DM60m is swapped back for the £20m with the German company.

� The German company borrows DM60m in Frankfurt at an interest rate of 7% pa. It then swaps this principal with the UK company which pays the interest payments on the loan (at 7% interest) to the German company, which then pays its bank. At the end of the loan period, the principal of £20m is swapped back for the DM60m with the UK company.

The process is illustrated in Figure 15.1.

Page 393: libvolume5.xyzlibvolume5.xyz/law/ballb/3rdyear/semester5/corporatelaw/corporate... · ABE Advanced Diploma in Business Administration Study Manual CORPORATE FINANCE Contents Study

International Trade and Finance 385

© Licensed to ABE

Figure 15.1: Currency swap

Currency Futures A currency futures contract is an agreement to purchase or sell a standard quantity of foreign currency at a pre-determined date. As we saw previously, futures have standard quantities and delivery dates set by the exchange on which the contracts are traded. As we have also seen, the vast majority of these contracts are not delivered, but are closed out.

The process of hedging exchange rate risk through the futures market is the same as we examined in relation to interest rate exposure in the previous unit. Thus, a UK company exporting to the USA and invoicing in US dollars, would need to hedge against a rise in the exchange rate (sterling strengthening relative to the dollar) in the period before payment is received.

If we assume that payment is due in two months’ time, the exporter will need to sell dollars then in exchange for sterling. The strategy would be, therefore, to take out a three month sterling futures contract and close it out in two months’ time – i.e. buy sterling futures now, hold them for two months and then sell them to cancel out the obligation to deliver the underlying currency. Any profit on the contract (the difference between the buying and selling prices) will offset any loss on the dollars received from an exchange rate rise over the period.

We can illustrate the process in more detail by reference to the actions of a speculator who is anticipating a rise in the value of the $ against the pound. He will, therefore, take a position to sell sterling futures in anticipation that the future cost (in dollars) of buying the pounds necessary to meet the contract obligation will be less than the proceeds of the sale under the contract.

If the current spot rate is $1.700/£ and December sterling futures are trading at $1.675/£, what will be the gain or loss on five sterling futures contracts if the spot rate in December is $1.600/£? (The standard size of sterling futures is £62,500.)

� Sale of five December contracts (each of which is for £62,500) at the agreed rate of $1.675/£ results in proceeds of:

5 × £62,500 × $1.675 = $523,437.50

London bank Frankfurt bank

UK Company German Company

£20m

£20m

DM60m

DM60

5% interest

5% interest

7% interest

7% interest

UK company now has DM60m available for

investment at 7% interest

German company now has £20m available for

investment at 5% interest

Page 394: libvolume5.xyzlibvolume5.xyz/law/ballb/3rdyear/semester5/corporatelaw/corporate... · ABE Advanced Diploma in Business Administration Study Manual CORPORATE FINANCE Contents Study

386 International Trade and Finance

© Licensed to ABE

� Purchase of the equivalent amount in sterling in December at the spot rate of $1.6/£ results in an outlay of:

5 × £62,500 × $1.6 = $500,000

� The gain on the transaction is $23,437.50 or, converting this into pounds at the spot rate, £14,648.

The advantage for the speculator of using the futures contract compared to the alternative of buying sterling at the current spot rate is that he only needs to put down a small deposit (the margin account) as opposed to an “up front” investment of $531,250 (£312,500 x $1.7).

Hedging using futures and forwards contracts

We can also consider the difference between a hedge using forward contracts and a hedge using futures contracts.

In December, a UK exporter invoices its US customer for $407,500 payable on 1 February. The exporter needs to hedge against a change in exchange rates whereby sterling becomes stronger relative to the dollar and he receives less pounds than now upon exchange of the dollars received in February. To hedge this exchange rate exposure, the company could take out either a forward contract or a futures contract. Which would be more appropriate given the following rates?

In December:

Spot rate $1.6275 – 1.6295/£

February forward rate $1.6250 – 1.6275/£

March sterling futures contracts $1.6300/£ (Contract size is £62,500)

Those applying on 1 February:

Spot rate $1.6370 – 1.6390/£

March sterling futures contracts $1.6355

Using a forward contract would require the exporter to commit to the sale of the dollar receivables (i.e. $407,500) at the February forward price of $1.6275/£, resulting in proceeds of:

6275.1500,407$ = £250,384

The futures contract hedge would require the exporter to take a long position in sterling futures – i.e. a commitment to buy sterling at the rate of $1.6300/£ – with the intention of closing out the contract on 1 February, prior to the receipt of the dollars. If sterling does strengthen against the dollar, this position will result in a gain. However, if the exchange rate falls, then the exporter will lose on the futures contract, but gain in the cash market.

The number of sterling futures contracts necessary to cover the exposure is:

6300.1500,407$ × 500,62£

1 = 4 contracts

The gain/loss on the futures transaction is calculated as follows:

� Buy four March contracts in December at $1.6300/£:

4 x £62,500 × $1.6300 = $407,500

Page 395: libvolume5.xyzlibvolume5.xyz/law/ballb/3rdyear/semester5/corporatelaw/corporate... · ABE Advanced Diploma in Business Administration Study Manual CORPORATE FINANCE Contents Study

International Trade and Finance 387

© Licensed to ABE

� Sell four March futures contracts in February at $1.6355

4 x £62,500 × $1.6355 = $408,875

� Gain through closing out:

$408,875 – $407,500 = $1,375

Converting this into sterling at the February spot rate gives a gain of:

6390.1375,1$ = £839

The total proceeds from the futures hedge is calculated by adding this gain to the proceeds of the exchange of the dollars received on 1 February at the then current spot rate of 1.6390$/£:

6390.1500,407$ = £248,627 + £839 = £249,466

This is marginally worse than the hedge using the forward contract.

Currency Options A currency option gives the holder the right, but not the obligation, to buy (in the case of a call option) or sell (a put option) a specified amount of currency at an agreed exchange rate (the exercise price) at a specific future date.

The principles of currency options are the same as those discussed in the previous unit in respect of other types of option. Thus, using the options market to hedge exchange rate exposure sets a limit on the loss that can be made in the case of adverse movements in exchange rates, but also allows the holder to take advantage of favourable movements.

The following example illustrates their use.

At the beginning of July, a UK company purchased goods to the value of $250,000 from its US supplier on three months’ credit, payable at the end of September. The spot exchange rate is currently $1.62/£, and for the purposes of the example, we shall assume that it falls to $1.55/£ by the end of September, coinciding with the expiry of the option.

Because the company needs to pay for the goods in dollars, it needs a strategy which enables it to sell pounds and buy dollars. The two choices are a long put or a short call. The short call, though, can only provide protection against exchange rate losses up to the cost of the premium, so the favoured strategy would be a long put. (Check with the previous unit to ensure that you understand the various pay-off profiles for these different types of option.)

The relevant sterling options offered on the Philadelphia exchange (the major market for currency options) are at the following prices:

Strike price September puts

1.6 2.32

1.61 2.65

1.62 3.22

Contracts expire on a monthly basis and are for denominations of exactly half of those for futures contracts. Thus, the standard contract sizes for sterling options are £31,500.

Page 396: libvolume5.xyzlibvolume5.xyz/law/ballb/3rdyear/semester5/corporatelaw/corporate... · ABE Advanced Diploma in Business Administration Study Manual CORPORATE FINANCE Contents Study

388 International Trade and Finance

© Licensed to ABE

In this case, the company decides to buy a September put option with a strike price of $1.6/£. It could have opted for a different strike price, but this would have incurred higher premiums (albeit for a higher degree of protection).

The strategy works in the following way:

� The company needs to raise $250,000 which, at the exercise price of $1.6/£, equates to £156,250. To cover this amount, it will need to purchase five standard contracts. The premium paid will be:

$3,625 5 £31,250 x 100

32.2 =×

In sterling, at the current exchange rate, that is:

£2,238 62.1625,3 =

� Because the spot exchange rate has declined during the period (the dollar having strengthened), it is advantageous to exercise the put option – i.e. less pounds will need to be exchanged at the exercise price than at the spot price to buy the required amount of dollars.

Total proceeds from exercising all five option contracts:

5 × £31,250 × $1.6 = $250,000 (matching the liability)

The net sterling cost of the transaction will be:

£156,250 – £2,238 = £154,012

If the option was not exercised, then the liability in dollars would need to be realised by selling sterling on the spot market. The cost involved here would be:

£161,290 55.1

000,250$ =

Thus, using a long put results in a saving of:

£161,290 – £154,012 = £7,278

Money Market Hedge The money market can be used to hedge against exchange rate fluctuations by borrowing an amount in foreign currency equal to the value of, say, invoiced exported goods, exchanging it for the domestic currency at the spot rate, and then using the receipts from the customer to repay the loan.

Effectively, this method uses the matching principle we saw earlier in respect of internal hedging, but applies it to the creation of an asset/liability in the money market, to match the liability/asset which needs to be hedged.

Thus, a UK exporter due a sum of dollars in three months’ time may eliminate the exchange rate exposure by borrowing the sum of dollars at the outset – creating a matching liability. It can then exchange the dollars for sterling at the current spot rate, fixing the exchange rate on the transaction. The sterling can then be invested for the three months. If the money markets and the foreign exchange markets are in equilibrium, we can expect that interest rate parity holds and the interest earned on the sterling investment will offset any change in the exchange rate. The dollars received can be used to pay off the loan, plus interest accrued, in three months’ time. This should, then, provide the same result as a forward currency hedge.

Page 397: libvolume5.xyzlibvolume5.xyz/law/ballb/3rdyear/semester5/corporatelaw/corporate... · ABE Advanced Diploma in Business Administration Study Manual CORPORATE FINANCE Contents Study

International Trade and Finance 389

© Licensed to ABE

Companies which regularly operate this form of hedging usually hold different accounts with their banks for the major currencies in which they trade. In this way, money can be deposited easily, and interest earned when there are surplus funds, and borrowing (overdraft) facilities are readily available when necessary.

Consider the following example.

A UK company is due $500,000 in three months’ time from a customer in the USA. The interest rate is 6% pa and the spot exchange rate is $1.6/£.

The company stands to lose value in sterling on the asset (the $½m) if the dollar weakens against the pound. To hedge this in the money market involves creating a matching dollar liability – a loan equivalent to $½m in three months’ time – exchanging this for sterling (thereby fixing the exchange rate on the transaction) and investing the proceeds for three months, and then using the receipt of the $500,000 to pay off the loan, plus interest accrued. The process is as follows.

� We first need to calculate the amount of dollars to be borrowed now, so that it will grow, with interest, over the three months to $½m. We shall call this amount “Q”. The interest rate applied over the three month period is 1.5% (one quarter of the annual rate of 6%), so the sum borrowed will be:

Q × 1.015 = $500,000

= 015.1

000,500$

= $492,611

The UK company, therefore, only needs to borrow $492,611 now in order to create a liability of $½m in three months’ time.

� Now we convert this sum into sterling at today’s spot rate, thereby fixing the exchange rate and eliminating the exposure:

6.1611,492$ = £307,882

� This sum is now available to the company in sterling and can be put on deposit in the money market for three months, earning interest of 6% pa (or 1.5% over the period):

£307,882 × 1.015 = £312,500

This return, assuming interest rate parity holds, will equate with a sterling forward market hedge on the $½m.

� In three months’ time, the receipt of the $½m from the US company will be used to repay the bank for the $492,611 loan, plus the interest accrued (which should equate to $½m).

This is a highly simplified example, but it does, nevertheless, illustrate the process. We can show it diagrammatically as in Figure 15.2.

Page 398: libvolume5.xyzlibvolume5.xyz/law/ballb/3rdyear/semester5/corporatelaw/corporate... · ABE Advanced Diploma in Business Administration Study Manual CORPORATE FINANCE Contents Study

390 International Trade and Finance

© Licensed to ABE

Figure 15.2: Money market hedge

Liability (Bank loan)

Asset (Debtor) $500,000

$500,000

£312,500

Three months’ time

$492,611 Bank

(invest) £307,882@ 1.5%

US customer

UK Company

Today